Site Loader

Π‘ΠΎΠ΄Π΅Ρ€ΠΆΠ°Π½ΠΈΠ΅

Π—Π°ΠΊΠΎΠ½Ρ‹ постоянного Ρ‚ΠΎΠΊΠ° — Умскул Π£Ρ‡Π΅Π±Π½ΠΈΠΊ

На этой страницС Π²Ρ‹ ΡƒΠ·Π½Π°Π΅Ρ‚Π΅
  • Π§Ρ‚ΠΎ ΠΎΠ±Ρ‰Π΅Π³ΠΎ Ρƒ элСктричСского Ρ‚ΠΎΠΊΠ° с Π²ΠΎΠ΄ΠΎΠΉ?
  • Π’ Ρ‡Π΅ΠΌ ΠΎΡ‚Π»ΠΈΡ‡ΠΈΠ΅ сопротивлСния ΠΎΡ‚ ΡƒΠ΄Π΅Π»ΡŒΠ½ΠΎΠ³ΠΎ сопротивлСния?
  • ΠŸΠΎΡ‡Π΅ΠΌΡƒ Π½Π΅ΠΆΠ΅Π»Π°Ρ‚Π΅Π»ΡŒΠ½ΠΎ ΠΈΡΠΏΠΎΠ»ΡŒΠ·ΠΎΠ²Π°Ρ‚ΡŒ Ρ‚Π΅Π»Π΅Ρ„ΠΎΠ½, ΠΏΠΎΠ΄ΠΊΠ»ΡŽΡ‡Π΅Π½Π½Ρ‹ΠΉ ΠΊ зарядкС?
  • Ѐамилия ΠΊΠ°ΠΊΠΎΠ³ΠΎ ΡƒΡ‡Π΅Π½ΠΎΠ³ΠΎ стоит ΠΌΠΈΠ»Π»ΠΈΠΎΠ½?

«ВсС, ΠΊΠΈΠ½Π° Π½Π΅ Π±ΡƒΠ΄Π΅Ρ‚. ЭлСктричСство ΠΊΠΎΠ½Ρ‡ΠΈΠ»ΠΎΡΡŒΒ». НавСрноС, Π½ΠΈΠΊΠΎΠ³ΠΎ Π½Π΅ оставит Ρ€Π°Π²Π½ΠΎΠ΄ΡƒΡˆΠ½Ρ‹ΠΌ популярная Ρ„Ρ€Π°Π·Π° ΠΈΠ· ΡˆΠΈΡ€ΠΎΠΊΠΎ извСстного Ρ„ΠΈΠ»ΡŒΠΌΠ° Β«Π”ΠΆΠ΅Π½Ρ‚Π»ΡŒΠΌΠ΅Π½Ρ‹ ΡƒΠ΄Π°Ρ‡ΠΈΒ». Π’Π΅Π΄ΡŒ Π΄Π΅ΠΉΡΡ‚Π²ΠΈΡ‚Π΅Π»ΡŒΠ½ΠΎ: бСсит, ΠΊΠΎΠ³Π΄Π° сидишь Π·Π° просмотром любимого ΡΠ΅Ρ€ΠΈΠ°Π»ΡŒΡ‡ΠΈΠΊΠ°, Π²Π΄Ρ€ΡƒΠ³ β€” бамс! Π’Ρ‹Ρ€ΡƒΠ±ΠΈΠ»ΠΈ свСт, ΠΈ зарядки Π½ΠΎΡƒΡ‚Π°, ΠΊΠ°ΠΊ Π½Π°Π·Π»ΠΎ, Π½Π΅ Ρ…Π²Π°Ρ‚ΠΈΠ»ΠΎ. И Π½Π΅ Π²Ρ‹Ρ€Π°Π±ΠΎΡ‚Π°Π΅ΡˆΡŒ элСктричСство Π² Π΄ΠΎΠΌΠ°ΡˆΠ½ΠΈΡ… условиях, Π° ΠΆΠ°Π»ΡŒβ€¦ Но Π²ΠΎΡ‚ ΠΏΠΎΠ½ΡΡ‚ΡŒ, ΠΊΠ°ΠΊ ΠΎΠ½ΠΎ Ρ€Π°Π±ΠΎΡ‚Π°Π΅Ρ‚ β€” это ΠΌΡ‹ смоТСм ΡΠ΄Π΅Π»Π°Ρ‚ΡŒ Π² ΡΡ‚Π°Ρ‚ΡŒΠ΅.

ЭлСктричСский Ρ‚ΠΎΠΊ

Π’ нашС врСмя Ρ‚Ρ€ΡƒΠ΄Π½ΠΎ сСбС ΠΏΡ€Π΅Π΄ΡΡ‚Π°Π²ΠΈΡ‚ΡŒ Тизнь Π±Π΅Π· элСктричСства. Π’Π΅Π»Π΅Π²ΠΈΠ·ΠΎΡ€ Π½Π΅ ΠΏΠΎΡΠΌΠΎΡ‚Ρ€Π΅Ρ‚ΡŒ, Ρ‚Π΅Π»Π΅Ρ„ΠΎΠ½ Π½Π΅ Π·Π°Ρ€ΡΠ΄ΠΈΡ‚ΡŒ, Ρ‡Π°ΠΉ Π½Π΅ ΠΏΠΎΠΏΠΈΡ‚ΡŒβ€¦ Ни ΠΎΠ΄ΠΈΠ½ элСктроприбор Π² Π΄ΠΎΠΌΠ΅ Π½Π΅ Π±ΡƒΠ΄Π΅Ρ‚ Ρ€Π°Π±ΠΎΡ‚Π°Ρ‚ΡŒ Π±Π΅Π· элСктричСства. А объявлСниС ΠΎΠ± ΠΎΡ‚ΠΊΠ»ΡŽΡ‡Π΅Π½ΠΈΠΈ элСктроэнСргии, Π²Ρ‹Π·Ρ‹Π²Π°Π΅Ρ‚ Ρ‚ΠΈΡ…ΠΈΠΉ уТас.

ЭлСктричСство β€” это Ρ„ΠΎΡ€ΠΌΠ° энСргии, которая сущСствуСт Π² Π²ΠΈΠ΄Π΅ статичСских ΠΈΠ»ΠΈ ΠΏΠΎΠ΄Π²ΠΈΠΆΠ½Ρ‹Ρ… элСктричСских зарядов.

Π§Ρ‚ΠΎ ΠΎΠ±Ρ‰Π΅Π³ΠΎ Ρƒ элСктричСского Ρ‚ΠΎΠΊΠ° с Π²ΠΎΠ΄ΠΎΠΉ?

А Ρ‡Ρ‚ΠΎ ΠΆΠ΅ Ρ‚ΠΎΠ³Π΄Π° Ρ‚Π°ΠΊΠΎΠ΅ элСктричСский Ρ‚ΠΎΠΊ? Π’ ΠΏΡ€ΠΈΠ½Ρ†ΠΈΠΏΠ΅, Π΅Π³ΠΎ ΠΌΠΎΠΆΠ½ΠΎ ΡΡ€Π°Π²Π½ΠΈΡ‚ΡŒ с Π²ΠΎΠ΄ΠΎΠΉ. Из Ρ‡Π΅Π³ΠΎ состоит Π²ΠΎΠ΄Π°? Из ΠΌΠΎΠ»Π΅ΠΊΡƒΠ». Когда эти ΠΌΠΎΠ»Π΅ΠΊΡƒΠ»Ρ‹ двиТутся Π² ΠΎΠ΄Π½ΠΎΠΌ Π½Π°ΠΏΡ€Π°Π²Π»Π΅Π½ΠΈΠΈ, Ρ‚ΠΎ ΠΎΠ½ΠΈ ΠΎΠ±Ρ€Π°Π·ΡƒΡŽΡ‚ ΠΏΠΎΡ‚ΠΎΠΊ Π²ΠΎΠ΄Ρ‹, ΠΊΠΎΡ‚ΠΎΡ€Ρ‹ΠΉ Ρ‚Π΅Ρ‡Π΅Ρ‚, Π½Π°ΠΏΡ€ΠΈΠΌΠ΅Ρ€, ΠΏΠΎ Ρ‚Ρ€ΡƒΠ±Π°ΠΌ.

Π’Π°ΠΊ ΠΆΠ΅ ΠΈ элСктричСский Ρ‚ΠΎΠΊ. Он образуСтся ΠΏΠΎΡ‚ΠΎΠΊΠΎΠΌ заряТСнных частиц, ΠΊΠΎΡ‚ΠΎΡ€Ρ‹Π΅ двиТутся ΠΏΠΎ ΠΏΡ€ΠΎΠ²ΠΎΠ΄Π°ΠΌ.Β 

Π‘Ρ„ΠΎΡ€ΠΌΡƒΠ»ΠΈΡ€ΡƒΠ΅ΠΌ ΠΎΠΏΡ€Π΅Π΄Π΅Π»Π΅Π½ΠΈΠ΅:

ЭлСктричСский Ρ‚ΠΎΠΊ β€” это упорядочСнноС Π΄Π²ΠΈΠΆΠ΅Π½ΠΈΠ΅ заряТСнных частиц.

Π§Ρ‚ΠΎΠ±Ρ‹ элСктричСский Ρ‚ΠΎΠΊ сущСствовал, Π½Π΅ΠΎΠ±Ρ…ΠΎΠ΄ΠΈΠΌΠΎ Π²Ρ‹ΠΏΠΎΠ»Π½Π΅Π½ΠΈΠ΅ ΡΠ»Π΅Π΄ΡƒΡŽΡ‰ΠΈΡ… условий:

β€” Π½Π°Π»ΠΈΡ‡ΠΈΠ΅ свободных заряТСнных частиц;
β€” Π½Π°Π»ΠΈΡ‡ΠΈΠ΅ элСктричСского поля;
β€” Π½Π°Π»ΠΈΡ‡ΠΈΠ΅ Π·Π°ΠΌΠΊΠ½ΡƒΡ‚ΠΎΠΉ элСктричСской Ρ†Π΅ΠΏΠΈ.

ΠžΡΠ½ΠΎΠ²Π½Ρ‹ΠΌΠΈ количСствСнными характСристиками элСктричСского Ρ‚ΠΎΠΊΠ° ΡΠ²Π»ΡΡŽΡ‚ΡΡ сила Ρ‚ΠΎΠΊΠ° ΠΈ напряТСниС, основной элСктричСской характСристикой ΠΏΡ€ΠΎΠ²ΠΎΠ΄Π½ΠΈΠΊΠ° являСтся элСктричСскоС сопротивлСниС.

Π‘ΠΈΠ»Π° Ρ‚ΠΎΠΊΠ°

Π‘ΠΈΠ»Π° Ρ‚ΠΎΠΊΠ° β€” это физичСская Π²Π΅Π»ΠΈΡ‡ΠΈΠ½Π°, ΠΏΠΎΠΊΠ°Π·Ρ‹Π²Π°ΡŽΡ‰Π°Ρ, ΠΊΠ°ΠΊΠΎΠΉ заряд пСрСносится Ρ‡Π΅Ρ€Π΅Π· Ρ€Π°ΡΡΠΌΠ°Ρ‚Ρ€ΠΈΠ²Π°Π΅ΠΌΡƒΡŽ ΠΏΠ»ΠΎΡ‰Π°Π΄ΡŒ ΠΏΠΎΠΏΠ΅Ρ€Π΅Ρ‡Π½ΠΎΠ³ΠΎ сСчСния Π·Π° Π΅Π΄ΠΈΠ½ΠΈΡ†Ρƒ Π²Ρ€Π΅ΠΌΠ΅Π½ΠΈ .Β 

\(I = \frac{q}{t}\), Π³Π΄Π΅

I β€” сила Ρ‚ΠΎΠΊΠ° (А),
q β€” элСктричСский заряд (Кл),Β 
t β€” врСмя (с).

Π•Π΄ΠΈΠ½ΠΈΡ†Π° измСрСния I β€” А (Π°ΠΌΠΏΠ΅Ρ€) = \(\frac{Кл}{с}\).

ΠŸΡ€Π΅Π΄ΡΡ‚Π°Π²ΠΈΠΌ, Ρ‡Ρ‚ΠΎ Π²Π½ΡƒΡ‚Ρ€ΠΈ ΠΏΡ€ΠΎΠ²ΠΎΠ΄Π½ΠΈΠΊΠ° β€œΠ±Π΅Π³ΡƒΡ‚β€ Π² ΠΎΠ΄Π½ΠΎΠΌ Π½Π°ΠΏΡ€Π°Π²Π»Π΅Π½ΠΈΠΈ ΠΎΠ³Ρ€ΠΎΠΌΠ½ΠΎΠ΅ количСство заряТСнных частиц. Π’Π°ΠΊ Π²ΠΎΡ‚, Ρ‡Π΅ΠΌ большС этих частиц ΠΏΡ€ΠΎΠ±Π΅ΠΆΠΈΡ‚ Π·Π° Π΅Π΄ΠΈΠ½ΠΈΡ†Ρƒ Π²Ρ€Π΅ΠΌΠ΅Π½ΠΈ, Ρ‚Π΅ΠΌ большС Π±ΡƒΠ΄Π΅Ρ‚ Π·Π½Π°Ρ‡Π΅Π½ΠΈΠ΅ силы Ρ‚ΠΎΠΊΠ°.

ΠŸΡ€ΠΈΠ±ΠΎΡ€ для измСрСния силы Ρ‚ΠΎΠΊΠ° β€” Π°ΠΌΠΏΠ΅Ρ€ΠΌΠ΅Ρ‚Ρ€. Он Π²ΠΊΠ»ΡŽΡ‡Π°Π΅Ρ‚ΡΡ Π² Ρ†Π΅ΠΏΡŒ ΠΏΠΎΡΠ»Π΅Π΄ΠΎΠ²Π°Ρ‚Π΅Π»ΡŒΠ½ΠΎ. ΠŸΡ€ΠΈΠΌΠ΅Ρ€ ΠΏΠΎΠ΄ΠΊΠ»ΡŽΡ‡Π΅Π½ΠΈΡ прСдставлСн Π½Π° рисункС:

Π§Ρ‚ΠΎΠ±Ρ‹ Π²Π½ΡƒΡ‚Ρ€ΠΈ Ρ†Π΅ΠΏΠΈ сущСствовал элСктричСский Ρ‚ΠΎΠΊ, Ρ†Π΅ΠΏΡŒ Π΄ΠΎΠ»ΠΆΠ½Π° Π±Ρ‹Ρ‚ΡŒ Π·Π°ΠΌΠΊΠ½ΡƒΡ‚Π° ΠΈ ΠΌΠ΅ΠΆΠ΄Ρƒ ΠΊΠΎΠ½Ρ†Π°ΠΌΠΈ участка Ρ†Π΅ΠΏΠΈ Π΄ΠΎΠ»ΠΆΠ½ΠΎ ΡΡƒΡ‰Π΅ΡΡ‚Π²ΠΎΠ²Π°Ρ‚ΡŒ напряТСниС.

НапряТСниС

НапряТСниС Β β€” скалярная физичСская Π²Π΅Π»ΠΈΡ‡ΠΈΠ½Π°, равная ΠΎΡ‚Π½ΠΎΡˆΠ΅Π½ΠΈΡŽ ΠΏΠΎΠ»Π½ΠΎΠΉ Ρ€Π°Π±ΠΎΡ‚Ρ‹ кулоновских ΠΈ сторонних сил  ΠΏΡ€ΠΈ ΠΏΠ΅Ρ€Π΅ΠΌΠ΅Ρ‰Π΅Π½ΠΈΠΈ ΠΏΠΎΠ»ΠΎΠΆΠΈΡ‚Π΅Π»ΡŒΠ½ΠΎΠ³ΠΎ заряда Π½Π° участкС Ρ†Π΅ΠΏΠΈ ΠΊ Π·Π½Π°Ρ‡Π΅Π½ΠΈΡŽ этого заряда .

\(U = \frac{A}{q}\), Π³Π΄Π΅Β 

U β€” напряТСниС (Π’),
A β€” Ρ€Π°Π±ΠΎΡ‚Π° Ρ‚ΠΎΠΊΠ° Π½Π° участкС Ρ†Π΅ΠΏΠΈ (Π”ΠΆ),Β 
q β€” элСктричСский заряд (Кл).

ЭлСктричСский Ρ‚ΠΎΠΊ Π½Π΅ лСнится. Он Π»ΡŽΠ±ΠΈΡ‚ Ρ€Π°Π±ΠΎΡ‚Π°Ρ‚ΡŒ. А Ρ€Π°Π±ΠΎΡ‚Π°Ρ‚ΡŒ ΠΎΠ½ Π±ΡƒΠ΄Π΅Ρ‚ ΠΏΡ€ΠΈ Π΄Π²ΠΈΠΆΠ΅Π½ΠΈΠΈ частиц, Ρ‚ΠΎ Π΅ΡΡ‚ΡŒ ΠΊΠΎΠ³Π΄Π° частицы Π±ΡƒΠ΄ΡƒΡ‚ ΠΏΠ΅Ρ€Π΅ΠΌΠ΅Ρ‰Π°Ρ‚ΡŒΡΡ ΠΈΠ· ΠΎΠ΄Π½ΠΎΠ³ΠΎ ΠΊΠΎΠ½Ρ†Π° Ρ†Π΅ΠΏΠΈ Π² Π΄Ρ€ΡƒΠ³ΠΎΠΉ. И Ρ‡Π΅ΠΌ большС ΠΎΠ½ΠΈ Π±ΡƒΠ΄ΡƒΡ‚ Ρ€Π°Π±ΠΎΡ‚Π°Ρ‚ΡŒ, Ρ‚Π΅ΠΌ большСС напряТСниС получится.

Π•Π΄ΠΈΠ½ΠΈΡ†Π° измСрСния U β€” Π’ (Π’ΠΎΠ»ΡŒΡ‚) = \(\frac{Π”ΠΆ}{Кл}\)

ΠŸΡ€ΠΈΠ±ΠΎΡ€ для измСрСния силы Ρ‚ΠΎΠΊΠ° β€” Π²ΠΎΠ»ΡŒΡ‚ΠΌΠ΅Ρ‚Ρ€. Он Π²ΠΊΠ»ΡŽΡ‡Π°Π΅Ρ‚ΡΡ Π² Ρ†Π΅ΠΏΡŒ ΠΏΠ°Ρ€Π°Π»Π»Π΅Π»ΡŒΠ½ΠΎ.Β 

ΠŸΡ€ΠΈΠΌΠ΅Ρ€ ΠΏΠΎΠ΄ΠΊΠ»ΡŽΡ‡Π΅Π½ΠΈΡ прСдставлСн Π½Π° рисункС:

НаправлСниС Ρ‚ΠΎΠΊΠ° совпадаСт с Π½Π°ΠΏΡ€Π°Π²Π»Π΅Π½ΠΈΠ΅ΠΌ двиТСния ΠΏΠΎΠ»ΠΎΠΆΠΈΡ‚Π΅Π»ΡŒΠ½ΠΎ заряТСнных частиц..

ЭлСктричСскоС сопротивлСниС

Π‘ΠΎΠΏΡ€ΠΎΡ‚ΠΈΠ²Π»Π΅Π½ΠΈΠ΅ β€” физичСская Π²Π΅Π»ΠΈΡ‡ΠΈΠ½Π°, Ρ…Π°Ρ€Π°ΠΊΡ‚Π΅Ρ€ΠΈΠ·ΡƒΡŽΡ‰Π°Ρ элСктричСскиС свойства участка Ρ†Π΅ΠΏΠΈ.

\(R = \frac{pl}{S}\), Π³Π΄Π΅Β 

R β€” сопротивлСниС (Ом),
p β€” ΡƒΠ΄Π΅Π»ΡŒΠ½ΠΎΠ΅ сопротивлСниС ΠΏΡ€ΠΎΠ²ΠΎΠ΄Π½ΠΈΠΊΠ°,Β 
l β€” Π΄Π»ΠΈΠ½Π° ΠΏΡ€ΠΎΠ²ΠΎΠ΄Π½ΠΈΠΊΠ° (ΠΌ),S β€” ΠΏΠ»ΠΎΡ‰Π°Π΄ΡŒ ΠΏΠΎΠΏΠ΅Ρ€Π΅Ρ‡Π½ΠΎΠ³ΠΎ сСчСния ΠΏΡ€ΠΎΠ²ΠΎΠ΄Π½ΠΈΠΊΠ° (ΠΌΠΌΒ²).

Π•Π΄ΠΈΠ½ΠΈΡ†Π° измСрСния R β€” Ом.

УдСльноС сопротивлСниС ΠΏΡ€ΠΎΠ²ΠΎΠ΄Π½ΠΈΠΊΠ° ΠΌΠΎΠΆΠ½ΠΎ ΠΏΠΎΡΠΌΠΎΡ‚Ρ€Π΅Ρ‚ΡŒ Π² Ρ‚Π°Π±Π»ΠΈΡ†Π΅ Π½ΠΈΠΆΠ΅, для ΠΊΠ°ΠΆΠ΄ΠΎΠ³ΠΎ ΠΌΠ°Ρ‚Π΅Ρ€ΠΈΠ°Π»Π° Π±ΡƒΠ΄Π΅Ρ‚ своС Π·Π½Π°Ρ‡Π΅Π½ΠΈΠ΅.

Π’ Ρ‡Π΅ΠΌ ΠΎΡ‚Π»ΠΈΡ‡ΠΈΠ΅ сопротивлСния ΠΎΡ‚ ΡƒΠ΄Π΅Π»ΡŒΠ½ΠΎΠ³ΠΎ сопротивлСния?

Π‘ΠΎΠΏΡ€ΠΎΡ‚ΠΈΠ²Π»Π΅Π½ΠΈΠ΅ β€” это внСшнСС свойство, зависящСС ΠΎΡ‚ количСства ΠΏΡ€ΠΈΡΡƒΡ‚ΡΡ‚Π²ΡƒΡŽΡ‰Π΅Π³ΠΎ ΠΌΠ°Ρ‚Π΅Ρ€ΠΈΠ°Π»Π°.Β 
УдСльноС сопротивлСниС β€” это Π²Π½ΡƒΡ‚Ρ€Π΅Π½Π½Π΅Π΅ свойство ΠΏΡ€ΠΎΠ²ΠΎΠ΄Π½ΠΈΠΊΠ°, ΠΊΠΎΡ‚ΠΎΡ€ΠΎΠ΅ Π½Π΅ зависит ΠΎΡ‚ Π΅Π³ΠΎ Ρ€Π°Π·ΠΌΠ΅Ρ€Π°.

ΠŸΠΎΠ»ΡƒΡ‡Π°Π΅Ρ‚ΡΡ, Ρ‡Ρ‚ΠΎ ΠΏΡ€Π΅ΠΆΠ΄Π΅ всСго Π½Π° Ρ‚ΠΎ, ΠΊΠ°ΠΊΠΈΠΌ Π±ΡƒΠ΄Π΅Ρ‚ сопротивлСниС, Π²Π»ΠΈΡΡŽΡ‚ Ρ€Π°Π·ΠΌΠ΅Ρ€Ρ‹ ΠΏΡ€ΠΎΠ²ΠΎΠ΄Π½ΠΈΠΊΠ°, Π΅Π³ΠΎ Ρ„ΠΎΡ€ΠΌΠ°, ΠΌΠ°Ρ‚Π΅Ρ€ΠΈΠ°Π», ΠΈΠ· ΠΊΠΎΡ‚ΠΎΡ€ΠΎΠ³ΠΎ ΠΎΠ½ сдСлан.Β 

УдСльноС сопротивлСниС ΠΏΡ€ΠΎΠ²ΠΎΠ΄Π½ΠΈΠΊΠ° зависит Ρ‚Π°ΠΊΠΆΠ΅ ΠΎΡ‚ Ρ‚Π΅ΠΌΠΏΠ΅Ρ€Π°Ρ‚ΡƒΡ€Ρ‹. Когда Ρ‚Π΅ΠΌΠΏΠ΅Ρ€Π°Ρ‚ΡƒΡ€Π° Ρ‚Π²Π΅Ρ€Π΄Ρ‹Ρ… Ρ‚Π΅Π» увСличиваСтся, Ρ‚ΠΎ ΡƒΠ΄Π΅Π»ΡŒΠ½ΠΎΠ΅ сопротивлСниС возрастаСт. А Π² растворах ΠΈ расплавах β€” Π½Π°ΠΎΠ±ΠΎΡ€ΠΎΡ‚, ΡƒΠΌΠ΅Π½ΡŒΡˆΠ°Π΅Ρ‚ΡΡ.

НСкоторыС значСния ΠΏΡ€ΠΈΠ²Π΅Π΄Π΅Π½Ρ‹ Π² Ρ‚Π°Π±Π»ΠΈΡ†Π΅:

Π’Π°Π±Π»ΠΈΡ†Π° ΡƒΠ΄Π΅Π»ΡŒΠ½Ρ‹Ρ… сопротивлСний ΠΏΡ€ΠΎΠ²ΠΎΠ΄Π½ΠΈΠΊΠΎΠ²

ΠœΠ΅Ρ‚Π°Π»Π»Π£Π΄Π΅Π»ΡŒΠ½ΠΎΠ΅ сопротивлСниС, Ом * ΠΌΠΌ2/ ΠΌ
Π‘Π΅Ρ€Π΅Π±Ρ€ΠΎ0,0015
МСдь0,018
Π—ΠΎΠ»ΠΎΡ‚ΠΎ0,023
Алюминий0,029
Π’ΠΎΠ»ΡŒΡ„Ρ€Π°ΠΌ0,055
Π–Π΅Π»Π΅Π·ΠΎ0,098

Π”Π°Π²Π°ΠΉΡ‚Π΅ ΠΏΠΎΡ€Π°Π·ΠΌΡ‹ΡˆΠ»ΡΠ΅ΠΌ: Ρ‡Ρ‚ΠΎ Ρ‡Π΅ΠΌΡƒ сопротивляСтся?Β 

ΠŸΡ€ΠΈΡ‡ΠΈΠ½Π° элСктричСского сопротивлСния кроСтся Π²ΠΎ взаимодСйствии зарядов Ρ€Π°Π·Π½ΠΎΠ³ΠΎ Π·Π½Π°ΠΊΠ° ΠΏΡ€ΠΈ ΠΏΡ€ΠΎΡ‚Π΅ΠΊΠ°Π½ΠΈΠΈ Ρ‚ΠΎΠΊΠ° ΠΏΠΎ ΠΏΡ€ΠΎΠ²ΠΎΠ΄Π½ΠΈΠΊΡƒ. Π­Ρ‚ΠΎ взаимодСйствиС ΠΌΠΎΠΆΠ½ΠΎ ΡΡ€Π°Π²Π½ΠΈΡ‚ΡŒ с силой трСния, стрСмящСйся ΠΎΡΡ‚Π°Π½ΠΎΠ²ΠΈΡ‚ΡŒ Π΄Π²ΠΈΠΆΠ΅Π½ΠΈΠ΅ заряТСнных частиц.

Π§Π΅ΠΌ сильнСС взаимодСйствиС свободных элСктронов с ΠΏΠΎΠ»ΠΎΠΆΠΈΡ‚Π΅Π»ΡŒΠ½Ρ‹ΠΌΠΈ ΠΈΠΎΠ½Π°ΠΌΠΈ Π² ΡƒΠ·Π»Π°Ρ… кристалличСской Ρ€Π΅ΡˆΠ΅Ρ‚ΠΊΠΈ ΠΏΡ€ΠΎΠ²ΠΎΠ΄Π½ΠΈΠΊΠ°, Ρ‚Π΅ΠΌ большС сопротивлСниС ΠΏΡ€ΠΎΠ²ΠΎΠ΄Π½ΠΈΠΊΠ°.

ΠŸΡ€ΠΎΠ²ΠΎΠ΄Π½ΠΈΠΊ с ΠΎΠΏΡ€Π΅Π΄Π΅Π»Π΅Π½Π½Ρ‹ΠΌ постоянным сопротивлСниСм называСтся рСзистор.

ВСрнСмся ΠΊ ΡΡ€Π°Π²Π½Π΅Π½ΠΈΡŽ элСктричСского Ρ‚ΠΎΠΊΠ° с Π²ΠΎΠ΄ΠΎΠΉ: ΠΊΠ°ΠΊ ΠΌΠΎΠ»Π΅ΠΊΡƒΠ»Ρ‹ Π²ΠΎΠ΄Ρ‹ ΠΈΠ· ΠΊΡ€Π°Π½Π° двиТутся свСрху Π²Π½ΠΈΠ·, Ρ‚Π°ΠΊ ΠΈ элСктричСский Ρ‚ΠΎΠΊ Π½Π°ΠΏΡ€Π°Π²Π»Π΅Π½ ΠΎΡ‚Β  β€œ+” ΠΊ β€œβˆ’β€. ЭлСктричСский заряд соотвСтствуСт массС Π²ΠΎΠ΄Ρ‹, Π° напряТСниС β€” Π½Π°ΠΏΠΎΡ€Ρƒ Π²ΠΎΠ΄Ρ‹ ΠΈΠ· ΠΊΡ€Π°Π½Π°.

Π—Π°ΠΊΠΎΠ½ Ома

Π‘ΠΈΠ»Π° Ρ‚ΠΎΠΊΠ°, напряТСниС ΠΈ сопротивлСниС связаны ΠΌΠ΅ΠΆΠ΄Ρƒ собой ΡΠΎΠΎΡ‚Π½ΠΎΡˆΠ΅Π½ΠΈΠ΅ΠΌ, ΠΊΠΎΡ‚ΠΎΡ€ΠΎΠ΅ называСтся Π·Π°ΠΊΠΎΠ½ Ома:

Π‘ΠΈΠ»Π° Ρ‚ΠΎΠΊΠ° Π² ΠΎΠ΄Π½ΠΎΡ€ΠΎΠ΄Π½ΠΎΠΌ участкС Ρ†Π΅ΠΏΠΈ прямо ΠΏΡ€ΠΎΠΏΠΎΡ€Ρ†ΠΈΠΎΠ½Π°Π»ΡŒΠ½Π° Π½Π°ΠΏΡ€ΡΠΆΠ΅Π½ΠΈΡŽ ΠΏΡ€ΠΈ постоянном сопротивлСнии ΠΈ ΠΎΠ±Ρ€Π°Ρ‚Π½ΠΎ ΠΏΡ€ΠΎΠΏΠΎΡ€Ρ†ΠΈΠΎΠ½Π°Π»ΡŒΠ½Π° ΡΠΎΠΏΡ€ΠΎΡ‚ΠΈΠ²Π»Π΅Π½ΠΈΡŽ участка ΠΏΡ€ΠΈ постоянном напряТСнии.

\(I = \frac{U}{R}\) , Π³Π΄Π΅Β 

I β€” сила Ρ‚ΠΎΠΊΠ° (А),
U β€” напряТСниС (Π’),Β 
R β€” сопротивлСниС (Ом).

Π”Π°Π½Π½Ρ‹ΠΉ Π·Π°ΠΊΠΎΠ½ справСдлив для участка Ρ†Π΅ΠΏΠΈ, Π½Π° ΠΊΠΎΡ‚ΠΎΡ€Ρ‹ΠΉ Π½Π΅ Π΄Π΅ΠΉΡΡ‚Π²ΡƒΡŽΡ‚ сторонниС силы.

Π Π°Π±ΠΎΡ‚Π° ΠΈ ΠΌΠΎΡ‰Π½ΠΎΡΡ‚ΡŒ элСктричСского Ρ‚ΠΎΠΊΠ°

ВСрнСмся ΠΊ ΠΏΠΎΠ½ΡΡ‚ΠΈΡŽ Ρ€Π°Π±ΠΎΡ‚Ρ‹. ΠœΡ‹ Π³ΠΎΠ²ΠΎΡ€ΠΈΠ»ΠΈ, ΠΏΡ€ΠΈ ΠΏΠ΅Ρ€Π΅ΠΌΠ΅Ρ‰Π΅Π½ΠΈΠΈ заряда ΠΏΠΎ ΠΏΡ€ΠΎΠ²ΠΎΠ΄Π½ΠΈΠΊΡƒ элСктричСскоС ΠΏΠΎΠ»Π΅ ΡΠΎΠ²Π΅Ρ€ΡˆΠ°Π΅Ρ‚ Ρ€Π°Π±ΠΎΡ‚Ρƒ (А):

A = qU

Если ΠΌΡ‹ Π²Ρ‹Ρ€Π°Π·ΠΈΠΌ заряд ΠΈΠ· Ρ„ΠΎΡ€ΠΌΡƒΠ»Ρ‹ силы Ρ‚ΠΎΠΊΠ° q=It, Ρ‚ΠΎ ΠΏΠΎΠ»ΡƒΡ‡ΠΈΠΌ, Ρ„ΠΎΡ€ΠΌΡƒΠ»Ρƒ для расчСта Ρ€Π°Π±ΠΎΡ‚Ρ‹ элСктричСского поля (А) ΠΏΡ€ΠΈ ΠΏΡ€ΠΎΡ‚Π΅ΠΊΠ°Π½ΠΈΠΈ постоянного Ρ‚ΠΎΠΊΠ° (ΠΈΠ»ΠΈ просто Ρ€Π°Π±ΠΎΡ‚Π° Ρ‚ΠΎΠΊΠ°):

А = UIt , гдС

A β€” Ρ€Π°Π±ΠΎΡ‚Π° элСктричСского Ρ‚ΠΎΠΊΠ° (Π”ΠΆ),
U β€” напряТСниС (Π’),
I β€” сила Ρ‚ΠΎΠΊΠ° (А),
t β€” врСмя прохоТдСния Ρ‚ΠΎΠΊΠ° (с).

Π•Π΄ΠΈΠ½ΠΈΡ†Π° измСрСния А β€” Π”ΠΆ (Π”ΠΆΠΎΡƒΠ»ΡŒ).

Π’ Π±Ρ‹Ρ‚Ρƒ Ρ‚ΠΎΠΊ ΡΠΎΠ²Π΅Ρ€ΡˆΠ°Π΅Ρ‚ Ρ€Π°Π±ΠΎΡ‚Ρƒ Π΄Π»ΠΈΡ‚Π΅Π»ΡŒΠ½ΠΎΠ΅ врСмя, поэтому ΠΏΡ€ΠΈ ΠΎΠΏΡ€Π΅Π΄Π΅Π»Π΅Π½ΠΈΠΈ Π·Π°Ρ‚Ρ€Π°Ρ‡Π΅Π½Π½ΠΎΠΉ элСктричСской энСргии ΠΈΡΠΏΠΎΠ»ΡŒΠ·ΡƒΡŽΡ‚ Π΅Π΄ΠΈΠ½ΠΈΡ†Ρƒ измСрСния ΠΊΠ’Ρ‚ * Ρ‡. ΠšΠΈΠ»ΠΎΠ²Π°Ρ‚Ρ‚ Π² час β€” это энСргия, которая потрСбляСтся устройством ΠΌΠΎΡ‰Π½ΠΎΡΡ‚ΡŒΡŽ 1 ΠΊΠ’Ρ‚ Π² Ρ‚Π΅Ρ‡Π΅Π½ΠΈΠΈ 1 часа. Учитывая, Ρ‡Ρ‚ΠΎ 1 Ρ‡=3600 с, ΠΏΠΎΠ»ΡƒΡ‡ΠΈΠΌ:

1 ΠΊΠ’Ρ‚*Ρ‡ = 1000 Π’Ρ‚ * 3600 с = 3600000 Π”ΠΆ = 3600 ΠΊΠ”ΠΆ

Если ΠΆΠ΅ Ρ€Π°Π±ΠΎΡ‚Ρƒ Ρ‚ΠΎΠΊΠ° Ρ€Π°ΡΡΡ‡ΠΈΡ‚Π°Ρ‚ΡŒ Π·Π° Π΅Π΄ΠΈΠ½ΠΈΡ†Ρƒ Π²Ρ€Π΅ΠΌΠ΅Π½ΠΈ, Ρ‚ΠΎ ΠΌΡ‹ ΠΏΠΎΠ»ΡƒΡ‡ΠΈΠΌ ΠΌΠΎΡ‰Π½ΠΎΡΡ‚ΡŒ постоянного элСктричСского Ρ‚ΠΎΠΊΠ°.

ΠœΠΎΡ‰Π½ΠΎΡΡ‚ΡŒ β€” Π²Π΅Π»ΠΈΡ‡ΠΈΠ½Π°, ΠΎΠ±ΠΎΠ·Π½Π°Ρ‡Π°ΡŽΡ‰Π°Ρ ΠΈΠ½Ρ‚Π΅Π½ΡΠΈΠ²Π½ΠΎΡΡ‚ΡŒ ΠΏΠ΅Ρ€Π΅Π΄Π°Ρ‡ΠΈ элСктричСской энСргии.

\(P = \frac{A}{t}\) , Π³Π΄Π΅Β 

P β€” ΠΌΠΎΡ‰Π½ΠΎΡΡ‚ΡŒ (Π’Ρ‚),
A β€” Ρ€Π°Π±ΠΎΡ‚Π° элСктричСского Ρ‚ΠΎΠΊΠ° (Π”ΠΆ),Β 
t β€” врСмя прохоТдСния Ρ‚ΠΎΠΊΠ° (с).


Π•Π΄ΠΈΠ½ΠΈΡ†Π° измСрСния P β€” Π’Ρ‚ (Π’Π°Ρ‚Ρ‚).

БрСдняя ΠΌΠΎΡ‰Π½ΠΎΡΡ‚ΡŒ Ρ‚ΠΎΠΊΠ° Ρ€Π°Π²Π½Π°:

\(P = \frac{A}{t} = \frac{qU}{t} = IU = \frac{U^2}{R} = I^2R\)

ΠœΠΎΡ‰Π½ΠΎΡΡ‚ΡŒ элСктроприбора всСгда указываСтся Π² Π΄ΠΎΠΊΡƒΠΌΠ΅Π½Ρ‚Π°Ρ†ΠΈΠΈ, ΠΏΡ€ΠΈΠ»Π°Π³Π°ΡŽΡ‰Π΅ΠΉΡΡ ΠΊ Π½Π΅ΠΌΡƒ. ΠšΡ€ΠΎΠΌΠ΅ Ρ‚ΠΎΠ³ΠΎ, Π½Π΅Ρ€Π΅Π΄ΠΊΠΎ Π΅Π΅ ΠΏΠΈΡˆΡƒΡ‚ Π½Π° самом ΠΏΡ€ΠΈΠ±ΠΎΡ€Π΅. Π”Π°Π²Π°ΠΉΡ‚Π΅ посмотрим Π½Π° ΡƒΡ‚ΡŽΠ³, ΠΈΠ»ΠΈ ΡΡ‚ΠΈΡ€Π°Π»ΡŒΠ½ΡƒΡŽ ΠΌΠ°ΡˆΠΈΠ½Ρƒ Π΄ΠΎΠΌΠ°. ΠœΡ‹ ΡƒΠ²ΠΈΠ΄ΠΈΠΌ, Ρ‡Ρ‚ΠΎ ΡƒΡ‚ΡŽΠ³ ΠΈΠΌΠ΅Π΅Ρ‚ ΠΌΠΎΡ‰Π½ΠΎΡΡ‚ΡŒ 1000 Π’Ρ‚, Π° обычная ΡΠ½Π΅Ρ€Π³ΠΎΡΠ±Π΅Ρ€Π΅Π³Π°ΡŽΡ‰Π°Ρ Π»Π°ΠΌΠΏΠΎΡ‡ΠΊΠ°, всСго 40 Π’Ρ‚ (Π½Π° Ρ‚ΠΎ ΠΎΠ½Π° ΠΈ ΡΠ±Π΅Ρ€Π΅Π³Π°ΡŽΡ‰Π°Ρ). Π§Π΅ΠΌ большС ΠΌΠΎΡ‰Π½ΠΎΡΡ‚ΡŒ ΠΏΡ€ΠΈΠ±ΠΎΡ€Π°, Ρ‚Π΅ΠΌ большС энСргии ΠΎΠ½ Π±ΡƒΠ΄Π΅Ρ‚ ΠΏΠΎΡ‚Ρ€Π΅Π±Π»ΡΡ‚ΡŒ. ΠŸΡ€ΠΈΠΌΠ΅Ρ€Ρ‹ мощностСй Ρ€Π°Π·Π»ΠΈΡ‡Π½Ρ‹Ρ… ΠΏΡ€ΠΈΠ±ΠΎΡ€ΠΎΠ² прСдставлСны Π½Π° рисункС:

Π—Π°ΠΊΠΎΠ½ ДТоуля β€” Π›Π΅Π½Ρ†Π°
ΠŸΠΎΡ‡Π΅ΠΌΡƒ Π½Π΅ΠΆΠ΅Π»Π°Ρ‚Π΅Π»ΡŒΠ½ΠΎ ΠΈΡΠΏΠΎΠ»ΡŒΠ·ΠΎΠ²Π°Ρ‚ΡŒ Ρ‚Π΅Π»Π΅Ρ„ΠΎΠ½, ΠΏΠΎΠ΄ΠΊΠ»ΡŽΡ‡Π΅Π½Π½Ρ‹ΠΉ ΠΊ зарядкС?

Когда ΠΏΡ€ΠΈΠ±ΠΎΡ€Ρ‹ ΠΏΠΎΠ΄ΠΊΠ»ΡŽΡ‡Π΅Π½Ρ‹ Π² ΡΠ΅Ρ‚ΡŒ, ΠΌΡ‹ ΠΌΠΎΠΆΠ΅ΠΌ Π·Π°ΠΌΠ΅Ρ‚ΠΈΡ‚ΡŒ, Ρ‡Ρ‚ΠΎ ΠΎΠ½ΠΈ Π½Π°Π³Ρ€Π΅Π²Π°ΡŽΡ‚ΡΡ. ΠžΡ‡Π΅Π½ΡŒ часто это Π½Π°Π±Π»ΡŽΠ΄Π°Π΅Ρ‚ΡΡ, ΠΊΠΎΠ³Π΄Π° Ρ‚Π΅Π»Π΅Ρ„ΠΎΠ½ ΠΏΠΎΠ΄ΠΊΠ»ΡŽΡ‡Π΅Π½ Π½Π° зарядку, Π° ΠΌΡ‹ ΠΏΡ€ΠΎΠ΄ΠΎΠ»ΠΆΠ°Π΅ΠΌ ΠΏΠΎ Π½Π΅ΠΌΡƒ Π·Π²ΠΎΠ½ΠΈΡ‚ΡŒ, ΠΈΡΠΏΠΎΠ»ΡŒΠ·ΠΎΠ²Π°Ρ‚ΡŒ ΠΈΠ½Ρ‚Π΅Ρ€Π½Π΅Ρ‚ ΠΈ ΠΏΡ€ΠΎΡ‡Π΅Π΅. Π­Ρ‚ΠΎ ΠΏΠ»ΠΎΡ…ΠΎ влияСт Π½Π° Ρ‚Π΅Π»Π΅Ρ„ΠΎΠ½: ΠΏΠ΅Ρ€Π΅Π³Ρ€Π΅Π² Π±Π°Ρ‚Π°Ρ€Π΅ΠΈ ΠΈ корпуса ΠΌΠΎΠ³ΡƒΡ‚ быстрСС привСсти дСвайс Π² Π½Π΅Π³ΠΎΠ΄Π½ΠΎΡΡ‚ΡŒ.Β 

ΠŸΠΎΡ‡Π΅ΠΌΡƒ Ρ‚Π°ΠΊ происходит?

ЭлСктричСский Ρ‚ΠΎΠΊ ΠΎΠΊΠ°Π·Ρ‹Π²Π°Π΅Ρ‚ Ρ‚Π΅ΠΏΠ»ΠΎΠ²ΠΎΠ΅ дСйствиС Π½Π° ΠΏΡ€ΠΎΠ²ΠΎΠ΄Π½ΠΈΠΊ. ΠšΠΎΠ»ΠΈΡ‡Π΅ΡΡ‚Π²ΠΎ Ρ‚Π΅ΠΏΠ»ΠΎΡ‚Ρ‹, ΠΊΠΎΡ‚ΠΎΡ€ΠΎΠ΅ ΠΏΡ€ΠΈ этом выдСляСтся, Π±ΡƒΠ΄Π΅Ρ‚ Ρ€Π°ΡΡΡ‡ΠΈΡ‚Ρ‹Π²Π°Ρ‚ΡŒΡΡ ΠΏΠΎ Π·Π°ΠΊΠΎΠ½Ρƒ ДТоуля β€” Π›Π΅Π½Ρ†Π°:

ΠšΠΎΠ»ΠΈΡ‡Π΅ΡΡ‚Π²ΠΎ Ρ‚Π΅ΠΏΠ»ΠΎΡ‚Ρ‹, выдСляСмоС Π·Π° врСмя Π² ΠΏΡ€ΠΎΠ²ΠΎΠ΄Π½ΠΈΠΊΠ΅ с Ρ‚ΠΎΠΊΠΎΠΌ, ΠΏΡ€ΠΎΠΏΠΎΡ€Ρ†ΠΈΠΎΠ½Π°Π»ΡŒΠ½ΠΎ ΠΏΡ€ΠΎΠΈΠ·Π²Π΅Π΄Π΅Π½ΠΈΡŽ ΠΊΠ²Π°Π΄Ρ€Π°Ρ‚Π° силы Ρ‚ΠΎΠΊΠ° Π½Π° этом участкС ΠΈ сопротивлСния ΠΏΡ€ΠΎΠ²ΠΎΠ΄Π½ΠΈΠΊΠ°:

Q = I2Rt , Π³Π΄Π΅Β 

Q β€” количСство Ρ‚Π΅ΠΏΠ»ΠΎΡ‚Ρ‹ (Π”ΠΆ),
I β€” Ρ€Π°Π±ΠΎΡ‚Π° элСктричСского Ρ‚ΠΎΠΊΠ° (Π”ΠΆ),Β 
R β€” сопротивлСниС (Ом),t β€” врСмя прохоТдСния Ρ‚ΠΎΠΊΠ° (с).

Π•Π΄ΠΈΠ½ΠΈΡ†Π° измСрСния Q β€” Π”ΠΆ (Π”ΠΆΠΎΡƒΠ»ΡŒ).

Π’ ΡΠ»Π΅ΠΊΡ‚Ρ€ΠΎΠ½Π°Π³Ρ€Π΅Π²Π°Ρ‚Π΅Π»ΡŒΠ½Ρ‹Ρ… ΠΏΡ€ΠΈΠ±ΠΎΡ€Π°Ρ… ΠΈΡΠΏΠΎΠ»ΡŒΠ·ΡƒΡŽΡ‚ΡΡ ΠΏΡ€ΠΎΠ²ΠΎΠ΄Π½ΠΈΠΊΠΈ с высоким сопротивлСниСм, Ρ‡Ρ‚ΠΎ обСспСчиваСт Π²Ρ‹Π΄Π΅Π»Π΅Π½ΠΈΠ΅ Ρ‚Π΅ΠΏΠ»Π° Π½Π° ΠΎΠΏΡ€Π΅Π΄Π΅Π»Π΅Π½Π½ΠΎΠΌ участкС. ΠŸΡ€ΠΎΠ²ΠΎΠ»ΠΎΠΊΡƒ ΠΈΠ· Π½ΠΈΡ…Ρ€ΠΎΠΌΠ° ΠΏΡ€ΠΈΠΌΠ΅Π½ΡΡŽΡ‚ Π² ΡΠ»Π΅ΠΊΡ‚Ρ€ΠΎΠ½Π°Π³Ρ€Π΅Π²Π°Ρ‚Π΅Π»ΡŒΠ½Ρ‹Ρ… элСмСнтах, Ρ€Π°Π±ΠΎΡ‚Π°ΡŽΡ‰ΠΈΡ… ΠΏΡ€ΠΈ Ρ‚Π΅ΠΌΠΏΠ΅Ρ€Π°Ρ‚ΡƒΡ€Π΅ Π΄ΠΎ 1000 ℃ .

Ѐамилия ΠΊΠ°ΠΊΠΎΠ³ΠΎ ΡƒΡ‡Π΅Π½ΠΎΠ³ΠΎ стоит ΠΌΠΈΠ»Π»ΠΈΠΎΠ½?

ΠŸΠ΅Ρ€Π²Ρ‹ΠΉ Π²Π²Π΅Π» понятиС β€œΡΠ»Π΅ΠΊΡ‚Ρ€ΠΈΡ‡Π΅ΡΠΊΠΈΠΉ ток” Π² Π½Π°ΡƒΠΊΡƒ АндрС β€” ΠœΠ°Ρ€ΠΈ АмпСр. Π’Π°ΠΊΠΎΠΉ вопрос Π±Ρ‹Π» послСдним Π² ΠΈΠ³Ρ€Π΅ Β«ΠšΡ‚ΠΎ Ρ…ΠΎΡ‡Π΅Ρ‚ ΡΡ‚Π°Ρ‚ΡŒ ΠΌΠΈΠ»Π»ΠΈΠΎΠ½Π΅Ρ€ΠΎΠΌ?Β» ΠΎΡ‚ 20 января 2018 Π³. Но участники Π½Π΅ смогли ΠΎΡ‚Π²Π΅Ρ‚ΠΈΡ‚ΡŒ Π½Π° этот вопрос, ΠΈ ΠΌΠ΅Ρ‡Ρ‚Π° ΠΏΠΎΠ»ΡƒΡ‡ΠΈΡ‚ΡŒ свой ΠΌΠΈΠ»Π»ΠΈΠΎΠ½, Π½Π΅ исполнилась.

Π•Ρ‰Π΅ Π½Π΅ΠΌΠ½ΠΎΠ³ΠΎ ΠΏΡ€ΠΎ элСктричСство…

  • ΠŸΠΎΡΡ‚ΠΎΡΠ½Π½Ρ‹ΠΉ элСктричСский Ρ‚ΠΎΠΊ ΠΈΡΠΏΠΎΠ»ΡŒΠ·ΡƒΠ΅Ρ‚ΡΡ Π² Ρ€Π°Π±ΠΎΡ‚Π΅ Π΄Π²ΠΈΠ³Π°Ρ‚Π΅Π»Π΅ΠΉ элСктротранспорта, схСмах Π°Π²Ρ‚ΠΎΠΌΠΎΠ±ΠΈΠ»Π΅ΠΉ, элСктроникС ΠΈ Π΄Ρ€.
  • ЭлСктричСство Π΅ΡΡ‚ΡŒ ΠΈ Π² нашСм ΠΎΡ€Π³Π°Π½ΠΈΠ·ΠΌΠ΅. ΠœΡ‹ΡˆΠ΅Ρ‡Π½Ρ‹Π΅ ΠΊΠ»Π΅Ρ‚ΠΊΠΈ сСрдца ΠΏΡ€ΠΈ сокращСнии производят ΡΠ»Π΅ΠΊΡ‚Ρ€ΠΎΡΠ½Π΅Ρ€Π³ΠΈΡŽ, эти ΠΈΠΌΠΏΡƒΠ»ΡŒΡΡ‹ ΠΌΠΎΠΆΠ½ΠΎ ΠΈΠ·ΠΌΠ΅Ρ€ΠΈΡ‚ΡŒ с ΠΏΠΎΠΌΠΎΡ‰ΡŒΡŽ элСктрокардиограммы (Π­ΠšΠ“).
  • Π‘Π΅Π½Π΄ΠΆΠ°ΠΌΠΈΠ½ Π€Ρ€Π°Π½ΠΊΠ»ΠΈΠ½ (Π΄Π°β€”Π΄Π°, ΠΏΡ€Π΅Π·ΠΈΠ΄Π΅Π½Ρ‚ АмСрики) ΠΏΡ€ΠΎΠ²Π΅Π» мноТСство ΠΎΠΏΡ‹Ρ‚ΠΎΠ² Π² 18 Π²Π΅ΠΊΠ΅ ΠΈ создал Π³Ρ€ΠΎΠΌΠΎΠΎΡ‚Π²ΠΎΠ΄.
  • Π’ дрСвности люди считали, Ρ‡Ρ‚ΠΎ Ссли молния ΡƒΠ΄Π°Ρ€ΠΈΠ»Π° Π² ΠΊΡƒΡ€Π³Π°Π½ β€” Π·Π½Π°Ρ‡ΠΈΡ‚ Ρ‚Π°ΠΌ Π·Π°Ρ€Ρ‹Ρ‚ΠΎ сокровищС.
  • ΠŸΡ‚ΠΈΡ‡ΠΊΡƒ, ΡΠΈΠ΄ΡΡ‰ΡƒΡŽ Π½Π° ΠΏΡ€ΠΎΠ²ΠΎΠ΄Π°Ρ…, Ρ‚ΠΎΠΊ Π½Π΅ Π±ΡŒΠ΅Ρ‚. Но Ссли эта ΠΏΡ‚ΠΈΡ‡ΠΊΠ° коснСтся Π·Π΅ΠΌΠ»ΠΈ, Ρ‚ΠΎ Π΅ΠΉ ΠΌΠ°Π»ΠΎ Π½Π΅ покаТСтся. ИмСнно поэтому Π½Π΅ΠΎΠ±Ρ…ΠΎΠ΄ΠΈΠΌΠΎ Π·Π°Π·Π΅ΠΌΠ»Π΅Π½ΠΈΠ΅ ΠΏΡ€ΠΈΠ±ΠΎΡ€ΠΎΠ².

Π’Π΅Ρ€ΠΌΠΈΠ½Ρ‹

Π˜ΡΡ‚ΠΎΡ‡Π½ΠΈΠΊ Ρ‚ΠΎΠΊΠ° β€” устройство, Ρ€Π°Π·Π΄Π΅Π»ΡΡŽΡ‰Π΅Π΅ ΠΏΠΎΠ»ΠΎΠΆΠΈΡ‚Π΅Π»ΡŒΠ½Ρ‹Π΅ ΠΈ ΠΎΡ‚Ρ€ΠΈΡ†Π°Ρ‚Π΅Π»ΡŒΠ½Ρ‹Π΅ заряды.

Π‘Ρ‚ΠΎΡ€ΠΎΠ½Π½ΠΈΠ΅ силы β€” силы нСэлСктричСского происхоТдСния, Π²Ρ‹Π·Ρ‹Π²Π°ΡŽΡ‰ΠΈΠ΅ Ρ€Π°Π·Π΄Π΅Π»Π΅Π½ΠΈΠ΅ зарядов Π² источникС Ρ‚ΠΎΠΊΠ°.

Π€Π°ΠΊΡ‚Ρ‡Π΅ΠΊ
  • Π‘ΠΈΠ»Π° Ρ‚ΠΎΠΊΠ° β€” это физичСская Π²Π΅Π»ΠΈΡ‡ΠΈΠ½Π°, ΠΏΠΎΠΊΠ°Π·Ρ‹Π²Π°ΡŽΡ‰Π°Ρ, ΠΊΠ°ΠΊΠΎΠΉ заряд пСрСносится Ρ‡Π΅Ρ€Π΅Π· Ρ€Π°ΡΡΠΌΠ°Ρ‚Ρ€ΠΈΠ²Π°Π΅ΠΌΡƒΡŽ ΠΏΠ»ΠΎΡ‰Π°Π΄ΡŒ ΠΏΠΎΠΏΠ΅Ρ€Π΅Ρ‡Π½ΠΎΠ³ΠΎ сСчСния Π·Π° Π΅Π΄ΠΈΠ½ΠΈΡ†Ρƒ Π²Ρ€Π΅ΠΌΠ΅Π½ΠΈ: \(I = \frac{q}{t}\)
  • НапряТСниС β€” скалярная физичСская Π²Π΅Π»ΠΈΡ‡ΠΈΠ½Π°, равная ΠΎΡ‚Π½ΠΎΡˆΠ΅Π½ΠΈΡŽ ΠΏΠΎΠ»Π½ΠΎΠΉ Ρ€Π°Π±ΠΎΡ‚Ρ‹ кулоновских ΠΈ сторонних сил А ΠΏΡ€ΠΈ ΠΏΠ΅Ρ€Π΅ΠΌΠ΅Ρ‰Π΅Π½ΠΈΠΈ ΠΏΠΎΠ»ΠΎΠΆΠΈΡ‚Π΅Π»ΡŒΠ½ΠΎΠ³ΠΎ заряда Π½Π° участкС Ρ†Π΅ΠΏΠΈ ΠΊ Π·Π½Π°Ρ‡Π΅Π½ΠΈΡŽ этого заряда: \(U = \frac{A}{q}\)
  • Π‘ΠΎΠΏΡ€ΠΎΡ‚ΠΈΠ²Π»Π΅Π½ΠΈΠ΅ β€” физичСская Π²Π΅Π»ΠΈΡ‡ΠΈΠ½Π°, Ρ…Π°Ρ€Π°ΠΊΡ‚Π΅Ρ€ΠΈΠ·ΡƒΡŽΡ‰Π°Ρ элСктричСскиС свойства участка Ρ†Π΅ΠΏΠΈ: \(R = \frac{pl}{S}\)
  • ΠœΠΎΡ‰Π½ΠΎΡΡ‚ΡŒΒ  β€” Π²Π΅Π»ΠΈΡ‡ΠΈΠ½Π°, ΠΎΠ±ΠΎΠ·Π½Π°Ρ‡Π°ΡŽΡ‰Π°Ρ ΠΈΠ½Ρ‚Π΅Π½ΡΠΈΠ²Π½ΠΎΡΡ‚ΡŒ ΠΏΠ΅Ρ€Π΅Π΄Π°Ρ‡ΠΈ элСктричСской энСргии: \(P = \frac{A}{t}\)
  • Π—Π°ΠΊΠΎΠ½ Ома: сила Ρ‚ΠΎΠΊΠ° Π² ΠΎΠ΄Π½ΠΎΡ€ΠΎΠ΄Π½ΠΎΠΌ участкС Ρ†Π΅ΠΏΠΈ прямо ΠΏΡ€ΠΎΠΏΠΎΡ€Ρ†ΠΈΠΎΠ½Π°Π»ΡŒΠ½Π° Π½Π°ΠΏΡ€ΡΠΆΠ΅Π½ΠΈΡŽ ΠΏΡ€ΠΈ постоянном сопротивлСнии ΠΈ ΠΎΠ±Ρ€Π°Ρ‚Π½ΠΎ ΠΏΡ€ΠΎΠΏΠΎΡ€Ρ†ΠΈΠΎΠ½Π°Π»ΡŒΠ½Π° ΡΠΎΠΏΡ€ΠΎΡ‚ΠΈΠ²Π»Π΅Π½ΠΈΡŽ участка ΠΏΡ€ΠΈ постоянном напряТСнии: \(I = \frac{U}{R}\).
  • Π—Π°ΠΊΠΎΠ½ ДТоуля— Π›Π΅Π½Ρ†Π°: количСство Ρ‚Π΅ΠΏΠ»ΠΎΡ‚Ρ‹ Q, выдСляСмоС Π·Π° врСмя tΒ  Π² ΠΏΡ€ΠΎΠ²ΠΎΠ΄Π½ΠΈΠΊΠ΅ с Ρ‚ΠΎΠΊΠΎΠΌ, ΠΏΡ€ΠΎΠΏΠΎΡ€Ρ†ΠΈΠΎΠ½Π°Π»ΡŒΠ½ΠΎ ΠΏΡ€ΠΎΠΈΠ·Π²Π΅Π΄Π΅Π½ΠΈΡŽ ΠΊΠ²Π°Π΄Ρ€Π°Ρ‚Π° силы Ρ‚ΠΎΠΊΠ° I Π½Π° этом участкС ΠΈ сопротивлСния R ΠΏΡ€ΠΎΠ²ΠΎΠ΄Π½ΠΈΠΊΠ°: Q = I2Rt.
  • Π Π°Π±ΠΎΡ‚Π° элСктричСского поля ΠΏΡ€ΠΈ ΠΏΡ€ΠΎΡ‚Π΅ΠΊΠ°Π½ΠΈΠΈ постоянного Ρ‚ΠΎΠΊΠ° (ΠΈΠ»ΠΈ просто Ρ€Π°Π±ΠΎΡ‚Π° Ρ‚ΠΎΠΊΠ°): А = UIt.

ΠŸΡ€ΠΎΠ²Π΅Ρ€ΡŒ сСбя

Π—Π°Π΄Π°Π½ΠΈΠ΅ 1.
УпорядочСнноС Π΄Π²ΠΈΠΆΠ΅Π½ΠΈΠ΅ заряТСнных частиц β€” это:

  1. ЭлСктричСскоС ΠΏΠΎΠ»Π΅
  2. ЭлСктричСский Ρ‚ΠΎΠΊ
  3. ЭлСктричСская ΠΌΠΎΡ‰Π½ΠΎΡΡ‚ΡŒ
  4. Π Π°Π±ΠΎΡ‚Π° Ρ‚ΠΎΠΊΠ°

Π—Π°Π΄Π°Π½ΠΈΠ΅ 2.
УдСльноС сопротивлСниС ΠΏΡ€ΠΎΠ²ΠΎΠ΄Π½ΠΈΠΊΠ°:

  1. Зависит ΠΎΡ‚ Ρ‚Π΅ΠΌΠΏΠ΅Ρ€Π°Ρ‚ΡƒΡ€Ρ‹
  2. НС зависит ΠΎΡ‚ Ρ‚Π΅ΠΌΠΏΠ΅Ρ€Π°Ρ‚ΡƒΡ€Ρ‹Β 

Π—Π°Π΄Π°Π½ΠΈΠ΅ 3.
Π€ΠΎΡ€ΠΌΡƒΠ»Π° для расчСта силы Ρ‚ΠΎΠΊΠ°:

  1. I = Ut
  2. I = UIt
  3. I = I2Rt
  4. \(I = \frac{q}{t}\)

Π—Π°Π΄Π°Π½ΠΈΠ΅ 4.Β 
Π§Ρ‚ΠΎ Ρ‚Π°ΠΊΠΎΠ΅ ΠΌΠΎΡ‰Π½ΠΎΡΡ‚ΡŒ элСктричСского Ρ‚ΠΎΠΊΠ°:

  1. Π Π°Π±ΠΎΡ‚Π° Π·Π° Π΅Π΄ΠΈΠ½ΠΈΡ†Ρƒ Π²Ρ€Π΅ΠΌΠ΅Π½ΠΈ
  2. ΠžΡ‚Π½ΠΎΡˆΠ΅Π½ΠΈΠ΅ заряда ΠΊ Π΅Π΄ΠΈΠ½ΠΈΡ†Π΅ Π²Ρ€Π΅ΠΌΠ΅Π½ΠΈ
  3. ΠŸΡ€ΠΎΠΈΠ·Π²Π΅Π΄Π΅Π½ΠΈΠ΅ силы Ρ‚ΠΎΠΊΠ° Π½Π° сопротивлСниС
  4. Π’Π΅ΠΏΠ»ΠΎ, выдСляСмоС Π½Π° рСзисторС

Π—Π°Π΄Π°Π½ΠΈΠ΅ 5. Β 
ΠŸΡ€ΠΈΡ‡ΠΈΠ½Π° элСктричСского сопротивлСния:

  1. Π’ΠΎ взаимодСйствии зарядов ΠΎΠ΄ΠΈΠ½Π°ΠΊΠΎΠ²ΠΎΠ³ΠΎ Π·Π½Π°ΠΊΠ°
  2. Π’ отсутствии взаимодСйствия ΠΌΠ΅ΠΆΠ΄Ρƒ зарядами
  3. Π’ΠΎ взаимодСйствии зарядов Ρ€Π°Π·Π½ΠΎΠ³ΠΎ Π·Π½Π°ΠΊΠ°
  4. Π’ ΠΏΠ΅Ρ€Π΅Π΄Π°Ρ‡Π΅ Ρ‚Π΅ΠΏΠ»Π°

ΠžΡ‚Π²Π΅Ρ‚Ρ‹: 1.β€” 2; 2. β€” 1; 3.β€” 4; 4.β€” 1; 5. β€” 3

Π‘ΠΈΠ»Π° Ρ‚ΠΎΠΊΠ°. НапряТСниС. Π‘ΠΎΠΏΡ€ΠΎΡ‚ΠΈΠ²Π»Π΅Π½ΠΈΠ΅. Π—Π°ΠΊΠΎΠ½ Ома 8 класс прСзСнтация, Π΄ΠΎΠΊΠ»Π°Π΄, ΠΏΡ€ΠΎΠ΅ΠΊΡ‚

Π‘Π»Π°ΠΉΠ΄ 1
ВСкст слайда:

ΠšΠΎΠ½ΠΊΡƒΡ€Ρ ΠΏΡ€Π΅Π·Π΅Π½Ρ‚Π°Ρ†ΠΈΠΉ Β«Π˜Π½Ρ‚Π΅Ρ€Π°ΠΊΡ‚ΠΈΠ²Π½Π°Ρ ΠΌΠΎΠ·Π°ΠΈΠΊΠ°Β» ΠΈΠ½Ρ‚Π΅Ρ€Π°ΠΊΡ‚ΠΈΠ²Π½ΠΎΠ΅ пособиС

БСляСва НадСТда Π’Π»Π°Π΄ΠΈΠΌΠΈΡ€ΠΎΠ²Π½Π°
МАОУ «БОШ β„–140Β» Π³. ΠŸΠ΅Ρ€ΠΌΠΈ
ΡƒΡ‡ΠΈΡ‚Π΅Π»ΡŒ ΠΈΠ½Ρ„ΠΎΡ€ΠΌΠ°Ρ‚ΠΈΠΊΠΈ


Π‘Π»Π°ΠΉΠ΄ 2
ВСкст слайда:

Π‘ΠΈΠ»Π° Ρ‚ΠΎΠΊΠ° НапряТСниС Π‘ΠΎΠΏΡ€ΠΎΡ‚ΠΈΠ²Π»Π΅Π½ΠΈΠ΅ Π—Π°ΠΊΠΎΠ½ Ома

Ѐизика, 8 класс


Π‘Π»Π°ΠΉΠ΄ 3
ВСкст слайда:

Π‘ΠΎΠ΄Π΅Ρ€ΠΆΠ°Π½ΠΈΠ΅:

ЭлСктричСский Ρ‚ΠΎΠΊ
Π‘ΠΈΠ»Π° Ρ‚ΠΎΠΊΠ°
АмпСрмСтр
НапряТСниС
Π’ΠΎΠ»ΡŒΡ‚ΠΌΠ΅Ρ‚Ρ€
Π‘ΠΎΠΏΡ€ΠΎΡ‚ΠΈΠ²Π»Π΅Π½ΠΈΠ΅
ΠžΠΌΠΌΠ΅Ρ‚Ρ€
РСостат, рСзистор, ΠΌΠ°Π³Π°Π·ΠΈΠ½ сопротивлСния
Π—Π°ΠΊΠΎΠ½ Ома
Лабораторная Ρ€Π°Π±ΠΎΡ‚Π°
Вопросы для самоконтроля
Бписок источников


Π‘Π»Π°ΠΉΠ΄ 4
ВСкст слайда:

ΠšΡ€ΠΈΡΡ‚Π°Π»Π»ΠΈΡ‡Π΅ΡΠΊΠ°Ρ Ρ€Π΅ΡˆΠ΅Ρ‚ΠΊΠ° ΠΌΠ΅Ρ‚Π°Π»Π»Π°

Π’ ΡƒΠ·Π»Π°Ρ… кристалличСской Ρ€Π΅ΡˆΠ΅Ρ‚ΠΊΠΈ располоТСны Β«+Β» ΠΈΠΎΠ½Ρ‹, ΠΌΠ΅ΠΆΠ΄Ρƒ ΠΊΠΎΡ‚ΠΎΡ€Ρ‹ΠΌΠΈ Ρ…Π°ΠΎΡ‚ΠΈΡ‡Π½ΠΎ двиТутся свободныС элСктроны

ΠœΠ΅Ρ‚Π°Π»Π»Ρ‹ ΡΠ²Π»ΡΡŽΡ‚ΡΡ Ρ…ΠΎΡ€ΠΎΡˆΠΈΠΌΠΈ ΠΏΡ€ΠΎΠ²ΠΎΠ΄Π½ΠΈΠΊΠ°ΠΌΠΈ благодаря свободным заряТСнным частицам – элСктронам


Π‘Π»Π°ΠΉΠ΄ 5
ВСкст слайда:

ЭлСктричСский Ρ‚ΠΎΠΊ — упорядочСнноС (Π½Π°ΠΏΡ€Π°Π²Π»Π΅Π½Π½ΠΎΠ΅) Π΄Π²ΠΈΠΆΠ΅Π½ΠΈΠ΅ заряТСнных частиц

ЭлСктричСский Ρ‚ΠΎΠΊ

Условия возникновСния элСктричСского Ρ‚ΠΎΠΊΠ° Π² ΠΏΡ€ΠΎΠ²ΠΎΠ΄Π½ΠΈΠΊΠ΅:

Π½Π°Π»ΠΈΡ‡ΠΈΠ΅ свободных заряТСнных частиц (элСктронов, ΠΈΠΎΠ½ΠΎΠ²)
элСктричСскоС ΠΏΠΎΠ»Π΅

НаправлСниС элСктричСского Ρ‚ΠΎΠΊΠ°: ΠΎΡ‚ + ΠΊ –


Π‘Π»Π°ΠΉΠ΄ 6
ВСкст слайда:

Π² ΠΌΠ΅Ρ‚Π°Π»Π»Π΅:
элСктроны двиТутся ΠΎΡ‚ – ΠΊ +
Ρ‚ΠΎΠΊ Π½Π°ΠΏΡ€Π°Π²Π»Π΅Π½ Π² сторону, ΠΏΡ€ΠΎΡ‚ΠΈΠ²ΠΎΠΏΠΎΠ»ΠΎΠΆΠ½ΡƒΡŽ Π½Π°ΠΏΡ€Π°Π²Π»Π΅Π½ΠΈΡŽ двиТСния элСктронов


Π‘Π»Π°ΠΉΠ΄ 7
ВСкст слайда:

Π‘ΠΈΠ»Π° Ρ‚ΠΎΠΊΠ°

Π‘ΠΈΠ»Π° Ρ‚ΠΎΠΊΠ°Β — физичСская Π²Π΅Π»ΠΈΡ‡ΠΈΠ½Π°, равная заряду, ΠΏΡ€ΠΎΡˆΠ΅Π΄ΡˆΠ΅ΠΌΡƒ Ρ‡Π΅Ρ€Π΅Π· ΠΏΠΎΠΏΠ΅Ρ€Π΅Ρ‡Π½ΠΎΠ΅ сСчСниС ΠΏΡ€ΠΎΠ²ΠΎΠ΄Π½ΠΈΠΊΠ° Π·Π° Π΅Π΄ΠΈΠ½ΠΈΡ†Ρƒ Π²Ρ€Π΅ΠΌΠ΅Π½ΠΈ

ΠžΠ±ΠΎΠ·Π½Π°Ρ‡Π΅Π½ΠΈΠ΅: I
Π•Π΄ΠΈΠ½ΠΈΡ†Π° измСрСния: 1А (АмпСр)

Π€ΠΎΡ€ΠΌΡƒΠ»Π°:

Π˜Π·ΠΌΠ΅Ρ€ΠΈΡ‚Π΅Π»ΡŒΠ½Ρ‹ΠΉ ΠΏΡ€ΠΈΠ±ΠΎΡ€: Π°ΠΌΠΏΠ΅Ρ€ΠΌΠ΅Ρ‚Ρ€


Π‘Π»Π°ΠΉΠ΄ 8
ВСкст слайда:

Π‘Π»Π°ΠΉΠ΄ 9
ВСкст слайда:

АмпСрмСтр


Π‘Π»Π°ΠΉΠ΄ 10
ВСкст слайда:

АмпСрмСтр Π²ΠΊΠ»ΡŽΡ‡Π°Π΅Ρ‚ΡΡ ΠΏ ΠΎ с Π» Π΅ Π΄ ΠΎ Π² Π° Ρ‚ Π΅ Π» ь Π½ ΠΎ


Π‘Π»Π°ΠΉΠ΄ 11
ВСкст слайда:

ΠŸΡ€ΠΈ Π²ΠΊΠ»ΡŽΡ‡Π΅Π½ΠΈΠΈ Π°ΠΌΠΏΠ΅Ρ€ΠΌΠ΅Ρ‚Ρ€Π° Π² Ρ†Π΅ΠΏΡŒ Π½Π΅ ΠΈΠΌΠ΅Π΅Ρ‚ значСния, с ΠΊΠ°ΠΊΠΎΠΉ стороны (слСва ΠΈΠ»ΠΈ справа) ΠΎΡ‚ исслСдуСмого элСмСнта Π΅Π³ΠΎ ΠΏΠΎΠ΄ΠΊΠ»ΡŽΡ‡Π°Ρ‚ΡŒ.

Β 


Π‘Π»Π°ΠΉΠ΄ 12
ВСкст слайда:

Π˜Π·ΠΌΠ΅Ρ€Π΅Π½ΠΈΠ΅ силы Ρ‚ΠΎΠΊΠ°
(Π²ΠΈΠ΄Π΅ΠΎΡ„Ρ€Π°Π³ΠΌΠ΅Π½Ρ‚ ΠΎΠΏΡ‹Ρ‚Π°)

ΠœΡƒΠ»ΡŒΡ‚ΠΈΠΌΠ΅Π΄ΠΈΠΉΠ½ΠΎΠ΅ ΠΏΡ€ΠΈΠ»ΠΎΠΆΠ΅Π½ΠΈΠ΅ ΠΊ ΡƒΡ‡Π΅Π±Π½ΠΈΠΊΡƒ Π‘.Π’. Π“Ρ€ΠΎΠΌΠΎΠ²Π° ΠΈ Н.А. Π ΠΎΠ΄ΠΈΠ½ΠΎΠΉ Β«Π€ΠΈΠ·ΠΈΠΊΠ°. 9 класс» (ΠŸΡ€ΠΎΡΠ²Π΅Ρ‰Π΅Π½ΠΈΠ΅ МСдиа, Новый диск)


Π‘Π»Π°ΠΉΠ΄ 13
ВСкст слайда:

АмпСрмСтр Π»Π°Π±ΠΎΡ€Π°Ρ‚ΠΎΡ€Π½Ρ‹ΠΉ


Π‘Π»Π°ΠΉΠ΄ 14
ВСкст слайда:

Π‘Π»Π°ΠΉΠ΄ 15
ВСкст слайда:

Π‘Π»Π°ΠΉΠ΄ 16
ВСкст слайда:

Π‘Π»Π°ΠΉΠ΄ 17
ВСкст слайда:

Π¨ΠΊΠ°Π»Π° Π°ΠΌΠΏΠ΅Ρ€ΠΌΠ΅Ρ‚Ρ€Π°


Π‘Π»Π°ΠΉΠ΄ 18
ВСкст слайда:

Π¦Π΅Π½Π° дСлСния ΠΈ ΠΏΡ€Π΅Π΄Π΅Π»Ρ‹ измСрСния ΠΏΡ€ΠΈΠ±ΠΎΡ€Π°


Π‘Π»Π°ΠΉΠ΄ 19
ВСкст слайда:

ΠŸΡ€ΠΈΠ½Ρ†ΠΈΠΏ дСйствия ΠΏΡ€ΠΈΠ±ΠΎΡ€Π°


Π‘Π»Π°ΠΉΠ΄ 20
ВСкст слайда:

Π¨ΡƒΠ½Ρ‚ – ΠΏΡ€ΠΎΠ²ΠΎΠ΄Π½ΠΈΠΊ, ΠΏΠΎΠ΄ΠΊΠ»ΡŽΡ‡Π°Π΅ΠΌΡ‹ΠΉ ΠΏΠ°Ρ€Π°Π»Π»Π΅Π»ΡŒΠ½ΠΎ Π°ΠΌΠΏΠ΅Ρ€ΠΌΠ΅Ρ‚Ρ€Ρƒ для Ρ€Π°ΡΡˆΠΈΡ€Π΅Π½ΠΈΡ ΠΏΡ€Π΅Π΄Π΅Π»ΠΎΠ² Π΅Π³ΠΎ ΠΈΠ·ΠΌΠ΅Ρ€Π΅Π½ΠΈΠΉ.

Π§Π°ΡΡ‚ΡŒ измСряСмого Ρ‚ΠΎΠΊΠ° отвСтвляСтся ΠΈ Ρ‡Π΅Ρ€Π΅Π· Π°ΠΌΠΏΠ΅Ρ€ΠΌΠ΅Ρ‚Ρ€ Π±ΡƒΠ΄Π΅Ρ‚ ΠΈΠ΄Ρ‚ΠΈ Ρ‚ΠΎΠΊ мСньшС измСряСмого


Π‘Π»Π°ΠΉΠ΄ 21
ВСкст слайда:

Π‘Π»Π°ΠΉΠ΄ 22
ВСкст слайда:

НапряТСниС  – скалярная физичСская Π²Π΅Π»ΠΈΡ‡ΠΈΠ½Π°, равная Ρ€Π°Π±ΠΎΡ‚Π΅ элСктричСского поля ΠΏΠΎ ΠΏΠ΅Ρ€Π΅ΠΌΠ΅Ρ‰Π΅Π½ΠΈΡŽ Π΅Π΄ΠΈΠ½ΠΈΡ‡Π½ΠΎΠ³ΠΎ ΠΏΠΎΠ»ΠΎΠΆΠΈΡ‚Π΅Π»ΡŒΠ½ΠΎΠ³ΠΎ заряда

ΠžΠ±ΠΎΠ·Π½Π°Ρ‡Π΅Π½ΠΈΠ΅: U

Π•Π΄ΠΈΠ½ΠΈΡ†Π° измСрСния Π² БИ: 1Π’ (Π²ΠΎΠ»ΡŒΡ‚)

НапряТСниС

Π€ΠΎΡ€ΠΌΡƒΠ»Π°:

Π˜Π·ΠΌΠ΅Ρ€ΠΈΡ‚Π΅Π»ΡŒΠ½Ρ‹ΠΉ ΠΏΡ€ΠΈΠ±ΠΎΡ€: Π²ΠΎΠ»ΡŒΡ‚ΠΌΠ΅Ρ‚Ρ€


Π‘Π»Π°ΠΉΠ΄ 23
ВСкст слайда:

Π‘Π»Π°ΠΉΠ΄ 24
ВСкст слайда:

Π’ΠΎΠ»ΡŒΡ‚ΠΌΠ΅Ρ‚Ρ€


Π‘Π»Π°ΠΉΠ΄ 25
ВСкст слайда:

Π’ΠΎΠ»ΡŒΡ‚ΠΌΠ΅Ρ‚Ρ€ Π²ΠΊΠ»ΡŽΡ‡Π°Π΅Ρ‚ΡΡ ΠΏ Π° Ρ€ Π° Π» Π» Π΅ Π» ь Π½ ΠΎ


Π‘Π»Π°ΠΉΠ΄ 26
ВСкст слайда:

Π˜Π·ΠΌΠ΅Ρ€Π΅Π½ΠΈΠ΅ напряТСния
(Π²ΠΈΠ΄Π΅ΠΎΡ„Ρ€Π°Π³ΠΌΠ΅Π½Ρ‚ ΠΎΠΏΡ‹Ρ‚Π°)

ΠœΡƒΠ»ΡŒΡ‚ΠΈΠΌΠ΅Π΄ΠΈΠΉΠ½ΠΎΠ΅ ΠΏΡ€ΠΈΠ»ΠΎΠΆΠ΅Π½ΠΈΠ΅ ΠΊ ΡƒΡ‡Π΅Π±Π½ΠΈΠΊΡƒ Π‘. Π’. Π“Ρ€ΠΎΠΌΠΎΠ²Π° ΠΈ Н.А. Π ΠΎΠ΄ΠΈΠ½ΠΎΠΉ Β«Π€ΠΈΠ·ΠΈΠΊΠ°. 9 класс» (ΠŸΡ€ΠΎΡΠ²Π΅Ρ‰Π΅Π½ΠΈΠ΅ МСдиа, Новый диск)


Π‘Π»Π°ΠΉΠ΄ 27
ВСкст слайда:

Π’ΠΎΠ»ΡŒΡ‚ΠΌΠ΅Ρ‚Ρ€ Π»Π°Π±ΠΎΡ€Π°Ρ‚ΠΎΡ€Π½Ρ‹ΠΉ


Π‘Π»Π°ΠΉΠ΄ 28
ВСкст слайда:

Π’ΠΎΠ»ΡŒΡ‚ΠΌΠ΅Ρ‚Ρ€ Π‘Π‘Π‘Π , 1940 Π³ΠΎΠ΄


Π‘Π»Π°ΠΉΠ΄ 29
ВСкст слайда:

Π‘Π»Π°ΠΉΠ΄ 30
ВСкст слайда:

Π‘Π»Π°ΠΉΠ΄ 31
ВСкст слайда:

Π¨ΠΊΠ°Π»Π° Π²ΠΎΠ»ΡŒΡ‚ΠΌΠ΅Ρ‚Ρ€Π°


Π‘Π»Π°ΠΉΠ΄ 32
ВСкст слайда:

Π¦Π΅Π½Π° дСлСния ΠΈ ΠΏΡ€Π΅Π΄Π΅Π»Ρ‹ измСрСния ΠΏΡ€ΠΈΠ±ΠΎΡ€Π°


Π‘Π»Π°ΠΉΠ΄ 33
ВСкст слайда:

ΠŸΡ€ΠΈΠ½Ρ†ΠΈΠΏ дСйствия ΠΏΡ€ΠΈΠ±ΠΎΡ€Π°


Π‘Π»Π°ΠΉΠ΄ 34
ВСкст слайда:

Π”ΠΎΠΏΠΎΠ»Π½ΠΈΡ‚Π΅Π»ΡŒΠ½ΠΎΠ΅ сопротивлСниС – ΠΏΡ€ΠΎΠ²ΠΎΠ΄Π½ΠΈΠΊ, ΠΏΠΎΠ΄ΠΊΠ»ΡŽΡ‡Π°Π΅ΠΌΡ‹ΠΉ ΠΏΠΎΡΠ»Π΅Π΄ΠΎΠ²Π°Ρ‚Π΅Π»ΡŒΠ½ΠΎ с Π²ΠΎΠ»ΡŒΡ‚ΠΌΠ΅Ρ‚Ρ€ΠΎΠΌ для Ρ€Π°ΡΡˆΠΈΡ€Π΅Π½ΠΈΡ ΠΏΡ€Π΅Π΄Π΅Π»ΠΎΠ² Π΅Π³ΠΎ ΠΈΠ·ΠΌΠ΅Ρ€Π΅Π½ΠΈΠΉ


Π‘Π»Π°ΠΉΠ΄ 35
ВСкст слайда:

Π’ΠΎΠΊ Ρ‡Π΅Ρ€Π΅Π· Π»Π°ΠΌΠΏΠΎΡ‡ΠΊΡƒ ΠΈ напряТСниС Π½Π° Π½Π΅ΠΉ


Π‘Π»Π°ΠΉΠ΄ 36
ВСкст слайда:

А

V


Π‘Π»Π°ΠΉΠ΄ 37
ВСкст слайда:

Π‘ΠΎΠΏΡ€ΠΎΡ‚ΠΈΠ²Π»Π΅Π½ΠΈΠ΅

Π‘ΠΎΠΏΡ€ΠΎΡ‚ΠΈΠ²Π»Π΅Π½ΠΈΠ΅ – скалярная физичСская Π²Π΅Π»ΠΈΡ‡ΠΈΠ½Π°, Ρ…Π°Ρ€Π°ΠΊΡ‚Π΅Ρ€ΠΈΠ·ΡƒΡŽΡ‰Π°Ρ свойство ΠΏΡ€ΠΎΠ²ΠΎΠ΄Π½ΠΈΠΊΠ° ΠΏΡ€ΠΎΡ‚ΠΈΠ²ΠΎΠ΄Π΅ΠΉΡΡ‚Π²ΠΎΠ²Π°Ρ‚ΡŒ элСктричСскому Ρ‚ΠΎΠΊΡƒ

ΠžΠ±ΠΎΠ·Π½Π°Ρ‡Π΅Π½ΠΈΠ΅: R

Π•Π΄ΠΈΠ½ΠΈΡ†Π° измСрСния: 1Ом (Ом)

Π˜Π·ΠΌΠ΅Ρ€ΠΈΡ‚Π΅Π»ΡŒΠ½Ρ‹ΠΉ ΠΏΡ€ΠΈΠ±ΠΎΡ€: ΠžΠΌΠΌΠ΅Ρ‚Ρ€


Π‘Π»Π°ΠΉΠ΄ 38
ВСкст слайда:

ΠžΠΌΠΌΠ΅Ρ‚Ρ€


Π‘Π»Π°ΠΉΠ΄ 39
ВСкст слайда:

Π‘Π»Π°ΠΉΠ΄ 40
ВСкст слайда:

Π‘Π»Π°ΠΉΠ΄ 41
ВСкст слайда:

Π‘Π»Π°ΠΉΠ΄ 42
ВСкст слайда:

Π˜Π·ΠΌΠ΅Ρ€Π΅Π½ΠΈΠ΅ сопротивлСния
Ρ†ΠΈΡ„Ρ€ΠΎΠ²Ρ‹ΠΌ ΠΌΡƒΠ»ΡŒΡ‚ΠΈΠΌΠ΅Ρ‚Ρ€ΠΎΠΌ


Π‘Π»Π°ΠΉΠ΄ 43
ВСкст слайда:

ΠŸΡ€ΠΈΡ‡ΠΈΠ½Π° элСктричСского сопротивлСния:
взаимодСйствиС элСктронов ΠΏΡ€ΠΈ ΠΈΡ… Π΄Π²ΠΈΠΆΠ΅Π½ΠΈΠΈ ΠΏΠΎ ΠΏΡ€ΠΎΠ²ΠΎΠ΄Π½ΠΈΠΊΡƒ с ΠΈΠΎΠ½Π°ΠΌΠΈ кристалличСской Ρ€Π΅ΡˆΠ΅Ρ‚ΠΊΠΈ.


Π‘Π»Π°ΠΉΠ΄ 44
ВСкст слайда:

НаправлСнному двиТСнию элСктронов ΠΌΠ΅ΡˆΠ°ΡŽΡ‚ ΠΈΡ… столкновСния с ΠΊΠΎΠ»Π΅Π±Π»ΡŽΡ‰ΠΈΠΌΠΈΡΡ тяТСлыми ΠΈ большими ΠΈΠΎΠ½Π°ΠΌΠΈ кристалличСской Ρ€Π΅ΡˆΠ΅Ρ‚ΠΊΠΈ. Π­Ρ‚ΠΎ ΠΈ создаСт сопротивлСниС двиТСнию элСктронов β€” Π²Ρ‹Π·Ρ‹Π²Π°Π΅Ρ‚ элСктричСскоС сопротивлСниС ΠΌΠ΅Ρ‚Π°Π»Π»Π°.


Π‘Π»Π°ΠΉΠ΄ 45
ВСкст слайда:

Π—Π°Π²ΠΈΡΠΈΠΌΠΎΡΡ‚ΡŒ сопротивлСния ΠΏΡ€ΠΎΠ²ΠΎΠ΄Π½ΠΈΠΊΠ° ΠΎΡ‚ Π΅Π³ΠΎ Π΄Π»ΠΈΠ½Ρ‹
(Π²ΠΈΠ΄Π΅ΠΎΡ„Ρ€Π°Π³ΠΌΠ΅Π½Ρ‚ ΠΎΠΏΡ‹Ρ‚Π°)

ΠœΡƒΠ»ΡŒΡ‚ΠΈΠΌΠ΅Π΄ΠΈΠΉΠ½ΠΎΠ΅ ΠΏΡ€ΠΈΠ»ΠΎΠΆΠ΅Π½ΠΈΠ΅ ΠΊ ΡƒΡ‡Π΅Π±Π½ΠΈΠΊΡƒ Π‘.Π’. Π“Ρ€ΠΎΠΌΠΎΠ²Π° ΠΈ Н.А. Π ΠΎΠ΄ΠΈΠ½ΠΎΠΉ Β«Π€ΠΈΠ·ΠΈΠΊΠ°. 9 класс» (ΠŸΡ€ΠΎΡΠ²Π΅Ρ‰Π΅Π½ΠΈΠ΅ МСдиа, Новый диск)


Π‘Π»Π°ΠΉΠ΄ 46
ВСкст слайда:

ЭлСктричСскоС сопротивлСниС ΠΌΠ΅Ρ‚Π°Π»Π»ΠΎΠ² прямо ΠΏΡ€ΠΎΠΏΠΎΡ€Ρ†ΠΈΠΎΠ½Π°Π»ΡŒΠ½ΠΎ Π΄Π»ΠΈΠ½Π΅ ΠΏΡ€ΠΎΠ²ΠΎΠ΄Π½ΠΈΠΊΠ° ΠΈ ΠΎΠ±Ρ€Π°Ρ‚Π½ΠΎ ΠΏΡ€ΠΎΠΏΠΎΡ€Ρ†ΠΈΠΎΠ½Π°Π»ΡŒΠ½ΠΎ ΠΏΠ»ΠΎΡ‰Π°Π΄ΠΈ Π΅Π³ΠΎ ΠΏΠΎΠΏΠ΅Ρ€Π΅Ρ‡Π½ΠΎΠ³ΠΎ сСчСния:

ρ – ΡƒΠ΄Π΅Π»ΡŒΠ½ΠΎΠ΅ сопротивлСниС
l – Π΄Π»ΠΈΠ½Π° ΠΏΡ€ΠΎΠ²ΠΎΠ΄Π½ΠΈΠΊΠ°
S – ΠΏΠ»ΠΎΡ‰Π°Π΄ΡŒ ΠΏΠΎΠΏΠ΅Ρ€Π΅Ρ‡Π½ΠΎΠ³ΠΎ сСчСния ΠΏΡ€ΠΎΠ²ΠΎΠ΄Π½ΠΈΠΊΠ°


Π‘Π»Π°ΠΉΠ΄ 47
ВСкст слайда:

Зависит ΠΎΡ‚ вСщСства ΠΈ Π΅Π³ΠΎ состояния (Ρ‚Π΅ΠΌΠΏΠ΅Ρ€Π°Ρ‚ΡƒΡ€Ρ‹)

Π•Π΄ΠΈΠ½ΠΈΡ†Π° измСрСния: 1 Ом β‹… ΠΌ

УдСльноС сопротивлСниС – скалярная физичСская Π²Π΅Π»ΠΈΡ‡ΠΈΠ½Π°, числСнно равная ΡΠΎΠΏΡ€ΠΎΡ‚ΠΈΠ²Π»Π΅Π½ΠΈΡŽ цилиндричСского ΠΏΡ€ΠΎΠ²ΠΎΠ΄Π½ΠΈΠΊΠ° Π΅Π΄ΠΈΠ½ΠΈΡ‡Π½ΠΎΠΉ Π΄Π»ΠΈΠ½Ρ‹ ΠΈ Π΅Π΄ΠΈΠ½ΠΈΡ‡Π½ΠΎΠΉ ΠΏΠ»ΠΎΡ‰Π°Π΄ΠΈ ΠΏΠΎΠΏΠ΅Ρ€Π΅Ρ‡Π½ΠΎΠ³ΠΎ сСчСния


Π‘Π»Π°ΠΉΠ΄ 48
ВСкст слайда:

Π‘Π»Π°ΠΉΠ΄ 49
ВСкст слайда:

РСзистор – устройство с постоянным сопротивлСниСм.


Π‘Π»Π°ΠΉΠ΄ 50
ВСкст слайда:

Π‘Π»Π°ΠΉΠ΄ 51
ВСкст слайда:

РСостат – устройство с ΠΏΠ΅Ρ€Π΅ΠΌΠ΅Π½Π½Ρ‹ΠΌ сопротивлСниСм, ΠΏΡ€Π΅Π΄Π½Π°Π·Π½Π°Ρ‡Π΅Π½Π½ΠΎΠ΅ для рСгулирования силы Ρ‚ΠΎΠΊΠ° ΠΈ напряТСния Π² элСктричСской Ρ†Π΅ΠΏΠΈ.

Π€ΠΈΠ·ΠΈΠΊΠ°, 7-11 ΠΊΠ». Π‘ΠΈΠ±Π»ΠΈΠΎΡ‚Π΅ΠΊΠ° наглядных пособий (1Π‘)


Π‘Π»Π°ΠΉΠ΄ 52
ВСкст слайда:

Π‘Π»Π°ΠΉΠ΄ 53
ВСкст слайда:

РСостат
(Π²ΠΈΠ΄Π΅ΠΎΡ„Ρ€Π°Π³ΠΌΠ΅Π½Ρ‚ ΠΎΠΏΡ‹Ρ‚Π°)

ΠœΡƒΠ»ΡŒΡ‚ΠΈΠΌΠ΅Π΄ΠΈΠΉΠ½ΠΎΠ΅ ΠΏΡ€ΠΈΠ»ΠΎΠΆΠ΅Π½ΠΈΠ΅ ΠΊ ΡƒΡ‡Π΅Π±Π½ΠΈΠΊΡƒ Π‘.Π’. Π“Ρ€ΠΎΠΌΠΎΠ²Π° ΠΈ Н.А. Π ΠΎΠ΄ΠΈΠ½ΠΎΠΉ Β«Π€ΠΈΠ·ΠΈΠΊΠ°. 9 класс» (ΠŸΡ€ΠΎΡΠ²Π΅Ρ‰Π΅Π½ΠΈΠ΅ МСдиа, Новый диск)


Π‘Π»Π°ΠΉΠ΄ 54
ВСкст слайда:

Магазин сопротивлСний


Π‘Π»Π°ΠΉΠ΄ 55
ВСкст слайда:

Π—Π°Π²ΠΈΡΠΈΠΌΠΎΡΡ‚ΡŒ силы Ρ‚ΠΎΠΊΠ°

ΠΎΡ‚ напряТСния ΠΈ сопротивлСния


Π‘Π»Π°ΠΉΠ΄ 56
ВСкст слайда:

R =

Π’Π°Π±Π»ΠΈΡ†Π° 1

Π Π΅Π·ΡƒΠ»ΡŒΡ‚Π°Ρ‚Ρ‹


Π‘Π»Π°ΠΉΠ΄ 57
ВСкст слайда:

Π—Π°ΠΊΠΎΠ½ Ома

Π‘ΠΈΠ»Π° Ρ‚ΠΎΠΊΠ° Π½Π° участкС Ρ†Π΅ΠΏΠΈ прямо ΠΏΡ€ΠΎΠΏΠΎΡ€Ρ†ΠΈΠΎΠ½Π°Π»ΡŒΠ½Π° Π½Π°ΠΏΡ€ΡΠΆΠ΅Π½ΠΈΡŽ Π½Π° Π΅Π³ΠΎ ΠΊΠΎΠ½Ρ†Π°Ρ… ΠΈ ΠΎΠ±Ρ€Π°Ρ‚Π½ΠΎ ΠΏΡ€ΠΎΠΏΠΎΡ€Ρ†ΠΈΠΎΠ½Π°Π»ΡŒΠ½Π° Π΅Π³ΠΎ ΡΠΎΠΏΡ€ΠΎΡ‚ΠΈΠ²Π»Π΅Π½ΠΈΡŽ


Π‘Π»Π°ΠΉΠ΄ 58
ВСкст слайда:

Π‘Π»Π°ΠΉΠ΄ 59
ВСкст слайда:

Π—Π°ΠΊΠΎΠ½Β ΠžΠΌΠ°Β Π½Π°Π³Π»ΡΠ΄Π½ΠΎ


Π‘Π»Π°ΠΉΠ΄ 60
ВСкст слайда:

АмпСр АндрС ΠœΠ°Ρ€ΠΈ
1775-1836

АлСсандро Π’ΠΎΠ»ΡŒΡ‚Π°
1745 — 1827

Π“Π΅ΠΎΡ€Π³ Ом
1787 — 1854


Π‘Π»Π°ΠΉΠ΄ 61
ВСкст слайда:

Β«Π’ΠΈΡ€Ρ‚ΡƒΠ°Π»ΡŒΠ½Π°Ρ Ρ„ΠΈΠ·ΠΈΠΊΠ°Β» (Π”. Π’. Баяндин, О.И. ΠœΡƒΡ…ΠΈΠ½, РЦИ ΠŸΠ“Π’Π£).Β 

Π‘ΠΎΠ΄Π΅Ρ€ΠΆΠ°Π½ΠΈΠ΅ ΡƒΡ‡Π΅Π±Π½ΠΎΠΉ Ρ€Π°Π±ΠΎΡ‚Ρ‹:Β ΠΎΠΏΡ€Π΅Π΄Π΅Π»Π΅Π½ΠΈΠ΅ ΠΏΡ€Π΅Π΄Π΅Π»ΠΎΠ² измСрСния, Ρ†Π΅Π½Ρ‹ дСлСния, ΠΏΠΎΠ³Ρ€Π΅ΡˆΠ½ΠΎΡΡ‚ΠΈ измСрСния ΠΈ ΠΏΠΎΠΊΠ°Π·Π°Π½ΠΈΠΉ ΠΏΡ€ΠΈΠ±ΠΎΡ€Π°; запись Ρ€Π΅Π·ΡƒΠ»ΡŒΡ‚Π°Ρ‚Π° измСрСния с ΡƒΡ‡Π΅Ρ‚ΠΎΠΌ ΠΏΠΎΠ³Ρ€Π΅ΡˆΠ½ΠΎΡΡ‚ΠΈ

Π’ΠΈΡ€Ρ‚ΡƒΠ°Π»ΡŒΠ½Ρ‹ΠΉ Ρ‚Ρ€Π΅Π½Π°ΠΆΠ΅Ρ€


Π‘Π»Π°ΠΉΠ΄ 62
ВСкст слайда:

Π’ΠΈΡ€Ρ‚ΡƒΠ°Π»ΡŒΠ½Π°Ρ лабораторная Ρ€Π°Π±ΠΎΡ‚Π°

Π‘ΠΎΠ΄Π΅Ρ€ΠΆΠ°Π½ΠΈΠ΅ ΡƒΡ‡Π΅Π±Π½ΠΎΠΉ Ρ€Π°Π±ΠΎΡ‚Ρ‹:Β 
Π‘Π±ΠΎΡ€ΠΊΠ° элСктричСской Ρ†Π΅ΠΏΠΈ
Π˜Π·ΠΌΠ΅Ρ€Π΅Π½ΠΈΠ΅ силы, Ρ‚ΠΎΠΊΠ°, напряТСния, сопротивлСния с ΠΏΠΎΠΌΠΎΡ‰ΡŒΡŽ Ρ†ΠΈΡ„Ρ€ΠΎΠ²ΠΎΠ³ΠΎ ΠΌΡƒΠ»ΡŒΡ‚ΠΈΠΌΠ΅Ρ‚Ρ€Π°.
ИсслСдованиС зависимости силы Ρ‚ΠΎΠΊΠ° ΠΎΡ‚ напряТСния ΠΈ сопротивлСния.

Начала элСктроники

ΠžΡ‚ΠΊΡ€Ρ‹Ρ‚Π°Ρ Ρ„ΠΈΠ·ΠΈΠΊΠ°: Π§Π°ΡΡ‚ΡŒ 2 ( Β«Π€ΠΈΠ·ΠΈΠΊΠΎΠ½Β»)


Π‘Π»Π°ΠΉΠ΄ 63
ВСкст слайда:

Π”Π°ΠΉΡ‚Π΅ ΠΎΠΏΡ€Π΅Π΄Π΅Π»Π΅Π½ΠΈΠ΅ элСктричСского Ρ‚ΠΎΠΊΠ°.
ΠŸΡ€ΠΈ ΠΊΠ°ΠΊΠΈΡ… условиях Π²ΠΎΠ·Π½ΠΈΠΊΠ°Π΅Ρ‚ элСктричСский Ρ‚ΠΎΠΊ?
Π§Π΅ΠΌ отличаСтся Π΄Π²ΠΈΠΆΠ΅Π½ΠΈΠ΅ заряТСнных частиц Π² ΠΏΡ€ΠΎΠ²ΠΎΠ΄Π½ΠΈΠΊΠ΅ Π² отсутствиС ΠΈ ΠΏΡ€ΠΈ Π½Π°Π»ΠΈΡ‡ΠΈΠΈ внСшнСго элСктричСского поля?
Как Π½Π°ΠΏΡ€Π°Π²Π»Π΅Π½ элСктричСский Ρ‚ΠΎΠΊ?
Π’ ΠΊΠ°ΠΊΠΎΠΌ Π½Π°ΠΏΡ€Π°Π²Π»Π΅Π½ΠΈΠΈ двиТутся элСктроны Π² мСталличСском ΠΏΡ€ΠΎΠ²ΠΎΠ΄Π½ΠΈΠΊΠ΅, ΠΏΠΎ ΠΊΠΎΡ‚ΠΎΡ€ΠΎΠΌΡƒ ΠΏΡ€ΠΎΡ‚Π΅ΠΊΠ°Π΅Ρ‚ элСктричСский Ρ‚ΠΎΠΊ?
Π§Ρ‚ΠΎ Π½Π°Π·Ρ‹Π²Π°ΡŽΡ‚ силой Ρ‚ΠΎΠΊΠ°?
Какова Π΅Π΄ΠΈΠ½ΠΈΡ†Π° измСрСния силы Ρ‚ΠΎΠΊΠ°?
Каким ΠΏΡ€ΠΈΠ±ΠΎΡ€ΠΎΠΌ ΠΈΠ·ΠΌΠ΅Ρ€ΡΡŽΡ‚ силу Ρ‚ΠΎΠΊΠ°? Как ΠΎΠ½ ΠΏΠΎΠ΄ΠΊΠ»ΡŽΡ‡Π°Π΅Ρ‚ΡΡ?
Π§Ρ‚ΠΎ Ρ‚Π°ΠΊΠΎΠ΅ напряТСниС?
Какова Π΅Π΄ΠΈΠ½ΠΈΡ†Π° напряТСния?
Каким ΠΏΡ€ΠΈΠ±ΠΎΡ€ΠΎΠΌ ΠΈΠ·ΠΌΠ΅Ρ€ΡΡŽΡ‚ напряТСниС? Как ΠΎΠ½ ΠΏΠΎΠ΄ΠΊΠ»ΡŽΡ‡Π°Π΅Ρ‚ΡΡ?
Π§Ρ‚ΠΎ Ρ‚Π°ΠΊΠΎΠ΅ сопротивлСниС? Какова ΠΏΡ€ΠΈΡ‡ΠΈΠ½Π° сопротивлСния?
Какова Π΅Π΄ΠΈΠ½ΠΈΡ†Π° сопротивлСния?
Каким ΠΏΡ€ΠΈΠ±ΠΎΡ€ΠΎΠΌ ΠΈΠ·ΠΌΠ΅Ρ€ΡΡŽΡ‚ сопротивлСниС? Как ΠΎΠ½ ΠΏΠΎΠ΄ΠΊΠ»ΡŽΡ‡Π°Π΅Ρ‚ΡΡ?
Π‘Ρ„ΠΎΡ€ΠΌΡƒΠ»ΠΈΡ€ΡƒΠΉΡ‚Π΅ Π·Π°ΠΊΠΎΠ½ Ома для участка Ρ†Π΅ΠΏΠΈ.

Вопросы для самоконтроля


Π‘Π»Π°ΠΉΠ΄ 64
ВСкст слайда:

Бписок Π»ΠΈΡ‚Π΅Ρ€Π°Ρ‚ΡƒΡ€Ρ‹

Бписок элСктронных ΡƒΡ‡Π΅Π±Π½Ρ‹Ρ… ΠΈΠ·Π΄Π°Π½ΠΈΠΉ

ΠŸΠ΅Ρ€Ρ‹ΡˆΠΊΠΈΠ½ А.Π’. Π€ΠΈΠ·ΠΈΠΊΠ°. 8 класс.
МякишСв Π“.Π―., Π‘ΡƒΡ…ΠΎΠ²Ρ†Π΅Π² Π‘.Π‘., Ботский Н.Н. Π€ΠΈΠ·ΠΈΠΊΠ°. 10 класс.

Π€ΠΈΠ·ΠΈΠΊΠ°, 7-11 ΠΊΠ». Π‘ΠΈΠ±Π»ΠΈΠΎΡ‚Π΅ΠΊΠ° наглядных пособий (1Π‘)
ΠžΡ‚ΠΊΡ€Ρ‹Ρ‚Π°Ρ Ρ„ΠΈΠ·ΠΈΠΊΠ°, Ρ‡.2 (Π€ΠΈΠ·ΠΈΠΊΠΎΠ½)
ΠœΡƒΠ»ΡŒΡ‚ΠΈΠΌΠ΅Π΄ΠΈΠΉΠ½ΠΎΠ΅ ΠΏΡ€ΠΈΠ»ΠΎΠΆΠ΅Π½ΠΈΠ΅ ΠΊ ΡƒΡ‡Π΅Π±Π½ΠΈΠΊΡƒ Π‘.Π’. Π“Ρ€ΠΎΠΌΠΎΠ²Π° ΠΈ Н.А.
Π ΠΎΠ΄ΠΈΠ½ΠΎΠΉ Β«Π€ΠΈΠ·ΠΈΠΊΠ°. 9 класс» (ΠŸΡ€ΠΎΡΠ²Π΅Ρ‰Π΅Π½ΠΈΠ΅ МСдиа, Новый диск)
Β«Π’ΠΈΡ€Ρ‚ΡƒΠ°Π»ΡŒΠ½Π°Ρ Ρ„ΠΈΠ·ΠΈΠΊΠ°Β» (Π”.Π’. Баяндин, О.И. ΠœΡƒΡ…ΠΈΠ½, РЦИ ΠŸΠ“Π’Π£).Β 
Начала элСктроники (http://zeus.malishich.com )


Π‘Π»Π°ΠΉΠ΄ 65
ВСкст слайда:

http://www.fizika.ru
http://school.xvatit.com
http://fizportal.ru
http://slovari.yandex.ru
http://cxem.pp.ua
http://www.td-medstar.ru
http://microschemes.pp.ua
http://base.eworld.ru
http://www.ww2.ru
http://portal.etherway.ru

Бписок рСсурсов Π˜Π½Ρ‚Π΅Ρ€Π½Π΅Ρ‚

http://www. go-radio.ru
http://hystory.ru
http://radionostalgia.ca
http://www.pribortorg.by
http://www.avito.ru
http://omop.su
http://www.proshkolu.ru
http://masteram.com.ua
http://solo-project.com

ΠΈΠ»Π»ΡŽΡΡ‚Ρ€Π°Ρ†ΠΈΠΈ


Π£Ρ€ΠΎΠΊ Ρ„ΠΈΠ·ΠΈΠΊΠΈ Π² 8-ΠΌ классС ΠΏΠΎ Ρ‚Π΅ΠΌΠ΅ «Π—Π°Π²ΠΈΡΠΈΠΌΠΎΡΡ‚ΡŒ силы Ρ‚ΠΎΠΊΠ° ΠΎΡ‚ напряТСния. Π—Π°ΠΊΠΎΠ½ Ома для участка Ρ†Π΅ΠΏΠΈ»

Π’Π΅ΠΌΠ° ΡƒΡ€ΠΎΠΊΠ°: Π—Π°Π²ΠΈΡΠΈΠΌΠΎΡΡ‚ΡŒ силы Ρ‚ΠΎΠΊΠ° ΠΎΡ‚ напряТСния. Π—Π°ΠΊΠΎΠ½ Ома для участка Ρ†Π΅ΠΏΠΈ.

ЦСль ΡƒΡ€ΠΎΠΊΠ°: Π£ΡΡ‚Π°Π½ΠΎΠ²ΠΈΡ‚ΡŒ Π·Π°Π²ΠΈΡΠΈΠΌΠΎΡΡ‚ΡŒ ΠΌΠ΅ΠΆΠ΄Ρƒ силой Ρ‚ΠΎΠΊΠ°, напряТСниСм Π½Π° ΠΎΠ΄Π½ΠΎΡ€ΠΎΠ΄Π½ΠΎΠΌ участкС элСктричСской Ρ†Π΅ΠΏΠΈ ΠΈ сопротивлСниСм этого участка.

Π—Π°Π΄Π°Ρ‡ΠΈ ΡƒΡ€ΠΎΠΊΠ°:

  • Π’Ρ‹ΡΡΠ½ΠΈΡ‚ΡŒ, Ρ‡Ρ‚ΠΎ сила Ρ‚ΠΎΠΊΠ° Π² участкС Ρ†Π΅ΠΏΠΈ ΠΎΠ±Ρ€Π°Ρ‚Π½ΠΎ ΠΏΡ€ΠΎΠΏΠΎΡ€Ρ†ΠΈΠΎΠ½Π°Π»ΡŒΠ½Π° Π΅Π³ΠΎ ΡΠΎΠΏΡ€ΠΎΡ‚ΠΈΠ²Π»Π΅Π½ΠΈΡŽ, Ссли ΠΏΡ€ΠΈ этом напряТСниС остаСтся постоянным
  • Π²Ρ‹ΡΡΠ½ΠΈΡ‚ΡŒ, Ρ‡Ρ‚ΠΎ сила Ρ‚ΠΎΠΊΠ° прямо ΠΏΡ€ΠΎΠΏΠΎΡ€Ρ†ΠΈΠΎΠ½Π°Π»ΡŒΠ½Π° Π½Π°ΠΏΡ€ΡΠΆΠ΅Π½ΠΈΡŽ Π½Π° ΠΊΠΎΠ½Ρ†Π°Ρ… ΠΏΡ€ΠΎΠ²ΠΎΠ΄Π½ΠΈΠΊΠ°, Ссли ΠΏΡ€ΠΈ этом сопротивлСниС Π½Π΅ мСняСтся.
  • Π½Π°ΡƒΡ‡ΠΈΡ‚ΡŒΡΡ ΠΏΡ€ΠΈΠΌΠ΅Π½ΡΡ‚ΡŒ Π·Π°ΠΊΠΎΠ½ Ома для участка Ρ†Π΅ΠΏΠΈ ΠΏΡ€ΠΈ Ρ€Π΅ΡˆΠ΅Π½ΠΈΠΈ Π·Π°Π΄Π°Ρ‡.
  • Π½Π°ΡƒΡ‡ΠΈΡ‚ΡŒΡΡ ΠΎΠΏΡ€Π΅Π΄Π΅Π»ΡΡ‚ΡŒ силу Ρ‚ΠΎΠΊΠ°, напряТСниС ΠΏΠΎ Π³Ρ€Π°Ρ„ΠΈΠΊΡƒ зависимости ΠΌΠ΅ΠΆΠ΄Ρƒ этими Π²Π΅Π»ΠΈΡ‡ΠΈΠ½Π°ΠΌΠΈ, Π° Ρ‚Π°ΠΊΠΆΠ΅ сопротивлСниС.

ΠžΠ±ΠΎΡ€ΡƒΠ΄ΠΎΠ²Π°Π½ΠΈΠ΅: Π­ΠΊΡ€Π°Π½,Β  дСмонстрационныС Π°ΠΌΠΏΠ΅Ρ€ΠΌΠ΅Ρ‚Ρ€ ΠΈ Π²ΠΎΠ»ΡŒΡ‚ΠΌΠ΅Ρ‚Ρ€, источник Ρ‚ΠΎΠΊΠ°, ΠΊΠ»ΡŽΡ‡, ΡΠΎΠ΅Π΄ΠΈΠ½ΠΈΡ‚Π΅Π»ΡŒΠ½Ρ‹Π΅ ΠΏΡ€ΠΎΠ²ΠΎΠ΄Π°, дСмонстрационный ΠΌΠ°Π³Π°Π·ΠΈΠ½ сопротивлСний, ВБО, ΠΏΠΎΡ€Ρ‚Ρ€Π΅Ρ‚Ρ‹ ΡƒΡ‡Π΅Π½Ρ‹Ρ….

План ΡƒΡ€ΠΎΠΊΠ°

  1. ΠžΡ€Π³Π°Π½ΠΈΠ·Π°Ρ†ΠΈΠΎΠ½Π½Ρ‹ΠΉ ΠΌΠΎΠΌΠ΅Π½Ρ‚.
  2. Ρ†Π΅Π»ΡŒΡŽ ΠΏΠΎΠ΄Π³ΠΎΡ‚ΠΎΠ²ΠΊΠΈ ΠΊ Π²ΠΎΡΠΏΡ€ΠΈΡΡ‚ΠΈΡŽ Π½ΠΎΠ²ΠΎΠ³ΠΎ ΠΌΠ°Ρ‚Π΅Ρ€ΠΈΠ°Π»Π°.
  3. Π˜Π·ΡƒΡ‡Π΅Π½ΠΈΠ΅ Π½ΠΎΠ²ΠΎΠ³ΠΎ ΠΌΠ°Ρ‚Π΅Ρ€ΠΈΠ°Π»Π°.
  4. Π—Π°ΠΊΡ€Π΅ΠΏΠ»Π΅Π½ΠΈΠ΅ Π·Π½Π°Π½ΠΈΠΉ, ΡƒΠΌΠ΅Π½ΠΈΠΉ ΠΈ Π½Π°Π²Ρ‹ΠΊΠΎΠ².
  5. Π”ΠΎΠΌΠ°ΡˆΠ½Π΅Π΅ Π·Π°Π΄Π°Π½ΠΈΠ΅.
  6. ПодвСдСниС ΠΈΡ‚ΠΎΠ³ΠΎΠ² ΡƒΡ€ΠΎΠΊΠ°.

Π₯ΠžΠ” УРОКА

1. ΠžΡ€Π³Π°Π½ΠΈΠ·Π°Ρ†ΠΈΠΎΠ½Π½Ρ‹ΠΉ ΠΌΠΎΠΌΠ΅Π½Ρ‚

Π£Ρ‡ΠΈΡ‚Π΅Π»ΡŒ: По словам русского поэта Π₯IΠ₯ Π²Π΅ΠΊΠ° Π―ΠΊΠΎΠ²Π° ΠŸΠ΅Ρ‚Ρ€ΠΎΠ²ΠΈΡ‡Π° Полонского,

Царство Π½Π°ΡƒΠΊΠΈ Π½Π΅ Π·Π½Π°Π΅Ρ‚ ΠΏΡ€Π΅Π΄Π΅Π»Π° –
Π’ΡΡŽΠ΄Ρƒ слСды Π΅Π΅ Π²Π΅Ρ‡Π½Ρ‹Ρ… ΠΏΠΎΠ±Π΅Π΄,
Π Π°Π·ΡƒΠΌΠ° слово ΠΈ Π΄Π΅Π»ΠΎ,
Π‘ΠΈΠ»Π° ΠΈ свСт.

Π­Ρ‚ΠΈ слова ΠΏΠΎ ΠΏΡ€Π°Π²Ρƒ ΠΌΠΎΠΆΠ½ΠΎ отнСсти  ΠΊ Ρ‚Π΅ΠΌΠ΅, ΠΊΠΎΡ‚ΠΎΡ€ΡƒΡŽ ΠΌΡ‹ сСйчас ΠΈΠ·ΡƒΡ‡Π°Π΅ΠΌ — элСктричСскиС явлСния.  Они ΠΏΠΎΠ΄Π°Ρ€ΠΈΠ»ΠΈ Π½Π°ΠΌ ΠΌΠ½ΠΎΠ³ΠΎ ΠΎΡ‚ΠΊΡ€Ρ‹Ρ‚ΠΈΠΉ, ΠΎΡΠ²Π΅Ρ‚ΠΈΠ²ΡˆΠΈΡ… Π½Π°ΡˆΡƒ Тизнь Π² прямом ΠΈ пСрСносном смыслС. А сколько Π΅Ρ‰Π΅ Π½Π΅ΠΎΠΏΠΎΠ·Π½Π°Π½Π½ΠΎΠ³ΠΎ Π²ΠΎΠΊΡ€ΡƒΠ³! КакоС ΠΏΠΎΠ»Π΅ Π΄Π΅ΡΡ‚Π΅Π»ΡŒΠ½ΠΎΡΡ‚ΠΈ для ΠΏΡ‹Ρ‚Π»ΠΈΠ²ΠΎΠ³ΠΎ ΡƒΠΌΠ°, ΡƒΠΌΠ΅Π»Ρ‹Ρ… Ρ€ΡƒΠΊ ΠΈ Π»ΡŽΠ±ΠΎΠ·Π½Π°Ρ‚Π΅Π»ΡŒΠ½ΠΎΠΉ Π½Π°Ρ‚ΡƒΡ€Ρ‹. Π’Π°ΠΊ Ρ‡Ρ‚ΠΎ запускайтС свой Β«Π²Π΅Ρ‡Π½Ρ‹ΠΉ Π΄Π²ΠΈΠ³Π°Ρ‚Π΅Π»ΡŒΒ», ΠΈ Π²ΠΏΠ΅Ρ€Π΅Π΄!
Вспомним, Ρ‡Ρ‚ΠΎ изучая Ρ‚Π΅ΠΌΡƒ «ЭлСктричСскиС явлСния», Π²Ρ‹ ΡƒΠ·Π½Π°Π»ΠΈ основныС Π²Π΅Π»ΠΈΡ‡ΠΈΠ½Ρ‹, Ρ…Π°Ρ€Π°ΠΊΡ‚Π΅Ρ€ΠΈΠ·ΡƒΡŽΡ‰ΠΈΠ΅ элСктричСскиС Ρ†Π΅ΠΏΠΈ.

2. Актуализация Π·Π½Π°Π½ΠΈΠΉ учащихся

Π£Ρ‡ΠΈΡ‚Π΅Π»ΡŒ: Π’ Π½Π°Ρ‡Π°Π»Π΅,Β  поТалуйста, пСрСчислим основныС Π²Π΅Π»ΠΈΡ‡ΠΈΠ½Ρ‹, Ρ…Π°Ρ€Π°ΠΊΡ‚Π΅Ρ€ΠΈΠ·ΡƒΡŽΡ‰ΠΈΠ΅ элСктричСскиС Ρ†Π΅ΠΏΠΈ.

Π£Ρ‡Π΅Π½ΠΈΠΊΠΈ: Π‘ΠΈΠ»Π° Ρ‚ΠΎΠΊΠ°, напряТСниС ΠΈ сопротивлСниС.

Π£Ρ‡ΠΈΡ‚Π΅Π»ΡŒ:  А Ρ‚Π΅ΠΏΠ΅Ρ€ΡŒ, Π΄Π°ΠΉΡ‚Π΅ Π½Π΅Π±ΠΎΠ»ΡŒΡˆΡƒΡŽ характСристику ΠΊΠ°ΠΆΠ΄ΠΎΠΉ ΠΈΠ· этих Π²Π΅Π»ΠΈΡ‡ΠΈΠ½, ΠΏΠΎ ΡΠ»Π΅Π΄ΡƒΡŽΡ‰Π΅ΠΌΡƒ ΠΏΠ»Π°Π½Ρƒ:

  1. НазваниС Π²Π΅Π»ΠΈΡ‡ΠΈΠ½Ρ‹.
  2. Π§Ρ‚ΠΎ Ρ…Π°Ρ€Π°ΠΊΡ‚Π΅Ρ€ΠΈΠ·ΡƒΠ΅Ρ‚ данная Π²Π΅Π»ΠΈΡ‡ΠΈΠ½Π°?
  3. По ΠΊΠ°ΠΊΠΎΠΉ Ρ„ΠΎΡ€ΠΌΡƒΠ»Π΅ находится?
  4. Π’ ΠΊΠ°ΠΊΠΈΡ… Π΅Π΄ΠΈΠ½ΠΈΡ†Π°Ρ… измСряСтся?
  5. Каким ΠΏΡ€ΠΈΠ±ΠΎΡ€ΠΎΠΌ измСряСтся ΠΈΠ»ΠΈ измСняСтся?

Π£Ρ‡Π΅Π½ΠΈΠΊΠΈ:
Π‘ΠΈΠ»Π° Ρ‚ΠΎΠΊΠ° – Ρ…Π°Ρ€Π°ΠΊΡ‚Π΅Ρ€ΠΈΠ·ΡƒΠ΅Ρ‚ элСктричСский Ρ‚ΠΎΠΊ Π² ΠΏΡ€ΠΎΠ²ΠΎΠ΄Π½ΠΈΠΊΠ΅.
– Ρ„ΠΎΡ€ΠΌΡƒΠ»Π° для нахоТдСния силы Ρ‚ΠΎΠΊΠ°, Π³Π΄Π΅ q-заряд, проходящий Ρ‡Π΅Ρ€Π΅Π· ΠΏΠΎΠΏΠ΅Ρ€Π΅Ρ‡Π½ΠΎΠ΅ сСчСниС ΠΏΡ€ΠΎΠ²ΠΎΠ΄Π½ΠΈΠΊΠ°, t-врСмя прохоТдСния заряда. Π•Π΄ΠΈΠ½ΠΈΡ†Π° измСрСния – Π°ΠΌΠΏΠ΅Ρ€. Π˜Π·ΠΌΠ΅Ρ€ΡΠ΅Ρ‚ΡΡ сила Ρ‚ΠΎΠΊΠ° – Π°ΠΌΠΏΠ΅Ρ€ΠΌΠ΅Ρ‚Ρ€ΠΎΠΌ.
НапряТСниС-Π²Π΅Π»ΠΈΡ‡ΠΈΠ½Π°, которая Ρ…Π°Ρ€Π°ΠΊΡ‚Π΅Ρ€ΠΈΠ·ΡƒΠ΅Ρ‚ элСктричСскоС ΠΏΠΎΠ»Π΅.
– Ρ„ΠΎΡ€ΠΌΡƒΠ»Π° для нахоТдСния напряТСния, Π³Π΄Π΅ А- Ρ€Π°Π±ΠΎΡ‚Π° ΠΏΠΎ пСрСносу заряда Ρ‡Π΅Ρ€Π΅Π· ΠΏΠΎΠΏΠ΅Ρ€Π΅Ρ‡Π½ΠΎΠ΅ сСчСниС ΠΏΡ€ΠΎΠ²ΠΎΠ΄Π½ΠΈΠΊΠ°, q-заряд. Π•Π΄ΠΈΠ½ΠΈΡ†Π° измСрСния – Π²ΠΎΠ»ΡŒΡ‚. НапряТСниС измСряСтся Β Π²ΠΎΠ»ΡŒΡ‚ΠΌΠ΅Ρ‚Ρ€ΠΎΠΌ.
Π‘ΠΎΠΏΡ€ΠΎΡ‚ΠΈΠ²Π»Π΅Π½ΠΈΠ΅ Ρ…Π°Ρ€Π°ΠΊΡ‚Π΅Ρ€ΠΈΠ·ΡƒΠ΅Ρ‚ сам ΠΏΡ€ΠΎΠ²ΠΎΠ΄Π½ΠΈΠΊ, обозначаСтся – R, Π΅Π΄ΠΈΠ½ΠΈΡ†Π° измСрСния 1Ом.

Π£Ρ‡ΠΈΡ‚Π΅Π»ΡŒ:Β  Π½Π° доскС заполняСм Ρ‚Π°Π±Π»ΠΈΡ†Ρƒ 1:

Π’Π°Π±Π»ΠΈΡ†Π° 1

ЀизичСская Π²Π΅Π»ΠΈΡ‡ΠΈΠ½Π°

Π‘ΠΈΠ»Π° Ρ‚ΠΎΠΊΠ°

НапряТСниС

Π‘ΠΎΠΏΡ€ΠΎΡ‚ΠΈΠ²Π»Π΅Π½ΠΈΠ΅

ΠžΠ±ΠΎΠ·Π½Π°Ρ‡Π΅Π½ΠΈΠ΅ Β  Β  Β 
Π•Π΄ΠΈΠ½ΠΈΡ†Π° измСрСния Β  Β  Β 
ΠŸΡ€ΠΈΠ±ΠΎΡ€ для измСрСния ΠΈΠ»ΠΈ измСнСния Β  Β  Β 

ΠŸΡ€Π°Π²ΠΈΠ»ΡŒΠ½ΠΎ, заполнСнная Ρ‚Π°Π±Π»ΠΈΡ†Π° 1:

Π’Π°Π±Π»ΠΈΡ†Π° 1

ЀизичСская Π²Π΅Π»ΠΈΡ‡ΠΈΠ½Π°

Π‘ΠΈΠ»Π° Ρ‚ΠΎΠΊΠ°

НапряТСниС

Π‘ΠΎΠΏΡ€ΠΎΡ‚ΠΈΠ²Π»Π΅Π½ΠΈΠ΅

ΠžΠ±ΠΎΠ·Π½Π°Ρ‡Π΅Π½ΠΈΠ΅

I

U

R

Π•Π΄ΠΈΠ½ΠΈΡ†Π° измСрСния

1 А
(1 АмпСр)

1 Π’
(1 Π’ΠΎΠ»ΡŒΡ‚)

1 Ом

ΠŸΡ€ΠΈΠ±ΠΎΡ€ для измСрСния ΠΈΠ»ΠΈ измСнСния

АмпСрмСтр

Π’ΠΎΠ»ΡŒΡ‚ΠΌΠ΅Ρ‚Ρ€

ΠŸΡ€ΠΎΠ²ΠΎΠ΄Π½ΠΈΠΊ

Β Π£Ρ‡ΠΈΡ‚Π΅Π»ΡŒ: РСбята, Π° Ρ‡Ρ‚ΠΎ Π²Ρ‹ Π·Π½Π°Π΅Ρ‚Π΅ ΠΎΠ± ΡƒΡ‡Π΅Π½Ρ‹Ρ…,Β  ΠΎΡ‚ΠΊΡ€Ρ‹Π²ΡˆΠΈΡ… силу Ρ‚ΠΎΠΊΠ°, напряТСниС, сопротивлСниС?

(Π£Ρ‡Π΅Π½ΠΈΠΊΠΈ ΠΏΡ€ΠΈΠ³ΠΎΡ‚ΠΎΠ²ΠΈΠ»ΠΈ сообщСния ΠΎΠ± ΡƒΡ‡Π΅Π½Ρ‹Ρ… Ρ„ΠΈΠ·ΠΈΠΊΠ°Ρ…)

Π£Ρ‡Π΅Π½ΠΈΠΊΠΈ: Π•Π΄ΠΈΠ½ΠΈΡ†Ρ‹ измСрСния физичСских Π²Π΅Π»ΠΈΡ‡ΠΈΠ½ силы Ρ‚ΠΎΠΊΠ°, напряТСния ΠΈ сопротивлСния, Π½Π°Π·Π²Π°Π½Ρ‹ Π² Ρ‡Π΅ΡΡ‚ΡŒ ΡƒΡ‡Π΅Π½Ρ‹Ρ… ΠΎΡ‚ΠΊΡ€Ρ‹Π²ΡˆΠΈΡ… ΠΈΡ…. АмпСр, Π’ΠΎΠ»ΡŒΡ‚ ΠΈ Ом.

АндрС-ΠœΠ°Ρ€ΠΈ АмпСр – Π½Π° Π΅Π³ΠΎ памятникС высСчСна надпись:Β  «Он Π±Ρ‹Π» Ρ‚Π°ΠΊΠΆΠ΅ Π΄ΠΎΠ±Ρ€ ΠΈ Ρ‚Π°ΠΊΠΆΠ΅ прост, ΠΊΠ°ΠΊ ΠΈ Π²Π΅Π»ΠΈΠΊΒ». Блавился своСй Ρ€Π°ΡΡΠ΅ΡΠ½Π½ΠΎΡΡ‚ΡŒΡŽ. ΠŸΡ€ΠΎ Π½Π΅Π³ΠΎ рассказывали, Ρ‡Ρ‚ΠΎ ΠΎΠ΄Π½Π°ΠΆΠ΄Ρ‹ ΠΎΠ½ с сосрСдоточСнным Π²ΠΈΠ΄ΠΎΠΌ Π²Π°Ρ€ΠΈΠ» Π² Π²ΠΎΠ΄Π΅ свои часы 3 ΠΌΠΈΠ½ΡƒΡ‚Ρ‹, Π΄Π΅Ρ€ΠΆΠ° яйцо Π² Ρ€ΡƒΠΊΠ΅.

АлСссандро Π’ΠΎΠ»ΡŒΡ‚Π° – Π±Ρ‹Π» Ρ€Ρ‹Ρ†Π°Ρ€Π΅ΠΌ ΠΏΠΎΡ‡Π΅Ρ‚Π½ΠΎΠ³ΠΎ Π»Π΅Π³ΠΈΠΎΠ½Π°, ΠΏΠΎΠ»ΡƒΡ‡ΠΈΠ» Π·Π²Π°Π½ΠΈΠ΅ сСнатора ΠΈ Π³Ρ€Π°Ρ„Π°. НаполСон Π½Π΅ упускал случая ΠΏΠΎΡΠ΅Ρ‚ΠΈΡ‚ΡŒ засСдания Ѐранцузской Π°ΠΊΠ°Π΄Π΅ΠΌΠΈΠΈ Π½Π°ΡƒΠΊ, Π³Π΄Π΅ ΠΎΠ½ выступал. Π˜Π·ΠΎΠ±Ρ€Π΅Π» ΡΠ»Π΅ΠΊΡ‚Ρ€ΠΈΡ‡Π΅ΡΠΊΡƒΡŽ Π±Π°Ρ‚Π°Ρ€Π΅ΡŽ, ΠΏΡ‹ΡˆΠ½ΠΎ Π½Π°Π·Π²Π°Π½Π½ΡƒΡŽ Β«ΠΊΠΎΡ€ΠΎΠ½ΠΎΠΉ сосудов».

Π“Π΅ΠΎΡ€Π³ Ом – Π½Π΅ΠΌΠ΅Ρ†ΠΊΠΈΠΉ Ρ„ΠΈΠ·ΠΈΠΊ. ΠžΠΏΡ‹Ρ‚Ρ‹ ΠΈ тСорСтичСскиС Π΄ΠΎΠΊΠ°Π·Π°Ρ‚Π΅Π»ΡŒΡΡ‚Π²Π° Π±Ρ‹Π»ΠΈ описаны ΠΈΠΌ Π² Π³Π»Π°Π²Π½ΠΎΠΌ Ρ‚Ρ€ΡƒΠ΄Π΅ Β«Π“Π°Π»ΡŒΠ²Π°Π½ΠΈΡ‡Π΅ΡΠΊΠ°Ρ Ρ†Π΅ΠΏΡŒ, разработанная матСматичСски», Π²Ρ‹ΡˆΠ΅Π΄ΡˆΠ΅ΠΌ Π² 1827Β Π³.

Π Π°Π·Π½ΠΎΡƒΡ€ΠΎΠ²Π½Π΅Π²Ρ‹Π΅ задания:

Π—Π°Π΄Π°Π½ΠΈΠ΅ β„–1

1. Бколько Π°ΠΌΠΏΠ΅Ρ€ Π² 250 мА?

А) 250 А;
Π‘) 25 А;
Π’) 2,5 А;
Π“) 0,25 А.

2. Π’ΡΡ‚Π°Π²ΡŒΡ‚Π΅ ΠΏΡ€ΠΎΠΏΡƒΡ‰Π΅Π½Π½ΠΎΠ΅ ΠΎΠΏΡ€Π΅Π΄Π΅Π»Π΅Π½ΠΈΠ΅:

Π’Π΅Π»ΠΈΡ‡ΠΈΠ½Π°, равная … называСтся элСктричСским напряТСниСм.

А) ΠΏΡ€ΠΎΠΈΠ·Π²Π΅Π΄Π΅Π½ΠΈΡŽ мощности Π½Π° силу Ρ‚ΠΎΠΊΠ°;
Π‘) ΠΎΡ‚Π½ΠΎΡˆΠ΅Π½ΠΈΡŽ мощности ΠΊ силС Ρ‚ΠΎΠΊΠ°;
Π’) ΠΎΡ‚Π½ΠΎΡˆΠ΅Π½ΠΈΡŽ Ρ€Π°Π±ΠΎΡ‚Ρ‹ ΠΊ Π²Π΅Π»ΠΈΡ‡ΠΈΠ½Π΅ элСктричСского заряда.

3. НачСртитС схСму элСктричСской Ρ†Π΅ΠΏΠΈ: источник Ρ‚ΠΎΠΊΠ°, ΠΊΠ»ΡŽΡ‡, Π°ΠΌΠΏΠ΅Ρ€ΠΌΠ΅Ρ‚Ρ€, ΡΠΎΠ΅Π΄ΠΈΠ½ΠΈΡ‚Π΅Π»ΡŒΠ½Ρ‹Π΅ ΠΏΡ€ΠΎΠ²ΠΎΠ΄Π°, Β Π΄Π²Π΅ Π»Π°ΠΌΠΏΠΎΡ‡ΠΊΠΈ ΠΈ Π²ΠΎΠ»ΡŒΡ‚ΠΌΠ΅Ρ‚Ρ€, ΠΈΠ·ΠΌΠ΅Ρ€ΡΡŽΡ‰ΠΈΠΉ напряТСниС Π½Π° ΠΎΠ΄Π½ΠΎΠΉ ΠΈΠ· Π»Π°ΠΌΠΏΠΎΡ‡Π΅ΠΊ.

ΠžΡ‚Π²Π΅Ρ‚: (1 – Π“; 2 – Π’; 3 – Рис.1)

Рис. 1

Π—Π°Π΄Π°Π½ΠΈΠ΅ β„–2

1. Бколько ΠΊΠΈΠ»ΠΎΠ²ΠΎΠ»ΡŒΡ‚Β  Π² 750 Π’?

А) 750000 ΠΊΠ’;
Π‘) 0,75 ΠΊΠ’;
Π’) 75 ΠΊΠ’;
Π“) 7,5 ΠΊΠ’.

2. Π’ΡΡ‚Π°Π²ΡŒΡ‚Π΅ ΠΏΡ€ΠΎΠΏΡƒΡ‰Π΅Π½Π½ΠΎΠ΅ ΠΎΠΏΡ€Π΅Π΄Π΅Π»Π΅Π½ΠΈΠ΅:

Π’Π΅Π»ΠΈΡ‡ΠΈΠ½Π°, равная … называСтся силой Ρ‚ΠΎΠΊΠ°.

А) ΠΎΡ‚Π½ΠΎΡˆΠ΅Π½ΠΈΡŽ Ρ€Π°Π±ΠΎΡ‚Ρ‹ ΠΊ Π²Π΅Π»ΠΈΡ‡ΠΈΠ½Π΅ элСктричСского заряда;
Π‘) ΠΎΡ‚Π½ΠΎΡˆΠ΅Π½ΠΈΡŽ элСктричСского заряда ΠΊΠΎ Π²Ρ€Π΅ΠΌΠ΅Π½ΠΈ;
Π’) ΠΏΡ€ΠΎΠΈΠ·Π²Π΅Π΄Π΅Π½ΠΈΡŽ Ρ€Π°Π±ΠΎΡ‚Ρ‹ Β Π½Π° врСмя.

3.  НачСртитС схСму элСктричСской Ρ†Π΅ΠΏΠΈ: источник Ρ‚ΠΎΠΊΠ°, ΠΊΠ»ΡŽΡ‡, Π°ΠΌΠΏΠ΅Ρ€ΠΌΠ΅Ρ‚Ρ€, ΡΠΎΠ΅Π΄ΠΈΠ½ΠΈΡ‚Π΅Π»ΡŒΠ½Ρ‹Π΅ ΠΏΡ€ΠΎΠ²ΠΎΠ΄Π°, Π΄Π²Π΅ Π»Π°ΠΌΠΏΠΎΡ‡ΠΊΠΈ ΠΈ Π²ΠΎΠ»ΡŒΡ‚ΠΌΠ΅Ρ‚Ρ€, ΠΈΠ·ΠΌΠ΅Ρ€ΡΡŽΡ‰ΠΈΠΉ напряТСниС Π½Π° Β Π΄Π²ΡƒΡ… Π»Π°ΠΌΠΏΠΎΡ‡ΠΊΠ°Ρ….

ΠžΡ‚Π²Π΅Ρ‚: (1 – Π‘; 2 – Π‘; 3 – Рис.2)

Рис. 2

3. Π˜Π·ΡƒΡ‡Π΅Π½ΠΈΠ΅ Π½ΠΎΠ²ΠΎΠ³ΠΎ ΠΌΠ°Ρ‚Π΅Ρ€ΠΈΠ°Π»Π°

Π£Ρ‡ΠΈΡ‚Π΅Π»ΡŒ: На ΠΏΡ€ΠΎΡˆΠ»Ρ‹Ρ… ΡƒΡ€ΠΎΠΊΠ°Ρ… рСбята, ΠΌΡ‹ ΠΈΠ·ΡƒΡ‡Π°Π»ΠΈ силу Ρ‚ΠΎΠΊΠ°, напряТСниС ΠΈ сопротивлСниС Π² ΠΎΡ‚Π΄Π΅Π»ΡŒΠ½ΠΎΡΡ‚ΠΈ. БСгодня ΠΌΡ‹ ΠΏΠ΅Ρ€Π΅Π΄ собой поставили Ρ†Π΅Π»ΡŒ: Ρ€Π°ΡΠΊΡ€Ρ‹Ρ‚ΡŒ Π²Π·Π°ΠΈΠΌΠΎΠ·Π°Π²ΠΈΡΠΈΠΌΠΎΡΡ‚ΡŒ силы Ρ‚ΠΎΠΊΠ°, напряТСния ΠΈ сопротивлСния Π½Π° участкС элСктричСской Ρ†Π΅ΠΏΠΈ. Выясним, Β ΠΊΠ°ΠΊ зависит сила Ρ‚ΠΎΠΊΠ° ΠΎΡ‚ сопротивлСния, Ссли напряТСниС остаСтся постоянным.
ΠžΠ±Ρ€Π°Ρ‚ΠΈΠΌΡΡ ΠΊ ΠΎΠΏΡ‹Ρ‚Ρƒ:

1. Π‘ΠΎΠ±Π΅Ρ€Π΅ΠΌ Ρ†Π΅ΠΏΡŒ, ΡΠΎΡΡ‚ΠΎΡΡ‰ΡƒΡŽ: источника Ρ‚ΠΎΠΊΠ°, Π°ΠΌΠΏΠ΅Ρ€ΠΌΠ΅Ρ‚Ρ€Π°, Π²ΠΎΠ»ΡŒΡ‚ΠΌΠ΅Ρ‚Ρ€Π°, ΠΏΡ€ΠΎΠ²ΠΎΠ΄Π½ΠΈΠΊΠΎΠ² сопротивлСниСм 1 Ом, 2 Ом, 4 Ом.

2. Π’ Ρ†Π΅ΠΏΡŒ ΠΏΠΎ ΠΎΡ‡Π΅Ρ€Π΅Π΄ΠΈ Π²ΠΊΠ»ΡŽΡ‡Π°Π΅ΠΌ ΠΏΡ€ΠΎΠ²ΠΎΠ΄Π½ΠΈΠΊΠΈ, ΠΎΠ±Π»Π°Π΄Π°ΡŽΡ‰ΠΈΠ΅ Ρ€Π°Π·Π»ΠΈΡ‡Π½Ρ‹ΠΌ сопротивлСниСм. НапряТСниС Π½Π° ΠΊΠΎΠ½Ρ†Π°Ρ… ΠΏΡ€ΠΎΠ²ΠΎΠ΄Π½ΠΈΠΊΠ° Π²ΠΎ врСмя ΠΎΠΏΡ‹Ρ‚Π° поддСрТиваСтся  постоянным. Π‘ΠΈΠ»Ρƒ Ρ‚ΠΎΠΊΠ° Π² Ρ†Π΅ΠΏΠΈ измСряСм Π°ΠΌΠΏΠ΅Ρ€ΠΌΠ΅Ρ‚Ρ€ΠΎΠΌ.

Π Π΅Π·ΡƒΠ»ΡŒΡ‚Π°Ρ‚Ρ‹ ΠΈΠ·ΠΌΠ΅Ρ€Π΅Π½ΠΈΠΉ помСстим Π² Ρ‚Π°Π±Π»ΠΈΡ†Ρƒ 2:

Π’Π°Π±Π»ΠΈΡ†Π° 2

β„–Β ΠΎΠΏΡ‹Ρ‚Π° НапряТСниС Π½Π° ΠΊΠΎΠ½Ρ†Π°Ρ… ΠΏΡ€ΠΎΠ²ΠΎΠ΄Π½ΠΈΠΊΠ°, Π’ Π‘ΠΎΠΏΡ€ΠΎΡ‚ΠΈΠ²Π»Π΅Π½ΠΈΠ΅ ΠΏΡ€ΠΎΠ²ΠΎΠ΄Π½ΠΈΠΊΠ°, Ом Π‘ΠΈΠ»Π° Ρ‚ΠΎΠΊΠ° Π² Ρ†Π΅ΠΏΠΈ, А

1

2

1

2

2

2

2

1

3

2

4

0,5

Π£Ρ‡ΠΈΡ‚Π΅Π»ΡŒ: Π§Ρ‚ΠΎ Π²Ρ‹ наблюдали?

Π£Ρ‡Π΅Π½ΠΈΠΊΠΈ: Π‘ ΡƒΠ²Π΅Π»ΠΈΡ‡Π΅Π½ΠΈΠ΅ΠΌ сопротивлСния сила Ρ‚ΠΎΠΊΠ° ΡƒΠΌΠ΅Π½ΡŒΡˆΠ°Π΅Ρ‚ΡΡ.

Π£Ρ‡ΠΈΡ‚Π΅Π»ΡŒ: Какой Π²Ρ‹Π²ΠΎΠ΄ ΠΌΠΎΠΆΠ½ΠΎ ΡΠ΄Π΅Π»Π°Ρ‚ΡŒ ΠΈΠ· этого?

Π£Ρ‡Π΅Π½ΠΈΠΊΠΈ: Π‘ΠΈΠ»Π° Ρ‚ΠΎΠΊΠ° Π² ΠΏΡ€ΠΎΠ²ΠΎΠ΄Π½ΠΈΠΊΠ΅ ΠΎΠ±Ρ€Π°Ρ‚Π½ΠΎ ΠΏΡ€ΠΎΠΏΠΎΡ€Ρ†ΠΈΠΎΠ½Π°Π»ΡŒΠ½Π° ΡΠΎΠΏΡ€ΠΎΡ‚ΠΈΠ²Π»Π΅Π½ΠΈΡŽ ΠΏΡ€ΠΎΠ²ΠΎΠ΄Π½ΠΈΠΊΠ°.

Π£Ρ‡ΠΈΡ‚Π΅Π»ΡŒ: Выясним, Β ΠΊΠ°ΠΊ зависит сила Ρ‚ΠΎΠΊΠ° ΠΎΡ‚ напряТСния, Ссли ΠΏΡ€ΠΈ этом сопротивлСниС Π½Π΅ мСняСтся. ΠžΠ±Ρ€Π°Ρ‚ΠΈΠΌΡΡ ΠΊ ΠΎΠΏΡ‹Ρ‚Ρƒ:

1. Π‘ΠΎΠ±Π΅Ρ€Π΅ΠΌ Ρ†Π΅ΠΏΡŒ, ΡΠΎΡΡ‚ΠΎΡΡ‰ΡƒΡŽ ΠΈΠ· источника Ρ‚ΠΎΠΊΠ° - аккумулятора, Π°ΠΌΠΏΠ΅Ρ€ΠΌΠ΅Ρ‚Ρ€Π°, спирали ΠΈΠ· Π½ΠΈΠΊΠ΅Π»ΠΈΠ½ΠΎΠ²ΠΎΠΉ ΠΏΡ€ΠΎΠ²ΠΎΠ»ΠΎΠΊΠΈ (ΠΏΡ€ΠΎΠ²ΠΎΠ΄Π½ΠΈΠΊΠ°), ΠΊΠ»ΡŽΡ‡Π° ΠΈ ΠΏΠ°Ρ€Π°Π»Π»Π΅Π»ΡŒΠ½ΠΎ присоСдинСнного ΠΊ спирали Π²ΠΎΠ»ΡŒΡ‚ΠΌΠ΅Ρ‚Ρ€Π°.

2. ΠŸΡ€ΠΈΡΠΎΠ΅Π΄ΠΈΠ½ΡΠ΅ΠΌ ΠΊ ΠΏΠ΅Ρ€Π²ΠΎΠΌΡƒ аккумулятору Π²Ρ‚ΠΎΡ€ΠΎΠΉ, Π·Π°Ρ‚Π΅ΠΌ Ρ‚Ρ€Π΅Ρ‚ΠΈΠΉ Ρ‚Π°ΠΊΠΎΠΉ ΠΆΠ΅, Π·Π°ΠΌΡ‹ΠΊΠ°Π΅ΠΌ Ρ†Π΅ΠΏΡŒ ΠΈ ΠΎΡ‚ΠΌΠ΅Ρ‡Π°Π΅ΠΌ ΠΏΠΎΠΊΠ°Π·Π°Π½ΠΈΠ΅ ΠΏΡ€ΠΈΠ±ΠΎΡ€ΠΎΠ² ΠΏΡ€ΠΈ ΠΊΠ°ΠΆΠ΄ΠΎΠΌ ΠΏΠΎΠ΄ΠΊΠ»ΡŽΡ‡Π΅Π½ΠΈΠΈΒ  Π΄ΠΎΠΏΠΎΠ»Π½ΠΈΡ‚Π΅Π»ΡŒΠ½ΠΎΠ³ΠΎ аккумулятора.

Π Π΅Π·ΡƒΠ»ΡŒΡ‚Π°Ρ‚Ρ‹ ΠΈΠ·ΠΌΠ΅Ρ€Π΅Π½ΠΈΠΉ помСстим Π² Ρ‚Π°Π±Π»ΠΈΡ†Ρƒ 3:

Π’Π°Π±Π»ΠΈΡ†Π° 3

β„–Β ΠΎΠΏΡ‹Ρ‚Π°

НапряТСниС Π½Π° ΠΊΠΎΠ½Ρ†Π°Ρ… ΠΏΡ€ΠΎΠ²ΠΎΠ΄Π½ΠΈΠΊΠ°, Π’

Π‘ΠΎΠΏΡ€ΠΎΡ‚ΠΈΠ²Π»Π΅Π½ΠΈΠ΅ ΠΏΡ€ΠΎΠ²ΠΎΠ΄Π½ΠΈΠΊΠ°, Ом Π‘ΠΈΠ»Π° Ρ‚ΠΎΠΊΠ° Π² Ρ†Π΅ΠΏΠΈ, А

1

2

4

0,5

2

4

4

1

3

6

4

1,5

Π£Ρ‡ΠΈΡ‚Π΅Π»ΡŒ: Π§Ρ‚ΠΎ Π²Ρ‹ наблюдали?

Π£Ρ‡Π΅Π½ΠΈΠΊΠΈ:Β  ΠŸΡ€ΠΈ ΡƒΠ²Π΅Π»ΠΈΡ‡Π΅Π½ΠΈΠΈ напряТСния Π² Π΄Π²Π° Ρ€Π°Π·Π°, сила Ρ‚ΠΎΠΊΠ° ΡƒΠ²Π΅Π»ΠΈΡ‡ΠΈΠ»Π°ΡΡŒ Π²Π΄Π²ΠΎΠ΅. ΠŸΡ€ΠΈ Ρ‚Ρ€Π΅Ρ… аккумуляторах напряТСниС Π½Π° спирали ΡƒΠ²Π΅Π»ΠΈΡ‡ΠΈΠ»ΠΎΡΡŒ Π²Ρ‚Ρ€ΠΎΠ΅,Β  Π²ΠΎ ΡΡ‚ΠΎΠ»ΡŒΠΊΠΎ ΠΆΠ΅ Ρ€Π°Π· ΡƒΠ²Π΅Π»ΠΈΡ‡ΠΈΠ»Π°ΡΡŒ сила Ρ‚ΠΎΠΊΠ°.

Π£Ρ‡ΠΈΡ‚Π΅Π»ΡŒ: Какой Π²Ρ‹Π²ΠΎΠ΄ ΠΈΠ· этого ΠΌΠΎΠΆΠ½ΠΎ ΡΠ΄Π΅Π»Π°Ρ‚ΡŒ?

Π£Ρ‡Π΅Π½ΠΈΠΊΠΈ: Π‘ΠΈΠ»Π° Ρ‚ΠΎΠΊΠ° Π² ΠΏΡ€ΠΎΠ²ΠΎΠ΄Π½ΠΈΠΊΠ΅ прямо ΠΏΡ€ΠΎΠΏΠΎΡ€Ρ†ΠΈΠΎΠ½Π°Π»ΡŒΠ½Π° Π½Π°ΠΏΡ€ΡΠΆΠ΅Π½ΠΈΡŽ Π½Π° ΠΊΠΎΠ½Ρ†Π°Ρ… ΠΏΡ€ΠΎΠ²ΠΎΠ΄Π½ΠΈΠΊΠ°.

Π£Ρ‡ΠΈΡ‚Π΅Π»ΡŒ: Π˜ΡΠΏΠΎΠ»ΡŒΠ·ΡƒΡ Ρ€Π΅Π·ΡƒΠ»ΡŒΡ‚Π°Ρ‚Ρ‹ ΠΎΠΏΡ‹Ρ‚ΠΎΠ², ΠΈ Π²Ρ‹Π²ΠΎΠ΄Ρ‹ сдСланныС ΠΈΠ· Π½ΠΈΡ…,  установим Π·Π°Π²ΠΈΡΠΈΠΌΠΎΡΡ‚ΡŒΒ  силы Ρ‚ΠΎΠΊΠ°, напряТСния ΠΈ сопротивлСния.

Вакая запись носит Π½Π°Π·Π²Π°Π½ΠΈΠ΅ Π·Π°ΠΊΠΎΠ½Π° Ома для участка Ρ†Π΅ΠΏΠΈ.

Π‘ΠΈΠ»Π° Ρ‚ΠΎΠΊΠ° Π² участкС Ρ†Π΅ΠΏΠΈ прямо ΠΏΡ€ΠΎΠΏΠΎΡ€Ρ†ΠΈΠΎΠ½Π°Π»ΡŒΠ½Π° Π½Π°ΠΏΡ€ΡΠΆΠ΅Π½ΠΈΡŽ Π½Π° ΠΊΠΎΠ½Ρ†Π°Ρ… этого участка ΠΈ ΠΎΠ±Ρ€Π°Ρ‚Π½ΠΎ ΠΏΡ€ΠΎΠΏΠΎΡ€Ρ†ΠΈΠΎΠ½Π°Π»ΡŒΠ½Π° Π΅Π³ΠΎ ΡΠΎΠΏΡ€ΠΎΡ‚ΠΈΠ²Π»Π΅Π½ΠΈΡŽ.

Π£Ρ‡ΠΈΡ‚Π΅Π»ΡŒ:

Π˜ΡΡ‚ΠΎΡ€ΠΈΡ‡Π΅ΡΠΊΠ°Ρ справка: Π­Ρ‚ΠΎΡ‚ Π·Π°ΠΊΠΎΠ½ ΠΎΡ‚ΠΊΡ€Ρ‹Π» Π½Π΅ΠΌΠ΅Ρ†ΠΊΠΈΠΉ Ρ„ΠΈΠ·ΠΈΠΊ Π“Π΅ΠΎΡ€Π³ Ом Π² Β 1827 Π³ΠΎΠ΄Ρƒ. ЀранцузскиС школьники ΠΈΠ·ΡƒΡ‡Π°ΡŽΡ‚ этот Π·Π°ΠΊΠΎΠ½ ΠΏΠΎΠ΄ ΠΈΠΌΠ΅Π½Π΅ΠΌ ΠŸΡƒΠΉΠ΅ — французского Ρ„ΠΈΠ·ΠΈΠΊΠ°, ΡƒΡΡ‚Π°Π½ΠΎΠ²ΠΈΠ²ΡˆΠ΅Π³ΠΎ этот ΠΆΠ΅ Π·Π°ΠΊΠΎΠ½, Π½ΠΎ спустя 10 Π»Π΅Ρ‚.
Π£Ρ‡ΠΈΡ‚Π΅Π»ΡŒ: Для Ρ‚ΠΎΠ³ΠΎ, Ρ‡Ρ‚ΠΎΠ±Ρ‹ Π²Π°ΠΌ Π±Ρ‹Π»ΠΎ Π»Π΅Π³Ρ‡Π΅ Π·Π°ΠΏΠΎΠΌΠ½ΠΈΡ‚ΡŒ Ρ„ΠΎΡ€ΠΌΡƒΠ»Ρƒ Π·Π°ΠΊΠΎΠ½Π° Ома ΠΌΠΎΠΆΠ½ΠΎ Π²ΠΎΡΠΏΠΎΠ»ΡŒΠ·ΠΎΠ²Π°Ρ‚ΡŒΡΡ ΡΠ»Π΅Π΄ΡƒΡŽΡ‰ΠΈΠΌ способом Π΅Ρ‘ записи. (рис. 3) Β 

Β 

Рис. 3

Β Β Β Β Β Β Β Β Β Β  ;Β Β Β Β Β  Β Β Β Β Β Β Β Β Β Β Β Β ;Β Β Β Β Β Β Β Β Β Β  Β Β Β Β Β Β Β Β U = I * R

ЀизичСская ΠΏΠ°ΡƒΠ·Π°

Π£Ρ‡ΠΈΡ‚Π΅Π»ΡŒ: ΠŸΡ€Π΅ΠΆΠ΄Π΅ Ρ‡Π΅ΠΌ ΠΏΡ€ΠΈΡΡ‚ΡƒΠΏΠΈΡ‚ΡŒ ΠΊ Ρ€Π΅ΡˆΠ΅Π½ΠΈΡŽ Π·Π°Π΄Π°Ρ‡ ΠΏΡ€ΠΎΠ²Π΅Π΄Π΅ΠΌ Ρ„ΠΈΠ·ΠΈΡ‡Π΅ΡΠΊΡƒΡŽ ΠΏΠ°ΡƒΠ·Ρƒ. ΠŸΡ€Π΅Π΄ΡΡ‚Π°Π²ΠΈΠΌ,Β  Ρ‡Ρ‚ΠΎ ΠΌΡ‹ с Π²Π°ΠΌΠΈ пассаТиры автобуса…

  • автобус Ρ€Π΅Π·ΠΊΠΎ трогаСтся с мСста – ΡƒΡ‡Π΅Π½ΠΈΠΊΠΈ Π΄ΠΎΠ»ΠΆΠ½Ρ‹ Π½Π°ΠΊΠ»ΠΎΠ½ΠΈΡ‚ΡŒΡΡ Π½Π°Π·Π°Π΄.
  • автобус Ρ‚ΠΎΡ€ΠΌΠΎΠ·ΠΈΡ‚ – отклонится Π²ΠΏΠ΅Ρ€Π΅Π΄.
  • автобус ΠΏΠΎΠ²ΠΎΡ€Π°Ρ‡ΠΈΠ²Π°Π΅Ρ‚ Π½Π°ΠΏΡ€Π°Π²ΠΎ – Π½Π°ΠΊΠ»ΠΎΠ½ΡΡŽΡ‚ΡΡ Π²Π»Π΅Π²ΠΎ.
  • автобус ΠΏΠΎΠ²ΠΎΡ€Π°Ρ‡ΠΈΠ²Π°Π΅Ρ‚ Π½Π°Π»Π΅Π²ΠΎ – Π½Π°ΠΊΠ»ΠΎΠ½ΡΡŽΡ‚ΡΡ Π²ΠΏΡ€Π°Π²ΠΎ.

Π£Ρ‡ΠΈΡ‚Π΅Π»ΡŒ: КакоС физичСскоС явлСниС Π²Ρ‹ ΠΈΠ·ΠΎΠ±Ρ€Π°ΠΆΠ°Π»ΠΈ?

Π£Ρ‡Π΅Π½ΠΈΠΊΠΈ: Π˜Π½Π΅Ρ€Ρ†ΠΈΡ – явлСниС сохранСния скорости Ρ‚Π΅Π»Π°, ΠΊΠΎΠ³Π΄Π° Π½Π° это Ρ‚Π΅Π»ΠΎ Π½Π΅ Π΄Π΅ΠΉΡΡ‚Π²ΡƒΡŽΡ‚ внСшниС силы.

4. Π—Π°ΠΊΡ€Π΅ΠΏΠ»Π΅Π½ΠΈΠ΅ ΡƒΠΌΠ΅Π½ΠΈΠΉ ΠΈ Π½Π°Π²Ρ‹ΠΊΠΎΠ²

Π˜ΡΠΏΠΎΠ»ΡŒΠ·ΡƒΡ Π·Π°ΠΊΠΎΠ½ Ома для участка Ρ†Π΅ΠΏΠΈ, Ρ€Π΅ΡˆΠΈΠΌ Π·Π°Π΄Π°Ρ‡Ρƒ.

Π—Π°Π΄Π°Ρ‡Π° 1.

НапряТСниС Π½Π° Π·Π°ΠΆΠΈΠΌΠ°Ρ… элСктричСского ΡƒΡ‚ΡŽΠ³Π° 220 Π’, сопротивлСниС Π½Π°Π³Ρ€Π΅Π²Π°Ρ‚Π΅Π»ΡŒΠ½ΠΎΠ³ΠΎ элСмСнта ΡƒΡ‚ΡŽΠ³Π° 50 Ом. Π§Π΅ΠΌΡƒ Ρ€Π°Π²Π½Π° сила Ρ‚ΠΎΠΊΠ° Π² Π½Π°Π³Ρ€Π΅Π²Π°Ρ‚Π΅Π»ΡŒΠ½ΠΎΠΌ элСмСнтС?

Π”Π°Π½ΠΎ:

U = 220 Π’
R = 50 Ом

I – ?

РСшСниС:

ΠžΡ‚Π²Π΅Ρ‚: 4,4 А

Π—Π°Π΄Π°Ρ‡Π° 2.

На рис.4 Β ΠΈΠ·ΠΎΠ±Ρ€Π°ΠΆΠ΅Π½ Π³Ρ€Π°Ρ„ΠΈΠΊ зависимости силы Ρ‚ΠΎΠΊΠ° ΠΎΡ‚ напряТСния для Π΄Π²ΡƒΡ… ΠΏΡ€ΠΎΠ²ΠΎΠ΄Π½ΠΈΠΊΠΎΠ² А ΠΈ Π’. ΠžΠΏΡ€Π΅Π΄Π΅Π»ΠΈΡ‚Π΅ сопротивлСниС ΠΊΠ°ΠΆΠ΄ΠΎΠ³ΠΎ ΠΈΠ· ΠΏΡ€ΠΎΠ²ΠΎΠ΄Π½ΠΈΠΊΠΎΠ². Какой ΠΈΠ· этих ΠΏΡ€ΠΎΠ²ΠΎΠ΄Π½ΠΈΠΊΠΎΠ² ΠΎΠ±Π»Π°Π΄Π°Π΅Ρ‚ большим сопротивлСниСм?

Β 

Рис. 4

Π£Ρ‡ΠΈΡ‚Π΅Π»ΡŒ:Β  РСшаСм эту Π·Π°Π΄Π°Ρ‡Ρƒ ΠΏΠΎ Π²Π°Ρ€ΠΈΠ°Π½Ρ‚Π°ΠΌ. Π’Π°Ρ€ΠΈΠ°Π½Ρ‚ 1 – Π½Π°Ρ…ΠΎΠ΄ΠΈΡ‚ сопротивлСния ΠΏΡ€ΠΎΠ²ΠΎΠ΄Π½ΠΈΠΊΠ° А. Π’Π°Ρ€ΠΈΠ°Π½Ρ‚ 2 – Π½Π°Ρ…ΠΎΠ΄ΠΈΡ‚ сопротивлСниС ΠΏΡ€ΠΎΠ²ΠΎΠ΄Π½ΠΈΠΊΠ° Π’.

Π’Π°Ρ€ΠΈΠ°Π½Ρ‚ 1.

Π”Π°Π½ΠΎ:

U = 6B
I = 3A

RA –  ?

РСшСниС:

;

ΠžΡ‚Π²Π΅Ρ‚: 2 Ом

Π’Π°Ρ€ΠΈΠ°Π½Ρ‚ 2.

Π”Π°Π½ΠΎ:

U = 4B
I = 1A

RB – ?

РСшСниС:

;

;

ΠžΡ‚Π²Π΅Ρ‚: 4 Ом.

Π£Ρ‡Π΅Π½ΠΈΠΊΠΈ:Β  2 Ом < 4 Ом , Π·Π½Π°Ρ‡ΠΈΡ‚Β Β  RA <Β  RB,Β  сопротивлСниС ΠΏΡ€ΠΎΠ²ΠΎΠ΄Π½ΠΈΠΊΠ° А мСньшС, Ρ‡Π΅ΠΌ сопротивлСниС ΠΏΡ€ΠΎΠ²ΠΎΠ΄Π½ΠΈΠΊΠ° Π’. Β Β 

5. Π”ΠΎΠΌΠ°ΡˆΠ½Π΅Π΅ Π·Π°Π΄Π°Π½ΠΈΠ΅: ΠΏ.42–44, ΡƒΠΏΡ€.19 β„– 3,4

6. ПодвСдСниС ΠΈΡ‚ΠΎΠ³ΠΎΠ² ΡƒΡ€ΠΎΠΊΠ°, ΠΎΡ†Π΅Π½ΠΊΠΈ Ρ€Π°Π±ΠΎΡ‚Ρ‹ учащихся

Π£Ρ‡ΠΈΡ‚Π΅Π»ΡŒ: Β ΠœΠΎΠ»ΠΎΠ΄Ρ†Ρ‹ рСбята, ΠΎΡ‡Π΅Π½ΡŒ Ρ…ΠΎΡ€ΠΎΡˆΠΎ ΠΏΠΎΡ‚Ρ€ΡƒΠ΄ΠΈΠ»ΠΈΡΡŒ, Ρ…ΠΎΡ€ΠΎΡˆΠΎ Ρ€Π΅ΡˆΠ°Π»ΠΈ Π·Π°Π΄Π°Ρ‡ΠΈ, Π²Π½ΠΈΠΌΠ°Ρ‚Π΅Π»ΡŒΠ½ΠΎ ΡΠ»ΡƒΡˆΠ°Π»ΠΈ ΠΈ ΠΏΡ€ΠΈΠ½ΠΈΠΌΠ°Π»ΠΈ Π°ΠΊΡ‚ΠΈΠ²Π½ΠΎΠ΅ участиС Π² Π²Ρ‹Π²ΠΎΠ΄Π΅ Π·Π°ΠΊΠΎΠ½Π° Ома. Как для ΠΊΠ°ΠΆΠ΄ΠΎΠ³ΠΎ ΠΏΡ€ΠΎΡˆΠ΅Π» ΡƒΡ€ΠΎΠΊ, ΠΌΡ‹ сСйчас ΡƒΠ²ΠΈΠ΄ΠΈΠΌ ΠΏΠΎ Ρ€Π΅Π·ΡƒΠ»ΡŒΡ‚Π°Ρ‚Π°ΠΌ самодиагностики.

Бамодиагностика (учащиСся ΠΏΠΎΠ΄Π½ΠΈΠΌΠ°ΡŽΡ‚ ΠΎΠ΄Π½Ρƒ ΠΈΠ· Ρ‚Ρ€Π΅Ρ… ΠΊΠ°Ρ€Ρ‚ΠΎΡ‡Π΅ΠΊ, Π»Π΅ΠΆΠ°Ρ‰ΠΈΡ… Ρƒ Π½ΠΈΡ… Π½Π° ΠΏΠ°Ρ€Ρ‚Π΅).

  • ΠšΡ€Π°ΡΠ½Π°Ρ ΠΊΠ°Ρ€Ρ‚ΠΎΡ‡ΠΊΠ° – ΡƒΠ΄ΠΎΠ²Π»Π΅Ρ‚Π²ΠΎΡ€Π΅Π½ ΡƒΡ€ΠΎΠΊΠΎΠΌ, ΡƒΡ€ΠΎΠΊ ΠΏΠΎΠ»Π΅Π·Π΅Π½ для мСня, я Ρ€Π°Π±ΠΎΡ‚Π°Π» ΠΈ ΠΏΠΎΠ»ΡƒΡ‡ΠΈΠ» Π·Π°ΡΠ»ΡƒΠΆΠ΅Π½Π½ΡƒΡŽ ΠΎΡ†Π΅Π½ΠΊΡƒ; я ΠΏΠΎΠ½ΠΈΠΌΠ°Π» всС, ΠΎ Ρ‡Π΅ΠΌ Π³ΠΎΠ²ΠΎΡ€ΠΈΠ»ΠΎΡΡŒ.
  • ЖСлтая ΠΊΠ°Ρ€Ρ‚ΠΎΡ‡ΠΊΠ° – ΡƒΡ€ΠΎΠΊ Π±Ρ‹Π» интСрСсСн, я ΠΎΡ‚Π²Π΅Ρ‡Π°Π» с мСста, сумСл Π²Ρ‹ΠΏΠΎΠ»Π½ΠΈΡ‚ΡŒ ряд Π·Π°Π΄Π°Π½ΠΈΠΉ. МнС Π½Π° ΡƒΡ€ΠΎΠΊΠ΅ достаточно ΠΊΠΎΠΌΡ„ΠΎΡ€Ρ‚Π½ΠΎ.
  • ЗСлСная ΠΊΠ°Ρ€Ρ‚ΠΎΡ‡ΠΊΠ° – ΠΏΠΎΠ»ΡŒΠ·Ρ‹ ΠΎΡ‚ ΡƒΡ€ΠΎΠΊΠ° я ΠΏΠΎΠ»ΡƒΡ‡ΠΈΠ» ΠΌΠ°Π»ΠΎ, я Π½Π΅ ΠΎΡ‡Π΅Π½ΡŒ ΠΏΠΎΠ½ΠΈΠΌΠ°Π», ΠΎ Ρ‡Π΅ΠΌ ΠΈΠ΄Π΅Ρ‚ Ρ€Π΅Ρ‡ΡŒ, ΠΊ ΠΎΡ‚Π²Π΅Ρ‚Ρƒ Π½Π° ΡƒΡ€ΠΎΠΊΠ΅ я Π±Ρ‹Π» Π½Π΅ Π³ΠΎΡ‚ΠΎΠ².

Π—Π°Π΄Π°Ρ‡ΠΈ ΠΏΠΎ Ρ‚Π΅ΠΌΠ°ΠΌ «Π‘ΠΈΠ»Π° Ρ‚ΠΎΠΊΠ°. НапряТСниС. Π‘ΠΎΠΏΡ€ΠΎΡ‚ΠΈΠ²Π»Π΅Π½ΠΈΠ΅.» 8 класс

Π‘ΠΈΠ»Π° Π’ΠΎΠΊΠ°. НапряТСниС, Π Π°Π±ΠΎΡ‚Π° Ρ‚ΠΎΠΊΠ°. 8 класс

  1. Какой заряд ΠΏΡ€ΠΎΡ…ΠΎΠ΄ΠΈΡ‚ Ρ‡Π΅Ρ€Π΅Π· ΠΏΠΎΠΏΠ΅Ρ€Π΅Ρ‡Π½ΠΎΠ΅ сСчСниС ΠΏΡ€ΠΎΠ²ΠΎΠ΄Π° Π·Π° 50 с, Ссли сила Ρ‚ΠΎΠΊΠ° Π² Π½Π΅ΠΌ 2,5 А?

  2. Π—Π° ΠΊΠ°ΠΊΠΎΠ΅ врСмя Ρ‡Π΅Ρ€Π΅Π· ΠΏΠΎΠΏΠ΅Ρ€Π΅Ρ‡Π½ΠΎΠ΅ сСчСниС ΠΏΡ€ΠΎΠ²ΠΎΠ΄Π° ΠΏΡ€ΠΎΠΉΠ΄Π΅Ρ‚ 72 Кл элСктричСства ΠΏΡ€ΠΈ силС Ρ‚ΠΎΠΊΠ° Π² Π½Π΅ΠΌ 1,2 А?

  3. Π’ Ρ†Π΅ΠΏΠΈ с напряТСниСм 100 Π’ ΠΏΡ€ΠΎΡ…ΠΎΠ΄ΠΈΡ‚ Ρ‚ΠΎΠΊ силой 1 А. Π½Π°ΠΉΠ΄ΠΈΡ‚Π΅ Ρ€Π°Π±ΠΎΡ‚Ρƒ, ΡΠΎΠ²Π΅Ρ€ΡˆΠ°Π΅ΠΌΡƒΡŽ элСктричСскими силами ΠΏΠΎ ΠΏΠ΅Ρ€Π΅ΠΌΠ΅Ρ‰Π΅Π½ΠΈΡŽ заряда, Π·Π° 1 ΠΌΠΈΠ½.

  4. По ΠΏΡ€ΠΎΠ²ΠΎΠ΄Π½ΠΈΠΊΡƒ, ΠΊ ΠΊΠΎΠ½Ρ†Π°ΠΌ ΠΊΠΎΡ‚ΠΎΡ€ΠΎΠ³ΠΎ ΠΏΡ€ΠΈΠ»ΠΎΠΆΠ΅Π½ΠΎ напряТСниС 5 Π’, ΠΏΡ€ΠΎΡˆΠ»ΠΎ 100 Кл элСктричСства. ΠžΠΏΡ€Π΅Π΄Π΅Π»ΠΈΡ‚Π΅ Ρ€Π°Π±ΠΎΡ‚Ρƒ Ρ‚ΠΎΠΊΠ°

  5. ЭлСктричСская Π»Π°ΠΌΠΏΠΎΡ‡ΠΊΠ° Π²ΠΊΠ»ΡŽΡ‡Π΅Π½Π° Π² Ρ†Π΅ΠΏΡŒ напряТСниСм 10 Π’. Π’ΠΎΠΊΠΎΠΌ Π±Ρ‹Π»Π° ΡΠΎΠ²Π΅Ρ€ΡˆΠ΅Π½Π° Ρ€Π°Π±ΠΎΡ‚Π° 150 Π”ΠΆ. КакоС количСство элСктричСства ΠΏΡ€ΠΎΡˆΠ»ΠΎ Ρ‡Π΅Ρ€Π΅Π· Π½ΠΈΡ‚ΡŒ Π½Π°ΠΊΠ°Π»Π° Π»Π°ΠΌΠΏΠΎΡ‡ΠΊΠΈ?

  6. ΠšΠ°ΠΊΡƒΡŽ Ρ€Π°Π±ΠΎΡ‚Ρƒ ΡΠΎΠ²Π΅Ρ€ΡˆΠΈΡ‚ Ρ‚ΠΎΠΊ силой 3 А Π·Π° 10 ΠΌΠΈΠ½ ΠΏΡ€ΠΈ напряТСнии Π² Ρ†Π΅ΠΏΠΈ 15 Π’?

  7. Π’ Π»Π°ΠΌΠΏΠΎΡ‡ΠΊΠ΅ ΠΊΠ°Ρ€ΠΌΠ°Π½Π½ΠΎΠ³ΠΎ фонаря сила Ρ‚ΠΎΠΊΠ° Ρ€Π°Π²Π½Π° 0,2 А. ВычислитС ΡΠ»Π΅ΠΊΡ‚Ρ€ΠΈΡ‡Π΅ΡΠΊΡƒΡŽ ΡΠ½Π΅Ρ€Π³ΠΈΡŽ, ΠΏΠΎΠ»ΡƒΡ‡Π°Π΅ΠΌΡƒΡŽ Π»Π°ΠΌΠΏΠΎΡ‡ΠΊΠΎΠΉ Π·Π° ΠΊΠ°ΠΆΠ΄Ρ‹Π΅ 2 ΠΌΠΈΠ½, Ссли напряТСниС Π½Π° Π»Π°ΠΌΠΏΠΎΡ‡ΠΊΠ΅ составляСт 3,6 Π’.

  8. НачСртитС схСму элСктричСской Ρ†Π΅ΠΏΠΈ, состоящСй ΠΈΠ· источника Ρ‚ΠΎΠΊΠ° ΠΈ Π΄Π²ΡƒΡ… ΠΏΠΎΡΠ»Π΅Π΄ΠΎΠ²Π°Ρ‚Π΅Π»ΡŒΠ½ΠΎ соСдинСнных Π»Π°ΠΌΠΏ. Π˜Π·ΠΎΠ±Ρ€Π°Π·ΠΈΡ‚Π΅, ΠΊΠ°ΠΊ Π½Π°Π΄ΠΎ ΠΏΠΎΠ΄ΠΊΠ»ΡŽΡ‡ΠΈΡ‚ΡŒ Π°ΠΌΠΏΠ΅Ρ€ΠΌΠ΅Ρ‚Ρ€ ΠΈ Π²ΠΎΠ»ΡŒΡ‚ΠΌΠ΅Ρ‚Ρ€ для измСрСния силы Ρ‚ΠΎΠΊΠ° Π² ΠΊΠ°ΠΆΠ΄ΠΎΠΉ Π»Π°ΠΌΠΏΠ΅ ΠΈ напряТСния Π½Π° ΠΊΠ°ΠΆΠ΄ΠΎΠΉ Π»Π°ΠΌΠΏΠ΅.

Π”.Π—._________________________________________

  1. По спирали элСктролампы ΠΏΡ€ΠΎΡ…ΠΎΠ΄ΠΈΡ‚ 540 Кл элСктричСства Π·Π° ΠΊΠ°ΠΆΠ΄Ρ‹Π΅ 5 ΠΌΠΈΠ½. Π§Π΅ΠΌΡƒ Ρ€Π°Π²Π½Π° сила Ρ‚ΠΎΠΊΠ° Π² Π»Π°ΠΌΠΏΠ΅?

  2. Π§Π΅ΠΌΡƒ Ρ€Π°Π²Π½ΠΎ напряТСниС Π½Π° участкС Ρ†Π΅ΠΏΠΈ, Π½Π° ΠΊΠΎΡ‚ΠΎΡ€ΠΎΠΌ ΡΠΎΠ²Π΅Ρ€ΡˆΠ΅Π½Π° Ρ€Π°Π±ΠΎΡ‚Π° 500 Π”ΠΆ ΠΏΡ€ΠΈ ΠΏΡ€ΠΎΡ…ΠΎΠΆΠ΄Π΅Π½ΠΈΠΈ 25 Кл элСктричСства?

  3. ВычислитС Ρ€Π°Π±ΠΎΡ‚Ρƒ, которая ΡΠΎΠ²Π΅Ρ€ΡˆΠ°Π΅Ρ‚ΡΡ ΠΏΡ€ΠΈ ΠΏΡ€ΠΎΡ…ΠΎΠΆΠ΄Π΅Π½ΠΈΠΈ Ρ‡Π΅Ρ€Π΅Π· ΡΠΏΠΈΡ€Π°Π»ΡŒ элСктроплитки 15 Кл элСктричСства, Ссли ΠΎΠ½Π° Π²ΠΊΠ»ΡŽΡ‡Π΅Π½Π° Π² ΡΠ΅Ρ‚ΡŒ напряТСниСм 220 Π’?

  4. ΠŸΡ€ΠΈ элСктросваркС сила Ρ‚ΠΎΠΊΠ° достигаСт 200 А. Какой элСктричСский заряд ΠΏΡ€ΠΎΡ…ΠΎΠ΄ΠΈΡ‚ Ρ‡Π΅Ρ€Π΅Π· ΠΏΠΎΠΏΠ΅Ρ€Π΅Ρ‡Π½ΠΎΠ΅ сСчСниС элСктрода Π·Π° 1 ΠΌΠΈΠ½?

  5. ΠžΠΏΡ€Π΅Π΄Π΅Π»ΠΈΡ‚Π΅ напряТСниС Π½Π° участкС Ρ†Π΅ΠΏΠΈ, Ссли ΠΏΡ€ΠΈ ΠΏΡ€ΠΎΡ…ΠΎΠΆΠ΄Π΅Π½ΠΈΠΈ ΠΏΠΎ Π½Π΅ΠΌΡƒ заряда Π² 15 Кл Ρ‚ΠΎΠΊΠΎΠΌ Π±Ρ‹Π»Π° ΡΠΎΠ²Π΅Ρ€ΡˆΠ΅Π½Π° Ρ€Π°Π±ΠΎΡ‚Π° 9 ΠΊΠ”ΠΆ.

Π‘ΠΎΠΏΡ€ΠΎΡ‚ΠΈΠ²Π»Π΅Π½ΠΈΠ΅. 8 класс

  1. Π”Π»ΠΈΠ½Π° алюминиСвого ΠΏΡ€ΠΎΠ²ΠΎΠ΄Π° 500 ΠΌ, ΠΏΠ»ΠΎΡ‰Π°Π΄ΡŒ Π΅Π³ΠΎ ΠΏΠΎΠΏΠ΅Ρ€Π΅Ρ‡Π½ΠΎΠ³ΠΎ сСчСния 4 ΠΌΠΌ2Β , Π§Π΅ΠΌΡƒ Ρ€Π°Π²Π½ΠΎ сопротивлСниС ΠΏΡ€ΠΎΠ²ΠΎΠ΄Π°?

  2. ΠœΠ΅Π΄Π½Ρ‹ΠΉ ΠΏΡ€ΠΎΠ²ΠΎΠ΄ с ΠΏΠ»ΠΎΡ‰Π°Π΄ΡŒΡŽ ΠΏΠΎΠΏΠ΅Ρ€Π΅Ρ‡Π½ΠΎΠ³ΠΎ сСчСния 0,85Β ΠΌΠΌ2Β ΠΎΠ±Π»Π°Π΄Π°Π΅Ρ‚ сопротивлСниСм 4 Ом. Какова Π΄Π»ΠΈΠ½Π° ΠΏΡ€ΠΎΠ²ΠΎΠ΄Π°?

  3. Π”Π»ΠΈΠ½Π° сСрСбряного ΠΏΡ€ΠΎΠ²ΠΎΠ΄Π° 0,6 ΠΌ, Π° сопротивлСниС 0,015 Ом. ΠžΠΏΡ€Π΅Π΄Π΅Π»ΠΈΡ‚Π΅ ΠΏΠ»ΠΎΡ‰Π°Π΄ΡŒ ΠΏΠΎΠΏΠ΅Ρ€Π΅Ρ‡Π½ΠΎΠ³ΠΎ сСчСния ΠΏΡ€ΠΎΠ²ΠΎΠ΄Π°.

  4. Π‘ΠΏΠΈΡ€Π°Π»ΡŒ элСктричСской ΠΏΠ»ΠΈΡ‚ΠΊΠΈ ΠΈΠ·Π³ΠΎΡ‚ΠΎΠ²Π»Π΅Π½Π° ΠΈΠ· Π½ΠΈΡ…Ρ€ΠΎΠΌΠΎΠ²ΠΎΠΉ ΠΏΡ€ΠΎΠ²ΠΎΠ»ΠΎΠΊΠΈ Π΄Π»ΠΈΠ½ΠΎΠΉ 13,75 ΠΌ ΠΈ ΠΏΠ»ΠΎΡ‰Π°Π΄ΡŒΡŽ ΠΏΠΎΠΏΠ΅Ρ€Π΅Ρ‡Π½ΠΎΠ³ΠΎ сСчСния 0,1 ΠΌΠΌ2. ΠŸΠ»ΠΈΡ‚ΠΊΠ° рассчитана Π½Π° напряТСниС 220 Π’. ΠžΠΏΡ€Π΅Π΄Π΅Π»ΠΈΡ‚Π΅ силу Ρ‚ΠΎΠΊΠ° Π² спирали ΠΏΠ»ΠΈΡ‚ΠΊΠΈ.

  5. Π‘ΠΈΠ»Π° Ρ‚ΠΎΠΊΠ° Π² ΠΆΠ΅Π»Π΅Π·Π½ΠΎΠΌ ΠΏΡ€ΠΎΠ²ΠΎΠ΄Π½ΠΈΠΊΠ΅ Π΄Π»ΠΈΠ½ΠΎΠΉ 150 ΠΌΠΌ ΠΈ ΠΏΠ»ΠΎΡ‰Π°Π΄ΡŒΡŽ ΠΏΠΎΠΏΠ΅Ρ€Π΅Ρ‡Π½ΠΎΠ³ΠΎ сСчСния 0,02 ΠΌΠΌ2 Ρ€Π°Π²Π½Π° 250 мА. Каково напряТСниС Π½Π° ΠΊΠΎΠ½Ρ†Π°Ρ… ΠΏΡ€ΠΎΠ²ΠΎΠ΄Π½ΠΈΠΊΠ°?

  6. ΠžΠΏΡ€Π΅Π΄Π΅Π»ΠΈΡ‚Π΅ силу Ρ‚ΠΎΠΊΠ°, проходящСго Ρ‡Π΅Ρ€Π΅Π· рСостат, ΠΈΠ·Π³ΠΎΡ‚ΠΎΠ²Π»Π΅Π½Π½Ρ‹ΠΉ ΠΈΠ· Π½ΠΈΠΊΠ΅Π»ΠΈΠ½ΠΎΠ²ΠΎΠΉ ΠΏΡ€ΠΎΠ²ΠΎΠ»ΠΎΠΊΠΈ Π΄Π»ΠΈΠ½ΠΎΠΉ 50 ΠΌ ΠΈ ΠΏΠ»ΠΎΡ‰Π°Π΄ΡŒΡŽ ΠΏΠΎΠΏΠ΅Ρ€Π΅Ρ‡Π½ΠΎΠ³ΠΎ сСчСния 1 ΠΌΠΌ2, Ссли напряТСниС Π½Π° Π·Π°ΠΆΠΈΠΌΠ°Ρ… рСостата Ρ€Π°Π²Π½ΠΎ 45 Π’.

Π”.Π—. Π£ΠΏΡ€ 20(ΡƒΡ‡Π΅Π±Π½ΠΈΠΊ)

Π‘ΠΈΠ»Π° Π’ΠΎΠΊΠ°. НапряТСниС, Π Π°Π±ΠΎΡ‚Π° Ρ‚ΠΎΠΊΠ°. 8 класс

  1. Какой заряд ΠΏΡ€ΠΎΡ…ΠΎΠ΄ΠΈΡ‚ Ρ‡Π΅Ρ€Π΅Π· ΠΏΠΎΠΏΠ΅Ρ€Π΅Ρ‡Π½ΠΎΠ΅ сСчСниС ΠΏΡ€ΠΎΠ²ΠΎΠ΄Π° Π·Π° 50 с, Ссли сила Ρ‚ΠΎΠΊΠ° Π² Π½Π΅ΠΌ 2,5 А?

  2. Π—Π° ΠΊΠ°ΠΊΠΎΠ΅ врСмя Ρ‡Π΅Ρ€Π΅Π· ΠΏΠΎΠΏΠ΅Ρ€Π΅Ρ‡Π½ΠΎΠ΅ сСчСниС ΠΏΡ€ΠΎΠ²ΠΎΠ΄Π° ΠΏΡ€ΠΎΠΉΠ΄Π΅Ρ‚ 72 Кл элСктричСства ΠΏΡ€ΠΈ силС Ρ‚ΠΎΠΊΠ° Π² Π½Π΅ΠΌ 1,2 А?

  3. Π’ Ρ†Π΅ΠΏΠΈ с напряТСниСм 100 Π’ ΠΏΡ€ΠΎΡ…ΠΎΠ΄ΠΈΡ‚ Ρ‚ΠΎΠΊ силой 1 А. Π½Π°ΠΉΠ΄ΠΈΡ‚Π΅ Ρ€Π°Π±ΠΎΡ‚Ρƒ, ΡΠΎΠ²Π΅Ρ€ΡˆΠ°Π΅ΠΌΡƒΡŽ элСктричСскими силами ΠΏΠΎ ΠΏΠ΅Ρ€Π΅ΠΌΠ΅Ρ‰Π΅Π½ΠΈΡŽ заряда, Π·Π° 1 ΠΌΠΈΠ½.

  4. По ΠΏΡ€ΠΎΠ²ΠΎΠ΄Π½ΠΈΠΊΡƒ, ΠΊ ΠΊΠΎΠ½Ρ†Π°ΠΌ ΠΊΠΎΡ‚ΠΎΡ€ΠΎΠ³ΠΎ ΠΏΡ€ΠΈΠ»ΠΎΠΆΠ΅Π½ΠΎ напряТСниС 5 Π’, ΠΏΡ€ΠΎΡˆΠ»ΠΎ 100 Кл элСктричСства. ΠžΠΏΡ€Π΅Π΄Π΅Π»ΠΈΡ‚Π΅ Ρ€Π°Π±ΠΎΡ‚Ρƒ Ρ‚ΠΎΠΊΠ°

  5. ЭлСктричСская Π»Π°ΠΌΠΏΠΎΡ‡ΠΊΠ° Π²ΠΊΠ»ΡŽΡ‡Π΅Π½Π° Π² Ρ†Π΅ΠΏΡŒ напряТСниСм 10 Π’. Π’ΠΎΠΊΠΎΠΌ Π±Ρ‹Π»Π° ΡΠΎΠ²Π΅Ρ€ΡˆΠ΅Π½Π° Ρ€Π°Π±ΠΎΡ‚Π° 150 Π”ΠΆ. КакоС количСство элСктричСства ΠΏΡ€ΠΎΡˆΠ»ΠΎ Ρ‡Π΅Ρ€Π΅Π· Π½ΠΈΡ‚ΡŒ Π½Π°ΠΊΠ°Π»Π° Π»Π°ΠΌΠΏΠΎΡ‡ΠΊΠΈ?

  6. ΠšΠ°ΠΊΡƒΡŽ Ρ€Π°Π±ΠΎΡ‚Ρƒ ΡΠΎΠ²Π΅Ρ€ΡˆΠΈΡ‚ Ρ‚ΠΎΠΊ силой 3 А Π·Π° 10 ΠΌΠΈΠ½ ΠΏΡ€ΠΈ напряТСнии Π² Ρ†Π΅ΠΏΠΈ 15 Π’?

  7. Π’ Π»Π°ΠΌΠΏΠΎΡ‡ΠΊΠ΅ ΠΊΠ°Ρ€ΠΌΠ°Π½Π½ΠΎΠ³ΠΎ фонаря сила Ρ‚ΠΎΠΊΠ° Ρ€Π°Π²Π½Π° 0,2 А. ВычислитС ΡΠ»Π΅ΠΊΡ‚Ρ€ΠΈΡ‡Π΅ΡΠΊΡƒΡŽ ΡΠ½Π΅Ρ€Π³ΠΈΡŽ, ΠΏΠΎΠ»ΡƒΡ‡Π°Π΅ΠΌΡƒΡŽ Π»Π°ΠΌΠΏΠΎΡ‡ΠΊΠΎΠΉ Π·Π° ΠΊΠ°ΠΆΠ΄Ρ‹Π΅ 2 ΠΌΠΈΠ½, Ссли напряТСниС Π½Π° Π»Π°ΠΌΠΏΠΎΡ‡ΠΊΠ΅ составляСт 3,6 Π’.

  8. НачСртитС схСму элСктричСской Ρ†Π΅ΠΏΠΈ, состоящСй ΠΈΠ· источника Ρ‚ΠΎΠΊΠ° ΠΈ Π΄Π²ΡƒΡ… ΠΏΠΎΡΠ»Π΅Π΄ΠΎΠ²Π°Ρ‚Π΅Π»ΡŒΠ½ΠΎ соСдинСнных Π»Π°ΠΌΠΏ. Π˜Π·ΠΎΠ±Ρ€Π°Π·ΠΈΡ‚Π΅, ΠΊΠ°ΠΊ Π½Π°Π΄ΠΎ ΠΏΠΎΠ΄ΠΊΠ»ΡŽΡ‡ΠΈΡ‚ΡŒ Π°ΠΌΠΏΠ΅Ρ€ΠΌΠ΅Ρ‚Ρ€ ΠΈ Π²ΠΎΠ»ΡŒΡ‚ΠΌΠ΅Ρ‚Ρ€ для измСрСния силы Ρ‚ΠΎΠΊΠ° Π² ΠΊΠ°ΠΆΠ΄ΠΎΠΉ Π»Π°ΠΌΠΏΠ΅ ΠΈ напряТСния Π½Π° ΠΊΠ°ΠΆΠ΄ΠΎΠΉ Π»Π°ΠΌΠΏΠ΅.

Π”.Π—._________________________________________

  1. По спирали элСктролампы ΠΏΡ€ΠΎΡ…ΠΎΠ΄ΠΈΡ‚ 540 Кл элСктричСства Π·Π° ΠΊΠ°ΠΆΠ΄Ρ‹Π΅ 5 ΠΌΠΈΠ½. Π§Π΅ΠΌΡƒ Ρ€Π°Π²Π½Π° сила Ρ‚ΠΎΠΊΠ° Π² Π»Π°ΠΌΠΏΠ΅?

  2. Π§Π΅ΠΌΡƒ Ρ€Π°Π²Π½ΠΎ напряТСниС Π½Π° участкС Ρ†Π΅ΠΏΠΈ, Π½Π° ΠΊΠΎΡ‚ΠΎΡ€ΠΎΠΌ ΡΠΎΠ²Π΅Ρ€ΡˆΠ΅Π½Π° Ρ€Π°Π±ΠΎΡ‚Π° 500 Π”ΠΆ ΠΏΡ€ΠΈ ΠΏΡ€ΠΎΡ…ΠΎΠΆΠ΄Π΅Π½ΠΈΠΈ 25 Кл элСктричСства?

  3. ВычислитС Ρ€Π°Π±ΠΎΡ‚Ρƒ, которая ΡΠΎΠ²Π΅Ρ€ΡˆΠ°Π΅Ρ‚ΡΡ ΠΏΡ€ΠΈ ΠΏΡ€ΠΎΡ…ΠΎΠΆΠ΄Π΅Π½ΠΈΠΈ Ρ‡Π΅Ρ€Π΅Π· ΡΠΏΠΈΡ€Π°Π»ΡŒ элСктроплитки 15 Кл элСктричСства, Ссли ΠΎΠ½Π° Π²ΠΊΠ»ΡŽΡ‡Π΅Π½Π° Π² ΡΠ΅Ρ‚ΡŒ напряТСниСм 220 Π’?

  4. ΠŸΡ€ΠΈ элСктросваркС сила Ρ‚ΠΎΠΊΠ° достигаСт 200 А. Какой элСктричСский заряд ΠΏΡ€ΠΎΡ…ΠΎΠ΄ΠΈΡ‚ Ρ‡Π΅Ρ€Π΅Π· ΠΏΠΎΠΏΠ΅Ρ€Π΅Ρ‡Π½ΠΎΠ΅ сСчСниС элСктрода Π·Π° 1 ΠΌΠΈΠ½?

  5. ΠžΠΏΡ€Π΅Π΄Π΅Π»ΠΈΡ‚Π΅ напряТСниС Π½Π° участкС Ρ†Π΅ΠΏΠΈ, Ссли ΠΏΡ€ΠΈ ΠΏΡ€ΠΎΡ…ΠΎΠΆΠ΄Π΅Π½ΠΈΠΈ ΠΏΠΎ Π½Π΅ΠΌΡƒ заряда Π² 15 Кл Ρ‚ΠΎΠΊΠΎΠΌ Π±Ρ‹Π»Π° ΡΠΎΠ²Π΅Ρ€ΡˆΠ΅Π½Π° Ρ€Π°Π±ΠΎΡ‚Π° 9 ΠΊΠ”ΠΆ.

Π‘ΠΎΠΏΡ€ΠΎΡ‚ΠΈΠ²Π»Π΅Π½ΠΈΠ΅. 8 класс

  1. Π”Π»ΠΈΠ½Π° алюминиСвого ΠΏΡ€ΠΎΠ²ΠΎΠ΄Π° 500 ΠΌ, ΠΏΠ»ΠΎΡ‰Π°Π΄ΡŒ Π΅Π³ΠΎ ΠΏΠΎΠΏΠ΅Ρ€Π΅Ρ‡Π½ΠΎΠ³ΠΎ сСчСния 4 ΠΌΠΌ2Β , Π§Π΅ΠΌΡƒ Ρ€Π°Π²Π½ΠΎ сопротивлСниС ΠΏΡ€ΠΎΠ²ΠΎΠ΄Π°?

  2. ΠœΠ΅Π΄Π½Ρ‹ΠΉ ΠΏΡ€ΠΎΠ²ΠΎΠ΄ с ΠΏΠ»ΠΎΡ‰Π°Π΄ΡŒΡŽ ΠΏΠΎΠΏΠ΅Ρ€Π΅Ρ‡Π½ΠΎΠ³ΠΎ сСчСния 0,85Β ΠΌΠΌ2Β ΠΎΠ±Π»Π°Π΄Π°Π΅Ρ‚ сопротивлСниСм 4 Ом. Какова Π΄Π»ΠΈΠ½Π° ΠΏΡ€ΠΎΠ²ΠΎΠ΄Π°?

  3. Π”Π»ΠΈΠ½Π° сСрСбряного ΠΏΡ€ΠΎΠ²ΠΎΠ΄Π° 0,6 ΠΌ, Π° сопротивлСниС 0,015 Ом. ΠžΠΏΡ€Π΅Π΄Π΅Π»ΠΈΡ‚Π΅ ΠΏΠ»ΠΎΡ‰Π°Π΄ΡŒ ΠΏΠΎΠΏΠ΅Ρ€Π΅Ρ‡Π½ΠΎΠ³ΠΎ сСчСния ΠΏΡ€ΠΎΠ²ΠΎΠ΄Π°.

  4. Π‘ΠΏΠΈΡ€Π°Π»ΡŒ элСктричСской ΠΏΠ»ΠΈΡ‚ΠΊΠΈ ΠΈΠ·Π³ΠΎΡ‚ΠΎΠ²Π»Π΅Π½Π° ΠΈΠ· Π½ΠΈΡ…Ρ€ΠΎΠΌΠΎΠ²ΠΎΠΉ ΠΏΡ€ΠΎΠ²ΠΎΠ»ΠΎΠΊΠΈ Π΄Π»ΠΈΠ½ΠΎΠΉ 13,75 ΠΌ ΠΈ ΠΏΠ»ΠΎΡ‰Π°Π΄ΡŒΡŽ ΠΏΠΎΠΏΠ΅Ρ€Π΅Ρ‡Π½ΠΎΠ³ΠΎ сСчСния 0,1 ΠΌΠΌ2. ΠŸΠ»ΠΈΡ‚ΠΊΠ° рассчитана Π½Π° напряТСниС 220 Π’. ΠžΠΏΡ€Π΅Π΄Π΅Π»ΠΈΡ‚Π΅ силу Ρ‚ΠΎΠΊΠ° Π² спирали ΠΏΠ»ΠΈΡ‚ΠΊΠΈ.

  5. Π‘ΠΈΠ»Π° Ρ‚ΠΎΠΊΠ° Π² ΠΆΠ΅Π»Π΅Π·Π½ΠΎΠΌ ΠΏΡ€ΠΎΠ²ΠΎΠ΄Π½ΠΈΠΊΠ΅ Π΄Π»ΠΈΠ½ΠΎΠΉ 150 ΠΌΠΌ ΠΈ ΠΏΠ»ΠΎΡ‰Π°Π΄ΡŒΡŽ ΠΏΠΎΠΏΠ΅Ρ€Π΅Ρ‡Π½ΠΎΠ³ΠΎ сСчСния 0,02 ΠΌΠΌ2 Ρ€Π°Π²Π½Π° 250 мА. Каково напряТСниС Π½Π° ΠΊΠΎΠ½Ρ†Π°Ρ… ΠΏΡ€ΠΎΠ²ΠΎΠ΄Π½ΠΈΠΊΠ°?

  6. ΠžΠΏΡ€Π΅Π΄Π΅Π»ΠΈΡ‚Π΅ силу Ρ‚ΠΎΠΊΠ°, проходящСго Ρ‡Π΅Ρ€Π΅Π· рСостат, ΠΈΠ·Π³ΠΎΡ‚ΠΎΠ²Π»Π΅Π½Π½Ρ‹ΠΉ ΠΈΠ· Π½ΠΈΠΊΠ΅Π»ΠΈΠ½ΠΎΠ²ΠΎΠΉ ΠΏΡ€ΠΎΠ²ΠΎΠ»ΠΎΠΊΠΈ Π΄Π»ΠΈΠ½ΠΎΠΉ 50 ΠΌ ΠΈ ΠΏΠ»ΠΎΡ‰Π°Π΄ΡŒΡŽ ΠΏΠΎΠΏΠ΅Ρ€Π΅Ρ‡Π½ΠΎΠ³ΠΎ сСчСния 1 ΠΌΠΌ2, Ссли напряТСниС Π½Π° Π·Π°ΠΆΠΈΠΌΠ°Ρ… рСостата Ρ€Π°Π²Π½ΠΎ 45 Π’.

Π”.Π—. Π£ΠΏΡ€ 20(ΡƒΡ‡Π΅Π±Π½ΠΈΠΊ)

Π’Π°Ρ€ΠΈΠ°Π½Ρ‚ β„–1

  1. ΠžΠΏΡ€Π΅Π΄Π΅Π»ΠΈΡ‚Π΅ силу Ρ‚ΠΎΠΊΠ° Π² элСктричСской Π»Π°ΠΌΠΏΠΎΡ‡ΠΊΠ΅, Ссли Ρ‡Π΅Ρ€Π΅Π· Π΅Π΅ Π½ΠΈΡ‚ΡŒ Π½Π°ΠΊΠ°Π»Π° Π·Π° 10 ΠΌΠΈΠ½ΡƒΡ‚ ΠΏΡ€ΠΎΡ…ΠΎΠ΄ΠΈΡ‚ элСктричСский заряд 300 Кл.

  2. Π‘ΠΈΠ»Π° Ρ‚ΠΎΠΊΠ° Π² ΡƒΡ‚ΡŽΠ³Π΅ 0,2 А. Какой элСктричСский заряд ΠΏΡ€ΠΎΠΉΠ΄Π΅Ρ‚ Ρ‡Π΅Ρ€Π΅Π· ΡΠΏΠΈΡ€Π°Π»ΡŒ Ρ‡Π΅Ρ€Π΅Π· 5 ΠΌΠΈΠ½ΡƒΡ‚?

  3. ΠŸΡ€ΠΈ элСктросваркС сила Ρ‚ΠΎΠΊΠ° достигаСт 200 А. Π—Π° ΠΊΠ°ΠΊΠΎΠ΅ врСмя Ρ‡Π΅Ρ€Π΅Π· ΠΏΠΎΠΏΠ΅Ρ€Π΅Ρ‡Π½ΠΎΠ΅ сСчСниС элСктрода ΠΏΡ€ΠΎΡ…ΠΎΠ΄ΠΈΡ‚ заряд 60000 Кл?

  4. Π§Π΅ΠΌΡƒ Ρ€Π°Π²Π½ΠΎ напряТСниС Π½Π° участкС Ρ†Π΅ΠΏΠΈ, Π½Π° ΠΊΠΎΡ‚ΠΎΡ€ΠΎΠΌ элСктричСскоС ΠΏΠΎΠ»Π΅ ΡΠΎΠ²Π΅Ρ€ΡˆΠΈΠ»ΠΎ Ρ€Π°Π±ΠΎΡ‚Ρƒ 500 Π”ΠΆ ΠΏΡ€ΠΈ ΠΏΡ€ΠΎΡ…ΠΎΠΆΠ΄Π΅Π½ΠΈΠΈ заряда 25 Кл?

  5. НапряТСниС Π½Π° Π»Π°ΠΌΠΏΠΎΡ‡ΠΊΠ΅ 220 Π’. ΠšΠ°ΠΊΡƒΡŽ Ρ€Π°Π±ΠΎΡ‚Ρƒ ΡΠΎΠ²Π΅Ρ€ΡˆΠ°Π΅Ρ‚ элСктричСскоС ΠΏΠΎΠ»Π΅ ΠΏΡ€ΠΈ ΠΏΡ€ΠΎΡ…ΠΎΠΆΠ΄Π΅Π½ΠΈΠΈ Ρ‡Π΅Ρ€Π΅Π· Π½ΠΈΡ‚ΡŒ Π½Π°ΠΊΠ°Π»Π° Π»Π°ΠΌΠΏΠΎΡ‡ΠΊΠΈ заряда 7 Кл?

  6. Π‘ΠΏΠΈΡ€Π°Π»ΡŒ элСктричСской ΠΏΠ»ΠΈΡ‚ΠΊΠΈ ΠΈΠ·Π³ΠΎΡ‚ΠΎΠ²Π»Π΅Π½Π° ΠΈΠ·Β Π½ΠΈΠΊΠ΅Π»ΠΈΠ½ΠΎΠ²ΠΎΠΉΒ ΠΏΡ€ΠΎΠ²ΠΎΠ»ΠΎΠΊΠΈ, Π΄Π»ΠΈΠ½Π° ΠΊΠΎΡ‚ΠΎΡ€ΠΎΠΉ Ρ€Π°Π²Π½Π°Β 6,2Β ΠΌ, Π°Β ΠΏΠ»ΠΎΡ‰Π°Π΄ΡŒ ΠΏΠΎΠΏΠ΅Ρ€Π΅Ρ‡Π½ΠΎΠ³ΠΎ сСчСния β€”Β 0,5Β ΠΌΠΌ2. НайдитС сопротивлСниС спирали, Ссли ΡƒΠ΄Π΅Π»ΡŒΠ½ΠΎΠ΅ сопротивлСниС Π½ΠΈΠΊΠ΅Π»ΠΈΠ½Π° Ρ€Π°Π²Π½ΠΎΒ 0,42Β .Β 

  7. ΠžΠΏΡ€Π΅Π΄Π΅Π»ΠΈΡ‚Π΅ напряТСниС Π½Π° ΠΊΠΎΠ½Ρ†Π°Ρ… ΡΡ‚Π°Π»ΡŒΠ½ΠΎΠ³ΠΎ ΠΏΡ€ΠΎΠ²ΠΎΠ΄Π½ΠΈΠΊΠ° Π΄Π»ΠΈΠ½ΠΎΠΉ 140 см ΠΈ ΠΏΠ»ΠΎΡ‰Π°Π΄ΡŒΡŽ ΠΏΠΎΠΏΠ΅Ρ€Π΅Ρ‡Π½ΠΎΠ³ΠΎ сСчСния 0,2 ΠΌΠΌ2, Π² ΠΊΠΎΡ‚ΠΎΡ€ΠΎΠΌ сила Ρ‚ΠΎΠΊΠ° 250 мА.

Π’Π°Ρ€ΠΈΠ°Π½Ρ‚β„–2

  1. Π§Π΅Ρ€Π΅Π· ΡΠΏΠΈΡ€Π°Π»ΡŒ элСктроплитки Π·Π° 2 ΠΌΠΈΠ½ΡƒΡ‚Ρ‹ ΠΏΡ€ΠΎΡˆΠ΅Π» заряд Π² 600 Кл. ΠžΠΏΡ€Π΅Π΄Π΅Π»ΠΈΡ‚Π΅ силу Ρ‚ΠΎΠΊΠ°.

  2. Π—Π° ΠΊΠ°ΠΊΠΎΠ΅ врСмя Ρ‡Π΅Ρ€Π΅Π· ΠΏΠΎΠΏΠ΅Ρ€Π΅Ρ‡Π½ΠΎΠ΅ сСчСниС ΠΏΡ€ΠΎΠ²ΠΎΠ΄Π½ΠΈΠΊΠ° ΠΏΡ€ΠΎΠΉΠ΄Π΅Ρ‚ заряд, Ρ€Π°Π²Π½Ρ‹ΠΉ 30 Кл, Ρ‚ΠΎΠΊΠ° Π² спирали. Ρ†Π΅ΠΏΠΈ 0,2 А? ΠΏΡ€ΠΈ силС Ρ‚ΠΎΠΊΠ° 200 мА?

  3. ΠŸΡ€ΠΈ ΠΏΡ€ΠΎΡ…ΠΎΠΆΠ΄Π΅Π½ΠΈΠΈ элСктричСского заряда 12 Кл ΡΠΎΠ²Π΅Ρ€ΡˆΠ°Π΅Ρ‚ΡΡ Ρ€Π°Π±ΠΎΡ‚Π° 600 Π”ΠΆ. Π§Π΅ΠΌΡƒ Ρ€Π°Π²Π½ΠΎ напряТСниС Π½Π° ΠΊΠΎΠ½Ρ†Π°Ρ… этого ΠΏΡ€ΠΎΠ²ΠΎΠ΄Π½ΠΈΠΊΠ°?

  4. ВычислитС Ρ€Π°Π±ΠΎΡ‚Ρƒ, которая ΡΠΎΠ²Π΅Ρ€ΡˆΠ°Π΅Ρ‚ΡΡ ΠΏΡ€ΠΈ ΠΏΡ€ΠΎΡ…ΠΎΠΆΠ΄Π΅Π½ΠΈΠΈ Ρ‡Π΅Ρ€Π΅Π· ΡΠΏΠΈΡ€Π°Π»ΡŒ элСктроплитки заряда 15 Кл, Ссли ΠΎΠ½Π° Π²ΠΊΠ»ΡŽΡ‡Π΅Π½Π° Π² ΡΠ΅Ρ‚ΡŒ с напряТСниСм 220 Π’.

  5. НапряТСниС Π½Π° Π»Π°ΠΌΠΏΠ΅ накаливания 220 Π’. Какой заряд ΠΏΡ€ΠΎΡˆΠ΅Π» Ρ‡Π΅Ρ€Π΅Π· Π½ΠΈΡ‚ΡŒ Π½Π°ΠΊΠ°Π»Π° Π»Π°ΠΌΠΏΡ‹, Ссли ΠΏΡ€ΠΈ этом Π±Ρ‹Π»Π° ΡΠΎΠ²Π΅Ρ€ΡˆΠ΅Π½Π° Ρ€Π°Π±ΠΎΡ‚Π° 4400 Π”ΠΆ?

  6. ΠŸΡ€ΠΈ ΠΈΠ·Π³ΠΎΡ‚ΠΎΠ²Π»Π΅Π½ΠΈΠΈ ΠΊΠΈΠΏΡΡ‚ΠΈΠ»ΡŒΠ½ΠΈΠΊΠ° ΠΈΡΠΏΠΎΠ»ΡŒΠ·ΠΎΠ²Π°Π»ΠΈΒ Π°Π»ΡŽΠΌΠΈΠ½ΠΈΠ΅Π²ΡƒΡŽΒ ΠΏΡ€ΠΎΠ²ΠΎΠ»ΠΎΠΊΡƒ, Π΄Π»ΠΈΠ½Π° ΠΊΠΎΡ‚ΠΎΡ€ΠΎΠΉ Ρ€Π°Π²Π½Π°Β 4,7Β ΠΌ, Π°Β ΠΏΠ»ΠΎΡ‰Π°Π΄ΡŒ ΠΏΠΎΠΏΠ΅Ρ€Π΅Ρ‡Π½ΠΎΠ³ΠΎ сСчСния β€”Β 0,6Β ΠΌΠΌ2. Найди сопротивлСниС спирали ΠΊΠΈΠΏΡΡ‚ΠΈΠ»ΡŒΠ½ΠΈΠΊΠ°, Ссли ΡƒΠ΄Π΅Π»ΡŒΠ½ΠΎΠ΅ сопротивлСниС алюминия Ρ€Π°Π²Π½ΠΎΒ 0,028Β 

  7. ΠžΠΏΡ€Π΅Π΄Π΅Π»ΠΈΡ‚Π΅ напряТСниС Π½Π° ΠΊΠΎΠ½Ρ†Π°Ρ… ΡΡ‚Π°Π»ΡŒΠ½ΠΎΠ³ΠΎ ΠΏΡ€ΠΎΠ²ΠΎΠ΄Π½ΠΈΠΊΠ° Π΄Π»ΠΈΠ½ΠΎΠΉ 140 см ΠΈ ΠΏΠ»ΠΎΡ‰Π°Π΄ΡŒΡŽ ΠΏΠΎΠΏΠ΅Ρ€Π΅Ρ‡Π½ΠΎΠ³ΠΎ сСчСния 0,2 ΠΌΠΌ2, Π² ΠΊΠΎΡ‚ΠΎΡ€ΠΎΠΌ сила Ρ‚ΠΎΠΊΠ° 250 мА.

Π’Π°Ρ€ΠΈΠ°Π½Ρ‚ β„–1

  1. ΠžΠΏΡ€Π΅Π΄Π΅Π»ΠΈΡ‚Π΅ силу Ρ‚ΠΎΠΊΠ° Π² элСктричСской Π»Π°ΠΌΠΏΠΎΡ‡ΠΊΠ΅, Ссли Ρ‡Π΅Ρ€Π΅Π· Π΅Π΅ Π½ΠΈΡ‚ΡŒ Π½Π°ΠΊΠ°Π»Π° Π·Π° 10 ΠΌΠΈΠ½ΡƒΡ‚ ΠΏΡ€ΠΎΡ…ΠΎΠ΄ΠΈΡ‚ элСктричСский заряд 300 Кл.

  2. Π‘ΠΈΠ»Π° Ρ‚ΠΎΠΊΠ° Π² ΡƒΡ‚ΡŽΠ³Π΅ 0,2 А. Какой элСктричСский заряд ΠΏΡ€ΠΎΠΉΠ΄Π΅Ρ‚ Ρ‡Π΅Ρ€Π΅Π· ΡΠΏΠΈΡ€Π°Π»ΡŒ Ρ‡Π΅Ρ€Π΅Π· 5 ΠΌΠΈΠ½ΡƒΡ‚?

  3. ΠŸΡ€ΠΈ элСктросваркС сила Ρ‚ΠΎΠΊΠ° достигаСт 200 А. Π—Π° ΠΊΠ°ΠΊΠΎΠ΅ врСмя Ρ‡Π΅Ρ€Π΅Π· ΠΏΠΎΠΏΠ΅Ρ€Π΅Ρ‡Π½ΠΎΠ΅ сСчСниС элСктрода ΠΏΡ€ΠΎΡ…ΠΎΠ΄ΠΈΡ‚ заряд 60000 Кл?

  4. Π§Π΅ΠΌΡƒ Ρ€Π°Π²Π½ΠΎ напряТСниС Π½Π° участкС Ρ†Π΅ΠΏΠΈ, Π½Π° ΠΊΠΎΡ‚ΠΎΡ€ΠΎΠΌ элСктричСскоС ΠΏΠΎΠ»Π΅ ΡΠΎΠ²Π΅Ρ€ΡˆΠΈΠ»ΠΎ Ρ€Π°Π±ΠΎΡ‚Ρƒ 500 Π”ΠΆ ΠΏΡ€ΠΈ ΠΏΡ€ΠΎΡ…ΠΎΠΆΠ΄Π΅Π½ΠΈΠΈ заряда 25 Кл?

  5. НапряТСниС Π½Π° Π»Π°ΠΌΠΏΠΎΡ‡ΠΊΠ΅ 220 Π’. ΠšΠ°ΠΊΡƒΡŽ Ρ€Π°Π±ΠΎΡ‚Ρƒ ΡΠΎΠ²Π΅Ρ€ΡˆΠ°Π΅Ρ‚ элСктричСскоС ΠΏΠΎΠ»Π΅ ΠΏΡ€ΠΈ ΠΏΡ€ΠΎΡ…ΠΎΠΆΠ΄Π΅Π½ΠΈΠΈ Ρ‡Π΅Ρ€Π΅Π· Π½ΠΈΡ‚ΡŒ Π½Π°ΠΊΠ°Π»Π° Π»Π°ΠΌΠΏΠΎΡ‡ΠΊΠΈ заряда 7 Кл?

  6. Π‘ΠΏΠΈΡ€Π°Π»ΡŒ элСктричСской ΠΏΠ»ΠΈΡ‚ΠΊΠΈ ΠΈΠ·Π³ΠΎΡ‚ΠΎΠ²Π»Π΅Π½Π° ΠΈΠ·Β Π½ΠΈΠΊΠ΅Π»ΠΈΠ½ΠΎΠ²ΠΎΠΉΒ ΠΏΡ€ΠΎΠ²ΠΎΠ»ΠΎΠΊΠΈ, Π΄Π»ΠΈΠ½Π° ΠΊΠΎΡ‚ΠΎΡ€ΠΎΠΉ Ρ€Π°Π²Π½Π°Β 6,2Β ΠΌ, Π°Β ΠΏΠ»ΠΎΡ‰Π°Π΄ΡŒ ΠΏΠΎΠΏΠ΅Ρ€Π΅Ρ‡Π½ΠΎΠ³ΠΎ сСчСния β€”Β 0,5Β ΠΌΠΌ2. НайдитС сопротивлСниС спирали, Ссли ΡƒΠ΄Π΅Π»ΡŒΠ½ΠΎΠ΅ сопротивлСниС Π½ΠΈΠΊΠ΅Π»ΠΈΠ½Π° Ρ€Π°Π²Π½ΠΎΒ 0,42Β .Β 

  7. ΠžΠΏΡ€Π΅Π΄Π΅Π»ΠΈΡ‚Π΅ напряТСниС Π½Π° ΠΊΠΎΠ½Ρ†Π°Ρ… ΡΡ‚Π°Π»ΡŒΠ½ΠΎΠ³ΠΎ ΠΏΡ€ΠΎΠ²ΠΎΠ΄Π½ΠΈΠΊΠ° Π΄Π»ΠΈΠ½ΠΎΠΉ 140 см ΠΈ ΠΏΠ»ΠΎΡ‰Π°Π΄ΡŒΡŽ ΠΏΠΎΠΏΠ΅Ρ€Π΅Ρ‡Π½ΠΎΠ³ΠΎ сСчСния 0,2 ΠΌΠΌ2, Π² ΠΊΠΎΡ‚ΠΎΡ€ΠΎΠΌ сила Ρ‚ΠΎΠΊΠ° 250 мА.

Π’Π°Ρ€ΠΈΠ°Π½Ρ‚β„–2

  1. Π§Π΅Ρ€Π΅Π· ΡΠΏΠΈΡ€Π°Π»ΡŒ элСктроплитки Π·Π° 2 ΠΌΠΈΠ½ΡƒΡ‚Ρ‹ ΠΏΡ€ΠΎΡˆΠ΅Π» заряд Π² 600 Кл. ΠžΠΏΡ€Π΅Π΄Π΅Π»ΠΈΡ‚Π΅ силу Ρ‚ΠΎΠΊΠ°.

  2. Π—Π° ΠΊΠ°ΠΊΠΎΠ΅ врСмя Ρ‡Π΅Ρ€Π΅Π· ΠΏΠΎΠΏΠ΅Ρ€Π΅Ρ‡Π½ΠΎΠ΅ сСчСниС ΠΏΡ€ΠΎΠ²ΠΎΠ΄Π½ΠΈΠΊΠ° ΠΏΡ€ΠΎΠΉΠ΄Π΅Ρ‚ заряд, Ρ€Π°Π²Π½Ρ‹ΠΉ 30 Кл, Ρ‚ΠΎΠΊΠ° Π² спирали. Ρ†Π΅ΠΏΠΈ 0,2 А? ΠΏΡ€ΠΈ силС Ρ‚ΠΎΠΊΠ° 200 мА?

  3. ΠŸΡ€ΠΈ ΠΏΡ€ΠΎΡ…ΠΎΠΆΠ΄Π΅Π½ΠΈΠΈ элСктричСского заряда 12 Кл ΡΠΎΠ²Π΅Ρ€ΡˆΠ°Π΅Ρ‚ΡΡ Ρ€Π°Π±ΠΎΡ‚Π° 600 Π”ΠΆ. Π§Π΅ΠΌΡƒ Ρ€Π°Π²Π½ΠΎ напряТСниС Π½Π° ΠΊΠΎΠ½Ρ†Π°Ρ… этого ΠΏΡ€ΠΎΠ²ΠΎΠ΄Π½ΠΈΠΊΠ°?

  4. ВычислитС Ρ€Π°Π±ΠΎΡ‚Ρƒ, которая ΡΠΎΠ²Π΅Ρ€ΡˆΠ°Π΅Ρ‚ΡΡ ΠΏΡ€ΠΈ ΠΏΡ€ΠΎΡ…ΠΎΠΆΠ΄Π΅Π½ΠΈΠΈ Ρ‡Π΅Ρ€Π΅Π· ΡΠΏΠΈΡ€Π°Π»ΡŒ элСктроплитки заряда 15 Кл, Ссли ΠΎΠ½Π° Π²ΠΊΠ»ΡŽΡ‡Π΅Π½Π° Π² ΡΠ΅Ρ‚ΡŒ с напряТСниСм 220 Π’.

  5. НапряТСниС Π½Π° Π»Π°ΠΌΠΏΠ΅ накаливания 220 Π’. Какой заряд ΠΏΡ€ΠΎΡˆΠ΅Π» Ρ‡Π΅Ρ€Π΅Π· Π½ΠΈΡ‚ΡŒ Π½Π°ΠΊΠ°Π»Π° Π»Π°ΠΌΠΏΡ‹, Ссли ΠΏΡ€ΠΈ этом Π±Ρ‹Π»Π° ΡΠΎΠ²Π΅Ρ€ΡˆΠ΅Π½Π° Ρ€Π°Π±ΠΎΡ‚Π° 4400 Π”ΠΆ?

  6. ΠŸΡ€ΠΈ ΠΈΠ·Π³ΠΎΡ‚ΠΎΠ²Π»Π΅Π½ΠΈΠΈ ΠΊΠΈΠΏΡΡ‚ΠΈΠ»ΡŒΠ½ΠΈΠΊΠ° ΠΈΡΠΏΠΎΠ»ΡŒΠ·ΠΎΠ²Π°Π»ΠΈΒ Π°Π»ΡŽΠΌΠΈΠ½ΠΈΠ΅Π²ΡƒΡŽΒ ΠΏΡ€ΠΎΠ²ΠΎΠ»ΠΎΠΊΡƒ, Π΄Π»ΠΈΠ½Π° ΠΊΠΎΡ‚ΠΎΡ€ΠΎΠΉ Ρ€Π°Π²Π½Π°Β 4,7Β ΠΌ, Π°Β ΠΏΠ»ΠΎΡ‰Π°Π΄ΡŒ ΠΏΠΎΠΏΠ΅Ρ€Π΅Ρ‡Π½ΠΎΠ³ΠΎ сСчСния β€”Β 0,6Β ΠΌΠΌ2. Найди сопротивлСниС спирали ΠΊΠΈΠΏΡΡ‚ΠΈΠ»ΡŒΠ½ΠΈΠΊΠ°, Ссли ΡƒΠ΄Π΅Π»ΡŒΠ½ΠΎΠ΅ сопротивлСниС алюминия Ρ€Π°Π²Π½ΠΎΒ 0,028Β 

  7. ΠžΠΏΡ€Π΅Π΄Π΅Π»ΠΈΡ‚Π΅ напряТСниС Π½Π° ΠΊΠΎΠ½Ρ†Π°Ρ… ΡΡ‚Π°Π»ΡŒΠ½ΠΎΠ³ΠΎ ΠΏΡ€ΠΎΠ²ΠΎΠ΄Π½ΠΈΠΊΠ° Π΄Π»ΠΈΠ½ΠΎΠΉ 140 см ΠΈ ΠΏΠ»ΠΎΡ‰Π°Π΄ΡŒΡŽ ΠΏΠΎΠΏΠ΅Ρ€Π΅Ρ‡Π½ΠΎΠ³ΠΎ сСчСния 0,2 ΠΌΠΌ2, Π² ΠΊΠΎΡ‚ΠΎΡ€ΠΎΠΌ сила Ρ‚ΠΎΠΊΠ° 250 мА.

Π—Π°Π²ΠΈΡΠΈΠΌΠΎΡΡ‚ΡŒ силы Ρ‚ΠΎΠΊΠ° ΠΎΡ‚ напряТСния – Π³Ρ€Π°Ρ„ΠΈΠΊ, Ρ„ΠΎΡ€ΠΌΡƒΠ»Π° Π·Π°ΠΊΠΎΠ½Π°

3.9

БрСдняя ΠΎΡ†Π΅Π½ΠΊΠ°: 3.9

ВсСго ΠΏΠΎΠ»ΡƒΡ‡Π΅Π½ΠΎ ΠΎΡ†Π΅Π½ΠΎΠΊ: 112.

3.9

БрСдняя ΠΎΡ†Π΅Π½ΠΊΠ°: 3.9

ВсСго ΠΏΠΎΠ»ΡƒΡ‡Π΅Π½ΠΎ ΠΎΡ†Π΅Π½ΠΎΠΊ: 112.

ЭлСктричСский Ρ‚ΠΎΠΊ β€” это упорядочСнноС Π΄Π²ΠΈΠΆΠ΅Π½ΠΈΠ΅ элСктричСских зарядов ΠΈΠ»ΠΈ заряТСнных макроскопичСских Ρ‚Π΅Π». НаправлСниС элСктричСского Ρ‚ΠΎΠΊΠ° I совпадаСт с Π½Π°ΠΏΡ€Π°Π²Π»Π΅Π½ΠΈΠ΅ΠΌ двиТСния ΠΏΠΎΠ»ΠΎΠΆΠΈΡ‚Π΅Π»ΡŒΠ½ΠΎ заряТСнных частиц: заряды двиТутся ΠΏΠΎΠ΄ воздСйствиСм элСктричСского поля, ΠΊΠΎΡ‚ΠΎΡ€ΠΎΠ΅ создаСтся Π² ΠΏΡ€ΠΎΠ²ΠΎΠ΄Π½ΠΈΠΊΠ΅ Π² Ρ€Π΅Π·ΡƒΠ»ΡŒΡ‚Π°Ρ‚Π΅ ΠΏΡ€ΠΈΠ»ΠΎΠΆΠ΅Π½Π½ΠΎΠ³ΠΎ ΠΊ ΠΊΠΎΠ½Ρ†Π°ΠΌ ΠΏΡ€ΠΎΠ²ΠΎΠ΄Π½ΠΈΠΊΠ° напряТСния U. Как Π²Π΅Π»ΠΈΡ‡ΠΈΠ½Π° силы Ρ‚ΠΎΠΊΠ° зависит ΠΎΡ‚ Π²Π΅Π»ΠΈΡ‡ΠΈΠ½Ρ‹ напряТСния? ΠŸΠΎΠΏΡ€ΠΎΠ±ΡƒΠ΅ΠΌ Ρ€Π°Π·ΠΎΠ±Ρ€Π°Ρ‚ΡŒΡΡ.

Π’Π΅Π»ΠΈΡ‡ΠΈΠ½Π° силы Ρ‚ΠΎΠΊΠ°

По ΠΎΠΏΡ€Π΅Π΄Π΅Π»Π΅Π½ΠΈΡŽ силой Ρ‚ΠΎΠΊΠ° называСтся физичСская Π²Π΅Π»ΠΈΡ‡ΠΈΠ½Π° равная Π²Π΅Π»ΠΈΡ‡ΠΈΠ½Π΅ заряда q, ΠΏΡ€ΠΎΡˆΠ΅Π΄ΡˆΠ΅Π³ΠΎ Ρ‡Π΅Ρ€Π΅Π· ΠΏΠΎΠΏΠ΅Ρ€Π΅Ρ‡Π½ΠΎΠ΅ сСчСниС ΠΏΡ€ΠΎΠ²ΠΎΠ΄Π½ΠΈΠΊΠ° Π·Π° врСмя t:

$$ I = { q\over t } $$

Если сила Ρ‚ΠΎΠΊΠ° Π½Π΅ зависит ΠΎΡ‚ Π²Ρ€Π΅ΠΌΠ΅Π½ΠΈ, Ρ‚ΠΎ Ρ‚Π°ΠΊΠΎΠΉ элСктричСский Ρ‚ΠΎΠΊ называСтся постоянным. Рассмотрим Π΄Π°Π»Π΅Π΅ ΠΈΠΌΠ΅Π½Π½ΠΎ Ρ‚Π°ΠΊΠΎΠΉ случай, ΠΊΠΎΠ³Π΄Π° Ρ‚ΠΎΠΊ постоянСн. Π˜Π·ΠΌΠ΅Ρ€ΠΈΡ‚ΡŒ Π²Π΅Π»ΠΈΡ‡ΠΈΠ½Ρƒ заряда Ρ‡Ρ€Π΅Π·Π²Ρ‹Ρ‡Π°ΠΉΠ½ΠΎ Ρ‚Ρ€ΡƒΠ΄Π½ΠΎ, поэтому Π² 1826 Π³. Π½Π΅ΠΌΠ΅Ρ†ΠΊΠΈΠΉ Ρ„ΠΈΠ·ΠΈΠΊ Π“Π΅ΠΎΡ€Π³ Ом поступил ΡΠ»Π΅Π΄ΡƒΡŽΡ‰ΠΈΠΌ ΠΎΠ±Ρ€Π°Π·ΠΎΠΌ: Π² элСктричСской Ρ†Π΅ΠΏΠΈ, состоящСй ΠΈΠ· источника напряТСния (Π±Π°Ρ‚Π°Ρ€Π΅ΠΈ) ΠΈ сопротивлСния, ΠΎΠ½ измСрял Π²Π΅Π»ΠΈΡ‡ΠΈΠ½Ρƒ Ρ‚ΠΎΠΊΠ° ΠΏΡ€ΠΈ Ρ€Π°Π·Π½Ρ‹Ρ… значСниях сопротивлСния. Π—Π°Ρ‚Π΅ΠΌ, Π½Π΅ мСняя Π²Π΅Π»ΠΈΡ‡ΠΈΠ½Ρƒ сопротивлСния, ΠΎΠ½ стал ΠΈΠ·ΠΌΠ΅Π½ΡΡ‚ΡŒ ΠΏΠ°Ρ€Π°ΠΌΠ΅Ρ‚Ρ€Ρ‹ источника напряТСния, ΠΏΠΎΠ΄ΠΊΠ»ΡŽΡ‡Π°Ρ сразу, Π½Π°ΠΏΡ€ΠΈΠΌΠ΅Ρ€, Π΄Π²Π°-Ρ‚Ρ€ΠΈ источника. Π˜Π·ΠΌΠ΅Ρ€ΡΡ Π²Π΅Π»ΠΈΡ‡ΠΈΠ½Ρƒ Ρ‚ΠΎΠΊΠ° Π² Ρ†Π΅ΠΏΠΈ, ΠΎΠ½ ΠΏΠΎΠ»ΡƒΡ‡ΠΈΠ» зависимости силы Ρ‚ΠΎΠΊΠ° ΠΎΡ‚ напряТСния U ΠΈ ΠΎΡ‚ сопротивлСния R.

Рис. 1. Π‘Ρ…Π΅ΠΌΠ° ΠΈΠ·ΠΌΠ΅Ρ€Π΅Π½ΠΈΠΉ Ρ‚ΠΎΠΊΠ° ΠΈ напряТСния Π“Π΅ΠΎΡ€Π³Π° Ома.

Π—Π°ΠΊΠΎΠ½ Ома

Π’ Ρ€Π΅Π·ΡƒΠ»ΡŒΡ‚Π°Ρ‚Π΅ ΠΏΡ€ΠΎΠ²Π΅Π΄Π΅Π½Π½Ρ‹Ρ… исслСдований Π“Π΅ΠΎΡ€Π³ Ом ΠΎΠ±Π½Π°Ρ€ΡƒΠΆΠΈΠ», Ρ‡Ρ‚ΠΎ ΠΎΡ‚Π½ΠΎΡˆΠ΅Π½ΠΈΠ΅ напряТСния U ΠΌΠ΅ΠΆΠ΄Ρƒ ΠΊΠΎΠ½Ρ†Π°ΠΌΠΈ мСталличСского ΠΏΡ€ΠΎΠ²ΠΎΠ΄Π½ΠΈΠΊΠ°, ΡΠ²Π»ΡΡŽΡ‰Π΅Π³ΠΎΡΡ участком элСктричСской Ρ†Π΅ΠΏΠΈ, ΠΊ силС Ρ‚ΠΎΠΊΠ° I Π² Ρ†Π΅ΠΏΠΈ Π΅ΡΡ‚ΡŒ Π²Π΅Π»ΠΈΡ‡ΠΈΠ½Π° постоянная:

$$ R= { U \over I } $$

Π³Π΄Π΅ R β€” элСктричСскоС сопротивлСниС. Данная Ρ„ΠΎΡ€ΠΌΡƒΠ»Π° называСтся Π·Π°ΠΊΠΎΠ½ΠΎΠΌ Ома, ΠΊΠΎΡ‚ΠΎΡ€Ρ‹ΠΉ Π΄ΠΎ сих ΠΏΠΎΡ€ являСтся основным расчСтным инструмСнтом ΠΏΡ€ΠΈ ΠΏΡ€ΠΎΠ΅ΠΊΡ‚ΠΈΡ€ΠΎΠ²Π°Π½ΠΈΠΈ элСктричСских ΠΈ элСктронных схСм.

Если ΠΏΠΎ оси абсцисс ΠΎΡ‚Π»ΠΎΠΆΠΈΡ‚ΡŒ значСния напряТСния, Π° ΠΏΠΎ оси ΠΎΡ€Π΄ΠΈΠ½Π°Ρ‚ β€” значСния Ρ‚ΠΎΠΊΠ° Π² Ρ†Π΅ΠΏΠΈ ΠΏΡ€ΠΈ Π΄Π°Π½Π½Ρ‹Ρ… значСниях напряТСния, Ρ‚ΠΎ получится Π³Ρ€Π°Ρ„ΠΈΠΊ зависимости силы Ρ‚ΠΎΠΊΠ° I ΠΎΡ‚ напряТСния U.

Рис. 2. Π“Ρ€Π°Ρ„ΠΈΠΊ зависимости силы Ρ‚ΠΎΠΊΠ° ΠΎΡ‚ напряТСния.

Из этого Π³Ρ€Π°Ρ„ΠΈΠΊΠ° Π²ΠΈΠ΄Π½ΠΎ, Ρ‡Ρ‚ΠΎ эта Π·Π°Π²ΠΈΡΠΈΠΌΠΎΡΡ‚ΡŒ линСйная. Π£Π³ΠΎΠ» Π½Π°ΠΊΠ»ΠΎΠ½Π° прямой зависит ΠΎΡ‚ Π²Π΅Π»ΠΈΡ‡ΠΈΠ½Ρ‹ сопротивлСния. Π§Π΅ΠΌ большС R, Ρ‚Π΅ΠΌ мСньшС ΡƒΠ³ΠΎΠ» Π½Π°ΠΊΠ»ΠΎΠ½Π°.

Рис. 3. Π“Ρ€Π°Ρ„ΠΈΠΊ зависимости силы Ρ‚ΠΎΠΊΠ° ΠΎΡ‚ сопротивлСния.

Если Π·Π°Ρ„ΠΈΠΊΡΠΈΡ€ΠΎΠ²Π°Ρ‚ΡŒ напряТСниС U ΠΈ ΠΏΠΎ оси абсцисс ΠΎΡ‚ΠΊΠ»Π°Π΄Ρ‹Π²Π°Ρ‚ΡŒ значСния R элСктричСского сопротивлСния, Ρ‚ΠΎ ΠΈΠ· ΠΏΠΎΠ»ΡƒΡ‡Π΅Π½Π½ΠΎΠ³ΠΎ Π³Ρ€Π°Ρ„ΠΈΠΊΠ° Π²ΠΈΠ΄Π½ΠΎ, Ρ‡Ρ‚ΠΎ эта Π·Π°Π²ΠΈΡΠΈΠΌΠΎΡΡ‚ΡŒ ΡƒΠΆΠ΅ нСлинСйная β€” с ростом сопротивлСния ΠΏΠΎΠ²Π΅Π΄Π΅Π½ΠΈΠ΅ Ρ‚ΠΎΠΊΠ° описываСт ΠΎΠ±Ρ€Π°Ρ‚Π½ΠΎ ΠΏΡ€ΠΎΠΏΠΎΡ€Ρ†ΠΈΠΎΠ½Π°Π»ΡŒΠ½ΠΎΠΉ Ρ„ΡƒΠ½ΠΊΡ†ΠΈΠ΅ΠΉ β€” Π³ΠΈΠΏΠ΅Ρ€Π±ΠΎΠ»ΠΎΠΉ.

Π—Π°ΠΊΠΎΠ½ Ома пСрСстаСт Ρ€Π°Π±ΠΎΡ‚Π°Ρ‚ΡŒ ΠΏΡ€ΠΈ Π±ΠΎΠ»ΡŒΡˆΠΈΡ… Π²Π΅Π»ΠΈΡ‡ΠΈΠ½Π°Ρ… Ρ‚ΠΎΠΊΠ°, Ρ‚Π°ΠΊ ΠΊΠ°ΠΊ Π½Π°Ρ‡ΠΈΠ½Π°ΡŽΡ‚ Ρ€Π°Π±ΠΎΡ‚Π°Ρ‚ΡŒ Π΄ΠΎΠΏΠΎΠ»Π½ΠΈΡ‚Π΅Π»ΡŒΠ½Ρ‹Π΅ эффСкты, связанныС с Ρ‚Π΅ΠΏΠ»ΠΎΠ²Ρ‹ΠΌ Ρ€Π°Π·ΠΎΠ³Ρ€Π΅Π²ΠΎΠΌ вСщСства, ростом Ρ‚Π΅ΠΌΠΏΠ΅Ρ€Π°Ρ‚ΡƒΡ€Ρ‹. Π’ Π³Π°Π·Π°Ρ… ΠΏΡ€ΠΈ Π±ΠΎΠ»ΡŒΡˆΠΈΡ… Ρ‚ΠΎΠΊΠ°Ρ… Π²ΠΎΠ·Π½ΠΈΠΊΠ°Π΅Ρ‚ ΠΏΡ€ΠΎΠ±ΠΎΠΉ, Ρ‚ΠΎΠΊ растСт Π»Π°Π²ΠΈΠ½ΠΎΠΎΠ±Ρ€Π°Π·Π½ΠΎ, ΠΎΡ‚ΠΊΠ»ΠΎΠ½ΡΡΡΡŒ ΠΎΡ‚ Π»ΠΈΠ½Π΅ΠΉΠ½ΠΎΠ³ΠΎ Π·Π°ΠΊΠΎΠ½Π°.

ΠžΡ‚ Ρ‡Π΅Π³ΠΎ зависит Π²Π΅Π»ΠΈΡ‡ΠΈΠ½Π° сопротивлСния

ЭкспСримСнты ΠΏΠΎΠΊΠ°Π·Ρ‹Π²Π°ΡŽΡ‚, Ρ‡Ρ‚ΠΎ сопротивлСниС ΠΏΡ€ΠΎΠ²ΠΎΠ΄Π½ΠΈΠΊΠ° прямо ΠΏΡ€ΠΎΠΏΠΎΡ€Ρ†ΠΈΠΎΠ½Π°Π»ΡŒΠ½ΠΎ Π΅Π³ΠΎ Π΄Π»ΠΈΠ½Π΅ L ΠΈ ΠΎΠ±Ρ€Π°Ρ‚Π½ΠΎ ΠΏΡ€ΠΎΠΏΠΎΡ€Ρ†ΠΈΠΎΠ½Π°Π»ΡŒΠ½ΠΎ ΠΏΠ»ΠΎΡ‰Π°Π΄ΠΈ ΠΏΠΎΠΏΠ΅Ρ€Π΅Ρ‡Π½ΠΎΠ³ΠΎ сСчСния S:

$$ R = ρ *{ L \over S } $$

Π³Π΄Π΅ ρ β€” ΡƒΠ΄Π΅Π»ΡŒΠ½ΠΎΠ΅ элСктричСскоС сопротивлСниС вСщСства.

Π•Π΄ΠΈΠ½ΠΈΡ†Ρ‹ измСрСния

Π’ ΠΌΠ΅ΠΆΠ΄ΡƒΠ½Π°Ρ€ΠΎΠ΄Π½ΠΎΠΉ систСмС Π΅Π΄ΠΈΠ½ΠΈΡ† БИ Π΅Π΄ΠΈΠ½ΠΈΡ†Π° измСрСния элСктричСского сопротивлСния называСтся β€œΠΎΠΌβ€ Π² Ρ‡Π΅ΡΡ‚ΡŒ Ρ„ΠΈΠ·ΠΈΠΊΠ° Π“Π΅ΠΎΡ€Π³Π° Ома. По ΠΎΠΏΡ€Π΅Π΄Π΅Π»Π΅Π½ΠΈΡŽ элСктричСским сопротивлСниСм 1 Ом ΠΎΠ±Π»Π°Π΄Π°Π΅Ρ‚ участок Ρ†Π΅ΠΏΠΈ, Π½Π° ΠΊΠΎΡ‚ΠΎΡ€ΠΎΠΌ ΠΏΠ°Π΄Π°Π΅Ρ‚ напряТСниС 1 Π’ ΠΏΡ€ΠΈ силС Ρ‚ΠΎΠΊΠ° 1 А.

$$ [1 Ом] ={ [1 Π’]\over [1 А] } $$

Π•Π΄ΠΈΠ½ΠΈΡ†Π° измСрСния ΡƒΠ΄Π΅Π»ΡŒΠ½ΠΎΠ³ΠΎ сопротивлСния получаСтся ΠΏΡ€ΠΎΠΈΠ·Π²ΠΎΠ΄Π½ΠΎΠΉ ΠΎΡ‚ Π΅Π΄ΠΈΠ½ΠΈΡ† Π²Π΅Π»ΠΈΡ‡ΠΈΠ½, входящих Π² Ρ„ΠΎΡ€ΠΈΡƒΠ»Ρƒ: сопротивлСния, Π΄Π»ΠΈΠ½Ρ‹ ΠΈ ΠΏΠ»ΠΎΡ‰Π°Π΄ΠΈ. Π’ΠΎ Π΅ΡΡ‚ΡŒ Π² систСмС БИ получатся, Ρ‡Ρ‚ΠΎ Ссли R = 1 Ом, S = 1 ΠΌ2, Π° L = 1 ΠΌ, Ρ‚ΠΎ ρ = 1 .

Π­Ρ‚ΠΎ ΠΈ Π΅ΡΡ‚ΡŒ Π΅Π΄ΠΈΠ½ΠΈΡ†Π° измСрСния ΡƒΠ΄Π΅Π»ΡŒΠ½ΠΎΠ³ΠΎ сопротивлСния. Но Π½Π° ΠΏΡ€Π°ΠΊΡ‚ΠΈΠΊΠ΅ оказалось, Ρ‡Ρ‚ΠΎ Ρƒ Ρ€Π΅Π°Π»ΡŒΠ½Ρ‹Ρ… ΠΏΡ€ΠΎΠ²ΠΎΠ΄ΠΎΠ² ΠΏΠ»ΠΎΡ‰Π°Π΄ΠΈ сСчСний Π³ΠΎΡ€Π°Π·Π΄ΠΎ мСньшС 1 ΠΌ2. 2] \over [ΠΌ]} $$

Π’Π΅Π»ΠΈΡ‡ΠΈΠ½Ρƒ Ρ‚ΠΎΠΊΠ° ΠΈΠ·ΠΌΠ΅Ρ€ΡΡŽΡ‚ Π°ΠΌΠΏΠ΅Ρ€ΠΌΠ΅Ρ‚Ρ€ΠΎΠΌ, Π° Π²Π΅Π»ΠΈΡ‡ΠΈΠ½Ρƒ напряТСния β€” Π²ΠΎΠ»ΡŒΡ‚ΠΌΠ΅Ρ‚Ρ€ΠΎΠΌ. ΠŸΡ€ΠΈ ΠΏΡ€ΠΎΠ²Π΅Π΄Π΅Π½ΠΈΠΈ ΠΎΡ‡Π΅Π½ΡŒ Ρ‚ΠΎΡ‡Π½Ρ‹Ρ… ΠΈΠ·ΠΌΠ΅Ρ€Π΅Π½ΠΈΠΉ, Π½Π΅ΠΎΠ±Ρ…ΠΎΠ΄ΠΈΠΌΠΎ ΡƒΡ‡ΠΈΡ‚Ρ‹Π²Π°Ρ‚ΡŒ Π²Π½ΡƒΡ‚Ρ€Π΅Π½Π½Π΅Π΅ сопротивлСниС этих ΠΏΡ€ΠΈΠ±ΠΎΡ€ΠΎΠ².

Π§Ρ‚ΠΎ ΠΌΡ‹ ΡƒΠ·Π½Π°Π»ΠΈ?

Π˜Ρ‚Π°ΠΊ, ΠΌΡ‹ ΡƒΠ·Π½Π°Π»ΠΈ, Ρ‡Ρ‚ΠΎ Π·Π°Π²ΠΈΡΠΈΠΌΠΎΡΡ‚ΡŒ силы Ρ‚ΠΎΠΊΠ° Π² элСктричСской Ρ†Π΅ΠΏΠΈ описываСтся с ΠΏΠΎΠΌΠΎΡ‰ΡŒΡŽ Π·Π°ΠΊΠΎΠ½Π° Ома. Π‘ΠΈΠ»Π° Ρ‚ΠΎΠΊΠ° I прямо ΠΏΡ€ΠΎΠΏΠΎΡ€Ρ†ΠΈΠΎΠ½Π°Π»ΡŒΠ½Π° Π²Π΅Π»ΠΈΡ‡ΠΈΠ½Π΅ U напряТСния, ΠΈ ΠΎΠ±Ρ€Π°Ρ‚Π½ΠΎ ΠΏΡ€ΠΎΠΏΠΎΡ€Ρ†ΠΈΠΎΠ½Π°Π»ΡŒΠ½Π° ΡΠΎΠΏΡ€ΠΎΡ‚ΠΈΠ²Π»Π΅Π½ΠΈΡŽ R.

ВСст ΠΏΠΎ Ρ‚Π΅ΠΌΠ΅

Доска ΠΏΠΎΡ‡Ρ‘Ρ‚Π°

Π§Ρ‚ΠΎΠ±Ρ‹ ΠΏΠΎΠΏΠ°ΡΡ‚ΡŒ сюда — ΠΏΡ€ΠΎΠΉΠ΄ΠΈΡ‚Π΅ тСст.

  • Im Posable

    10/10

ΠžΡ†Π΅Π½ΠΊΠ° Π΄ΠΎΠΊΠ»Π°Π΄Π°

3.9

БрСдняя ΠΎΡ†Π΅Π½ΠΊΠ°: 3.9

ВсСго ΠΏΠΎΠ»ΡƒΡ‡Π΅Π½ΠΎ ΠΎΡ†Π΅Π½ΠΎΠΊ: 112.


А какая ваша ΠΎΡ†Π΅Π½ΠΊΠ°?

Π—Π°ΠΊΠΎΠ½

Ом: связь ΠΌΠ΅ΠΆΠ΄Ρƒ напряТСниСм, сопротивлСниСм, ΠΌΠΎΡ‰Π½ΠΎΡΡ‚ΡŒΡŽ ΠΈ Ρ‚ΠΎΠΊΠΎΠΌ | ДТСйсон Π₯ΠΎΠ»Π»

Π’ Π½Π°Ρ‡Π°Π»Π΅ 19 Π²Π΅ΠΊΠ° Π½Π΅ΠΌΠ΅Ρ†ΠΊΠΈΠΉ Ρ„ΠΈΠ·ΠΈΠΊ Π“Π΅ΠΎΡ€Π³ Ом ΠΎΠ±Π½Π°Ρ€ΡƒΠΆΠΈΠ», Ρ‡Ρ‚ΠΎ сила Ρ‚ΠΎΠΊΠ° Π² ΠΏΡ€ΠΎΠ²ΠΎΠ΄Π½ΠΈΠΊΠ΅ прямо ΠΏΡ€ΠΎΠΏΠΎΡ€Ρ†ΠΈΠΎΠ½Π°Π»ΡŒΠ½Π° Π½Π°ΠΏΡ€ΡΠΆΠ΅Π½ΠΈΡŽ Π½Π° ΠΏΡ€ΠΎΠ²ΠΎΠ΄Π½ΠΈΠΊΠ΅. Π­Ρ‚ΠΎ ΠΎΡ‚ΠΊΡ€Ρ‹Ρ‚ΠΈΠ΅ Π»Π΅Π³Π»ΠΎ Π² основу Ρ‚ΠΎΠ³ΠΎ, Ρ‡Ρ‚ΠΎ стало извСстно ΠΊΠ°ΠΊ Π·Π°ΠΊΠΎΠ½ Ома, ΠΊΠΎΡ‚ΠΎΡ€Ρ‹ΠΉ описываСт взаимосвязь ΠΌΠ΅ΠΆΠ΄Ρƒ напряТСниСм (V), Ρ‚ΠΎΠΊΠΎΠΌ (I) ΠΈ сопротивлСниСм (R):

Π—Π°ΠΊΠΎΠ½ Ома Π»Π΅Π³ΠΊΠΎ Π·Π°ΠΏΠΎΠΌΠ½ΠΈΡ‚ΡŒ, Ссли Π²Ρ‹ Π·Π°ΠΏΠΎΠΌΠ½ΠΈΡ‚Π΅ ΠΏΠΈΡ€Π°ΠΌΠΈΠ΄Ρƒ Π½ΠΈΠΆΠ΅. ΠŸΡ€ΠΎΡΡ‚ΠΎ Π·Π°ΠΊΡ€ΠΎΠΉΡ‚Π΅ Π·Π½Π°Ρ‡Π΅Π½ΠΈΠ΅, ΠΊΠΎΡ‚ΠΎΡ€ΠΎΠ΅ Π²Ρ‹ Ρ…ΠΎΡ‚ΠΈΡ‚Π΅ Π½Π°ΠΉΡ‚ΠΈ, ΠΈ Π±ΡƒΠ΄Π΅Ρ‚ ΠΏΠΎΠΊΠ°Π·Π°Π½ Ρ‚Ρ€Π΅Π±ΡƒΠ΅ΠΌΡ‹ΠΉ расчСт. ΠŸΡ€ΠΈΠΌΠ΅Ρ€Ρ‹: Ρ‡Ρ‚ΠΎΠ±Ρ‹ Π½Π°ΠΉΡ‚ΠΈ напряТСниС, Π·Π°ΠΊΡ€ΠΎΠΉΡ‚Π΅ V, ΠΈ ΠΎΡ‚Π²Π΅Ρ‚ Π±ΡƒΠ΄Π΅Ρ‚ I x R. Π§Ρ‚ΠΎΠ±Ρ‹ Π½Π°ΠΉΡ‚ΠΈ Ρ‚ΠΎΠΊ, Π·Π°ΠΊΡ€ΠΎΠΉΡ‚Π΅ I, ΠΈ ΠΎΡ‚Π²Π΅Ρ‚ Π±ΡƒΠ΄Π΅Ρ‚ V/R. Π§Ρ‚ΠΎΠ±Ρ‹ Π½Π°ΠΉΡ‚ΠΈ сопротивлСниС, Π·Π°ΠΊΡ€ΠΎΠΉΡ‚Π΅ R, ΠΈ ΠΎΡ‚Π²Π΅Ρ‚ Π±ΡƒΠ΄Π΅Ρ‚ V/I.

КолСсо Ρ„ΠΎΡ€ΠΌΡƒΠ» Π·Π°ΠΊΠΎΠ½Π° Ома ΠΏΠΎΠΊΠ°Π·Ρ‹Π²Π°Π΅Ρ‚ расчСты, Π½Π΅ΠΎΠ±Ρ…ΠΎΠ΄ΠΈΠΌΡ‹Π΅ для опрСдСлСния напряТСния, Ρ‚ΠΎΠΊΠ°, сопротивлСния, Π° Ρ‚Π°ΠΊΠΆΠ΅ мощности. ΠœΠΎΡ‰Π½ΠΎΡΡ‚ΡŒ – это ΡΠΊΠΎΡ€ΠΎΡΡ‚ΡŒ, с ΠΊΠΎΡ‚ΠΎΡ€ΠΎΠΉ элСктричСская энСргия пСрСдаСтся ΠΏΠΎ элСктричСской Ρ†Π΅ΠΏΠΈ.

Π§Ρ‚ΠΎΠ±Ρ‹ ΠΈΡΠΏΠΎΠ»ΡŒΠ·ΠΎΠ²Π°Ρ‚ΡŒ колСсо Ρ„ΠΎΡ€ΠΌΡƒΠ», Π²Π°ΠΌ Π½ΡƒΠΆΠ½ΠΎ Π·Π½Π°Ρ‚ΡŒ Ρ‚ΠΎΠ»ΡŒΠΊΠΎ Π΄Π²Π΅ ΠΏΠ΅Ρ€Π΅ΠΌΠ΅Π½Π½Ρ‹Π΅, Ρ‡Ρ‚ΠΎΠ±Ρ‹ Π½Π°ΠΉΡ‚ΠΈ Ρ‚Ρ€Π΅Ρ‚ΡŒΡŽ. НапримСр, Ссли Π²Ρ‹ Π·Π½Π°Π΅Ρ‚Π΅ Ρ‚ΠΎΠΊ ΠΈ сопротивлСниС Π² Ρ†Π΅ΠΏΠΈ, Π²Ρ‹ ΠΌΠΎΠΆΠ΅Ρ‚Π΅ Ρ€Π°ΡΡΡ‡ΠΈΡ‚Π°Ρ‚ΡŒ ΠΌΠΎΡ‰Π½ΠΎΡΡ‚ΡŒ, ΠΏΡ€ΠΎΠΈΠ·Π²ΠΎΠ΄ΠΈΠΌΡƒΡŽ ΠΏΡƒΡ‚Π΅ΠΌ возвСдСния Ρ‚ΠΎΠΊΠ° Π² ΠΊΠ²Π°Π΄Ρ€Π°Ρ‚ ΠΈ умноТСния Π΅Π³ΠΎ Π½Π° сопротивлСниС: IΒ² x R.

ЭлСктродвиТущая сила, ΠΈΠ»ΠΈ Π­Π”Π‘, ΠΌΠΎΠΆΠ΅Ρ‚ ΡΡ‡ΠΈΡ‚Π°Ρ‚ΡŒΡΡ Π΄Π°Π²Π»Π΅Π½ΠΈΠ΅ΠΌ, ΠΊΠΎΡ‚ΠΎΡ€ΠΎΠ΅ заставляСт элСктроны Ρ‚Π΅Ρ‡ΡŒ Π² элСктричСской Ρ†Π΅ΠΏΠΈ. Π­Π”Π‘ ΠΌΠΎΠΆΠ΅Ρ‚ ΡƒΠΏΠΎΠΌΠΈΠ½Π°Ρ‚ΡŒΡΡ ΠΊΠ°ΠΊ напряТСниС, Ρ€Π°Π·Π½ΠΎΡΡ‚ΡŒ элСктричСских ΠΏΠΎΡ‚Π΅Π½Ρ†ΠΈΠ°Π»ΠΎΠ², элСктричСскоС Π΄Π°Π²Π»Π΅Π½ΠΈΠ΅ ΠΈΠ»ΠΈ элСктричСскоС напряТСниС. Π’ ΠœΠ΅ΠΆΠ΄ΡƒΠ½Π°Ρ€ΠΎΠ΄Π½ΠΎΠΉ систСмС Π΅Π΄ΠΈΠ½ΠΈΡ† Π­Π”Π‘ измСряСтся Π² Π²ΠΎΠ»ΡŒΡ‚Π°Ρ… ( Π’ ).

ЭлСктричСскоС сопротивлСниС ( R ) ΠΌΠ°Ρ‚Π΅Ρ€ΠΈΠ°Π»Π° являСтся ΠΌΠ΅Ρ€ΠΎΠΉ Π΅Π³ΠΎ сопротивлСния ΠΏΡ€ΠΎΡ‚Π΅ΠΊΠ°Π½ΠΈΡŽ Ρ‚ΠΎΠΊΠ°. Π•Π΄ΠΈΠ½ΠΈΡ†Π΅ΠΉ сопротивлСния Π² систСмС БИ являСтся ΠΎΠΌ. Π₯отя всС ΠΌΠ°Ρ‚Π΅Ρ€ΠΈΠ°Π»Ρ‹ ΡΠΎΠΏΡ€ΠΎΡ‚ΠΈΠ²Π»ΡΡŽΡ‚ΡΡ элСктричСскому Ρ‚ΠΎΠΊΡƒ, Π½Π΅ΠΊΠΎΡ‚ΠΎΡ€Ρ‹Π΅ ΠΌΠ°Ρ‚Π΅Ρ€ΠΈΠ°Π»Ρ‹ ΠΈΠΌΠ΅ΡŽΡ‚ ΠΎΡ‡Π΅Π½ΡŒ Π½ΠΈΠ·ΠΊΠΎΠ΅ сопротивлСниС (ΠΏΡ€ΠΎΠ²ΠΎΠ΄Π½ΠΈΠΊΠΈ), Π° Π΄Ρ€ΡƒΠ³ΠΈΠ΅ ΠΈΠΌΠ΅ΡŽΡ‚ ΠΎΡ‡Π΅Π½ΡŒ высокоС сопротивлСниС (изоляторы).

Π€Π°ΠΊΡ‚ΠΎΡ€Π°ΠΌΠΈ, Π²Π»ΠΈΡΡŽΡ‰ΠΈΠΌΠΈ Π½Π° сопротивлСниС ΠΌΠ°Ρ‚Π΅Ρ€ΠΈΠ°Π»Π°, ΡΠ²Π»ΡΡŽΡ‚ΡΡ Π΅Π³ΠΎ ΡƒΠ΄Π΅Π»ΡŒΠ½ΠΎΠ΅ сопротивлСниС, Π΅Π³ΠΎ ΡˆΠΈΡ€ΠΈΠ½Π°, Π΄Π»ΠΈΠ½Π° ΠΈ Ρ‚Π΅ΠΌΠΏΠ΅Ρ€Π°Ρ‚ΡƒΡ€Π°. ΠšΠ°ΠΆΠ΄Ρ‹ΠΉ ΠΌΠ°Ρ‚Π΅Ρ€ΠΈΠ°Π» ΠΈΠΌΠ΅Π΅Ρ‚ ΠΎΠΏΡ€Π΅Π΄Π΅Π»Π΅Π½Π½ΠΎΠ΅ ΡƒΠ΄Π΅Π»ΡŒΠ½ΠΎΠ΅ сопротивлСниС, ΠΊΠΎΡ‚ΠΎΡ€ΠΎΠ΅ являСтся ΠΌΠ΅Ρ€ΠΎΠΉ Ρ‚ΠΎΠ³ΠΎ, насколько сильно ΠΎΠ½ сопротивляСтся ΠΈΠ»ΠΈ ΠΏΡ€ΠΎΠ²ΠΎΠ΄ΠΈΡ‚ элСктричСский Ρ‚ΠΎΠΊ. Π¨ΠΈΡ€ΠΎΠΊΠΈΠΉ ΠΏΡ€ΠΎΠ²ΠΎΠ΄Π½ΠΈΠΊ ΠΈΠΌΠ΅Π΅Ρ‚ мСньшСС сопротивлСниС, Ρ‡Π΅ΠΌ ΡƒΠ·ΠΊΠΈΠΉ ΠΏΡ€ΠΎΠ²ΠΎΠ΄Π½ΠΈΠΊ, Π° Π΄Π»ΠΈΠ½Π½Ρ‹ΠΉ ΠΏΡ€ΠΎΠ²ΠΎΠ΄Π½ΠΈΠΊ ΠΈΠΌΠ΅Π΅Ρ‚ большСС сопротивлСниС, Ρ‡Π΅ΠΌ Π±ΠΎΠ»Π΅Π΅ ΠΊΠΎΡ€ΠΎΡ‚ΠΊΠΈΠΉ (см. Π½ΠΈΠΆΠ΅ аналогию с Π²ΠΎΠ΄ΠΎΠΉ). Π₯ΠΎΠ»ΠΎΠ΄Π½Ρ‹ΠΉ ΠΏΡ€ΠΎΠ²ΠΎΠ΄Π½ΠΈΠΊ ΠΈΠΌΠ΅Π΅Ρ‚ мСньшСС сопротивлСниС, Ρ‡Π΅ΠΌ Ρ‚Π΅ΠΏΠ»Ρ‹ΠΉ, ΠΈΠ·-Π·Π° влияния Ρ‚Π΅ΠΌΠΏΠ΅Ρ€Π°Ρ‚ΡƒΡ€Ρ‹ Π½Π° элСктроны Π²Π½ΡƒΡ‚Ρ€ΠΈ ΠΏΡ€ΠΎΠ²ΠΎΠ΄Π½ΠΈΠΊΠ°: Π² Ρ…ΠΎΠ»ΠΎΠ΄Π½ΠΎΠΉ срСдС элСктроны мСньшС ΡΡ‚Π°Π»ΠΊΠΈΠ²Π°ΡŽΡ‚ΡΡ Π΄Ρ€ΡƒΠ³ с Π΄Ρ€ΡƒΠ³ΠΎΠΌ, Ρ‡Π΅ΠΌ ΠΏΡ€ΠΈ Π±ΠΎΠ»Π΅Π΅ высокой Ρ‚Π΅ΠΌΠΏΠ΅Ρ€Π°Ρ‚ΡƒΡ€Π΅, Ρ‡Ρ‚ΠΎ позволяСт ΠΈΠΌ ΠΏΠ΅Ρ€Π΅ΠΌΠ΅Ρ‰Π°Ρ‚ΡŒΡΡ ΠΎΡ‚ Π°Ρ‚ΠΎΠΌΠ° ΠΊ Π°Ρ‚ΠΎΠΌ, нСсущий свой заряд Π±ΠΎΠ»Π΅Π΅ эффСктивно.

ЭлСктричСская ΠΌΠΎΡ‰Π½ΠΎΡΡ‚ΡŒ ( P ) β€” ΡΠΊΠΎΡ€ΠΎΡΡ‚ΡŒ ΠΏΠ΅Ρ€Π΅Π΄Π°Ρ‡ΠΈ элСктричСской энСргии ΠΏΠΎ элСктричСской Ρ†Π΅ΠΏΠΈ. Π”Ρ€ΡƒΠ³ΠΎΠΉ способ Π΄ΡƒΠΌΠ°Ρ‚ΡŒ ΠΎΠ± элСктроэнСргии состоит Π² Ρ‚ΠΎΠΌ, Ρ‡Ρ‚ΠΎ это ΡΠΊΠΎΡ€ΠΎΡΡ‚ΡŒ выполнСния Ρ€Π°Π±ΠΎΡ‚Ρ‹, измСряСмая Π² Π²Π°Ρ‚Ρ‚Π°Ρ…. Один Π²Π°Ρ‚Ρ‚ β€” это ΡΠΊΠΎΡ€ΠΎΡΡ‚ΡŒ, с ΠΊΠΎΡ‚ΠΎΡ€ΠΎΠΉ ΡΠΎΠ²Π΅Ρ€ΡˆΠ°Π΅Ρ‚ΡΡ Ρ€Π°Π±ΠΎΡ‚Π°, ΠΊΠΎΠ³Π΄Π° Ρ‚ΠΎΠΊ Π² ΠΎΠ΄ΠΈΠ½ Π°ΠΌΠΏΠ΅Ρ€ ΠΏΡ€ΠΎΡ‚Π΅ΠΊΠ°Π΅Ρ‚ ΠΏΠΎ Ρ†Π΅ΠΏΠΈ с элСктричСским ΠΏΠΎΡ‚Π΅Π½Ρ†ΠΈΠ°Π»ΠΎΠΌ (напряТСниСм) Π² ΠΎΠ΄ΠΈΠ½ Π²ΠΎΠ»ΡŒΡ‚. ΠœΠΎΡ‰Π½ΠΎΡΡ‚ΡŒ ΠΎΠ±Ρ‹Ρ‡Π½ΠΎ измСряСтся Π² ΠΊΠΈΠ»ΠΎΠ²Π°Ρ‚Ρ‚Π°Ρ… Π² час ΠΈΠ»ΠΈ ΠΊΠΈΠ»ΠΎΠ²Π°Ρ‚Ρ‚-часах (ΠΊΠ’Ρ‚Ρ‡).

ЭлСктричСский Ρ‚ΠΎΠΊ ( I ) ΡΠΊΠΎΡ€ΠΎΡΡ‚ΡŒ ΠΏΠΎΡ‚ΠΎΠΊΠ° элСктричСского заряда. Π•Π΄ΠΈΠ½ΠΈΡ†Π΅ΠΉ силы Ρ‚ΠΎΠΊΠ° Π² систСмС БИ являСтся Π°ΠΌΠΏΠ΅Ρ€ ΠΈΠ»ΠΈ Π°ΠΌΠΏΠ΅Ρ€. АмпСр опрСдСляСтся ΠΊΠ°ΠΊ ΠΏΠΎΡ‚ΠΎΠΊ элСктричСского заряда Π² ΠΎΠ΄ΠΈΠ½ ΠΊΡƒΠ»ΠΎΠ½ Π² сСкунду, Π³Π΄Π΅ ΠΊΡƒΠ»ΠΎΠ½ являСтся Π΅Π΄ΠΈΠ½ΠΈΡ†Π΅ΠΉ элСктричСского заряда Π² систСмС БИ.

Как Π±Ρ‹Π»ΠΎ сказано Ρ€Π°Π½Π΅Π΅, элСктричСский Ρ‚ΠΎΠΊ β€” это ΡΠΊΠΎΡ€ΠΎΡΡ‚ΡŒ ΠΏΠΎΡ‚ΠΎΠΊΠ° заряда. Заряд β€” это Ρ„ΡƒΠ½Π΄Π°ΠΌΠ΅Π½Ρ‚Π°Π»ΡŒΠ½ΠΎΠ΅ свойство элСктричСства, ΠΏΡ€ΠΈΡΡƒΡ‚ΡΡ‚Π²ΡƒΡŽΡ‰Π΅Π΅ Π² элСктронах ΠΈ ΠΏΡ€ΠΎΡ‚ΠΎΠ½Π°Ρ…. Из ΡƒΡ€ΠΎΠΊΠΎΠ² Ρ„ΠΈΠ·ΠΈΠΊΠΈ ΠΌΡ‹ Π·Π½Π°Π΅ΠΌ, Ρ‡Ρ‚ΠΎ ΠΏΡ€ΠΎΡ‚ΠΎΠ½Ρ‹ заряТСны ΠΏΠΎΠ»ΠΎΠΆΠΈΡ‚Π΅Π»ΡŒΠ½ΠΎ, Π° элСктроны β€” ΠΎΡ‚Ρ€ΠΈΡ†Π°Ρ‚Π΅Π»ΡŒΠ½ΠΎ. Когда эти частицы двиТутся, Π²ΠΎΠ·Π½ΠΈΠΊΠ°Π΅Ρ‚ элСктричСский Ρ‚ΠΎΠΊ.

Π§Ρ‚ΠΎΠ±Ρ‹ Π»ΡƒΡ‡ΡˆΠ΅ ΠΏΠΎΠ½ΡΡ‚ΡŒ Ρ€Π°Π·Π½ΠΈΡ†Ρƒ ΠΌΠ΅ΠΆΠ΄Ρƒ напряТСниСм, сопротивлСниСм, Ρ‚ΠΎΠΊΠΎΠΌ ΠΈ ΠΌΠΎΡ‰Π½ΠΎΡΡ‚ΡŒΡŽ, ΠΏΠΎΠ»Π΅Π·Π½ΠΎ ΠΏΡ€Π΅Π΄ΡΡ‚Π°Π²ΠΈΡ‚ΡŒ сСбС элСктричСство ΠΊΠ°ΠΊ Π²ΠΎΠ΄Ρƒ, Ρ‚Π΅ΠΊΡƒΡ‰ΡƒΡŽ ΠΏΠΎ Ρ‚Ρ€ΡƒΠ±Π΅. НапряТСниС ΠΏΠΎΡ…ΠΎΠΆΠ΅ Π½Π° Π΄Π°Π²Π»Π΅Π½ΠΈΠ΅, ΠΊΠΎΡ‚ΠΎΡ€ΠΎΠ΅ Π³ΠΎΠ½ΠΈΡ‚ Π²ΠΎΠ΄Ρƒ ΠΏΠΎ Ρ‚Ρ€ΡƒΠ±Π΅, Π² Ρ‚ΠΎ врСмя ΠΊΠ°ΠΊ сопротивлСниС ΠΌΠΎΠΆΠ½ΠΎ Ρ€Π°ΡΡΠΌΠ°Ρ‚Ρ€ΠΈΠ²Π°Ρ‚ΡŒ ΠΊΠ°ΠΊ ΠΎΠ³Ρ€Π°Π½ΠΈΡ‡Π΅Π½ΠΈΠ΅ ΠΏΠΎΡ‚ΠΎΠΊΠ° Π²ΠΎΠ΄Ρ‹ Π²Π½ΡƒΡ‚Ρ€ΠΈ Ρ‚Ρ€ΡƒΠ±Ρ‹ ΠΈΠ·-Π·Π° Ρ‚Π°ΠΊΠΈΡ… Ρ„Π°ΠΊΡ‚ΠΎΡ€ΠΎΠ², ΠΊΠ°ΠΊ ΡˆΠΈΡ€ΠΈΠ½Π° Ρ‚Ρ€ΡƒΠ±Ρ‹ ΠΈ Π΄Π»ΠΈΠ½Π° Ρ‚Ρ€ΡƒΠ±Ρ‹. Узкая Ρ‚Ρ€ΡƒΠ±Π° создаСт большСС сопротивлСниС, Ρ‡Π΅ΠΌ Π±ΠΎΠ»Π΅Π΅ ΡˆΠΈΡ€ΠΎΠΊΠ°Ρ Ρ‚Ρ€ΡƒΠ±Π°, Π° Π±ΠΎΠ»Π΅Π΅ длинная Ρ‚Ρ€ΡƒΠ±Π° создаСт большСС сопротивлСниС, Ρ‡Π΅ΠΌ Π±ΠΎΠ»Π΅Π΅ короткая. Π’ΠΎΠΊ β€” это расход Π²ΠΎΠ΄Ρ‹, Π³Π΄Π΅ ΠΌΠΎΡ‰Π½ΠΎΡΡ‚ΡŒ ΠΌΠΎΠΆΠ½ΠΎ Ρ€Π°ΡΡΠΌΠ°Ρ‚Ρ€ΠΈΠ²Π°Ρ‚ΡŒ ΠΊΠ°ΠΊ Ρ€Π°Π±ΠΎΡ‚Ρƒ, ΠΊΠΎΡ‚ΠΎΡ€ΡƒΡŽ ΠΌΠΎΠΆΠ΅Ρ‚ Π²Ρ‹ΠΏΠΎΠ»Π½ΡΡ‚ΡŒ ΠΏΠΎΡ‚ΠΎΠΊ Π²ΠΎΠ΄Ρ‹, Π½Π°ΠΏΡ€ΠΈΠΌΠ΅Ρ€, Π²Ρ€Π°Ρ‰Π΅Π½ΠΈΠ΅ ΠΊΡ€Ρ‹Π»ΡŒΡ‡Π°Ρ‚ΠΊΠΈ Π² насосС.

  • Π—Π°ΠΊΠΎΠ½ Ома, V = IR описываСт взаимосвязь ΠΌΠ΅ΠΆΠ΄Ρƒ напряТСниСм, Ρ‚ΠΎΠΊΠΎΠΌ ΠΈ сопротивлСниСм.
  • НапряТСниС Ρ‚Π°ΠΊΠΆΠ΅ извСстно ΠΊΠ°ΠΊ Ρ€Π°Π·Π½ΠΎΡΡ‚ΡŒ элСктричСских ΠΏΠΎΡ‚Π΅Π½Ρ†ΠΈΠ°Π»ΠΎΠ² (Π­Π”Π‘), элСктричСскоС Π΄Π°Π²Π»Π΅Π½ΠΈΠ΅ ΠΈΠ»ΠΈ элСктричСскоС напряТСниС ΠΈ измСряСтся Π² Π²ΠΎΠ»ΡŒΡ‚Π°Ρ….
  • Π‘ΠΎΠΏΡ€ΠΎΡ‚ΠΈΠ²Π»Π΅Π½ΠΈΠ΅ являСтся ΠΌΠ΅Ρ€ΠΎΠΉ сопротивлСния ΠΌΠ°Ρ‚Π΅Ρ€ΠΈΠ°Π»ΠΎΠ² ΠΏΡ€ΠΎΡ‚Π΅ΠΊΠ°Π½ΠΈΡŽ Ρ‚ΠΎΠΊΠ° ΠΈ измСряСтся Π² ΠΎΠΌΠ°Ρ…. На сопротивлСниС влияСт ΡƒΠ΄Π΅Π»ΡŒΠ½ΠΎΠ΅ сопротивлСниС ΠΌΠ°Ρ‚Π΅Ρ€ΠΈΠ°Π»Π°, Π΅Π³ΠΎ Ρ„ΠΎΡ€ΠΌΠ° ΠΈ Ρ‚Π΅ΠΌΠΏΠ΅Ρ€Π°Ρ‚ΡƒΡ€Π°.
  • ЭлСктричСская ΠΌΠΎΡ‰Π½ΠΎΡΡ‚ΡŒ β€” это ΡΠΊΠΎΡ€ΠΎΡΡ‚ΡŒ, с ΠΊΠΎΡ‚ΠΎΡ€ΠΎΠΉ элСктричСская энСргия пСрСдаСтся ΠΏΠΎ Ρ†Π΅ΠΏΠΈ, ΠΈ измСряСтся Π² Π²Π°Ρ‚Ρ‚Π°Ρ…. Π•Π³ΠΎ Ρ‚Π°ΠΊΠΆΠ΅ ΠΌΠΎΠΆΠ½ΠΎ Ρ€Π°ΡΡΠΌΠ°Ρ‚Ρ€ΠΈΠ²Π°Ρ‚ΡŒ ΠΊΠ°ΠΊ ΡΠΊΠΎΡ€ΠΎΡΡ‚ΡŒ выполнСния Ρ€Π°Π±ΠΎΡ‚Ρ‹.
  • Π’ΠΎΠΊ β€” это ΡΠΊΠΎΡ€ΠΎΡΡ‚ΡŒ протСкания элСктричСского заряда, измСряСмая Π² Π°ΠΌΠΏΠ΅Ρ€Π°Ρ….
  • ЭлСктричСский заряд пСрСносится ΠΎΡ‚Ρ€ΠΈΡ†Π°Ρ‚Π΅Π»ΡŒΠ½ΠΎ заряТСнными элСктронами ΠΈ ΠΏΠΎΠ»ΠΎΠΆΠΈΡ‚Π΅Π»ΡŒΠ½ΠΎ заряТСнными ΠΏΡ€ΠΎΡ‚ΠΎΠ½Π°ΠΌΠΈ Π²Π½ΡƒΡ‚Ρ€ΠΈ Π°Ρ‚ΠΎΠΌΠ°.
  • ПовСдСниС элСктричСства Π°Π½Π°Π»ΠΎΠ³ΠΈΡ‡Π½ΠΎ повСдСнию Π²ΠΎΠ΄Ρ‹ Π² Ρ‚Ρ€ΡƒΠ±Π΅.

Руководство для Π½ΠΎΠ²ΠΈΡ‡ΠΊΠΎΠ² ΠΏΠΎ Π½Π°ΠΏΡ€ΡΠΆΠ΅Π½ΠΈΡŽ, Ρ‚ΠΎΠΊΡƒ ΠΈ ΡΠΎΠΏΡ€ΠΎΡ‚ΠΈΠ²Π»Π΅Π½ΠΈΡŽ

Π₯ΠΎΡ‚ΠΈΡ‚Π΅ ΡƒΠ·Π½Π°Ρ‚ΡŒ большС ΠΎΠ± элСктричСствС? Π§Ρ‚ΠΎ ΠΆ, Ρ‚ΠΎΠ³Π΄Π° Π²Π°ΠΌ Π½ΡƒΠΆΠ½ΠΎ Π±ΡƒΠ΄Π΅Ρ‚ ΡƒΠ·Π½Π°Ρ‚ΡŒ ΠΎ напряТСнии, Ρ‚ΠΎΠΊΠ΅ ΠΈ сопротивлСнии, Π° Ρ‚Π°ΠΊΠΆΠ΅ ΠΎ Ρ€Π°Π·Π»ΠΈΡ‡Π½Ρ‹Ρ… ролях, ΠΊΠΎΡ‚ΠΎΡ€Ρ‹Π΅ ΠΎΠ½ΠΈ ΠΈΠ³Ρ€Π°ΡŽΡ‚ Π² элСктричСской Ρ†Π΅ΠΏΠΈ.

  TL;DR 
НапряТСниС β€” это ΠΈΠ·ΠΌΠ΅Ρ€Π΅Π½ΠΈΠ΅ «давлСния» Π² элСктричСской систСмС, измСряСмоС Π² Π²ΠΎΠ»ΡŒΡ‚Π°Ρ….
Π’ΠΎΠΊ β€” это ΡΠΊΠΎΡ€ΠΎΡΡ‚ΡŒ прохоТдСния элСктронов Ρ‡Π΅Ρ€Π΅Π· Ρ‚ΠΎΡ‡ΠΊΡƒ, измСряСмая Π² Π°ΠΌΠΏΠ΅Ρ€Π°Ρ…. 
Π‘ΠΎΠΏΡ€ΠΎΡ‚ΠΈΠ²Π»Π΅Π½ΠΈΠ΅ β€” это ΠΌΠ΅Ρ€Π° противодСйствия ΠΏΡ€ΠΎΡ‚Π΅ΠΊΠ°Π½ΠΈΡŽ Ρ‚ΠΎΠΊΠ° Π² Ρ†Π΅ΠΏΠΈ. 

Π­Ρ‚ΠΈ ΠΊΠΎΠ½Ρ†Π΅ΠΏΡ†ΠΈΠΈ ΠΌΠΎΠ³ΡƒΡ‚ Π±Ρ‹Ρ‚ΡŒ ΠΎΡ‡Π΅Π½ΡŒ слоТными для понимания, ΠΏΠΎΡΠΊΠΎΠ»ΡŒΠΊΡƒ напряТСниС, Ρ‚ΠΎΠΊ ΠΈ сопротивлСниС Π½Π΅ Π²ΠΈΠ΄Π½Ρ‹ чСловСчСскому Π³Π»Π°Π·Ρƒ, Π½ΠΎ это скорСС тСорСтичСскиС ΠΊΠΎΠ½Ρ†Π΅ΠΏΡ†ΠΈΠΈ, ΠΊΠΎΡ‚ΠΎΡ€Ρ‹Π΅ ΠΌΡ‹ Π΄ΠΎΠ»ΠΆΠ½Ρ‹ ΠΏΠΎΠΏΡ‹Ρ‚Π°Ρ‚ΡŒΡΡ Π²ΠΈΠ·ΡƒΠ°Π»ΠΈΠ·ΠΈΡ€ΠΎΠ²Π°Ρ‚ΡŒ.

Но всС становится ΠΏΡ€ΠΎΡ‰Π΅, ΠΊΠΎΠ³Π΄Π° Π²Ρ‹ Π½Π°Ρ‡ΠΈΠ½Π°Π΅Ρ‚Π΅ ΠΏΠΎΠ½ΠΈΠΌΠ°Ρ‚ΡŒ, Ρ‡Ρ‚ΠΎ Ρ‚Π°ΠΊΠΎΠ΅ элСктричСство. ΠŸΡ€ΠΎΡ‰Π΅ говоря, элСктричСство β€” это Π΄Π²ΠΈΠΆΠ΅Π½ΠΈΠ΅ элСктронов, Π° ΠΏΠΎΡΠΊΠΎΠ»ΡŒΠΊΡƒ двиТущиСся элСктроны ΡΠΎΠ·Π΄Π°ΡŽΡ‚ заряд, Π΄Π²ΠΈΠΆΠ΅Π½ΠΈΠ΅ элСктронов ΠΌΠΎΠΆΠ½ΠΎ ΠΈΡΠΏΠΎΠ»ΡŒΠ·ΠΎΠ²Π°Ρ‚ΡŒ для выполнСния Ρ€Π°Π±ΠΎΡ‚Ρ‹.

ΠŸΡ€ΠΎΡΡ‚Ρ‹Π΅ устройства, Ρ‚Π°ΠΊΠΈΠ΅ ΠΊΠ°ΠΊ Π»Π°ΠΌΠΏΠΎΡ‡ΠΊΠΈ, ΠΈ слоТная элСктроника, такая ΠΊΠ°ΠΊ ΠΊΠΎΠΌΠΏΡŒΡŽΡ‚Π΅Ρ€Ρ‹ ΠΈ Ρ‚Π΅Π»Π΅Ρ„ΠΎΠ½Ρ‹, Ρ„ΡƒΠ½ΠΊΡ†ΠΈΠΎΠ½ΠΈΡ€ΡƒΡŽΡ‚ благодаря двиТСнию элСктронов.

НапряТСниС, Ρ‚ΠΎΠΊ ΠΈ сопротивлСниС ΠΌΠΎΠΆΠ½ΠΎ ΠΎΠΏΠΈΡΠ°Ρ‚ΡŒ Π² Ρ‚Π΅Ρ€ΠΌΠΈΠ½Π°Ρ… двиТСния элСктронов. НачнСм с понимания Ρ‚ΠΎΠ³ΠΎ, Ρ‡Ρ‚ΠΎ Ρ‚Π°ΠΊΠΎΠ΅ напряТСниС.

1 Π Π°Π·Π½ΠΈΡ†Π° ΠΈ сравнСниС: напряТСниС ΠΈ Ρ‚ΠΎΠΊ

2 ПослСдниС мысли

3 ΠžΡΡ‚Π°Π»ΠΈΡΡŒ вопросы?

Π Π°Π·Π½ΠΈΡ†Π° ΠΈ сравнСниС : НапряТСниС ΠΈ Ρ‚ΠΎΠΊ

ΠšΠΎΡ€ΠΎΡ‡Π΅ говоря, напряТСниС β€” это сила «давлСния» Π² Ρ†Π΅ΠΏΠΈ. Π’ΠΎΡ‡Π½Π΅Π΅, это ΠΈΠ·ΠΌΠ΅Ρ€Π΅Π½ΠΈΠ΅ разности ΠΏΠΎΡ‚Π΅Π½Ρ†ΠΈΠ°Π»ΠΎΠ² энСргии ΠΌΠ΅ΠΆΠ΄Ρƒ двумя Ρ‚ΠΎΡ‡ΠΊΠ°ΠΌΠΈ. Π’ΠΎΠΊ β€” это ΠΈΠ·ΠΌΠ΅Ρ€Π΅Π½ΠΈΠ΅ количСства элСктронов, ΠΏΡ€ΠΎΡ‚Π΅ΠΊΠ°ΡŽΡ‰ΠΈΡ… Ρ‡Π΅Ρ€Π΅Π· Ρ‚ΠΎΡ‡ΠΊΡƒ Ρ†Π΅ΠΏΠΈ Π² сСкунду.

Π§Ρ‚ΠΎ Ρ‚Π°ΠΊΠΎΠ΅ напряТСниС?

ПониманиС опрСдСлСния напряТСния β€” это ΠΏΠ΅Ρ€Π²Ρ‹ΠΉ шаг ΠΊ пониманию Ρ‚ΠΎΠ³ΠΎ, ΠΊΠ°ΠΊ Ρ€Π°Π±ΠΎΡ‚Π°ΡŽΡ‚ элСктроприборы. Π’ любой Ρ†Π΅ΠΏΠΈ элСктроны двиТутся ΠΎΡ‚ ΠΎΡ‚Ρ€ΠΈΡ†Π°Ρ‚Π΅Π»ΡŒΠ½ΠΎΠ³ΠΎ ΠΊΠΎΠ½Ρ†Π° ΠΊ ΠΏΠΎΠ»ΠΎΠΆΠΈΡ‚Π΅Π»ΡŒΠ½ΠΎΠΌΡƒ ΠΊΠΎΠ½Ρ†Ρƒ источника.

НапряТСниС β€” это Π΄Π°Π²Π»Π΅Π½ΠΈΠ΅, с ΠΊΠΎΡ‚ΠΎΡ€Ρ‹ΠΌ источник питания ΠΏΡ€ΠΎΡ‚Π°Π»ΠΊΠΈΠ²Π°Π΅Ρ‚ элСктроны ΠΏΠΎ Ρ†Π΅ΠΏΠΈ.

Π˜Π·ΠΌΠ΅Ρ€ΡΠ΅Ρ‚ΡΡ Π² Π²ΠΎΠ»ΡŒΡ‚Π°Ρ… (Π΅Π΄ΠΈΠ½ΠΈΡ†Π° БИ Β«Π’Β») ΠΊΠ°ΠΊ Ρ€Π°Π·Π½ΠΈΡ†Π° заряда ΠΌΠ΅ΠΆΠ΄Ρƒ двумя Ρ‚ΠΎΡ‡ΠΊΠ°ΠΌΠΈ Ρ†Π΅ΠΏΠΈ. На Π·Π°Ρ€Π΅ элСктричСства напряТСниС Π½Π°Π·Ρ‹Π²Π°Π»ΠΎΡΡŒ элСктродвиТущСй силой (Π­Π”Π‘), поэтому напряТСниС прСдставлСно Π±ΡƒΠΊΠ²ΠΎΠΉ Β«Π•Β» Π² Ρ‚Π°ΠΊΠΈΡ… уравнСниях, ΠΊΠ°ΠΊ Π·Π°ΠΊΠΎΠ½ Ома.

НапряТСниС Π½Π°Π·Π²Π°Π½ΠΎ Π² Ρ‡Π΅ΡΡ‚ΡŒ АлСссандро Π’ΠΎΠ»ΡŒΡ‚Π°, ΠΈΡ‚Π°Π»ΡŒΡΠ½ΡΠΊΠΎΠ³ΠΎ Ρ„ΠΈΠ·ΠΈΠΊΠ°, ΠΊΠΎΡ‚ΠΎΡ€Ρ‹ΠΉ ΠΈΠ·ΠΎΠ±Ρ€Π΅Π» Π³Π°Π»ΡŒΠ²Π°Π½ΠΈΡ‡Π΅ΡΠΊΡƒΡŽ Π±Π°Ρ‚Π°Ρ€Π΅ΡŽ, которая ΠΏΡ€Π΅Π²Ρ€Π°Ρ‚ΠΈΠ»Π°ΡΡŒ Π² ΡΠΎΠ²Ρ€Π΅ΠΌΠ΅Π½Π½ΡƒΡŽ Π±Ρ‹Ρ‚ΠΎΠ²ΡƒΡŽ Π±Π°Ρ‚Π°Ρ€Π΅ΡŽ.

НапряТСниС ΠΈΡΠΏΠΎΠ»ΡŒΠ·ΡƒΠ΅Ρ‚ΡΡ взаимозамСняСмо с Ρ‚Π΅Ρ€ΠΌΠΈΠ½ΠΎΠΌ Β«Ρ€Π°Π·Π½ΠΎΡΡ‚ΡŒ ΠΏΠΎΡ‚Π΅Π½Ρ†ΠΈΠ°Π»ΠΎΠ²Β», ΠΊΠΎΡ‚ΠΎΡ€Ρ‹ΠΉ опрСдСляСтся ΠΊΠ°ΠΊ Ρ€Π°Π·Π½ΠΎΡΡ‚ΡŒ ΠΏΠΎΡ‚Π΅Π½Ρ†ΠΈΠ°Π»ΡŒΠ½ΠΎΠΉ энСргии ΠΌΠ΅ΠΆΠ΄Ρƒ двумя Ρ‚ΠΎΡ‡ΠΊΠ°ΠΌΠΈ. Π”Ρ€ΡƒΠ³ΠΈΠΌΠΈ словами, напряТСниС рассматриваСтся ΠΊΠ°ΠΊ ΠΌΠ΅Ρ€Π° элСктричСского ΠΏΠΎΡ‚Π΅Π½Ρ†ΠΈΠ°Π»Π° ΠΌΠ΅ΠΆΠ΄Ρƒ двумя Ρ‚ΠΎΡ‡ΠΊΠ°ΠΌΠΈ Ρ†Π΅ΠΏΠΈ.

Π’Π°ΠΆΠ½ΠΎ ΠΏΠΎΠΌΠ½ΠΈΡ‚ΡŒ, Ρ‡Ρ‚ΠΎ Ρ€Π°Π·Π½ΠΈΡ†Ρƒ Π² энСргии ΠΌΠΎΠΆΠ½ΠΎ ΠΈΠ·ΠΌΠ΅Ρ€ΠΈΡ‚ΡŒ Ρ‚ΠΎΠ»ΡŒΠΊΠΎ ΠΌΠ΅ΠΆΠ΄Ρƒ двумя Ρ‚ΠΎΡ‡ΠΊΠ°ΠΌΠΈ. Π‘Π΅Π· привязки ΠΊ Π΄Π²ΡƒΠΌ Ρ‚ΠΎΡ‡ΠΊΠ°ΠΌ Ρ†Π΅ΠΏΠΈ Π½Π΅Π²ΠΎΠ·ΠΌΠΎΠΆΠ½ΠΎ ΠΎΠΏΡ€Π΅Π΄Π΅Π»ΠΈΡ‚ΡŒ напряТСниС.

Π’Π΅Π»ΠΈΡ‡ΠΈΠ½Π° этой Ρ€Π°Π·Π½ΠΈΡ†Ρ‹ ΠΏΠΎΠΌΠΎΠ³Π°Π΅Ρ‚ ΠΎΠΏΡ€Π΅Π΄Π΅Π»ΠΈΡ‚ΡŒ объСм Ρ€Π°Π±ΠΎΡ‚Ρ‹, ΠΊΠΎΡ‚ΠΎΡ€ΡƒΡŽ ΠΌΠΎΠΆΠ΅Ρ‚ Π²Ρ‹ΠΏΠΎΠ»Π½ΠΈΡ‚ΡŒ схСма. НапримСр, типичная батарСя Ρ‚ΠΈΠΏΠ° АА ΠΈΠΌΠ΅Π΅Ρ‚ Ρ€Π°Π·Π½ΠΎΡΡ‚ΡŒ ΠΏΠΎΡ‚Π΅Π½Ρ†ΠΈΠ°Π»ΠΎΠ² 1,5 Π’. Напротив, элСктричСскиС Ρ€ΠΎΠ·Π΅Ρ‚ΠΊΠΈ Π² Π΄ΠΎΠΌΠ°Ρ… ΠΈΠΌΠ΅ΡŽΡ‚ Ρ€Π°Π·Π½ΠΎΡΡ‚ΡŒ ΠΏΠΎΡ‚Π΅Π½Ρ†ΠΈΠ°Π»ΠΎΠ² 120 Π’.

ΠŸΠΎΡΠΊΠΎΠ»ΡŒΠΊΡƒ элСктричСскиС Ρ€ΠΎΠ·Π΅Ρ‚ΠΊΠΈ ΠΈΠΌΠ΅ΡŽΡ‚ Π±ΠΎΠ»Π΅Π΅ высокоС Β«Π΄Π°Π²Π»Π΅Π½ΠΈΠ΅Β» двиТСния элСктронов, ΠΎΠ½ΠΈ ΠΌΠΎΠ³ΡƒΡ‚ ΠΏΠΈΡ‚Π°Ρ‚ΡŒ Π±ΠΎΠ»Π΅Π΅ ΠΊΡ€ΡƒΠΏΠ½Ρ‹Π΅ ΠΏΡ€ΠΈΠ±ΠΎΡ€Ρ‹, Ρ‡Π΅ΠΌ Π±Π°Ρ‚Π°Ρ€Π΅ΠΉΠΊΠΈ АА.

Π§Π΅ΠΌ ΠΏΠΎΠ»Π΅Π·Π½ΠΎ ΠΈΠ·ΠΌΠ΅Ρ€Π΅Π½ΠΈΠ΅ напряТСния?

НапряТСниС Ρ‡Π°Ρ‰Π΅ всСго измСряСтся для устранСния Π½Π΅ΠΏΠΎΠ»Π°Π΄ΠΎΠΊ Π² цСпях.

Π¦Π΅ΠΏΠΈ строятся с основной Ρ†Π΅Π»ΡŒΡŽ ΠΏΠΎΠ΄Π°Ρ‡ΠΈ энСргии Π½Π° Π½Π΅ΠΊΠΎΡ‚ΠΎΡ€ΡƒΡŽ Π½Π°Π³Ρ€ΡƒΠ·ΠΊΡƒ. Нагрузка ΠΌΠΎΠΆΠ΅Ρ‚ Π±Ρ‹Ρ‚ΡŒ нСбольшой, ΠΊΠ°ΠΊ Π±Ρ‹Ρ‚ΠΎΠ²ΠΎΠΉ ΠΏΡ€ΠΈΠ±ΠΎΡ€, ΠΈΠ»ΠΈ большой, ΠΊΠ°ΠΊ ΠΏΡ€ΠΎΠΌΡ‹ΡˆΠ»Π΅Π½Π½Π°Ρ машина.

ВСхничСскиС спСциалисты ΠΏΠΎΠ½ΠΈΠΌΠ°ΡŽΡ‚, ΠΊΠ°ΠΊ Π΄ΠΎΠ»ΠΆΠ½Π° Ρ€Π°Π±ΠΎΡ‚Π°Ρ‚ΡŒ Ρ†Π΅ΠΏΡŒ, ΠΏΠΎΡ‚ΠΎΠΌΡƒ Ρ‡Ρ‚ΠΎ напряТСниС ΠΈ Ρ‚ΠΎΠΊ Ρ†Π΅ΠΏΠΈ всСгда ΡƒΠΊΠ°Π·Π°Π½Ρ‹ Π½Π° Π½Π°Π³Ρ€ΡƒΠ·ΠΊΠ΅. Если Π½Π° Ρ†Π΅ΠΏΠΈ Π½Π΅Ρ‚ паспортной Ρ‚Π°Π±Π»ΠΈΡ‡ΠΊΠΈ, ΠΏΡ€ΠΎΠΈΠ·Π²ΠΎΠ΄ΠΈΡ‚Π΅Π»ΠΈ Π²ΠΊΠ»ΡŽΡ‡Π°ΡŽΡ‚ ΠΏΠΎΠ΄Ρ€ΠΎΠ±Π½ΡƒΡŽ схСму Ρ†Π΅ΠΏΠΈ, Ρ‡Ρ‚ΠΎΠ±Ρ‹ ΠΏΠΎΠΌΠΎΡ‡ΡŒ тСхничСским спСциалистам ΠΏΡ€Π°Π²ΠΈΠ»ΡŒΠ½ΠΎ Π΅Π΅ ΠΎΡ‚Ρ€Π΅ΠΌΠΎΠ½Ρ‚ΠΈΡ€ΠΎΠ²Π°Ρ‚ΡŒ.

Π¦ΠΈΡ„Ρ€Ρ‹ Π½Π° паспортной Ρ‚Π°Π±Π»ΠΈΡ‡ΠΊΠ΅ ΠΈΠ»ΠΈ Π² руководствС ΠΏΠΎΠΌΠΎΠ³Π°ΡŽΡ‚ тСхничСским спСциалистам ΠΏΠΎΠ½ΡΡ‚ΡŒ, ΠΊΠ°ΠΊΠΈΠ΅ показания слСдуСт ΠΎΠΆΠΈΠ΄Π°Ρ‚ΡŒ ΠΎΡ‚ Ρ†Π΅ΠΏΠΈ. Π˜Π·ΠΌΠ΅Ρ€Π΅Π½ΠΈΠ΅ напряТСния ΠΌΡƒΠ»ΡŒΡ‚ΠΈΠΌΠ΅Ρ‚Ρ€ΠΎΠΌ ΠΏΠΎΠΌΠΎΠ³Π°Π΅Ρ‚ тСхничСскому спСциалисту ΠΎΠ±ΡŠΠ΅ΠΊΡ‚ΠΈΠ²Π½ΠΎ ΠΏΠΎΠ½ΡΡ‚ΡŒ, Π² Ρ‡Π΅ΠΌ ΠΌΠΎΠΆΠ΅Ρ‚ Π±Ρ‹Ρ‚ΡŒ ΠΏΡ€ΠΎΠ±Π»Π΅ΠΌΠ°.

Как ΠΈΠ·ΠΌΠ΅Ρ€ΠΈΡ‚ΡŒ напряТСниС Π² простой Ρ†Π΅ΠΏΠΈ?

ΠœΡƒΠ»ΡŒΡ‚ΠΈΠΌΠ΅Ρ‚Ρ€ ΠΈΠ»ΠΈ Π²ΠΎΠ»ΡŒΡ‚ΠΌΠ΅Ρ‚Ρ€ Π»ΡƒΡ‡ΡˆΠ΅ всСго подходят для измСрСния напряТСния.

  1. ΠŸΠ΅Ρ€Π΅Π΄ Π²ΠΊΠ»ΡŽΡ‡Π΅Π½ΠΈΠ΅ΠΌ Ρ†ΠΈΡ„Ρ€ΠΎΠ²ΠΎΠ³ΠΎ ΠΌΡƒΠ»ΡŒΡ‚ΠΈΠΌΠ΅Ρ‚Ρ€Π° (DMM) подсоСдинитС Ρ‡Π΅Ρ€Π½Ρ‹ΠΉ ΠΏΡ€ΠΎΠ²ΠΎΠ΄ ΠΊ ΠΎΡ‚Ρ€ΠΈΡ†Π°Ρ‚Π΅Π»ΡŒΠ½ΠΎΠΌΡƒ Ρ€Π°Π·ΡŠΠ΅ΠΌΡƒ, Π° красный ΠΏΡ€ΠΎΠ²ΠΎΠ΄ ΠΊ ΠΏΠΎΠ»ΠΎΠΆΠΈΡ‚Π΅Π»ΡŒΠ½ΠΎΠΌΡƒ Ρ€Π°Π·ΡŠΠ΅ΠΌΡƒ.
  2. Π’ΠΊΠ»ΡŽΡ‡ΠΈΡ‚Π΅ Ρ†ΠΈΡ„Ρ€ΠΎΠ²ΠΎΠΉ ΠΌΡƒΠ»ΡŒΡ‚ΠΈΠΌΠ΅Ρ‚Ρ€ ΠΈ настройтС Π΅Π³ΠΎ Π½Π° максимальноС Π·Π½Π°Ρ‡Π΅Π½ΠΈΠ΅, ΠΎΠΆΠΈΠ΄Π°Π΅ΠΌΠΎΠ΅ ΠΎΡ‚ схСмы. Но ΠΈΠΌΠ΅ΠΉΡ‚Π΅ Π² Π²ΠΈΠ΄Ρƒ, Ρ‡Ρ‚ΠΎ Π»ΡƒΡ‡ΡˆΠ΅ всСго ΡƒΡΡ‚Π°Π½ΠΎΠ²ΠΈΡ‚ΡŒ Ρ†ΠΈΡ„Ρ€ΠΎΠ²ΠΎΠΉ ΠΌΡƒΠ»ΡŒΡ‚ΠΈΠΌΠ΅Ρ‚Ρ€ Π½Π° максимальноС Π·Π½Π°Ρ‡Π΅Π½ΠΈΠ΅, ΠΊΠΎΡ‚ΠΎΡ€ΠΎΠ΅ ΠΎΠ½ ΠΌΠΎΠΆΠ΅Ρ‚ ΠΈΠ·ΠΌΠ΅Ρ€ΠΈΡ‚ΡŒ.
  3. ΠŸΡ€ΠΈΠΊΠΎΡΠ½ΠΈΡ‚Π΅ΡΡŒ Ρ‡Π΅Ρ€Π½Ρ‹ΠΌ ΠΏΡ€ΠΎΠ²ΠΎΠ΄ΠΎΠΌ ΠΊ Ρ‚ΠΎΡ‡ΠΊΠ΅ с Π±ΠΎΠ»Π΅Π΅ Π½ΠΈΠ·ΠΊΠΈΠΌ напряТСниСм, Π° красным ΠΏΡ€ΠΎΠ²ΠΎΠ΄ΠΎΠΌ ΠΊ Ρ‚ΠΎΡ‡ΠΊΠ΅ с Π±ΠΎΠ»Π΅Π΅ высоким напряТСниСм.
  4. Π’ΠΎΠ·ΠΌΠΎΠΆΠ½ΠΎ, Π²Π°ΠΌ придСтся ΠΎΡ‚Ρ€Π΅Π³ΡƒΠ»ΠΈΡ€ΠΎΠ²Π°Ρ‚ΡŒ ΠΏΠ΅Ρ€Π΅ΠΊΠ»ΡŽΡ‡Π°Ρ‚Π΅Π»ΡŒ Π΄ΠΈΠ°ΠΏΠ°Π·ΠΎΠ½ΠΎΠ², Ρ‡Ρ‚ΠΎΠ±Ρ‹ ΠΏΠΎΠ»ΡƒΡ‡ΠΈΡ‚ΡŒ максимально Ρ‚ΠΎΡ‡Π½Ρ‹Π΅ показания. Однако Π½Π΅ΠΊΠΎΡ‚ΠΎΡ€Ρ‹Π΅ Ρ†ΠΈΡ„Ρ€ΠΎΠ²Ρ‹Π΅ ΠΌΡƒΠ»ΡŒΡ‚ΠΈΠΌΠ΅Ρ‚Ρ€Ρ‹ ΠΈΠΌΠ΅ΡŽΡ‚ Ρ„ΡƒΠ½ΠΊΡ†ΠΈΡŽ автоматичСского Π²Ρ‹Π±ΠΎΡ€Π° Π΄ΠΈΠ°ΠΏΠ°Π·ΠΎΠ½Π°, которая избавляСт вас ΠΎΡ‚ нСобходимости Π΄Π΅Π»Π°Ρ‚ΡŒ это.
  5. Π’Ρ‹ ΡƒΠ²ΠΈΠ΄ΠΈΡ‚Π΅ показания напряТСния Π½Π° ΠΌΡƒΠ»ΡŒΡ‚ΠΈΠΌΠ΅Ρ‚Ρ€Π΅.

Π˜Π·ΠΌΠ΅Ρ€ΠΈΡ‚ΡŒ напряТСниС Π² простых цСпях ΠΎΡ‡Π΅Π½ΡŒ просто. Но для измСрСния напряТСния Π² Π±ΠΎΠ»Π΅Π΅ ΡΠΎΠ²Π΅Ρ€ΡˆΠ΅Π½Π½Ρ‹Ρ… схСмах Π½Π΅ΠΎΠ±Ρ…ΠΎΠ΄ΠΈΠΌΠΎ ΠΏΠΎΠ½ΠΈΠΌΠ°Ρ‚ΡŒ, ΠΊΠ°ΠΊΠΎΠ΅ ΠΏΡ€ΠΈΠ»ΠΎΠΆΠ΅Π½ΠΎ напряТСниС.

Π’ΠΎΡ‡Π½ΠΎ Ρ‚Π°ΠΊ ΠΆΠ΅, ΠΊΠ°ΠΊ Π²Ρ‹ ΠΌΠΎΠΆΠ΅Ρ‚Π΅ ΠΈΠ·ΠΌΠ΅Ρ€ΠΈΡ‚ΡŒ напряТСниС Π½Π° Ρ†Π΅ΠΏΠΈ, Π²Ρ‹ Ρ‚Π°ΠΊΠΆΠ΅ ΠΌΠΎΠΆΠ΅Ρ‚Π΅ ΠΈΠ·ΠΌΠ΅Ρ€ΠΈΡ‚ΡŒ напряТСниС Π½Π° ΠΊΠΎΠΌΠΏΠΎΠ½Π΅Π½Ρ‚Π΅ Π² Ρ†Π΅ΠΏΠΈ. НапряТСниС Π½Π° ΠΊΠΎΠΌΠΏΠΎΠ½Π΅Π½Ρ‚Π΅ прСдставляСт собой Π±ΠΎΠ»Π΅Π΅ Π½ΠΈΠ·ΠΊΠΎΠ΅ напряТСниС, ΠΊΠΎΡ‚ΠΎΡ€ΠΎΠ΅ ΡƒΠΌΠ΅Π½ΡŒΡˆΠ°Π΅Ρ‚ΡΡ Π·Π° счСт Π²Π½ΡƒΡ‚Ρ€Π΅Π½Π½Π΅Π³ΠΎ сопротивлСния ΡƒΠΊΠ°Π·Π°Π½Π½ΠΎΠ³ΠΎ ΠΊΠΎΠΌΠΏΠΎΠ½Π΅Π½Ρ‚Π°.

Π‘ Π΄Ρ€ΡƒΠ³ΠΎΠΉ стороны, ΠΏΡ€ΠΈΠ»ΠΎΠΆΠ΅Π½Π½ΠΎΠ΅ напряТСниС β€” это напряТСниС, ΠΏΠΎΠ΄Π°Π²Π°Π΅ΠΌΠΎΠ΅ Π½Π° ΠΊΠΎΠΌΠΏΠΎΠ½Π΅Π½Ρ‚.

Зная эту Ρ€Π°Π·Π½ΠΈΡ†Ρƒ, Π²Ρ‹ смоТСтС ΠΏΠΎΠ½ΡΡ‚ΡŒ Ρ€Π°Π·Π½ΠΎΡΡ‚ΡŒ напряТСний Π² Ρ€Π°Π·Π½Ρ‹Ρ… Ρ‚ΠΎΡ‡ΠΊΠ°Ρ… Ρ†Π΅ΠΏΠΈ.

Как Ρ‚ΠΎΠΊ ΠΈ сопротивлСниС связаны с напряТСниСм?

Π’ΠΎΠΊ ΠΈ сопротивлСниС Π² Ρ†Π΅ΠΏΠΈ ΠΏΡ€Π΅Π΄ΡΡ‚Π°Π²Π»ΡΡŽΡ‚ собой ΡΠΊΠΎΡ€ΠΎΡΡ‚ΡŒ ΠΏΠΎΡ‚ΠΎΠΊΠ° элСктронов ΠΈ Ρ‚Ρ€Π΅Π½ΠΈΠ΅ ΠΏΡ€ΠΎΡ‚ΠΈΠ² ΠΏΠΎΡ‚ΠΎΠΊΠ° элСктронов Π² этой Ρ‚ΠΎΡ‡ΠΊΠ΅ Ρ†Π΅ΠΏΠΈ соотвСтствСнно.

Π‘ΠΈΠ»Π° Ρ‚ΠΎΠΊΠ° измСряСтся Π² АмпСрах (А), Π΅Π΄ΠΈΠ½ΠΈΡ†Π° измСрСния, названная Π² Ρ‡Π΅ΡΡ‚ΡŒ АндрС-ΠœΠ°Ρ€ΠΈ АмпСра. Π’ уравнСниях Ρ‚ΠΎΠΊ обозначаСтся Π±ΡƒΠΊΠ²ΠΎΠΉ Β«IΒ». ΠΎΠ·Π½Π°ΠΊΠΎΠΌΡŒΡ‚Π΅ΡΡŒ с нашСй ΠΏΠΎΠ΄Ρ€ΠΎΠ±Π½ΠΎΠΉ ΡΡ‚Π°Ρ‚ΡŒΠ΅ΠΉ ΠΎ Ρ‚ΠΎΠΌ, ΠΊΠ°ΠΊΠΎΠΉ Ρ€Π°Π·ΠΌΠ΅Ρ€ рСзистора для вашСго свСтодиода.

Π‘ Π΄Ρ€ΡƒΠ³ΠΎΠΉ стороны, сопротивлСниС измСряСтся Π² ΠΎΠΌΠ°Ρ… (Ом), Π΅Π΄ΠΈΠ½ΠΈΡ†Π°, названная Π² Ρ‡Π΅ΡΡ‚ΡŒ Π“Π΅ΠΎΡ€Π³Π° Π‘ΠΈΠΌΠΎΠ½Π° Ома. Π•ΠΌΡƒ ΠΏΡ€ΠΈΠΏΠΈΡΡ‹Π²Π°ΡŽΡ‚ ΠΎΠ±Π½Π°Ρ€ΡƒΠΆΠ΅Π½ΠΈΠ΅ взаимосвязи ΠΌΠ΅ΠΆΠ΄Ρƒ напряТСниСм, Ρ‚ΠΎΠΊΠΎΠΌ ΠΈ сопротивлСниСм Π² Ρ‚Π°ΠΊ Π½Π°Π·Ρ‹Π²Π°Π΅ΠΌΠΎΠΌ Π·Π°ΠΊΠΎΠ½Π΅ Ома. Π’ уравнСниях сопротивлСниС обозначаСтся Π±ΡƒΠΊΠ²ΠΎΠΉ R.

Π—Π°ΠΊΠΎΠ½ Ома гласит, Ρ‡Ρ‚ΠΎ Ρ‚ΠΎΠΊ, ΠΏΡ€ΠΎΡ‚Π΅ΠΊΠ°ΡŽΡ‰ΠΈΠΉ Π² Π΄Π°Π½Π½ΠΎΠΉ Ρ†Π΅ΠΏΠΈ, прямо ΠΏΡ€ΠΎΠΏΠΎΡ€Ρ†ΠΈΠΎΠ½Π°Π»Π΅Π½ ΠΏΡ€ΠΈΠ»ΠΎΠΆΠ΅Π½Π½ΠΎΠΌΡƒ Π½Π°ΠΏΡ€ΡΠΆΠ΅Π½ΠΈΡŽ Π² Ρ†Π΅ΠΏΠΈ ΠΈ ΠΎΠ±Ρ€Π°Ρ‚Π½ΠΎ ΠΏΡ€ΠΎΠΏΠΎΡ€Ρ†ΠΈΠΎΠ½Π°Π»Π΅Π½ ΡΠΎΠΏΡ€ΠΎΡ‚ΠΈΠ²Π»Π΅Π½ΠΈΡŽ Π² Ρ†Π΅ΠΏΠΈ. Π’Π°ΠΆΠ½ΠΎ ΠΎΡ‚ΠΌΠ΅Ρ‚ΠΈΡ‚ΡŒ, Ρ‡Ρ‚ΠΎ для примСнСния этого Π·Π°ΠΊΠΎΠ½Π° Ρ‚Π΅ΠΌΠΏΠ΅Ρ€Π°Ρ‚ΡƒΡ€Π° Π΄ΠΎΠ»ΠΆΠ½Π° ΠΎΡΡ‚Π°Π²Π°Ρ‚ΡŒΡΡ постоянной.

  Π’ΠΎΠΊ Π² Π°ΠΌΠΏΠ΅Ρ€Π°Ρ…  =  НапряТСниС  (Π² Π²ΠΎΠ»ΡŒΡ‚Π°Ρ…)  Π‘ΠΎΠΏΡ€ΠΎΡ‚ΠΈΠ²Π»Π΅Π½ΠΈΠ΅  (Π² ΠΎΠΌΠ°Ρ…) 

Или

 I = V/R 

Π’ΠΎΡ‚ ΠΊΠ°ΠΊ Π²Ρ‹ Π΄ΠΎΠ»ΠΆΠ½Ρ‹ ΠΏΠΎΠ½ΠΈΠΌΠ°Ρ‚ΡŒ это:

3 900 НапряТСниС Π² Ρ†Π΅ΠΏΠΈ увСличиваСтся , Ρ‚ΠΎΠΊ Π² Ρ†Π΅ΠΏΠΈ Ρ‚Π°ΠΊΠΆΠ΅ увСличится. Π‘ Π΄Ρ€ΡƒΠ³ΠΎΠΉ стороны, Ссли сопротивлСниС Π² Ρ†Π΅ΠΏΠΈ увСличится, Ρ‚ΠΎΠΊ Π² Ρ†Π΅ΠΏΠΈ ΡƒΠΌΠ΅Π½ΡŒΡˆΠΈΡ‚ΡΡ.

ΠœΡ‹ ΠΌΠΎΠΆΠ΅ΠΌ ΠΈΡΠΏΠΎΠ»ΡŒΠ·ΠΎΠ²Π°Ρ‚ΡŒ ΡƒΡ€Π°Π²Π½Π΅Π½ΠΈΠ΅ Π·Π°ΠΊΠΎΠ½Π° Ома для опрСдСлСния значСния Ρ‚ΠΎΠΊΠ°, напряТСния ΠΈΠ»ΠΈ сопротивлСния, Ссли Ρƒ нас Π΅ΡΡ‚ΡŒ Π΄Π²Π° Π΄Ρ€ΡƒΠ³ΠΈΡ… ΡΠΎΠΎΡ‚Π²Π΅Ρ‚ΡΡ‚Π²ΡƒΡŽΡ‰ΠΈΡ… значСния.

Для Ρ€Π΅ΡˆΠ΅Π½ΠΈΡ для напряТСния ΡƒΡ€Π°Π²Π½Π΅Π½ΠΈΠ΅ Π·Π°ΠΊΠΎΠ½Π° Ома ΠΌΠΎΠΆΠ½ΠΎ ΠΏΠ΅Ρ€Π΅ΡΡ‚Π°Π²ΠΈΡ‚ΡŒ, ΠΊΠ°ΠΊ ΠΈ SO:

НапряТСниС = Π’ΠΎΠΊ * Π‘ΠΎΠΏΡ€ΠΎΡ‚ΠΈΠ²Π»Π΅Π½ΠΈΠ΅.

Π‘ΠΎΠΏΡ€ΠΎΡ‚ΠΈΠ²Π»Π΅Π½ΠΈΠ΅ = НапряТСниС / Π’ΠΎΠΊ

Π—Π°ΠΊΠ»ΡŽΡ‡ΠΈΡ‚Π΅Π»ΡŒΠ½Ρ‹Π΅ мысли

Π²Ρ‹ Ρ…ΠΎΡ‚ΠΈΡ‚Π΅ ΠΏΠΎΡΡ‚Ρ€ΠΎΠΈΡ‚ΡŒ Ρ‡Ρ‚ΠΎ-Π½ΠΈΠ±ΡƒΠ΄ΡŒ элСктричСскоС ΠΈΠ»ΠΈ просто ΠΏΠΎΠ½ΡΡ‚ΡŒ, ΠΊΠ°ΠΊ Ρ€Π°Π±ΠΎΡ‚Π°ΡŽΡ‚ Ρ€ΠΎΠ·Π΅Ρ‚ΠΊΠΈ Π² вашСм Π΄ΠΎΠΌΠ΅.

ΠžΡΡ‚Π°Π»ΠΈΡΡŒ вопросы?

ΠΠ°ΠΏΠΈΡˆΠΈΡ‚Π΅ ΠΌΠ½Π΅ Π² коммСнтариях Π½ΠΈΠΆΠ΅, ΠΈ я сдСлаю всС Π²ΠΎΠ·ΠΌΠΎΠΆΠ½ΠΎΠ΅, Ρ‡Ρ‚ΠΎΠ±Ρ‹ ΠΏΠΎΠΌΠΎΡ‡ΡŒ Π²Π°ΠΌ!

Быстрый вопрос, Ссли Π²Ρ‹ Ρ‡ΠΈΡ‚Π°Π»ΠΈ ΠΌΠΎΠΉ пост ΠΈ Ρ‡Ρ‚ΠΎ-Ρ‚ΠΎ ΠΈΠ· Π½Π΅Π³ΠΎ ΡƒΠ·Π½Π°Π»ΠΈ, поТалуйста, ΠΏΠΎΠ΄Π΅Π»ΠΈΡ‚Π΅ΡΡŒ ΠΈΠΌ .

Π’ΠΎΠ»ΡŒΡ‚Π°ΠΌΠΏΠ΅Ρ€Π½Ρ‹Π΅ характСристики: ПояснСниС | StudySmarter

ΠŸΡ€ΠΈ ΠΈΠ·ΡƒΡ‡Π΅Π½ΠΈΠΈ элСктричСских Ρ†Π΅ΠΏΠ΅ΠΉ ΠΌΡ‹ часто ΠΈΡΠΏΠΎΠ»ΡŒΠ·ΡƒΠ΅ΠΌ Π·Π°ΠΊΠΎΠ½ Ома, ΠΊΠΎΡ‚ΠΎΡ€Ρ‹ΠΉ прСдставляСт собой ΡΠΎΠΎΡ‚Π½ΠΎΡˆΠ΅Π½ΠΈΠ΅ ΠΌΠ΅ΠΆΠ΄Ρƒ трСмя связанными Π²Π΅Π»ΠΈΡ‡ΠΈΠ½Π°ΠΌΠΈ. Для описания ΠΌΠ°Ρ‚Π΅Ρ€ΠΈΠ°Π»ΠΎΠ² ΠΈ схСм Π½Π΅ΠΎΠ±Ρ…ΠΎΠ΄ΠΈΠΌΠΎ ΠΈΠ·ΡƒΡ‡ΠΈΡ‚ΡŒ Π²ΠΎΠ»ΡŒΡ‚-Π°ΠΌΠΏΠ΅Ρ€Π½Ρ‹Π΅ характСристики ΠΈ ΠΈΡ… ΠΏΠΎΠ²Π΅Π΄Π΅Π½ΠΈΠ΅ для Ρ€Π°Π·Π»ΠΈΡ‡Π½Ρ‹Ρ… устройств ΠΈ установок.

ΠŸΡ€Π΅ΠΆΠ΄Π΅ всСго, Ρ‡Ρ‚ΠΎ Ρ‚Π°ΠΊΠΎΠ΅ Π·Π°ΠΊΠΎΠ½ Ома?

Π—Π°ΠΊΠΎΠ½ Ома гласит:

Для ΠΏΡ€ΠΎΠ²ΠΎΠ΄Π½ΠΈΠΊΠ° с постоянной Ρ‚Π΅ΠΌΠΏΠ΅Ρ€Π°Ρ‚ΡƒΡ€ΠΎΠΉ Ρ‚ΠΎΠΊ, проходящий Ρ‡Π΅Ρ€Π΅Π· Π½Π΅Π³ΠΎ, ΠΏΡ€ΠΎΠΏΠΎΡ€Ρ†ΠΈΠΎΠ½Π°Π»Π΅Π½ разности ΠΏΠΎΡ‚Π΅Π½Ρ†ΠΈΠ°Π»ΠΎΠ² Π½Π° Π½Π΅ΠΌ, учитывая, Ρ‡Ρ‚ΠΎ физичСскиС условия ΠΈ сопротивлСниС ΠΎΡΡ‚Π°ΡŽΡ‚ΡΡ постоянными.

Или, Π½Π° матСматичСском языкС:

Π’ β€” Ρ€Π°Π·Π½ΠΎΡΡ‚ΡŒ ΠΏΠΎΡ‚Π΅Π½Ρ†ΠΈΠ°Π»ΠΎΠ² (измСряСтся Π² Π²ΠΎΠ»ΡŒΡ‚Π°Ρ…, Π’), I β€” элСктричСский Ρ‚ΠΎΠΊ (измСряСтся Π² Π°ΠΌΠΏΠ΅Ρ€Π°Ρ…, А), R β€” элСктричСскоС сопротивлСниС (измСряСтся Π² ΠΎΠΌΠ°Ρ…, Ξ©) . Π­Ρ‚ΠΎ ΡƒΡ€Π°Π²Π½Π΅Π½ΠΈΠ΅ фиксируСт линСйная Π·Π°Π²ΠΈΡΠΈΠΌΠΎΡΡ‚ΡŒ ΠΌΠ΅ΠΆΠ΄Ρƒ Ρ€Π°Π·Π½ΠΎΡΡ‚ΡŒΡŽ ΠΏΠΎΡ‚Π΅Π½Ρ†ΠΈΠ°Π»ΠΎΠ² ΠΈ элСктричСским Ρ‚ΠΎΠΊΠΎΠΌ.

Но Ρ‡Ρ‚ΠΎ Ρ‚Π°ΠΊΠΎΠ΅ сопротивлСниС? ΠšΠΎΡ€ΠΎΡ‡Π΅ говоря, сопротивлСниС β€” это ΠΊΠΎΠ»Π»Π΅ΠΊΡ‚ΠΈΠ²Π½Ρ‹ΠΉ эффСкт срСды, ΠΏΡ€Π΅ΠΏΡΡ‚ΡΡ‚Π²ΡƒΡŽΡ‰Π΅ΠΉ двиТСнию зарядов (Ρ‚ΠΎΠΊ). Π‘ΠΎΠΏΡ€ΠΎΡ‚ΠΈΠ²Π»Π΅Π½ΠΈΠ΅ зависит ΠΎΡ‚ ΠΌΠ½ΠΎΠ³ΠΈΡ… Ρ„Π°ΠΊΡ‚ΠΎΡ€ΠΎΠ², Ρ‚Π°ΠΊΠΈΡ… ΠΊΠ°ΠΊ Ρ‚ΠΈΠΏ ΠΈΡΠΏΠΎΠ»ΡŒΠ·ΡƒΠ΅ΠΌΠΎΠ³ΠΎ ΠΌΠ°Ρ‚Π΅Ρ€ΠΈΠ°Π»Π° ΠΈ Ρ‚Π΅ΠΌΠΏΠ΅Ρ€Π°Ρ‚ΡƒΡ€Π° ΠΌΠ°Ρ‚Π΅Ρ€ΠΈΠ°Π»Π°.

ΠŸΠΎΡΠΊΠΎΠ»ΡŒΠΊΡƒ ΡƒΡΡ‚Π°Π½ΠΎΠ²ΠΈΡ‚ΡŒ Ρ€Π°Π·Π½ΠΎΡΡ‚ΡŒ ΠΏΠΎΡ‚Π΅Π½Ρ†ΠΈΠ°Π»ΠΎΠ² ΠΎΡ‚Π½ΠΎΡΠΈΡ‚Π΅Π»ΡŒΠ½ΠΎ просто, ΠΌΡ‹ ΠΌΠΎΠΆΠ΅ΠΌ Π³Π΅Π½Π΅Ρ€ΠΈΡ€ΠΎΠ²Π°Ρ‚ΡŒ ΠΎΠΏΡ€Π΅Π΄Π΅Π»Π΅Π½Π½Ρ‹ΠΉ элСктричСский Ρ‚ΠΎΠΊ, измСняя сопротивлСниС. ЭлСктричСский Ρ‚ΠΎΠΊ Π²ΠΎΠ·Π½ΠΈΠΊΠ°Π΅Ρ‚, ΠΊΠΎΠ³Π΄Π° ΠΌΡ‹ устанавливаСм Ρ€Π°Π·Π½ΠΎΡΡ‚ΡŒ ΠΏΠΎΡ‚Π΅Π½Ρ†ΠΈΠ°Π»ΠΎΠ² ΠΌΠ΅ΠΆΠ΄Ρƒ двумя сторонами ΠΏΡ€ΠΎΠ²ΠΎΠ΄Π½ΠΈΠΊΠ°. ΠŸΠΎΡΠΊΠΎΠ»ΡŒΠΊΡƒ ΠΌΡ‹ ΠΌΠΎΠΆΠ΅ΠΌ ΠΈΠ·ΠΌΠ΅Π½ΠΈΡ‚ΡŒ Ρ‚ΠΎΠΊ, ΠΈΠ·ΠΌΠ΅Π½ΠΈΠ² сопротивлСниС, интСрСсно ΠΈΠ·ΡƒΡ‡ΠΈΡ‚ΡŒ ΠΊΠ°ΠΊ это сопротивлСниС влияСт Π½Π° Ρ‚ΠΎΠΊ . ΠŸΠΎΡΡ‚ΠΎΠΌΡƒ стоит ΠΈΠ·ΡƒΡ‡ΠΈΡ‚ΡŒ ΠΏΠΎΠ²Π΅Π΄Π΅Π½ΠΈΠ΅ сопротивлСния ΠΌΠ°Ρ‚Π΅Ρ€ΠΈΠ°Π»ΠΎΠ² ΠΈ схСм для создания устройств, ΠΊΠΎΡ‚ΠΎΡ€Ρ‹Π΅ слуТат Ρ€Π°Π·Π½Ρ‹ΠΌ цСлям.

Π—Π°ΠΊΠΎΠ½ Ома гласит, Ρ‡Ρ‚ΠΎ Π·Π°Π²ΠΈΡΠΈΠΌΠΎΡΡ‚ΡŒ ΠΌΠ΅ΠΆΠ΄Ρƒ напряТСниСм Π² Ρ†Π΅ΠΏΠΈ ΠΈ Ρ‚ΠΎΠΊΠΎΠΌ, ΠΏΡ€ΠΎΡ‚Π΅ΠΊΠ°ΡŽΡ‰ΠΈΠΌ Ρ‡Π΅Ρ€Π΅Π· Π½Π΅Π΅, являСтся Π»ΠΈΠ½Π΅ΠΉΠ½ΠΎΠΉ ΠΈ, ΠΊΠ°ΠΊ ΠΏΡ€Π°Π²ΠΈΠ»ΠΎ, постоянной. Π­Ρ‚ΠΎ ΠΏΡ€ΠΈΠ±Π»ΠΈΠΆΠ΅Π½ΠΈΠ΅ ΠΊ повСдСнию Π±ΠΎΠ»ΡŒΡˆΠΈΠ½ΡΡ‚Π²Π° ΠΌΠ°Ρ‚Π΅Ρ€ΠΈΠ°Π»ΠΎΠ².

НСомичСскиС ΠΌΠ°Ρ‚Π΅Ρ€ΠΈΠ°Π»Ρ‹

Как ΠΏΡ€Π°Π²ΠΈΠ»ΠΎ, сопротивлСниС Π½Π΅ являСтся константой, ΠΏΠΎΠ»ΡƒΡ‡Π΅Π½Π½ΠΎΠΉ ΠΏΡƒΡ‚Π΅ΠΌ дСлСния разности ΠΏΠΎΡ‚Π΅Π½Ρ†ΠΈΠ°Π»ΠΎΠ² Π½Π° элСктричСский Ρ‚ΠΎΠΊ. Π‘ΠΎΠΏΡ€ΠΎΡ‚ΠΈΠ²Π»Π΅Π½ΠΈΠ΅ Π½Π° самом Π΄Π΅Π»Π΅ являСтся ΠΏΡ€ΠΎΠΈΠ·Π²ΠΎΠ»ΡŒΠ½ΠΎΠΉ Ρ„ΡƒΠ½ΠΊΡ†ΠΈΠ΅ΠΉ R(V, I) , Ρ‡Ρ‚ΠΎ зависит ΠΎΡ‚ разности ΠΏΠΎΡ‚Π΅Π½Ρ†ΠΈΠ°Π»ΠΎΠ² ΠΈ Ρ‚ΠΎΠΊΠ°. Π—Π°ΠΊΠΎΠ½ Ома являСтся Π»ΠΈΠ½Π΅ΠΉΠ½Ρ‹ΠΌ ΠΏΡ€ΠΈΠ±Π»ΠΈΠΆΠ΅Π½ΠΈΠ΅ΠΌ для нСбольшой области этого ΡΠΎΠΎΡ‚Π½ΠΎΡˆΠ΅Π½ΠΈΡ. Π’ нСомичСских ΠΌΠ°Ρ‚Π΅Ρ€ΠΈΠ°Π»Π°Ρ… сопротивлСниС Π½Π΅ соотвСтствуСт Π»ΠΈΠ½Π΅ΠΉΠ½ΠΎΠΌΡƒ ΠΏΡ€ΠΈΠ±Π»ΠΈΠΆΠ΅Π½ΠΈΡŽ.

Если Ρƒ нас Π΅ΡΡ‚ΡŒ связь ΠΌΠ΅ΠΆΠ΄Ρƒ Ρ‚ΠΎΠΊΠΎΠΌ I ΠΈ напряТСниСм V(I), ΠΌΡ‹ ΠΌΠΎΠΆΠ΅ΠΌ Ρ€Π°ΡΡΡ‡ΠΈΡ‚Π°Ρ‚ΡŒ сопротивлСниС ΡΠ»Π΅Π΄ΡƒΡŽΡ‰ΠΈΠΌ ΠΎΠ±Ρ€Π°Π·ΠΎΠΌ:

Если связь ΠΌΠ΅ΠΆΠ΄Ρƒ напряТСниСм ΠΈ Ρ‚ΠΎΠΊΠΎΠΌ Π»ΠΈΠ½Π΅ΠΉΠ½ΠΎ ΠΏΡ€ΠΎΠΏΠΎΡ€Ρ†ΠΈΠΎΠ½Π°Π»ΡŒΠ½Π°, Ρ‚. , производная ΠΏΠΎΠΊΠ°Π·Ρ‹Π²Π°Π΅Ρ‚, Ρ‡Ρ‚ΠΎ константой ΠΏΡ€ΠΎΠΏΠΎΡ€Ρ†ΠΈΠΎΠ½Π°Π»ΡŒΠ½ΠΎΡΡ‚ΠΈ являСтся сопротивлСниС. Для Π΄Ρ€ΡƒΠ³ΠΈΡ… Π²ΠΈΠ΄ΠΎΠ² зависимости ΠΌΡ‹ Π½Π°Ρ…ΠΎΠ΄ΠΈΠΌ Π΄Ρ€ΡƒΠ³ΠΈΠ΅ Ρ„ΡƒΠ½ΠΊΡ†ΠΈΠΈ. На Π³Ρ€Π°Ρ„ΠΈΠΊΠ΅ Π½ΠΈΠΆΠ΅ ΠΏΠΎΠΊΠ°Π·Π°Π½ΠΎ, ΠΏΠΎΡ‡Π΅ΠΌΡƒ Π·Π°ΠΊΠΎΠ½ Ома ΠΏΡ€ΠΈΠΌΠ΅Ρ€Π½ΠΎ справСдлив Π² нСбольшом Π΄ΠΈΠ°ΠΏΠ°Π·ΠΎΠ½Π΅ Π·Π½Π°Ρ‡Π΅Π½ΠΈΠΉ Ρ‚ΠΎΠΊΠ° ΠΈ разности ΠΏΠΎΡ‚Π΅Π½Ρ†ΠΈΠ°Π»ΠΎΠ².

ЛинСйная аппроксимация, commons. wikimedia.org

Для ΠΎΠ±Ρ‰Π΅ΠΉ Ρ„ΡƒΠ½ΠΊΡ†ΠΈΠΈ (Π·Π΅Π»Π΅Π½ΠΎΠΉ), которая Π½Π΅ являСтся прямой Π»ΠΈΠ½ΠΈΠ΅ΠΉ, ΠΌΡ‹ всСгда ΠΌΠΎΠΆΠ΅ΠΌ ΠΎΠ³Ρ€Π°Π½ΠΈΡ‡ΠΈΡ‚ΡŒΡΡ ΠΎΡ‡Π΅Π½ΡŒ нСбольшим Π΄ΠΈΠ°ΠΏΠ°Π·ΠΎΠ½ΠΎΠΌ, Π³Π΄Π΅ связь ΠΌΠΎΠΆΠ΅Ρ‚ Π±Ρ‹Ρ‚ΡŒ ΠΎΡ†Π΅Π½Π΅Π½Π° Π»ΠΈΠ½Π΅ΠΉΠ½ΠΎΠΉ Π·Π°Π²ΠΈΡΠΈΠΌΠΎΡΡ‚ΡŒΡŽ, Ρ‚. Π΅. прямой линия. Π§Π΅ΠΌ мСньшС Π΄ΠΈΠ°ΠΏΠ°Π·ΠΎΠ½, Ρ‚Π΅ΠΌ Π»ΡƒΡ‡ΡˆΠ΅ ΠΏΡ€ΠΈΠ±Π»ΠΈΠΆΠ΅Π½ΠΈΠ΅.

Если зСлСная функция Π²Ρ‹ΡˆΠ΅ ΠΎΡ‚Ρ€Π°ΠΆΠ°Π΅Ρ‚ Π·Π°Π²ΠΈΡΠΈΠΌΠΎΡΡ‚ΡŒ ΠΌΠ΅ΠΆΠ΄Ρƒ напряТСниСм ΠΈ элСктричСским Ρ‚ΠΎΠΊΠΎΠΌ, ΠΌΡ‹ Π²ΠΈΠ΄ΠΈΠΌ, Ρ‡Ρ‚ΠΎ для нСбольшого Π΄ΠΈΠ°ΠΏΠ°Π·ΠΎΠ½Π°, Π³Π΄Π΅ напряТСниС ΠΈ Ρ‚ΠΎΠΊ ΠΈΠ·ΠΌΠ΅Π½ΡΡŽΡ‚ΡΡ Π½Π΅Π·Π½Π°Ρ‡ΠΈΡ‚Π΅Π»ΡŒΠ½ΠΎ, функция аппроксимируСтся красной Π»ΠΈΠ½ΠΈΠ΅ΠΉ. Π—Π°Ρ‚Π΅ΠΌ ΠΌΡ‹ ΠΌΠΎΠΆΠ΅ΠΌ ΠΈΡΠΏΠΎΠ»ΡŒΠ·ΠΎΠ²Π°Ρ‚ΡŒ Π·Π°ΠΊΠΎΠ½ Ома для опрСдСлСния сопротивлСния Π±Π΅Π· нСобходимости Π΄ΠΈΡ„Ρ„Π΅Ρ€Π΅Π½Ρ†ΠΈΡ€ΠΎΠ²Π°Ρ‚ΡŒ.

Π’ΠΎΠ»ΡŒΡ‚-Π°ΠΌΠΏΠ΅Ρ€Π½Ρ‹Π΅ характСристики

Π’ΠΎΠ»ΡŒΡ‚-Π°ΠΌΠΏΠ΅Ρ€Π½Ρ‹Π΅ характСристики – это ΠΊΡ€ΠΈΠ²Ρ‹Π΅, ΠΎΠΏΡ€Π΅Π΄Π΅Π»ΡΡŽΡ‰ΠΈΠ΅ Π·Π°Π²ΠΈΡΠΈΠΌΠΎΡΡ‚ΡŒ ΠΌΠ΅ΠΆΠ΄Ρƒ элСктричСским Ρ‚ΠΎΠΊΠΎΠΌ ΠΈ Ρ€Π°Π·Π½ΠΎΡΡ‚ΡŒΡŽ ΠΏΠΎΡ‚Π΅Π½Ρ†ΠΈΠ°Π»ΠΎΠ² устройства.

Π”Π°Π²Π°ΠΉΡ‚Π΅ ΠΈΠ·ΡƒΡ‡ΠΈΠΌ нСсколько ΠΏΡ€ΠΈΠΌΠ΅Ρ€ΠΎΠ² этих ΠΊΡ€ΠΈΠ²Ρ‹Ρ… Π² Ρ€Π°Π·Π½Ρ‹Ρ… устройствах ΠΈ выясним, ΠΊΠ°ΠΊΠΈΠ΅ Π²Ρ‹Π²ΠΎΠ΄Ρ‹ ΠΌΡ‹ ΠΌΠΎΠΆΠ΅ΠΌ ΠΈΠ· Π½ΠΈΡ… ΡΠ΄Π΅Π»Π°Ρ‚ΡŒ.

Π’ΠΎΠ»ΡŒΡ‚-Π°ΠΌΠΏΠ΅Ρ€Π½Ρ‹Π΅ характСристики омичСского рСзистора

Π’ΠΎΠ»ΡŒΡ‚-Π°ΠΌΠΏΠ΅Ρ€Π½Ρ‹Π΅ характСристики омичСских рСзисторов :

  • Π“Ρ€Π°Ρ„ΠΈΠΊ ВАΠ₯ для омичСского рСзистора прСдставляСт собой ΠΏΡ€ΡΠΌΡƒΡŽ линию .
  • ΠšΡ€ΠΈΠ²Π°Ρ ΠΏΡ€ΠΎΡ…ΠΎΠ΄ΠΈΡ‚ Ρ‡Π΅Ρ€Π΅Π· Π½Π°Ρ‡Π°Π»ΠΎ ΠΊΠΎΠΎΡ€Π΄ΠΈΠ½Π°Ρ‚ , Ρ‡Ρ‚ΠΎ ΠΎΠ·Π½Π°Ρ‡Π°Π΅Ρ‚, Ρ‡Ρ‚ΠΎ ΠΏΡ€ΠΈ Π½ΡƒΠ»Π΅Π²ΠΎΠΉ разности ΠΏΠΎΡ‚Π΅Π½Ρ†ΠΈΠ°Π»ΠΎΠ² ΠΌΡ‹ ΠΈΠΌΠ΅Π΅ΠΌ Π½ΡƒΠ»Π΅Π²ΠΎΠΉ Ρ‚ΠΎΠΊ.
  • Π’ΠΎΠΊ прямо ΠΏΡ€ΠΎΠΏΠΎΡ€Ρ†ΠΈΠΎΠ½Π°Π»Π΅Π½ разности ΠΏΠΎΡ‚Π΅Π½Ρ†ΠΈΠ°Π»ΠΎΠ². ΠšΠΎΠ½ΡΡ‚Π°Π½Ρ‚Π° ΠΏΡ€ΠΎΠΏΠΎΡ€Ρ†ΠΈΠΎΠ½Π°Π»ΡŒΠ½ΠΎΡΡ‚ΠΈ – это сопротивлСниС.

Π“Ρ€Π°Ρ„ΠΈΠΊ ВАΠ₯ для омичСского рСзистора прСдставляСт собой ΠΏΡ€ΡΠΌΡƒΡŽ линию .

Π’ΠΎΠ»ΡŒΡ‚-Π°ΠΌΠΏΠ΅Ρ€Π½Ρ‹Π΅ характСристики Π½ΠΈΡ‚ΠΈ Π½Π°ΠΊΠ°Π»Π°

Нити β€” это ΠΌΠ°Ρ‚Π΅Ρ€ΠΈΠ°Π»Ρ‹, ΠΈΡΠΏΠΎΠ»ΡŒΠ·ΡƒΠ΅ΠΌΡ‹Π΅ Π² Π»Π°ΠΌΠΏΠ°Ρ… , ΠΊΠΎΡ‚ΠΎΡ€Ρ‹Π΅ состоят ΠΈΠ· ΠΌΠ΅Ρ‚Π°Π»Π»ΠΎΠ², ΠΊΠΎΡ‚ΠΎΡ€Ρ‹Π΅ свСтятся, ΠΊΠΎΠ³Π΄Π° Ρ‡Π΅Ρ€Π΅Π· Π½ΠΈΡ… ΠΏΡ€ΠΎΡ…ΠΎΠ΄ΠΈΡ‚ ΠΎΠΏΡ€Π΅Π΄Π΅Π»Π΅Π½Π½Ρ‹ΠΉ Ρ‚ΠΎΠΊ. Нити Π½Π°ΠΊΠ°Π»Π° — это Ρ‚ΠΈΠΏ элСктричСского устройства, Π½Π°Π·Ρ‹Π²Π°Π΅ΠΌΠΎΠ³ΠΎ тСрмистором , , ΠΊΠΎΡ‚ΠΎΡ€Ρ‹ΠΉ прСдставляСт собой ΠΌΠ°Ρ‚Π΅Ρ€ΠΈΠ°Π», сопротивлСниС ΠΊΠΎΡ‚ΠΎΡ€ΠΎΠ³ΠΎ зависит ΠΎΡ‚ Π΅Π³ΠΎ Ρ‚Π΅ΠΌΠΏΠ΅Ρ€Π°Ρ‚ΡƒΡ€Ρ‹.

ΠŸΠΎΡΠΊΠΎΠ»ΡŒΠΊΡƒ сопротивлСниС Ρ‡ΡƒΠ²ΡΡ‚Π²ΠΈΡ‚Π΅Π»ΡŒΠ½ΠΎ ΠΊ Ρ‚Π΅ΠΏΠ»Ρƒ, Π° Ρ‚ΠΎΠΊ Π½Π°Π³Ρ€Π΅Π²Π°Π΅Ρ‚ ΠΌΠ°Ρ‚Π΅Ρ€ΠΈΠ°Π», протСкая Ρ‡Π΅Ρ€Π΅Π· Π½Π΅Π³ΠΎ, сопротивлСниС измСнится. Π­Ρ‚ΠΎΡ‚ эффСкт Π½Π°Π±Π»ΡŽΠ΄Π°Π΅Ρ‚ΡΡ Π½Π° ВАΠ₯ Π½ΠΈΡ‚Π΅ΠΉ. ВСхничСски всС ΠΌΠ°Ρ‚Π΅Ρ€ΠΈΠ°Π»Ρ‹ Π²Π΅Π΄ΡƒΡ‚ сСбя Ρ‚Π°ΠΊΠΈΠΌ ΠΎΠ±Ρ€Π°Π·ΠΎΠΌ, Π½ΠΎ Π½Π΅ΠΊΠΎΡ‚ΠΎΡ€Ρ‹Π΅ Π²Π΅Π΄ΡƒΡ‚ сСбя Π² ΠΎΡ‡Π΅Π½ΡŒ ΡƒΠΌΠ΅Ρ€Π΅Π½Π½ΠΎΠΌ ΠΌΠ°ΡΡˆΡ‚Π°Π±Π΅, ΠΊΠΎΡ‚ΠΎΡ€Ρ‹ΠΉ ΠΌΡ‹ Π½Π΅ ΠΌΠΎΠΆΠ΅ΠΌ ΠΈΠ·ΠΌΠ΅Ρ€ΠΈΡ‚ΡŒ.

Π’ΠΎΠ»ΡŒΡ‚-Π°ΠΌΠΏΠ΅Ρ€Π½Ρ‹Π΅ характСристики Π»Π°ΠΌΠΏΡ‹ накаливания:

  • Π“Ρ€Π°Ρ„ΠΈΠΊ I-V ΠΏΠΎΠΊΠ°Π·Ρ‹Π²Π°Π΅Ρ‚ рост Ρ‚ΠΎΠΊΠ° с мСньшСй ΡΠΊΠΎΡ€ΠΎΡΡ‚ΡŒΡŽ, Ρ‡Π΅ΠΌ Ρ€Π°Π·Π½ΠΎΡΡ‚ΡŒ ΠΏΠΎΡ‚Π΅Π½Ρ†ΠΈΠ°Π»ΠΎΠ² (напряТСниС).
  • Π’ Π΄ΠΈΠ°ΠΏΠ°Π·ΠΎΠ½Π°Ρ…, Π³Π΄Π΅ напряТСниС Π½Π΅ слишком сильноС, Ρ‚ΠΎΠΊ Π½Π΅ ΠΎΡ‡Π΅Π½ΡŒ ΡΠΈΠ»ΡŒΠ½Ρ‹ΠΉ ΠΈ Ρ‚Π΅ΠΌΠΏΠ΅Ρ€Π°Ρ‚ΡƒΡ€Π° Π½Π΅ ΠΎΡ‡Π΅Π½ΡŒ высокая. Π­Ρ‚ΠΎ ΠΎΠ·Π½Π°Ρ‡Π°Π΅Ρ‚, Ρ‡Ρ‚ΠΎ сопротивлСниС Π½Π΅ высокоС ΠΈ Ρ‚ΠΎΠΊ ΠΌΠΎΠΆΠ΅Ρ‚ ΠΏΡ€ΠΎΡ‚Π΅ΠΊΠ°Ρ‚ΡŒ Π»Π΅Π³ΠΊΠΎ.
  • Π’ Π΄ΠΈΠ°ΠΏΠ°Π·ΠΎΠ½Π°Ρ… с высоким напряТСниСм (ΠΏΠΎΠ»ΠΎΠΆΠΈΡ‚Π΅Π»ΡŒΠ½Ρ‹ΠΌ ΠΈΠ»ΠΈ ΠΎΡ‚Ρ€ΠΈΡ†Π°Ρ‚Π΅Π»ΡŒΠ½Ρ‹ΠΌ) Π³Π΅Π½Π΅Ρ€ΠΈΡ€ΡƒΠ΅ΠΌΡ‹ΠΉ Ρ‚ΠΎΠΊ ΠΎΡ‡Π΅Π½ΡŒ Π²Π΅Π»ΠΈΠΊ ΠΈ Ρ‚Π΅ΠΌΠΏΠ΅Ρ€Π°Ρ‚ΡƒΡ€Π° быстро растСт. ΠŸΠΎΡΠΊΠΎΠ»ΡŒΠΊΡƒ Ρ‚Π΅ΠΌΠΏΠ΅Ρ€Π°Ρ‚ΡƒΡ€Π° ΠΏΠΎΠ²Ρ‹ΡˆΠ°Π΅Ρ‚ΡΡ, сопротивлСниС увСличиваСтся ΠΈ ΠΏΡ€ΠΎΡ‚Π΅ΠΊΠ°Π½ΠΈΠ΅ Ρ‚ΠΎΠΊΠ° ΡƒΠΌΠ΅Π½ΡŒΡˆΠ°Π΅Ρ‚ΡΡ. ΠŸΡ€ΠΈ достаточно высоком напряТСнии достигаСтся ΠΌΠ°ΠΊΡΠΈΠΌΠ°Π»ΡŒΠ½Ρ‹ΠΉ Ρ‚ΠΎΠΊ.

Для Π»Π°ΠΌΠΏ накаливания Π³Ρ€Π°Ρ„ΠΈΠΊ I-V ΠΏΠΎΠΊΠ°Π·Ρ‹Π²Π°Π΅Ρ‚ ΡƒΠ²Π΅Π»ΠΈΡ‡Π΅Π½ΠΈΠ΅ Ρ‚ΠΎΠΊΠ° с мСньшСй ΡΠΊΠΎΡ€ΠΎΡΡ‚ΡŒΡŽ , Ρ‡Π΅ΠΌ Ρ€Π°Π·Π½ΠΎΡΡ‚ΡŒ ΠΏΠΎΡ‚Π΅Π½Ρ†ΠΈΠ°Π»ΠΎΠ² (напряТСниС).

Π’ΠΎΠ»ΡŒΡ‚-ампСрная характСристика Π΄ΠΈΠΎΠ΄Π°

Π”ΠΈΠΎΠ΄ прСдставляСт собой ΠΏΠΎΠ»ΡƒΠΏΡ€ΠΎΠ²ΠΎΠ΄Π½ΠΈΠΊ , ΠΏΡ€ΠΎΠΏΡƒΡΠΊΠ°ΡŽΡ‰ΠΈΠΉ Ρ‚ΠΎΠΊ Π² ΠΎΠΏΡ€Π΅Π΄Π΅Π»Π΅Π½Π½ΠΎΠΌ Π½Π°ΠΏΡ€Π°Π²Π»Π΅Π½ΠΈΠΈ (Π½ΠΎ Π½Π΅ Π² ΠΏΡ€ΠΎΡ‚ΠΈΠ²ΠΎΠΏΠΎΠ»ΠΎΠΆΠ½ΠΎΠΌ). Он Ρ€Π°Π±ΠΎΡ‚Π°Π΅Ρ‚ ΠΊΠ°ΠΊ ΠΏΡ€ΠΎΠ²ΠΎΠ΄Π½ΠΈΠΊ ΠΈΠ»ΠΈ ΠΎΡ‡Π΅Π½ΡŒ Ρ…ΠΎΡ€ΠΎΡˆΠΈΠΉ рСзистор Π² зависимости ΠΎΡ‚ направлСния Ρ‚ΠΎΠΊΠ°.

Π’ΠΎΠ»ΡŒΡ‚-Π°ΠΌΠΏΠ΅Ρ€Π½Ρ‹Π΅ характСристики Π΄ΠΈΠΎΠ΄ΠΎΠ²:

  • ΠŸΡ€ΠΈ ΠΏΡ€ΠΎΡ‚Π΅ΠΊΠ°Π½ΠΈΠΈ Ρ‚ΠΎΠΊΠ° Π² Π½Π°ΠΏΡ€Π°Π²Π»Π΅Π½ΠΈΠΈ Ρ€Π°Π±ΠΎΡ‚Ρ‹ ΠΏΡ€ΠΎΠ²ΠΎΠ΄Π½ΠΈΠΊΠ° (ΠΏΠΎΠ»ΠΎΠΆΠΈΡ‚Π΅Π»ΡŒΠ½Π°Ρ Ρ€Π°Π·Π½ΠΎΡΡ‚ΡŒ ΠΏΠΎΡ‚Π΅Π½Ρ†ΠΈΠ°Π»ΠΎΠ²) происходит Ρ€Π΅Π·ΠΊΠΎΠ΅ ΡƒΠ²Π΅Π»ΠΈΡ‡Π΅Π½ΠΈΠ΅ Ρ‚ΠΎΠΊΠ° послС ΠΎΠΏΡ€Π΅Π΄Π΅Π»Π΅Π½Π½Ρ‹Ρ… Π·Π½Π°Ρ‡Π΅Π½ΠΈΠΉ напряТСния, Π° сопротивлСния Ρ€Π΅Π·ΠΊΠΎ сниТаСтся. Π­Ρ‚ΠΎ дСлаСтся для ΠΏΠΎΡ€ΠΎΠ³ΠΎΠ²ΠΎΠ³ΠΎ значСния, ΠΊΠΎΡ‚ΠΎΡ€ΠΎΠ΅ опрСдСляСт, ΠΊΠΎΠ³Π΄Π° Π΄ΠΈΠΎΠ΄ Π½Π°Ρ‡ΠΈΠ½Π°Π΅Ρ‚ ΠΏΡ€ΠΎΠ²ΠΎΠ΄ΠΈΡ‚ΡŒ элСктричСство.
  • Когда Ρ‚ΠΎΠΊ пытаСтся Ρ‚Π΅Ρ‡ΡŒ Π² Π½Π°ΠΏΡ€Π°Π²Π»Π΅Π½ΠΈΠΈ, ΠΊΠΎΡ‚ΠΎΡ€ΠΎΠ΅ Π²Π΅Π΄Π΅Ρ‚ сСбя ΠΊΠ°ΠΊ рСзистор (ΠΎΡ‚Ρ€ΠΈΡ†Π°Ρ‚Π΅Π»ΡŒΠ½Π°Ρ Ρ€Π°Π·Π½ΠΎΡΡ‚ΡŒ ΠΏΠΎΡ‚Π΅Π½Ρ†ΠΈΠ°Π»ΠΎΠ²), Ρ‚ΠΎΠΊ ΠΏΡ€ΠΈΠ±Π»ΠΈΠ·ΠΈΡ‚Π΅Π»ΡŒΠ½ΠΎ отсутствуСт. Π‘ΠΎΠΏΡ€ΠΎΡ‚ΠΈΠ²Π»Π΅Π½ΠΈΠ΅ Π±Π»ΠΈΠ·ΠΊΠΎ ΠΊ бСсконСчности.

Π”ΠΈΠΎΠ΄Ρ‹ ΠΌΠΎΠ³ΡƒΡ‚ Ρ€Π°Π±ΠΎΡ‚Π°Ρ‚ΡŒ ΠΊΠ°ΠΊ ΠΏΡ€ΠΎΠ²ΠΎΠ΄Π½ΠΈΠΊ ΠΈΠ»ΠΈ ΠΎΡ‡Π΅Π½ΡŒ Ρ…ΠΎΡ€ΠΎΡˆΠΈΠΉ рСзистор Π² зависимости ΠΎΡ‚ направлСния Ρ‚ΠΎΠΊΠ°.

Π’ΠΎΠ»ΡŒΡ‚-Π°ΠΌΠΏΠ΅Ρ€Π½Ρ‹Π΅ характСристики солнСчного фотоэлСктричСского элСмСнта

Π‘ΠΎΠ»Π½Π΅Ρ‡Π½Ρ‹ΠΉ фотоэлСктричСский элСмСнт прСдставляСт собой устройство, ΠΏΡ€Π΅ΠΎΠ±Ρ€Π°Π·ΡƒΡŽΡ‰Π΅Π΅ свСт Π² ΡΠ»Π΅ΠΊΡ‚Ρ€ΠΈΡ‡Π΅ΡΠΊΡƒΡŽ ΡΠ½Π΅Ρ€Π³ΠΈΡŽ . Π˜Ρ… Ρ€Π°Π±ΠΎΡ‚Π° основана Π½Π° фотоэлСктричСском эффСктС: высвобоТдСнии элСктронов ΠΌΠ°Ρ‚Π΅Ρ€ΠΈΠ°Π»ΠΎΠΌ ΠΏΠΎΠ΄ воздСйствиСм элСктромагнитного излучСния ΠΎΠΏΡ€Π΅Π΄Π΅Π»Π΅Π½Π½ΠΎΠ³ΠΎ частотного Π΄ΠΈΠ°ΠΏΠ°Π·ΠΎΠ½Π°. Π§Π΅ΠΌ Π²Ρ‹ΡˆΠ΅ частота свСта, Ρ‚Π΅ΠΌ интСнсивнСС ΠΈΠ½Π΄ΡƒΡ†ΠΈΡ€ΡƒΠ΅ΠΌΡ‹ΠΉ элСктричСский Ρ‚ΠΎΠΊ.

Π’ΠΎΠ»ΡŒΡ‚-Π°ΠΌΠΏΠ΅Ρ€Π½Ρ‹Π΅ характСристики солнСчных Ρ„ΠΎΡ‚ΠΎΠ³Π°Π»ΡŒΠ²Π°Π½ΠΈΡ‡Π΅ΡΠΊΠΈΡ… элСмСнтов Π½Π΅ΠΌΠ½ΠΎΠ³ΠΎ ΠΎΡ‚Π»ΠΈΡ‡Π°ΡŽΡ‚ΡΡ, ΠΏΠΎΡ‚ΠΎΠΌΡƒ Ρ‡Ρ‚ΠΎ Π² этом случаС ΠΌΡ‹ ΠΈΠΌΠ΅Π΅ΠΌ ΠΊΠΎΠ½Ρ‚Ρ€ΠΎΠ»ΡŒ Π½Π°Π΄ Π³Π΅Π½Π΅Ρ€ΠΈΡ€ΡƒΠ΅ΠΌΡ‹ΠΌ Ρ‚ΠΎΠΊΠΎΠΌ ed , ΠΈ наша Ρ†Π΅Π»ΡŒ состоит Π² Ρ‚ΠΎΠΌ, Ρ‡Ρ‚ΠΎΠ±Ρ‹ ΡΠΎΠ·Π΄Π°Ρ‚ΡŒ Ρ€Π°Π·Π½ΠΎΡΡ‚ΡŒ ΠΏΠΎΡ‚Π΅Π½Ρ†ΠΈΠ°Π»ΠΎΠ².

  • Π’ области ΠΏΠΎΠ»ΠΎΠΆΠΈΡ‚Π΅Π»ΡŒΠ½ΠΎΠΉ разности ΠΏΠΎΡ‚Π΅Π½Ρ†ΠΈΠ°Π»ΠΎΠ² Ρ‚ΠΎΠΊ ΠΌΠΎΠΆΠ΅Ρ‚ ΠΏΡ€ΠΎΠΈΠ·Π²ΠΎΠ»ΡŒΠ½ΠΎ Π²ΠΎΠ·Ρ€Π°ΡΡ‚Π°Ρ‚ΡŒ ΠΈ появится постоянная Ρ€Π°Π·Π½ΠΎΡΡ‚ΡŒ ΠΏΠΎΡ‚Π΅Π½Ρ†ΠΈΠ°Π»ΠΎΠ² e . ΠœΡ‹ Π½Π΅ ΠΌΠΎΠΆΠ΅ΠΌ эффСктивно ΠΈΡΠΏΠΎΠ»ΡŒΠ·ΠΎΠ²Π°Ρ‚ΡŒ Π΅Π³ΠΎ Π² этом Ρ€Π΅Π³ΠΈΠΎΠ½Π΅. Π­Ρ‚ΠΎ ΠΎΠ±Π»Π°ΡΡ‚ΡŒ, Π³Π΄Π΅ ΠΌΠ°Ρ‚Π΅Ρ€ΠΈΠ°Π» Π½Π΅ ΠΏΠΎΠ»ΡƒΡ‡Π°Π΅Ρ‚ свСта.
  • По ΠΌΠ΅Ρ€Π΅ Ρ‚ΠΎΠ³ΠΎ, ΠΊΠ°ΠΊ количСство ΠΏΠ°Π΄Π°ΡŽΡ‰Π΅Π³ΠΎ свСта Π½Π°Ρ‡ΠΈΠ½Π°Π΅Ρ‚ расти, Ρ‚ΠΎΠΊ становится всС Π±ΠΎΠ»Π΅Π΅ ΠΈ Π±ΠΎΠ»Π΅Π΅ ΠΎΡ‚Ρ€ΠΈΡ†Π°Ρ‚Π΅Π»ΡŒΠ½Ρ‹ΠΌ, ΠΈ появляСтся ΠΎΡ‚Ρ€ΠΈΡ†Π°Ρ‚Π΅Π»ΡŒΠ½Π°Ρ Ρ€Π°Π·Π½ΠΎΡΡ‚ΡŒ ΠΏΠΎΡ‚Π΅Π½Ρ†ΠΈΠ°Π»ΠΎΠ² , которая ΠΌΠΎΠΆΠ΅Ρ‚ расти ΠΏΡ€ΠΎΠΈΠ·Π²ΠΎΠ»ΡŒΠ½ΠΎ Π² зависимости ΠΎΡ‚ характСристик свСта ΠΈ ΠΌΠ°Ρ‚Π΅Ρ€ΠΈΠ°Π»Π°.

Π“Ρ€Π°Ρ„ΠΈΠΊΠΈ ВАΠ₯ для рСзистора, Π΄ΠΈΠΎΠ΄Π° ΠΈ Π±Π°Ρ‚Π°Ρ€Π΅ΠΈ, commons.wikimedia.org

Π’ΠΎΠ»ΡŒΡ‚-Π°ΠΌΠΏΠ΅Ρ€Π½Ρ‹Π΅ характСристики β€” основныС Π²Ρ‹Π²ΠΎΠ΄Ρ‹ линСйная ΠΈ, ΠΊΠ°ΠΊ ΠΏΡ€Π°Π²ΠΈΠ»ΠΎ, постоянная. Π­Ρ‚ΠΎ ΠΏΡ€ΠΈΠ±Π»ΠΈΠΆΠ΅Π½ΠΈΠ΅ ΠΊ повСдСнию Π±ΠΎΠ»ΡŒΡˆΠΈΠ½ΡΡ‚Π²Π° ΠΌΠ°Ρ‚Π΅Ρ€ΠΈΠ°Π»ΠΎΠ².

  • Π—Π°Π²ΠΈΡΠΈΠΌΠΎΡΡ‚ΡŒ ΠΌΠ΅ΠΆΠ΄Ρƒ напряТСниСм ΠΈ Ρ‚ΠΎΠΊΠΎΠΌ Π½Π΅Π»ΠΈΠ½Π΅ΠΉΠ½Π°. Он опрСдСляСтся сопротивлСниСм, ΠΊΠΎΡ‚ΠΎΡ€ΠΎΠ΅ измСряСт прСпятствиС срСды ΠΏΡ€ΠΎΡ‚Π΅ΠΊΠ°Π½ΠΈΡŽ Ρ‚ΠΎΠΊΠ°.
  • ПолСзно ΠΈΠ·ΡƒΡ‡ΠΈΡ‚ΡŒ Π²ΠΎΠ»ΡŒΡ‚Π°ΠΌΠΏΠ΅Ρ€Π½Ρ‹Π΅ характСристики ΠΈΠ»ΠΈ ВАΠ₯ Ρ€Π°Π·Π»ΠΈΡ‡Π½Ρ‹Ρ… устройств ΠΈ ΠΌΠ°Ρ‚Π΅Ρ€ΠΈΠ°Π»ΠΎΠ², Ρ‡Ρ‚ΠΎΠ±Ρ‹ ΠΏΠΎΠ½ΡΡ‚ΡŒ, ΠΊΠ°ΠΊ ΠΎΠ½ΠΈ Ρ€Π°Π±ΠΎΡ‚Π°ΡŽΡ‚.
  • Π”ΠΈΠΎΠ΄Ρ‹, Π½ΠΈΡ‚ΠΈ Π½Π°ΠΊΠ°Π»Π° ΠΈ Ρ„ΠΎΡ‚ΠΎΠ³Π°Π»ΡŒΠ²Π°Π½ΠΈΡ‡Π΅ΡΠΊΠΈΠ΅ элСмСнты ΡΠ²Π»ΡΡŽΡ‚ΡΡ Ρ…ΠΎΡ€ΠΎΡˆΠΈΠΌΠΈ ΠΏΡ€ΠΈΠΌΠ΅Ρ€Π°ΠΌΠΈ нСомичСских устройств, ΠΊΠΎΡ‚ΠΎΡ€Ρ‹Π΅ слуТат Ρ€Π°Π·Π»ΠΈΡ‡Π½Ρ‹ΠΌ цСлям.
  • ΠžΠ±Π·ΠΎΡ€ элСктричСского Ρ‚ΠΎΠΊΠ°, напряТСния ΠΈ сопротивлСния | Π’Ρ€ΠΈ основныС элСктричСскиС Π²Π΅Π»ΠΈΡ‡ΠΈΠ½Ρ‹

    Π₯ΠΎΡ‚ΠΈΡ‚Π΅ ΡΠΎΠ·Π΄Π°Ρ‚ΡŒ сайт? НайдитС бСсплатныС Ρ‚Π΅ΠΌΡ‹ ΠΈ ΠΏΠ»Π°Π³ΠΈΠ½Ρ‹ WordPress.

    ЭлСктричСство Π’ΠΎΠΊ, напряТСниС ΠΈ сопротивлСниС ΡΠ²Π»ΡΡŽΡ‚ΡΡ трСмя основными элСктричСскими свойствами. ΠŸΡ€ΠΎΡ‰Π΅ говоря,

    β€’ Ρ‚ΠΎΠΊ : это Π½Π°ΠΏΡ€Π°Π²Π»Π΅Π½Π½Ρ‹ΠΉ ΠΏΠΎΡ‚ΠΎΠΊ заряда Ρ‡Π΅Ρ€Π΅Π· ΠΏΡ€ΠΎΠ²ΠΎΠ΄Π½ΠΈΠΊ.
    β€’ НапряТСниС : сила, ΡΠΎΠ·Π΄Π°ΡŽΡ‰Π°Ρ Ρ‚ΠΎΠΊ.
    β€’ Π‘ΠΎΠΏΡ€ΠΎΡ‚ΠΈΠ²Π»Π΅Π½ΠΈΠ΅ : сопротивлСниС Ρ‚ΠΎΠΊΡƒ, обСспСчиваСмому ΠΌΠ°Ρ‚Π΅Ρ€ΠΈΠ°Π»ΠΎΠΌ, ΠΊΠΎΠΌΠΏΠΎΠ½Π΅Π½Ρ‚ΠΎΠΌ ΠΈΠ»ΠΈ схСмой.

    ЭлСктричСский Ρ‚ΠΎΠΊ, напряТСниС ΠΈ сопротивлСниС β€” Ρ‚Ρ€ΠΈ основных свойства элСктричСской Ρ†Π΅ΠΏΠΈ. ΠžΡ‚Π½ΠΎΡˆΠ΅Π½ΠΈΡ ΠΌΠ΅ΠΆΠ΄Ρƒ Π½ΠΈΠΌΠΈ ΠΎΠΏΡ€Π΅Π΄Π΅Π»ΡΡŽΡ‚ΡΡ основным Π·Π°ΠΊΠΎΠ½ΠΎΠΌ Ρ€Π°Π±ΠΎΡ‚Ρ‹ Ρ†Π΅ΠΏΠΈ, Π½Π°Π·Ρ‹Π²Π°Π΅ΠΌΡ‹ΠΌ 9.0031 Π—Π°ΠΊΠΎΠ½ Ома .

    ЭлСктричСский Ρ‚ΠΎΠΊ

    Как Π²Ρ‹ Π·Π½Π°Π΅Ρ‚Π΅, внСшняя сила ΠΌΠΎΠΆΠ΅Ρ‚ ΠΎΡ‚ΠΎΡ€Π²Π°Ρ‚ΡŒ элСктрон ΠΎΡ‚ Ρ€ΠΎΠ΄ΠΈΡ‚Π΅Π»ΡŒΡΠΊΠΎΠ³ΠΎ Π°Ρ‚ΠΎΠΌΠ°. Π’ ΠΌΠ΅Π΄ΠΈ (ΠΈ Π΄Ρ€ΡƒΠ³ΠΈΡ… ΠΌΠ΅Ρ‚Π°Π»Π»Π°Ρ…) трСбуСтся ΠΎΡ‡Π΅Π½ΡŒ нСбольшая внСшняя сила для образования свободных элСктронов. На самом Π΄Π΅Π»Π΅ тСпловая энСргия (Ρ‚Π΅ΠΏΠ»ΠΎ), ΠΏΡ€ΠΈΡΡƒΡ‚ΡΡ‚Π²ΡƒΡŽΡ‰Π°Ρ ΠΏΡ€ΠΈ ΠΊΠΎΠΌΠ½Π°Ρ‚Π½ΠΎΠΉ Ρ‚Π΅ΠΌΠΏΠ΅Ρ€Π°Ρ‚ΡƒΡ€Π΅ (22 0 C), ΠΌΠΎΠΆΠ΅Ρ‚ Π³Π΅Π½Π΅Ρ€ΠΈΡ€ΠΎΠ²Π°Ρ‚ΡŒ свободныС элСктроны. ΠšΠΎΠ»ΠΈΡ‡Π΅ΡΡ‚Π²ΠΎ Π³Π΅Π½Π΅Ρ€ΠΈΡ€ΡƒΠ΅ΠΌΡ‹Ρ… элСктронов Π½Π°ΠΏΡ€ΡΠΌΡƒΡŽ зависит ΠΎΡ‚ Ρ‚Π΅ΠΌΠΏΠ΅Ρ€Π°Ρ‚ΡƒΡ€Ρ‹. Π”Ρ€ΡƒΠ³ΠΈΠΌΠΈ словами, Π±ΠΎΠ»Π΅Π΅ высокиС Ρ‚Π΅ΠΌΠΏΠ΅Ρ€Π°Ρ‚ΡƒΡ€Ρ‹ Π³Π΅Π½Π΅Ρ€ΠΈΡ€ΡƒΡŽΡ‚ большС свободных элСктронов.

    Π”Π²ΠΈΠΆΠ΅Π½ΠΈΠ΅ свободных элСктронов Π² ΠΌΠ΅Π΄ΠΈ являСтся случайным, ΠΊΠΎΠ³Π΄Π° Π½Π΅ примСняСтся Π½Π°ΠΏΡ€Π°Π²Π»ΡΡŽΡ‰Π°Ρ сила. Π’ΠΎ Π΅ΡΡ‚ΡŒ свободныС элСктроны Π² ΠΌΠ΅Π΄ΠΈ Ρ…Π°ΠΎΡ‚ΠΈΡ‡Π½Ρ‹, ΠΊΠΎΠ³Π΄Π° Π½Π΅ примСняСтся Π½Π°ΠΏΡ€Π°Π²Π»ΡΡŽΡ‰Π°Ρ сила. Π’ΠΎ Π΅ΡΡ‚ΡŒ свободныС элСктроны двиТутся Π²ΠΎ всСх направлСниях, ΠΊΠ°ΠΊ ΠΏΠΎΠΊΠ°Π·Π°Π½ΠΎ Π½Π° рис. 1 . ΠŸΠΎΡΠΊΠΎΠ»ΡŒΠΊΡƒ свободныС элСктроны двиТутся Π²ΠΎ всСх направлСниях, этот суммарный ΠΏΠΎΡ‚ΠΎΠΊ элСктронов Π² любом Π½Π°ΠΏΡ€Π°Π²Π»Π΅Π½ΠΈΠΈ Ρ€Π°Π²Π΅Π½ Π½ΡƒΠ»ΡŽ.

    Рисунок 1 Π‘Π»ΡƒΡ‡Π°ΠΉΠ½ΠΎΠ΅ Π΄Π²ΠΈΠΆΠ΅Π½ΠΈΠ΅ элСктронов Π² ΠΌΠ΅Π΄ΠΈ

    Рисунок 2 ΠŸΠΎΠΊΠ°Π·Ρ‹Π²Π°Π΅Ρ‚, Ρ‡Ρ‚ΠΎ происходит, ΠΊΠΎΠ³Π΄Π° внСшняя сила заставляСт всС элСктроны Π΄Π²ΠΈΠ³Π°Ρ‚ΡŒΡΡ Π² ΠΎΠ΄Π½ΠΎΠΌ Π½Π°ΠΏΡ€Π°Π²Π»Π΅Π½ΠΈΠΈ. Π’ этом случаС ΠΊ ΠΎΠ΄Π½ΠΎΠΌΡƒ ΠΊΠΎΠ½Ρ†Ρƒ ΠΌΠ΅Π΄ΠΈ прикладываСтся ΠΎΡ‚Ρ€ΠΈΡ†Π°Ρ‚Π΅Π»ΡŒΠ½Ρ‹ΠΉ ΠΏΠΎΡ‚Π΅Π½Ρ†ΠΈΠ°Π», Π° ΠΊ Π΄Ρ€ΡƒΠ³ΠΎΠΌΡƒ β€” ΠΏΠΎΠ»ΠΎΠΆΠΈΡ‚Π΅Π»ΡŒΠ½Ρ‹ΠΉ. Π’ Ρ€Π΅Π·ΡƒΠ»ΡŒΡ‚Π°Ρ‚Π΅ всС свободныС элСктроны ΠΏΠ΅Ρ€Π΅ΠΌΠ΅Ρ‰Π°ΡŽΡ‚ΡΡ ΠΎΡ‚ ΠΎΡ‚Ρ€ΠΈΡ†Π°Ρ‚Π΅Π»ΡŒΠ½ΠΎΠ³ΠΎ ΠΊ ΠΏΠΎΠ»ΠΎΠΆΠΈΡ‚Π΅Π»ΡŒΠ½ΠΎΠΌΡƒ, ΠΈ ΠΌΡ‹ ΠΌΠΎΠΆΠ΅ΠΌ ΡΠΊΠ°Π·Π°Ρ‚ΡŒ, Ρ‡Ρ‚ΠΎ ΠΈΠΌΠ΅Π΅ΠΌ Π½Π°ΠΏΡ€Π°Π²Π»Π΅Π½Π½Ρ‹ΠΉ ΠΏΠΎΡ‚ΠΎΠΊ заряда (элСктронов). Π­Ρ‚ΠΎΡ‚ Π½Π°ΠΏΡ€Π°Π²Π»Π΅Π½Π½Ρ‹ΠΉ ΠΏΠΎΡ‚ΠΎΠΊ элСктронов называСтся элСктричСский Ρ‚ΠΎΠΊ .

    Рисунок 2 НаправлСнноС Π΄Π²ΠΈΠΆΠ΅Π½ΠΈΠ΅ элСктронов Π² ΠΌΠ΅Π΄ΠΈ.

    Π”Π°Π²Π°ΠΉΡ‚Π΅ посмотрим, Ρ‡Ρ‚ΠΎ происходит Π² Π±ΠΎΠ»Π΅Π΅ ΡˆΠΈΡ€ΠΎΠΊΠΎΠΌ ΠΌΠ°ΡΡˆΡ‚Π°Π±Π΅, ΠΊΠΎΠ³Π΄Π° Π΄Π²ΠΈΠΆΠ΅Π½ΠΈΠ΅ элСктрона направляСтся внСшнСй силой. Π’ Рисунок 3 ΠΎΡ‚Ρ€ΠΈΡ†Π°Ρ‚Π΅Π»ΡŒΠ½Ρ‹ΠΉ ΠΏΠΎΡ‚Π΅Π½Ρ†ΠΈΠ°Π» направляСт ΠΏΠΎΡ‚ΠΎΠΊ элСктронов (Ρ‚ΠΎΠΊ) ΠΊ ΠΏΠΎΠ»ΠΎΠΆΠΈΡ‚Π΅Π»ΡŒΠ½ΠΎΠΌΡƒ ΠΏΠΎΡ‚Π΅Π½Ρ†ΠΈΠ°Π»Ρƒ. Π’ΠΎΠΊ ΠΏΡ€ΠΎΡ…ΠΎΠ΄ΠΈΡ‚ Ρ‡Π΅Ρ€Π΅Π· Π»Π°ΠΌΠΏΡƒ, заставляя Π΅Π΅ ΠΏΡ€ΠΎΠΈΠ·Π²ΠΎΠ΄ΠΈΡ‚ΡŒ свСт ΠΈ Ρ‚Π΅ΠΏΠ»ΠΎ. Π§Π΅ΠΌ интСнсивнСС Ρ‚ΠΎΠΊ (Ρ‚ΠΎ Π΅ΡΡ‚ΡŒ большС Π΅Π³ΠΎ Π·Π½Π°Ρ‡Π΅Π½ΠΈΠ΅), Ρ‚Π΅ΠΌ большС свСта ΠΈ Ρ‚Π΅ΠΏΠ»Π° ΠΏΡ€ΠΎΠΈΠ·Π²ΠΎΠ΄ΠΈΡ‚ Π»Π°ΠΌΠΏΠΎΡ‡ΠΊΠ°.

    Рисунок 3 Π’ΠΎΠΊ Ρ‡Π΅Ρ€Π΅Π· ΠΎΡΠ½ΠΎΠ²Π½ΡƒΡŽ Ρ†Π΅ΠΏΡŒ Π»Π°ΠΌΠΏΡ‹.

    ЭлСктричСский Ρ‚ΠΎΠΊ обозначаСтся Π² Ρ„ΠΎΡ€ΠΌΡƒΠ»Π°Ρ… Π±ΡƒΠΊΠ²ΠΎΠΉ I (Для интСнсивности). Π‘ΠΈΠ»Π° Ρ‚ΠΎΠΊΠ° опрСдСляСтся количСством заряда, ΠΏΡ€ΠΎΡ‚Π΅ΠΊΠ°ΡŽΡ‰Π΅Π³ΠΎ Π² сСкунду. Π§Π΅ΠΌ большС ΠΏΠΎΡ‚ΠΎΠΊ заряда Π² сСкунду, Ρ‚Π΅ΠΌ интСнсивнСС Ρ‚ΠΎΠΊ.

    ΠšΡƒΠ»ΠΎΠ½Ρ‹ ΠΈ АмпСры

    ИзмСнСниС ΠΎΠ΄Π½ΠΎΠ³ΠΎ элСктрона нСдостаточно для получСния практичСской Π΅Π΄ΠΈΠ½ΠΈΡ†Ρ‹ измСрСния заряда. Π‘Π»Π΅Π΄ΠΎΠ²Π°Ρ‚Π΅Π»ΡŒΠ½ΠΎ, ΠšΡƒΠ»ΠΎΠ½ (C) ΠΈΡΠΏΠΎΠ»ΡŒΠ·ΡƒΠ΅Ρ‚ΡΡ ΠΊΠ°ΠΊ основная Π΅Π΄ΠΈΠ½ΠΈΡ†Π° заряда. Один ΠΊΡƒΠ»ΠΎΠ½ Ρ€Π°Π²Π΅Π½ ΠΎΠ±Ρ‰Π΅ΠΌΡƒ заряду 6,25 Γ— 10 18 элСктронов. Когда ΠΎΠ΄ΠΈΠ½ ΠΊΡƒΠ»ΠΎΠ½ заряда ΠΏΡ€ΠΎΡ…ΠΎΠ΄ΠΈΡ‚ Ρ‚ΠΎΡ‡ΠΊΡƒ Π·Π° ΠΎΠ΄Π½Ρƒ сСкунду, ΠΌΡ‹ ΠΈΠΌΠ΅Π΅ΠΌ ΠΎΠ΄ΠΈΠ½ АмпСр (А) элСктричСского Ρ‚ΠΎΠΊΠ°. Π”Ρ€ΡƒΠ³ΠΈΠΌΠΈ словами,

    $\begin{matrix}1\text{ Π°ΠΌΠΏΠ΅Ρ€=1 ΠΊΡƒΠ»ΠΎΠ½ Π² сСкунду} \\Or \\1\text{ A=1 Кл/с} \\\end{matrix}$

    ΠŸΠΎΠ»Π½Ρ‹ΠΉ Ρ‚ΠΎΠΊ, проходящий Ρ‡Π΅Ρ€Π΅Π· Ρ‚ΠΎΡ‡ΠΊΡƒ (Π² Π°ΠΌΠΏΠ΅Ρ€Π°Ρ…), ΠΌΠΎΠΆΠ½ΠΎ Π½Π°ΠΉΡ‚ΠΈ, Ρ€Π°Π·Π΄Π΅Π»ΠΈΠ² ΠΎΠ±Ρ‰ΠΈΠΉ заряд (Π² ΠΊΡƒΠ»ΠΎΠ½Π°Ρ…) Π½Π° врСмя (Π² сСкундах). По Π€ΠΎΡ€ΠΌΡƒΠ»Π΅

    \[\begin{matrix}I=\frac{Q}{t} & {} & \left( 1 \right) \\\end{matrix}\]

    Π“Π΄Π΅

    I= ΠΈΠ½Ρ‚Π΅Π½ΡΠΈΠ²Π½ΠΎΡΡ‚ΡŒ элСктричСского Ρ‚ΠΎΠΊ Π² Π°ΠΌΠΏΠ΅Ρ€Π°Ρ…

    Q= ΠΎΠ±Ρ‰ΠΈΠΉ заряд Π² ΠΊΡƒΠ»ΠΎΠ½Π°Ρ…

    T= врСмя прохоТдСния зарядом Ρ‚ΠΎΡ‡ΠΊΠΈ Π² сСкундах

    Π­Ρ‚ΠΎ ΡΠΎΠΎΡ‚Π½ΠΎΡˆΠ΅Π½ΠΈΠ΅ ΠΏΡ€ΠΎΠΈΠ»Π»ΡŽΡΡ‚Ρ€ΠΈΡ€ΠΎΠ²Π°Π½ΠΎ Π² ΠΏΡ€ΠΈΠΌΠ΅Ρ€Π΅ 1 .

    ΠŸΡ€ΠΈΠΌΠ΅Ρ€ 1

    Π’Ρ€ΠΈ ΠΊΡƒΠ»ΠΎΠ½Π° заряда проходят Ρ‡Π΅Ρ€Π΅Π· ΠΌΠ΅Π΄Π½Ρ‹ΠΉ ΠΏΡ€ΠΎΠ²ΠΎΠ΄ ΠΊΠ°ΠΆΠ΄ΡƒΡŽ сСкунду. Каково Π·Π½Π°Ρ‡Π΅Π½ΠΈΠ΅ элСктричСского Ρ‚ΠΎΠΊΠ°? 9{C}/{}_{s}=3A\]

    ΠŸΡ€ΠΈΠΌΠ΅Ρ€ 1 Π²ΠΊΠ»ΡŽΡ‡Π΅Π½ сюда, Ρ‡Ρ‚ΠΎΠ±Ρ‹ ΠΏΠΎΠΌΠΎΡ‡ΡŒ Π²Π°ΠΌ ΠΏΠΎΠ½ΡΡ‚ΡŒ взаимосвязь ΠΌΠ΅ΠΆΠ΄Ρƒ Π°ΠΌΠΏΠ΅Ρ€Π°ΠΌΠΈ, ΠΊΡƒΠ»ΠΎΠ½Π°ΠΌΠΈ ΠΈ сСкундами. На ΠΏΡ€Π°ΠΊΡ‚ΠΈΠΊΠ΅ элСктричСский Ρ‚ΠΎΠΊ Π½Π΅ рассчитываСтся с ΠΏΠΎΠΌΠΎΡ‰ΡŒΡŽ уравнСния 1 , ΠΏΠΎΡ‚ΠΎΠΌΡƒ Ρ‡Ρ‚ΠΎ Π²Ρ‹ Π½Π΅ ΠΌΠΎΠΆΠ΅Ρ‚Π΅ Π½Π°ΠΏΡ€ΡΠΌΡƒΡŽ ΠΈΠ·ΠΌΠ΅Ρ€ΠΈΡ‚ΡŒ ΠΊΡƒΠ»ΠΎΠ½Ρ‹ заряда. Как Π²Ρ‹ ΡƒΠ·Π½Π°Π΅Ρ‚Π΅, Π΅ΡΡ‚ΡŒ Π³ΠΎΡ€Π°Π·Π΄ΠΎ Π±ΠΎΠ»Π΅Π΅ ΠΏΡ€Π°ΠΊΡ‚ΠΈΡ‡Π½Ρ‹Π΅ способы расчСта Ρ‚ΠΎΠΊΠ°.

    Π”Π²Π΅ Ρ‚Π΅ΠΎΡ€ΠΈΠΈ: ΠΎΠ±Ρ‹Ρ‡Π½Ρ‹ΠΉ Ρ‚ΠΎΠΊ ΠΈ ΠΏΠΎΡ‚ΠΎΠΊ элСктронов.

    Π‘ΡƒΡ‰Π΅ΡΡ‚Π²ΡƒΡŽΡ‚ Π΄Π²Π΅ Ρ‚Π΅ΠΎΡ€ΠΈΠΈ, ΠΎΠΏΠΈΡΡ‹Π²Π°ΡŽΡ‰ΠΈΠ΅ элСктричСский Ρ‚ΠΎΠΊ, ΠΈ Π²Ρ‹ ΡΡ‚ΠΎΠ»ΠΊΠ½Π΅Ρ‚Π΅ΡΡŒ с ΠΎΠ±Π΅ΠΈΠΌΠΈ Π½Π° ΠΏΡ€Π°ΠΊΡ‚ΠΈΠΊΠ΅.

    ВСория ΠžΠ±Ρ‹Ρ‡Π½Ρ‹ΠΉ Ρ‚ΠΎΠΊ опрСдСляСт Ρ‚ΠΎΠΊ ΠΊΠ°ΠΊ ΠΏΠΎΡ‚ΠΎΠΊ заряда ΠΎΡ‚ ΠΏΠΎΠ»ΠΎΠΆΠΈΡ‚Π΅Π»ΡŒΠ½ΠΎΠ³ΠΎ ΠΊ ΠΎΡ‚Ρ€ΠΈΡ†Π°Ρ‚Π΅Π»ΡŒΠ½ΠΎΠΌΡƒ. Π­Ρ‚Π° тСория называСтся Β«Ρ‚Ρ€Π°Π΄ΠΈΡ†ΠΈΠΎΠ½Π½Ρ‹ΠΌ Ρ‚ΠΎΠΊΠΎΠΌΒ», ΠΏΠΎΡ‚ΠΎΠΌΡƒ Ρ‡Ρ‚ΠΎ это Π±ΠΎΠ»Π΅Π΅ старый ΠΈΠ· Π΄Π²ΡƒΡ… ΠΏΠΎΠ΄Ρ…ΠΎΠ΄ΠΎΠ² ΠΊ Ρ‚ΠΎΠΊΡƒ, ΠΈ Π² Ρ‚Π΅Ρ‡Π΅Π½ΠΈΠ΅ ΠΌΠ½ΠΎΠ³ΠΈΡ… Π»Π΅Ρ‚ ΠΎΠ½ Π±Ρ‹Π» СдинствСнным, ΠΊΠΎΡ‚ΠΎΡ€ΠΎΠΌΡƒ ΠΎΠ±ΡƒΡ‡Π°Π»ΠΈ Π·Π° ΠΏΡ€Π΅Π΄Π΅Π»Π°ΠΌΠΈ Π²ΠΎΠ΅Π½Π½Ρ‹Ρ… ΠΈ рСмСслСнных ΡƒΡ‡ΠΈΠ»ΠΈΡ‰.

    Π­Π»Π΅ΠΊΡ‚Ρ€ΠΎΠ½Π½Ρ‹ΠΉ ΠΏΠΎΡ‚ΠΎΠΊ β€” Π±ΠΎΠ»Π΅Π΅ новая ΠΈΠ· Π΄Π²ΡƒΡ… Ρ‚Π΅ΠΊΡƒΡ‰ΠΈΡ… Ρ‚Π΅ΠΎΡ€ΠΈΠΉ. ВСория ΠΏΠΎΡ‚ΠΎΠΊΠ° элСктронов опрСдСляСт Ρ‚ΠΎΠΊ ΠΊΠ°ΠΊ ΠΏΠΎΡ‚ΠΎΠΊ элСктронов ΠΎΡ‚ ΠΎΡ‚Ρ€ΠΈΡ†Π°Ρ‚Π΅Π»ΡŒΠ½ΠΎΠ³ΠΎ ΠΊ ΠΏΠΎΠ»ΠΎΠΆΠΈΡ‚Π΅Π»ΡŒΠ½ΠΎΠΌΡƒ 9.0012 .

    Π”Π²Π΅ Ρ‚Π΅ΠΎΡ€ΠΈΠΈ элСктричСского Ρ‚ΠΎΠΊΠ° противопоставлСны Π½Π° Рис. 4 . КаТдая Ρ†Π΅ΠΏΡŒ содСрТит Π±Π°Ρ‚Π°Ρ€Π΅ΡŽ ΠΈ Π»Π°ΠΌΠΏΡƒ. ΠžΠ±Ρ‹Ρ‡Π½Ρ‹ΠΉ Ρ‚ΠΎΠΊ начинаСтся с ΠΏΠΎΠ»ΠΎΠΆΠΈΡ‚Π΅Π»ΡŒΠ½ΠΎΠΉ ΠΊΠ»Π΅ΠΌΠΌΡ‹ аккумулятора, ΠΏΡ€ΠΎΡ…ΠΎΠ΄ΠΈΡ‚ Ρ‡Π΅Ρ€Π΅Π· ΠΊΠΎΠ»Π΅Π½ΠΈ ΠΈ возвращаСтся ΠΊ аккумулятору Ρ‡Π΅Ρ€Π΅Π· ΠΎΡ‚Ρ€ΠΈΡ†Π°Ρ‚Π΅Π»ΡŒΠ½ΡƒΡŽ ΠΊΠ»Π΅ΠΌΠΌΡƒ. ΠŸΠΎΡ‚ΠΎΠΊ элСктронов Π½Π°ΠΏΡ€Π°Π²Π»Π΅Π½ Π² ΠΏΡ€ΠΎΡ‚ΠΈΠ²ΠΎΠΏΠΎΠ»ΠΎΠΆΠ½ΠΎΠΌ Π½Π°ΠΏΡ€Π°Π²Π»Π΅Π½ΠΈΠΈ: ΠΎΠ½ начинаСтся с ΠΎΡ‚Ρ€ΠΈΡ†Π°Ρ‚Π΅Π»ΡŒΠ½ΠΎΠΉ ΠΊΠ»Π΅ΠΌΠΌΡ‹, ΠΏΡ€ΠΎΡ…ΠΎΠ΄ΠΈΡ‚ Ρ‡Π΅Ρ€Π΅Π· Π»Π°ΠΌΠΏΡƒ ΠΈ возвращаСтся ΠΊ Π±Π°Ρ‚Π°Ρ€Π΅Π΅ Ρ‡Π΅Ρ€Π΅Π· Π΅Π΅ ΠΏΠΎΠ»ΠΎΠΆΠΈΡ‚Π΅Π»ΡŒΠ½ΡƒΡŽ ΠΊΠ»Π΅ΠΌΠΌΡƒ.

    Рисунок 4 ΠžΠ±Ρ‹Ρ‡Π½Ρ‹ΠΉ Ρ‚ΠΎΠΊ ΠΈ ΠΏΠΎΡ‚ΠΎΠΊ элСктронов.

    НичСго Π½Π΅ стоит Ρ‚ΠΎ, Ρ‡Ρ‚ΠΎ Π΄Π²Π΅ схСмы Π² Рисунок 4 ΠΈΠ΄Π΅Π½Ρ‚ΠΈΡ‡Π½Ρ‹. ЕдинствСнная Ρ€Π°Π·Π½ΠΈΡ†Π° ΠΌΠ΅ΠΆΠ΄Ρƒ Π½ΠΈΠΌΠΈ Π·Π°ΠΊΠ»ΡŽΡ‡Π°Π΅Ρ‚ΡΡ Π² Ρ‚ΠΎΠΌ, ΠΊΠ°ΠΊ ΠΌΡ‹ описываСм элСктричСский Ρ‚ΠΎΠΊ. На ΠΏΡ€Π°ΠΊΡ‚ΠΈΠΊΠ΅ Ρ‚ΠΎ, ΠΊΠ°ΠΊ Π²Ρ‹ смотритС Π½Π° Ρ‚ΠΎΠΊ, Π½Π΅ влияСт Π½ΠΈ Π½Π° расчСты схСмы, Π½ΠΈ Π½Π° измСрСния, Π½ΠΈ Π½Π° ΠΏΡ€ΠΎΡ†Π΅Π΄ΡƒΡ€Ρ‹ испытаний. Π’Π΅ΠΌ Π½Π΅ ΠΌΠ΅Π½Π΅Π΅, Π²Ρ‹ Π΄ΠΎΠ»ΠΆΠ½Ρ‹ ΠΎΡΠ²ΠΎΠΈΡ‚ΡŒΡΡ с ΠΎΠ±Π΅ΠΈΠΌΠΈ Ρ‚ΠΎΡ‡ΠΊΠ°ΠΌΠΈ зрСния, ΠΏΠΎΡΠΊΠΎΠ»ΡŒΠΊΡƒ ΠΎΠ±Π΅ ΠΎΠ½ΠΈ ΠΈΡΠΏΠΎΠ»ΡŒΠ·ΡƒΡŽΡ‚ΡΡ ΠΌΠ½ΠΎΠ³ΠΈΠΌΠΈ ΠΈΠ½ΠΆΠ΅Π½Π΅Ρ€Π°ΠΌΠΈ ΠΈ Ρ‚Π΅Ρ…Π½ΠΈΠΊΠ°ΠΌΠΈ, Π° Ρ‚Π°ΠΊΠΆΠ΅ тСхничСскими публикациями. Π’ этом тСкстС ΠΌΡ‹ Π±Π΅Ρ€Π΅ΠΌ ΠΏΠΎΡ‚ΠΎΠΊ элСктронов ΠΏΡ€ΠΈΠ±Π»ΠΈΠΆΠ΅Π½ΠΈΠ΅ ΠΊ Ρ‚ΠΎΠΊΡƒ. Π’ΠΎ Π΅ΡΡ‚ΡŒ ΠΌΡ‹ Π±ΡƒΠ΄Π΅ΠΌ ΡΡ‡ΠΈΡ‚Π°Ρ‚ΡŒ, Ρ‡Ρ‚ΠΎ Ρ‚ΠΎΠΊ β€” это ΠΏΠΎΡ‚ΠΎΠΊ элСктронов ΠΎΡ‚ ΠΎΡ‚Ρ€ΠΈΡ†Π°Ρ‚Π΅Π»ΡŒΠ½ΠΎΠ³ΠΎ ΠΊ ΠΏΠΎΠ»ΠΎΠΆΠΈΡ‚Π΅Π»ΡŒΠ½ΠΎΠΌΡƒ.

    ΠŸΠΎΡΡ‚ΠΎΡΠ½Π½Ρ‹ΠΉ Ρ‚ΠΎΠΊ (DC) Π² сравнСнии с ΠΏΠ΅Ρ€Π΅ΠΌΠ΅Π½Π½Ρ‹ΠΌ Ρ‚ΠΎΠΊΠΎΠΌ (AC)

    Π’ΠΎΠΊ ΠΎΠ±Ρ‹Ρ‡Π½ΠΎ классифицируСтся ΠΊΠ°ΠΊ постоянный Ρ‚ΠΎΠΊ (DC) ΠΈΠ»ΠΈ ΠΏΠ΅Ρ€Π΅ΠΌΠ΅Π½Π½Ρ‹ΠΉ Ρ‚ΠΎΠΊ (AC). Различия ΠΌΠ΅ΠΆΠ΄Ρƒ постоянным ΠΈ ΠΏΠ΅Ρ€Π΅ΠΌΠ΅Π½Π½Ρ‹ΠΌ Ρ‚ΠΎΠΊΠΎΠΌ ΠΏΠΎΠΊΠ°Π·Π°Π½Ρ‹ Π½Π° рис. 5 .

    Рисунок 5 ΠŸΠΎΡΡ‚ΠΎΡΠ½Π½Ρ‹ΠΉ Ρ‚ΠΎΠΊ (DC) ΠΈ ΠΏΠ΅Ρ€Π΅ΠΌΠ΅Π½Π½Ρ‹ΠΉ Ρ‚ΠΎΠΊ (AC).

    ΠŸΠΎΡΡ‚ΠΎΡΠ½Π½Ρ‹ΠΉ Ρ‚ΠΎΠΊ ΠΎΠ΄Π½ΠΎΠ½Π°ΠΏΡ€Π°Π²Π»Π΅Π½Π½Ρ‹ΠΉ. Π’ΠΎ Π΅ΡΡ‚ΡŒ ΠΏΠΎΡ‚ΠΎΠΊ заряда всСгда Π² ΠΎΠ΄Π½ΠΎΠΌ Π½Π°ΠΏΡ€Π°Π²Π»Π΅Π½ΠΈΠΈ. Π’Π΅Ρ€ΠΌΠΈΠ½ постоянный Ρ‚ΠΎΠΊ ΠΎΠ±Ρ‹Ρ‡Π½ΠΎ ΠΏΠΎΠ΄Ρ€Π°Π·ΡƒΠΌΠ΅Π²Π°Π΅Ρ‚, Ρ‡Ρ‚ΠΎ Ρ‚ΠΎΠΊ ΠΈΠΌΠ΅Π΅Ρ‚ фиксированноС Π·Π½Π°Ρ‡Π΅Π½ΠΈΠ΅. НапримСр, Π½Π° Π³Ρ€Π°Ρ„ΠΈΠΊΠ΅ рис. 5Π° ΠΏΠΎΠΊΠ°Π·Π°Π½ΠΎ, Ρ‡Ρ‚ΠΎ Ρ‚ΠΎΠΊ ΠΈΠΌΠ΅Π΅Ρ‚ постоянноС Π·Π½Π°Ρ‡Π΅Π½ΠΈΠ΅ 1А. Π₯отя подразумСваСтся фиксированноС Π·Π½Π°Ρ‡Π΅Π½ΠΈΠ΅, Π·Π½Π°Ρ‡Π΅Π½ΠΈΠ΅ постоянного Ρ‚ΠΎΠΊΠ° ΠΌΠΎΠΆΠ΅Ρ‚ ΠΈΠ·ΠΌΠ΅Π½ΡΡ‚ΡŒΡΡ. Однако Π½Π°ΠΏΡ€Π°Π²Π»Π΅Π½ΠΈΠ΅ Ρ‚ΠΎΠΊΠ° Π½Π΅ мСняСтся.

    ΠŸΠ΅Ρ€Π΅ΠΌΠ΅Π½Π½Ρ‹ΠΉ Ρ‚ΠΎΠΊ β€” Π΄Π²ΡƒΠ½Π°ΠΏΡ€Π°Π²Π»Π΅Π½Π½Ρ‹ΠΉ. Π’ΠΎ Π΅ΡΡ‚ΡŒ Π½Π°ΠΏΡ€Π°Π²Π»Π΅Π½ΠΈΠ΅ Ρ‚ΠΎΠΊΠ° постоянно мСнялось. НапримСр, Π² рисунок 5b , Π³Ρ€Π°Ρ„ΠΈΠΊ ΠΏΠΎΠΊΠ°Π·Ρ‹Π²Π°Π΅Ρ‚, Ρ‡Ρ‚ΠΎ Ρ‚ΠΎΠΊ достигаСт ΠΏΠΈΠΊΠΎΠ²ΠΎΠ³ΠΎ значСния Π² ΠΎΠ΄Π½ΠΎΠΌ Π½Π°ΠΏΡ€Π°Π²Π»Π΅Π½ΠΈΠΈ, Π° Π·Π°Ρ‚Π΅ΠΌ достигаСт ΠΏΠΈΠΊΠΎΠ²ΠΎΠ³ΠΎ значСния Π² Π΄Ρ€ΡƒΠ³ΠΎΠΌ Π½Π°ΠΏΡ€Π°Π²Π»Π΅Π½ΠΈΠΈ. ΠžΠ±Ρ€Π°Ρ‚ΠΈΡ‚Π΅ Π²Π½ΠΈΠΌΠ°Π½ΠΈΠ΅, Ρ‡Ρ‚ΠΎ ΠΏΠ΅Ρ€Π΅ΠΌΠ΅Π½Π½Ρ‹ΠΉ Ρ‚ΠΎΠΊ, прСдставлСнный Π½Π° Π³Ρ€Π°Ρ„ΠΈΠΊΠ΅, Π½Π΅ Ρ‚ΠΎΠ»ΡŒΠΊΠΎ мСняСт Π½Π°ΠΏΡ€Π°Π²Π»Π΅Π½ΠΈΠ΅, Π½ΠΎ ΠΈ постоянно мСняСт своС Π·Π½Π°Ρ‡Π΅Π½ΠΈΠ΅.

    ЭлСктричСский Ρ‚ΠΎΠΊ ΠΏΡ€ΠΎΠΈΠ·Π²ΠΎΠ΄ΠΈΡ‚ Ρ‚Π΅ΠΏΠ»ΠΎ

    Всякий Ρ€Π°Π·, ΠΊΠΎΠ³Π΄Π° Ρ‡Π΅Ρ€Π΅Π· ΠΊΠΎΠΌΠΏΠΎΠ½Π΅Π½Ρ‚ ΠΈΠ»ΠΈ Ρ†Π΅ΠΏΡŒ гСнСрируСтся элСктричСский Ρ‚ΠΎΠΊ, выдСляСтся Ρ‚Π΅ΠΏΠ»ΠΎ. ΠšΠΎΠ»ΠΈΡ‡Π΅ΡΡ‚Π²ΠΎ Ρ‚Π΅ΠΏΠ»Π° зависит ΠΎΡ‚ уровня Ρ‚ΠΎΠΊΠ°: Ρ‡Π΅ΠΌ большС Ρ‚ΠΎΠΊ, Ρ‚Π΅ΠΌ большС Ρ‚Π΅ΠΏΠ»Π° ΠΎΠ½ ΠΏΡ€ΠΎΠΈΠ·Π²ΠΎΠ΄ΠΈΡ‚. Π’ΠΎΡ‚ ΠΏΠΎΡ‡Π΅ΠΌΡƒ ΠΌΠ½ΠΎΠ³ΠΈΠ΅ ΡΠΈΠ»ΡŒΠ½ΠΎΡ‚ΠΎΡ‡Π½Ρ‹Π΅ ΠΊΠΎΠΌΠΏΠΎΠ½Π΅Π½Ρ‚Ρ‹, Ρ‚Π°ΠΊΠΈΠ΅ ΠΊΠ°ΠΊ Π΄Π²ΠΈΠ³Π°Ρ‚Π΅Π»ΠΈ, Π½Π°Π³Ρ€Π΅Π²Π°ΡŽΡ‚ΡΡ Π²ΠΎ врСмя Ρ€Π°Π±ΠΎΡ‚Ρ‹. НСкоторыС ΡΠΈΠ»ΡŒΠ½ΠΎΡ‚ΠΎΡ‡Π½Ρ‹Π΅ Ρ†Π΅ΠΏΠΈ Π½Π°Π³Ρ€Π΅Π²Π°ΡŽΡ‚ΡΡ Π½Π°ΡΡ‚ΠΎΠ»ΡŒΠΊΠΎ, Ρ‡Ρ‚ΠΎ ΠΈΡ… Π½Π΅ΠΎΠ±Ρ…ΠΎΠ΄ΠΈΠΌΠΎ ΠΎΡ…Π»Π°ΠΆΠ΄Π°Ρ‚ΡŒ.

    Π’Π΅ΠΏΠ»ΠΎ, выдСляСмоС элСктричСским Ρ‚ΠΎΠΊΠΎΠΌ, ΠΈΠ½ΠΎΠ³Π΄Π° Π±Ρ‹Π²Π°Π΅Ρ‚ ΠΆΠ΅Π»Π°Ρ‚Π΅Π»ΡŒΠ½Ρ‹ΠΌ. ВостСры, элСктричСскиС ΠΏΠ»ΠΈΡ‚Ρ‹ ΠΈ Ρ‚Π΅ΠΏΠ»ΠΎΠ²Ρ‹Π΅ Π»Π°ΠΌΠΏΡ‹ β€” это ΠΎΠ±Ρ‹Ρ‡Π½Ρ‹Π΅ ΠΏΡ€Π΅Π΄ΠΌΠ΅Ρ‚Ρ‹, ΠΊΠΎΡ‚ΠΎΡ€Ρ‹Π΅ ΠΈΡΠΏΠΎΠ»ΡŒΠ·ΡƒΡŽΡ‚ Ρ‚Π΅ΠΏΠ»ΠΎ, выдСляСмоС Ρ‚ΠΎΠΊΠΎΠΌ.

    Рисунок 6 Высокий Ρ‚ΠΎΠΊ заставляСт Π³ΠΎΡ€Π΅Π»ΠΊΡƒ Π½Π°Π³Ρ€Π΅Π²Π°Ρ‚Π΅Π»ΡŒΠ½ΠΎΠ³ΠΎ элСмСнта ΠΏΠ΅Ρ‡ΠΈ ΡΠ²Π΅Ρ‚ΠΈΡ‚ΡŒΡΡ красным.

    Π‘ΠΎΠ±ΠΈΡ€Π°Π΅ΠΌ всС вмСстС

    Π‘Π²ΠΎΠ±ΠΎΠ΄Π½Ρ‹Π΅ элСктроны Π³Π΅Π½Π΅Ρ€ΠΈΡ€ΡƒΡŽΡ‚ΡΡ Π² ΠΌΠ΅Π΄ΠΈ ΠΏΡ€ΠΈ ΠΊΠΎΠΌΠ½Π°Ρ‚Π½ΠΎΠΉ Ρ‚Π΅ΠΌΠΏΠ΅Ρ€Π°Ρ‚ΡƒΡ€Π΅. Когда ΠΎΠ½ΠΈ Π½Π΅Π½Π°ΠΏΡ€Π°Π²Π»Π΅Π½Ρ‹, Π΄Π²ΠΈΠΆΠ΅Π½ΠΈΠ΅ этих свободных элСктронов являСтся случайным, ΠΈ суммарный ΠΏΠΎΡ‚ΠΎΠΊ элСктронов Π² любом Π½Π°ΠΏΡ€Π°Π²Π»Π΅Π½ΠΈΠΈ Ρ€Π°Π²Π΅Π½ Π½ΡƒΠ»ΡŽ.

    Под дСйствиСм внСшнСй силы свободныС элСктроны Π²Ρ‹Π½ΡƒΠΆΠ΄Π΅Π½Ρ‹ Π΄Π²ΠΈΠ³Π°Ρ‚ΡŒΡΡ Π² ΠΎΠ΄Π½ΠΎΠΌ Π½Π°ΠΏΡ€Π°Π²Π»Π΅Π½ΠΈΠΈ. Π­Ρ‚ΠΎΡ‚ Π½Π°ΠΏΡ€Π°Π²Π»Π΅Π½Π½Ρ‹ΠΉ ΠΏΠΎΡ‚ΠΎΠΊ заряда называСтся элСктричСским Ρ‚ΠΎΠΊΠΎΠΌ.

    ЭлСктричСский Ρ‚ΠΎΠΊ обозначаСтся Π±ΡƒΠΊΠ²ΠΎΠΉ I, Ρ‡Ρ‚ΠΎ ΠΎΠ·Π½Π°Ρ‡Π°Π΅Ρ‚ ΠΈΠ½Ρ‚Π΅Π½ΡΠΈΠ²Π½ΠΎΡΡ‚ΡŒ. Π‘ΠΈΠ»Π° Ρ‚ΠΎΠΊΠ° зависит ΠΎΡ‚ количСства ΠΏΠ΅Ρ€Π΅ΠΌΠ΅Ρ‰Π΅Π½Π½ΠΎΠ³ΠΎ заряда ΠΈ Π²Ρ€Π΅ΠΌΠ΅Π½ΠΈ, Π½Π΅ΠΎΠ±Ρ…ΠΎΠ΄ΠΈΠΌΠΎΠ³ΠΎ для Π΅Π³ΠΎ пСрСмСщСния.

    ЭлСктричСский Ρ‚ΠΎΠΊ измСряСтся Π² Π°ΠΌΠΏΠ΅Ρ€Π°Ρ… (А). Когда ΠΎΠ΄ΠΈΠ½ ΠΊΡƒΠ»ΠΎΠ½ Ρ‚ΠΎΠΊΠ° ΠΏΡ€ΠΎΡ…ΠΎΠ΄ΠΈΡ‚ Ρ‡Π΅Ρ€Π΅Π· Ρ‚ΠΎΡ‡ΠΊΡƒ ΠΊΠ°ΠΆΠ΄ΡƒΡŽ сСкунду, Ρƒ вас Π΅ΡΡ‚ΡŒ ΠΎΠ΄ΠΈΠ½ Π°ΠΌΠΏΠ΅Ρ€ Ρ‚ΠΎΠΊΠ°.

    Π•ΡΡ‚ΡŒ Π΄Π²Π΅ Ρ‚Π΅ΠΊΡƒΡ‰ΠΈΠ΅ Ρ‚Π΅ΠΎΡ€ΠΈΠΈ . ВСория ΠΏΠΎΡ‚ΠΎΠΊΠ° элСктронов описываСт Ρ‚ΠΎΠΊ ΠΊΠ°ΠΊ ΠΏΠΎΡ‚ΠΎΠΊ заряда (элСктронов) ΠΎΡ‚ ΠΎΡ‚Ρ€ΠΈΡ†Π°Ρ‚Π΅Π»ΡŒΠ½ΠΎΠ³ΠΎ ΠΊ ΠΏΠΎΠ»ΠΎΠΆΠΈΡ‚Π΅Π»ΡŒΠ½ΠΎΠΌΡƒ. Врадиционная тСория Ρ‚ΠΎΠΊΠ° описываСт Ρ‚ΠΎΠΊ ΠΊΠ°ΠΊ ΠΏΠΎΡ‚ΠΎΠΊ заряда ΠΎΡ‚ ΠΏΠΎΠ»ΠΎΠΆΠΈΡ‚Π΅Π»ΡŒΠ½ΠΎΠ³ΠΎ ΠΊ ΠΎΡ‚Ρ€ΠΈΡ†Π°Ρ‚Π΅Π»ΡŒΠ½ΠΎΠΌΡƒ. Оба ΠΏΠΎΠ΄Ρ…ΠΎΠ΄Π° ΡˆΠΈΡ€ΠΎΠΊΠΎ распространСны. Π’ΠΎ, ΠΊΠ°ΠΊ Π²Ρ‹ смотритС Π½Π° Ρ‚ΠΎΠΊ, Π½Π΅ влияСт Π½Π° Ρ€Π΅Π·ΡƒΠ»ΡŒΡ‚Π°Ρ‚ ΠΊΠ°ΠΊΠΈΡ…-Π»ΠΈΠ±ΠΎ расчСтов схСмы, ΠΈΠ·ΠΌΠ΅Ρ€Π΅Π½ΠΈΠΉ ΠΈΠ»ΠΈ тСстовых ΠΏΡ€ΠΎΡ†Π΅Π΄ΡƒΡ€.

    Π‘ΠΎΠ»ΡŒΡˆΠΈΠ½ΡΡ‚Π²ΠΎ элСктричСских ΠΈ элСктронных систСм содСрТат Ρ†Π΅ΠΏΠΈ ΠΊΠ°ΠΊ постоянного (DC), Ρ‚Π°ΠΊ ΠΈ ΠΏΠ΅Ρ€Π΅ΠΌΠ΅Π½Π½ΠΎΠ³ΠΎ Ρ‚ΠΎΠΊΠ° (AC). Π’ цСпях постоянного Ρ‚ΠΎΠΊΠ° Ρ‚ΠΎΠΊ всСгда ΠΈΠΌΠ΅Π΅Ρ‚ ΠΎΠ΄Π½ΠΎ ΠΈ Ρ‚ΠΎ ΠΆΠ΅ Π½Π°ΠΏΡ€Π°Π²Π»Π΅Π½ΠΈΠ΅. Π’ цСпях ΠΏΠ΅Ρ€Π΅ΠΌΠ΅Π½Π½ΠΎΠ³ΠΎ Ρ‚ΠΎΠΊΠ° Ρ‚ΠΎΠΊ постоянно мСняСт Π½Π°ΠΏΡ€Π°Π²Π»Π΅Π½ΠΈΠ΅.

    Вопросы для повторСния

    Как ΠΎΠ±Ρ€Π°Π·ΡƒΡŽΡ‚ΡΡ свободныС элСктроны Π² ΠΏΡ€ΠΎΠ²ΠΎΠ΄Π½ΠΈΠΊΠ΅ ΠΏΡ€ΠΈ ΠΊΠΎΠΌΠ½Π°Ρ‚Π½ΠΎΠΉ Ρ‚Π΅ΠΌΠΏΠ΅Ρ€Π°Ρ‚ΡƒΡ€Π΅?

    Π’Π΅ΠΏΠ»ΠΎΠ²ΠΎΠΉ энСргии (Ρ‚Π΅ΠΏΠ»Π°) ΠΏΡ€ΠΈ ΠΊΠΎΠΌΠ½Π°Ρ‚Π½ΠΎΠΉ Ρ‚Π΅ΠΌΠΏΠ΅Ρ€Π°Ρ‚ΡƒΡ€Π΅ достаточно для образования свободных элСктронов.

    Π§Ρ‚ΠΎ Ρ‚Π°ΠΊΠΎΠ΅ элСктричСский Ρ‚ΠΎΠΊ? КакиС Ρ„Π°ΠΊΡ‚ΠΎΡ€Ρ‹ Π²Π»ΠΈΡΡŽΡ‚ Π½Π° силу элСктричСского Ρ‚ΠΎΠΊΠ°?

    Π’ΠΎΠΊ – это Π½Π°ΠΏΡ€Π°Π²Π»Π΅Π½Π½Ρ‹ΠΉ ΠΏΠΎΡ‚ΠΎΠΊ элСктронов Π² ΠΌΠ°Ρ‚Π΅Ρ€ΠΈΠ°Π»Π΅. Π‘ΠΈΠ»Π° Ρ‚ΠΎΠΊΠ° зависит ΠΎΡ‚ количСства ΠΏΠ΅Ρ€Π΅ΠΌΠ΅Ρ‰Π΅Π½Π½ΠΎΠ³ΠΎ заряда ΠΈ Π²Ρ€Π΅ΠΌΠ΅Π½ΠΈ, Π½Π΅ΠΎΠ±Ρ…ΠΎΠ΄ΠΈΠΌΠΎΠ³ΠΎ для Π΅Π³ΠΎ пСрСмСщСния.

    Π§Ρ‚ΠΎ Ρ‚Π°ΠΊΠΎΠ΅ ΠΊΡƒΠ»ΠΎΠ½?

    Один ΠΊΡƒΠ»ΠΎΠ½ Ρ€Π°Π²Π΅Π½ ΠΎΠ±Ρ‰Π΅ΠΌΡƒ заряду 6,25 Γ— 10 18 элСктронов.

    Π§Ρ‚ΠΎ Ρ‚Π°ΠΊΠΎΠ΅ основная Π΅Π΄ΠΈΠ½ΠΈΡ†Π° элСктричСского Ρ‚ΠΎΠΊΠ°?

    АмпСр β€” основная Π΅Π΄ΠΈΠ½ΠΈΡ†Π° измСрСния элСктричСского Ρ‚ΠΎΠΊΠ°. Он опрСдСляСтся ΠΊΠ°ΠΊ 1 ΠΊΡƒΠ»ΠΎΠ½ Π² сСкунду ΠΈΠ»ΠΈ 1 А = 1 Кл/с.

    Π‘Ρ€Π°Π²Π½ΠΈΡ‚Π΅ Ρ‚Π΅ΠΎΡ€ΠΈΡŽ ΠΏΠΎΡ‚ΠΎΠΊΠ° элСктронов ΠΈ ΠΎΠ±Ρ‰Π΅ΠΏΡ€ΠΈΠ½ΡΡ‚ΡƒΡŽ Ρ‚Π΅ΠΎΡ€ΠΈΡŽ Ρ‚ΠΎΠΊΠ°.

    Врадиционная тСория Ρ‚ΠΎΠΊΠ° опрСдСляСт Ρ‚ΠΎΠΊ ΠΊΠ°ΠΊ ΠΏΠΎΡ‚ΠΎΠΊ заряда ΠΎΡ‚ ΠΏΠΎΠ»ΠΎΠΆΠΈΡ‚Π΅Π»ΡŒΠ½ΠΎΠ³ΠΎ ΠΊ ΠΎΡ‚Ρ€ΠΈΡ†Π°Ρ‚Π΅Π»ΡŒΠ½ΠΎΠΌΡƒ. ΠŸΠΎΡ‚ΠΎΠΊ элСктронов β€” это ΠΏΠΎΡ‚ΠΎΠΊ заряда ΠΎΡ‚ ΠΎΡ‚Ρ€ΠΈΡ†Π°Ρ‚Π΅Π»ΡŒΠ½ΠΎΠ³ΠΎ ΠΊ ΠΏΠΎΠ»ΠΎΠΆΠΈΡ‚Π΅Π»ΡŒΠ½ΠΎΠΌΡƒ.

    НапряТСниС

    НапряТСниС ΠΌΠΎΠΆΠ½ΠΎ ΠΎΠΏΠΈΡΠ°Ρ‚ΡŒ ΠΊΠ°ΠΊ силу, которая создаСт ΠΏΠΎΡ‚ΠΎΠΊ элСктронов (Ρ‚ΠΎΠΊ) Π² Ρ†Π΅ΠΏΠΈ. Π’ этом Ρ€Π°Π·Π΄Π΅Π»Π΅ ΠΌΡ‹ ΠΏΠΎΠ΄Ρ€ΠΎΠ±Π½ΠΎ рассмотрим напряТСниС ΠΈ Ρ‚ΠΎ, ΠΊΠ°ΠΊ ΠΎΠ½ΠΎ Π³Π΅Π½Π΅Ρ€ΠΈΡ€ΡƒΠ΅Ρ‚ Ρ‚ΠΎΠΊ.

    ГСнСрация Ρ‚ΠΎΠΊΠ° с ΠΏΠΎΠΌΠΎΡ‰ΡŒΡŽ Π±Π°Ρ‚Π°Ρ€Π΅ΠΈ

    БатарСя Π² Рисунок 7a ΠΈΠΌΠ΅Π΅Ρ‚ Π΄Π²Π΅ ΠΊΠ»Π΅ΠΌΠΌΡ‹. ΠŸΠΎΠ»ΠΎΠΆΠΈΡ‚Π΅Π»ΡŒΠ½Π°Ρ (+) ΠΊΠ»Π΅ΠΌΠΌΠ° ΠΈΠΌΠ΅Π΅Ρ‚ ΠΈΠ·Π±Ρ‹Ρ‚ΠΎΠΊ ΠΏΠΎΠ»ΠΎΠΆΠΈΡ‚Π΅Π»ΡŒΠ½Ρ‹Ρ… ΠΈΠΎΠ½ΠΎΠ² ΠΈ описываСтся ΠΊΠ°ΠΊ ΠΈΠΌΠ΅ΡŽΡ‰Π°Ρ ΠΏΠΎΠ»ΠΎΠΆΠΈΡ‚Π΅Π»ΡŒΠ½Ρ‹ΠΉ ΠΏΠΎΡ‚Π΅Π½Ρ†ΠΈΠ°Π». ΠžΡ‚Ρ€ΠΈΡ†Π°Ρ‚Π΅Π»ΡŒΠ½Π°Ρ (-) ΠΊΠ»Π΅ΠΌΠΌΠ° ΠΈΠΌΠ΅Π΅Ρ‚ ΠΈΠ·Π±Ρ‹Ρ‚ΠΎΠΊ элСктронов ΠΈ описываСтся ΠΊΠ°ΠΊ ΠΈΠΌΠ΅ΡŽΡ‰Π°Ρ ΠΎΡ‚Ρ€ΠΈΡ†Π°Ρ‚Π΅Π»ΡŒΠ½Ρ‹ΠΉ ΠΏΠΎΡ‚Π΅Π½Ρ†ΠΈΠ°Π».

    Рисунок 7 Π Π°Π·Π½ΠΎΡΡ‚ΡŒ ΠΏΠΎΡ‚Π΅Π½Ρ†ΠΈΠ°Π»ΠΎΠ² ΠΈ Ρ€Π΅Π·ΡƒΠ»ΡŒΡ‚ΠΈΡ€ΡƒΡŽΡ‰ΠΈΠΉ Ρ‚ΠΎΠΊ.

    Π’Π°ΠΊΠΈΠΌ ΠΎΠ±Ρ€Π°Π·ΠΎΠΌ, сущСствуСт Ρ€Π°Π·Π½ΠΈΡ†Π° ΠΏΠΎΡ‚Π΅Π½Ρ†ΠΈΠ°Π»ΠΎΠ² ΠΈΠ»ΠΈ напряТСния (Π’) ΠΌΠ΅ΠΆΠ΄Ρƒ двумя ΠΊΠ»Π΅ΠΌΠΌΠ°ΠΌΠΈ.

    Если ΠΌΡ‹ соСдиним Π΄Π²Π΅ ΠΊΠ»Π΅ΠΌΠΌΡ‹ Π±Π°Ρ‚Π°Ρ€Π΅ΠΈ с ΠΌΠ΅Π΄Π½Ρ‹ΠΌ ΠΏΡ€ΠΎΠ²ΠΎΠ΄ΠΎΠΌ ΠΈ Π»Π°ΠΌΠΏΠΎΠΉ ( Рисунок 7b) , Ρ‚ΠΎΠΊ Π±ΡƒΠ΄Π΅Ρ‚ ΠΏΡ€ΠΎΠΈΠ·Π²ΠΎΠ΄ΠΈΡ‚ΡŒΡΡ ΠΏΠΎ ΠΌΠ΅Ρ€Π΅ Ρ‚ΠΎΠ³ΠΎ, ΠΊΠ°ΠΊ элСктроны ΠΏΡ€ΠΈΡ‚ΡΠ³ΠΈΠ²Π°ΡŽΡ‚ΡΡ ΠΊ ΠΏΠΎΠ»ΠΎΠΆΠΈΡ‚Π΅Π»ΡŒΠ½ΠΎΠΉ ΠΊΠ»Π΅ΠΌΠΌΠ΅ Π±Π°Ρ‚Π°Ρ€Π΅ΠΈ. Π”Ρ€ΡƒΠ³ΠΈΠΌΠΈ словами, сущСствуСт Π½Π°ΠΏΡ€Π°Π²Π»Π΅Π½Π½Ρ‹ΠΉ ΠΏΠΎΡ‚ΠΎΠΊ элСктронов ΠΎΡ‚ ΠΎΡ‚Ρ€ΠΈΡ†Π°Ρ‚Π΅Π»ΡŒΠ½ΠΎΠΉ (-) ΠΊΠ»Π΅ΠΌΠΌΡ‹ ΠΊ ΠΏΠΎΠ»ΠΎΠΆΠΈΡ‚Π΅Π»ΡŒΠ½ΠΎΠΉ (+) ΠΊΠ»Π΅ΠΌΠΌΠ΅ Π±Π°Ρ‚Π°Ρ€Π΅ΠΈ.

    Π•ΡΡ‚ΡŒ нСсколько Π²Π°ΠΆΠ½Ρ‹Ρ… ΠΌΠΎΠΌΠ΅Π½Ρ‚ΠΎΠ², ΠΊΠΎΡ‚ΠΎΡ€Ρ‹Π΅ Π½Π΅ΠΎΠ±Ρ…ΠΎΠ΄ΠΈΠΌΠΎ ΡΠ΄Π΅Π»Π°Ρ‚ΡŒ:

    1. НапряТСниС – это сила, которая ΠΏΠ΅Ρ€Π΅ΠΌΠ΅Ρ‰Π°Π΅Ρ‚ элСктроны, ΠΏΠΎ этой ΠΏΡ€ΠΈΡ‡ΠΈΠ½Π΅ Π΅Π΅ часто Π½Π°Π·Ρ‹Π²Π°ΡŽΡ‚ элСктричСской силой (Π­) ΠΈΠ»ΠΈ элСктродвиТущСй силой (Π­Π”Π‘). .

    2. Π’ΠΎΠΊ ΠΈ напряТСниС Π½Π΅ ΠΎΠ΄Π½ΠΎ ΠΈ Ρ‚ΠΎ ΠΆΠ΅. Π’ΠΎΠΊ – это Π½Π°ΠΏΡ€Π°Π²Π»Π΅Π½Π½Ρ‹ΠΉ ΠΏΠΎΡ‚ΠΎΠΊ элСктронов ΠΎΡ‚ минуса ΠΊ ΠΏΠ»ΡŽΡΡƒ. НапряТСниС – это элСктричСская сила, ΡΠΎΠ·Π΄Π°ΡŽΡ‰Π°Ρ Ρ‚ΠΎΠΊ. Π”Ρ€ΡƒΠ³ΠΈΠΌΠΈ словами, Ρ‚ΠΎΠΊ Π²ΠΎΠ·Π½ΠΈΠΊΠ°Π΅Ρ‚ Π² Ρ€Π΅Π·ΡƒΠ»ΡŒΡ‚Π°Ρ‚Π΅ ΠΏΡ€ΠΈΠ»ΠΎΠΆΠ΅Π½Π½ΠΎΠ³ΠΎ напряТСния (элСктричСской силы). 9{J}/{}_{C} \\\end{align}\]

    ΠšΠΎΠ½Ρ‚Ρ€ΠΎΠ»ΡŒΠ½Ρ‹Π΅ вопросы

    Π§Ρ‚ΠΎ Ρ‚Π°ΠΊΠΎΠ΅ напряТСниС?

    НапряТСниС β€” это сила, ΡΠΎΠ·Π΄Π°ΡŽΡ‰Π°Ρ Ρ‚ΠΎΠΊ Π² Ρ†Π΅ΠΏΠΈ.

    Каким ΠΎΠ±Ρ€Π°Π·ΠΎΠΌ напряТСниС Π³Π΅Π½Π΅Ρ€ΠΈΡ€ΡƒΠ΅Ρ‚ Ρ‚ΠΎΠΊ Π² ΠΏΡ€ΠΎΠ²ΠΎΠ΄Π½ΠΈΠΊΠ΅?

    Π˜ΡΡ‚ΠΎΡ‡Π½ΠΈΠΊ напряТСния ΠΈΠΌΠ΅Π΅Ρ‚ ΠΈΠ·Π±Ρ‹Ρ‚ΠΎΠΊ элСктронов (ΠΎΡ‚Ρ€ΠΈΡ†Π°Ρ‚Π΅Π»ΡŒΠ½Ρ‹ΠΉ заряд) Π½Π° ΠΎΠ΄Π½ΠΎΠΌ Π²Ρ‹Π²ΠΎΠ΄Π΅ ΠΈ ΠΈΠ·Π±Ρ‹Ρ‚ΠΎΠΊ ΠΏΠΎΠ»ΠΎΠΆΠΈΡ‚Π΅Π»ΡŒΠ½Ρ‹Ρ… ΠΈΠΎΠ½ΠΎΠ² Π½Π° Π΄Ρ€ΡƒΠ³ΠΎΠΌ. Π­Ρ‚ΠΎ называСтся Ρ€Π°Π·Π½ΠΎΡΡ‚ΡŒΡŽ ΠΏΠΎΡ‚Π΅Π½Ρ†ΠΈΠ°Π»ΠΎΠ². Π˜Π·Π±Ρ‹Ρ‚ΠΎΡ‡Π½Ρ‹Π΅ элСктроны Π½Π° ΠΎΡ‚Ρ€ΠΈΡ†Π°Ρ‚Π΅Π»ΡŒΠ½ΠΎΠΌ полюсС ΠΏΡ€ΠΈΡ‚ΡΠ³ΠΈΠ²Π°ΡŽΡ‚ΡΡ ΠΏΠΎΠ»ΠΎΠΆΠΈΡ‚Π΅Π»ΡŒΠ½Ρ‹ΠΌΠΈ ΠΈΠΎΠ½Π°ΠΌΠΈ Π½Π° ΠΏΠΎΠ»ΠΎΠΆΠΈΡ‚Π΅Π»ΡŒΠ½ΠΎΠΌ полюсС. Π­Ρ‚ΠΎ ΠΏΡ€ΠΈΠ²ΠΎΠ΄ΠΈΡ‚ ΠΊ ΠΏΠΎΡ‚ΠΎΠΊΡƒ заряда Π² любом ΠΏΡ€ΠΎΠ²ΠΎΠ΄Π΅, ΡΠΎΠ΅Π΄ΠΈΠ½ΡΡŽΡ‰Π΅ΠΌ Π΄Π²Π΅ ΠΊΠ»Π΅ΠΌΠΌΡ‹ источника напряТСния.

    Какова Π΅Π΄ΠΈΠ½ΠΈΡ†Π° измСрСния напряТСния? Как это опрСдСляСтся?

    Π•Π΄ΠΈΠ½ΠΈΡ†Π΅ΠΉ измСрСния напряТСния являСтся Π²ΠΎΠ»ΡŒΡ‚. Один Π²ΠΎΠ»ΡŒΡ‚ β€” это количСство элСктричСской силы, которая ΠΈΡΠΏΠΎΠ»ΡŒΠ·ΡƒΠ΅Ρ‚ ΠΎΠ΄ΠΈΠ½ Π΄ΠΆΠΎΡƒΠ»ΡŒ (Π”ΠΆ) энСргии для пСрСмСщСния ΠΎΠ΄Π½ΠΎΠ³ΠΎ ΠΊΡƒΠ»ΠΎΠ½Π° (Кл) заряда. 1 Π’ = 1 Π”ΠΆ/Кл.

    Как Π±Ρ‹ Π²Ρ‹ ΠΎΠΏΡ€Π΅Π΄Π΅Π»ΠΈΠ»ΠΈ ΠΊΡƒΠ»ΠΎΠ½ с Ρ‚ΠΎΡ‡ΠΊΠΈ зрСния напряТСния ΠΈ энСргии?

    1 ΠΊΡƒΠ»ΠΎΠ½ Ρ€Π°Π²Π΅Π½ 1 Π΄ΠΆΠΎΡƒΠ»ΡŽ Π½Π° Π²ΠΎΠ»ΡŒΡ‚, 1 Кл = 1 Π”ΠΆ/Π’

    Как Π±Ρ‹ Π²Ρ‹ ΠΎΠΏΡ€Π΅Π΄Π΅Π»ΠΈΠ»ΠΈ Π΄ΠΆΠΎΡƒΠ»ΡŒ с Ρ‚ΠΎΡ‡ΠΊΠΈ зрСния напряТСния ΠΈ заряда?

    1 Π΄ΠΆΠΎΡƒΠ»ΡŒ Ρ€Π°Π²Π΅Π½ 1 Π’, ΡƒΠΌΠ½ΠΎΠΆΠ΅Π½Π½ΠΎΠΌΡƒ Π½Π° 1 ΠΊΡƒΠ»ΠΎΠ½, 1 Π”ΠΆ = 1 Π’ Γ— 1 Кл

    Π‘ΠΎΠΏΡ€ΠΎΡ‚ΠΈΠ²Π»Π΅Π½ΠΈΠ΅

    ВсС элСмСнты Π² Π½Π΅ΠΊΠΎΡ‚ΠΎΡ€ΠΎΠΉ стСпСни ΠΏΡ€ΠΎΡ‚ΠΈΠ²ΠΎΠ΄Π΅ΠΉΡΡ‚Π²ΡƒΡŽΡ‚ Ρ‚ΠΎΠΊΡƒ. Π­Ρ‚ΠΎ противодСйствиС Ρ‚ΠΎΠΊΡƒ называСтся сопротивлСниСм. Π§Π΅ΠΌ Π²Ρ‹ΡˆΠ΅ сопротивлСниС элСмСнта, ΠΊΠΎΠΌΠΏΠΎΠ½Π΅Π½Ρ‚Π° ΠΈΠ»ΠΈ Ρ†Π΅ΠΏΠΈ, Ρ‚Π΅ΠΌ мСньшС Ρ‚ΠΎΠΊ, создаваСмый Π΄Π°Π½Π½Ρ‹ΠΌ напряТСниСм.

    Π‘ΠΎΠΏΡ€ΠΎΡ‚ΠΈΠ²Π»Π΅Π½ΠΈΠ΅ (R) измСряСтся Π² Ом . ΠžΠΌΡ‹ прСдставлСны с ΠΏΠΎΠΌΠΎΡ‰ΡŒΡŽ грСчСской Π±ΡƒΠΊΠ²Ρ‹ ΠΎΠΌΠ΅Π³Π° (Ξ©). Π‘ тСхничСской Ρ‚ΠΎΡ‡ΠΊΠΈ зрСния ΠΎΠ΄ΠΈΠ½ ΠΎΠΌ β€” это Π²Π΅Π»ΠΈΡ‡ΠΈΠ½Π° сопротивлСния, которая ΠΎΠ³Ρ€Π°Π½ΠΈΡ‡ΠΈΠ²Π°Π΅Ρ‚ Ρ‚ΠΎΠΊ Π΄ΠΎ ΠΎΠ΄Π½ΠΎΠ³ΠΎ Π°ΠΌΠΏΠ΅Ρ€Π° ΠΏΡ€ΠΈ ΠΏΡ€ΠΈΠ»ΠΎΠΆΠ΅Π½ΠΈΠΈ элСктричСской силы Π² ΠΎΠ΄ΠΈΠ½ Π²ΠΎΠ»ΡŒΡ‚. Π­Ρ‚ΠΎ ΠΎΠΏΡ€Π΅Π΄Π΅Π»Π΅Π½ΠΈΠ΅ ΠΏΡ€ΠΎΠΈΠ»Π»ΡŽΡΡ‚Ρ€ΠΈΡ€ΠΎΠ²Π°Π½ΠΎ Π² Рисунок 8 .

    Рисунок 8 Основная элСктричСская Ρ†Π΅ΠΏΡŒ.

    БхСматичСская Π΄ΠΈΠ°Π³Ρ€Π°ΠΌΠΌΠ° На рис. 8 ΠΏΠΎΠΊΠ°Π·Π°Π½Π° батарСя, ΠΏΠΎΠ΄ΠΊΠ»ΡŽΡ‡Π΅Π½Π½Π°Ρ ΠΊ рСзистору. РСзистор — это ΠΊΠΎΠΌΠΏΠΎΠ½Π΅Π½Ρ‚, ΠΊΠΎΡ‚ΠΎΡ€Ρ‹ΠΉ обСспСчиваСт ΠΎΠΏΡ€Π΅Π΄Π΅Π»Π΅Π½Π½ΠΎΠ΅ сопротивлСниС. Как ΠΏΠΎΠΊΠ°Π·Π°Π½ΠΎ Π½Π° рисункС, сопротивлСниС 1 Ом ΠΎΠ³Ρ€Π°Π½ΠΈΡ‡ΠΈΠ²Π°Π΅Ρ‚ Ρ‚ΠΎΠΊ Π΄ΠΎ 1 А ΠΏΡ€ΠΈ ΠΏΠΎΠ΄Π°Ρ‡Π΅ 1 Π’. ΠžΠ±Ρ€Π°Ρ‚ΠΈΡ‚Π΅ Π²Π½ΠΈΠΌΠ°Π½ΠΈΠ΅, Ρ‡Ρ‚ΠΎ длинная полоса Π½Π° ΠΊΠΎΠ½Ρ†Π΅ схСмы символа Π±Π°Ρ‚Π°Ρ€Π΅ΠΈ прСдставляСт собой ΠΏΠΎΠ»ΠΎΠΆΠΈΡ‚Π΅Π»ΡŒΠ½ΡƒΡŽ ΠΊΠ»Π΅ΠΌΠΌΡƒ Π±Π°Ρ‚Π°Ρ€Π΅ΠΈ, Π° короткая полоса Π½Π° ΠΊΠΎΠ½Ρ†Π΅ прСдставляСт Π΅Π΅ ΠΎΡ‚Ρ€ΠΈΡ†Π°Ρ‚Π΅Π»ΡŒΠ½ΡƒΡŽ ΠΊΠ»Π΅ΠΌΠΌΡƒ.

    Π‘ΠΎΠ±ΠΈΡ€Π°Π΅ΠΌ всС вмСстС

    Π’Π΅ΠΏΠ΅Ρ€ΡŒ ΠΌΡ‹ ΠΎΠΏΡ€Π΅Π΄Π΅Π»ΠΈΠ»ΠΈ заряд, Ρ‚ΠΎΠΊ, напряТСниС ΠΈ сопротивлСниС. Для удобства эти элСктричСскиС свойства ΠΏΡ€ΠΈΠ²Π΅Π΄Π΅Π½Ρ‹ Π² Ρ‚Π°Π±Π»ΠΈΡ†Π΅ 1 .

    Π’Π°Π±Π»ΠΈΡ†Π° 1 : ΠžΡΠ½ΠΎΠ²Π½Ρ‹Π΅ элСктричСскиС свойства

    МногиС свойства, пСрСчислСнныС Π² Π’Π°Π±Π»ΠΈΡ†Π° 1 , ΠΌΠΎΠ³ΡƒΡ‚ Π±Ρ‹Ρ‚ΡŒ ΠΎΠΏΡ€Π΅Π΄Π΅Π»Π΅Π½Ρ‹ Ρ‡Π΅Ρ€Π΅Π· Π΄Ρ€ΡƒΠ³ΠΈΠ΅. НапримСр, Π² нашСм обсуТдСнии сопротивлСния ΠΌΡ‹ сказали, Ρ‡Ρ‚ΠΎ ΠΎΠ΄ΠΈΠ½ ΠΎΠΌ β€” это Π²Π΅Π»ΠΈΡ‡ΠΈΠ½Π° сопротивлСния, которая ΠΎΠ³Ρ€Π°Π½ΠΈΡ‡ΠΈΠ²Π°Π΅Ρ‚ Ρ‚ΠΎΠΊ Π΄ΠΎ ΠΎΠ΄Π½ΠΎΠ³ΠΎ Π°ΠΌΠΏΠ΅Ρ€Π° ΠΏΡ€ΠΈ ΠΏΡ€ΠΈΠ»ΠΎΠΆΠ΅Π½ΠΈΠΈ элСктричСской силы Π² ΠΎΠ΄ΠΈΠ½ Π²ΠΎΠ»ΡŒΡ‚. Π’ΠΎΡ‡Π½ΠΎ Ρ‚Π°ΠΊ ΠΆΠ΅ ΠΌΡ‹ ΠΌΠΎΠΆΠ΅ΠΌ ΠΏΠ΅Ρ€Π΅ΠΎΠΏΡ€Π΅Π΄Π΅Π»ΠΈΡ‚ΡŒ Π°ΠΌΠΏΠ΅Ρ€ ΠΈ Π²ΠΎΠ»ΡŒΡ‚ ΡΠ»Π΅Π΄ΡƒΡŽΡ‰ΠΈΠΌ ΠΎΠ±Ρ€Π°Π·ΠΎΠΌ:

    1. Один Π°ΠΌΠΏΠ΅Ρ€ – это сила Ρ‚ΠΎΠΊΠ°, которая гСнСрируСтся, ΠΊΠΎΠ³Π΄Π° элСктричСская сила Π² ΠΎΠ΄ΠΈΠ½ Π²ΠΎΠ»ΡŒΡ‚ прикладываСтся ΠΊ ΡΠΎΠΏΡ€ΠΎΡ‚ΠΈΠ²Π»Π΅Π½ΠΈΡŽ Π² ΠΎΠ΄ΠΈΠ½ ΠΎΠΌ.
    2. Один Π²ΠΎΠ»ΡŒΡ‚ β€” это Π²Π΅Π»ΠΈΡ‡ΠΈΠ½Π° элСктричСской силы, Π½Π΅ΠΎΠ±Ρ…ΠΎΠ΄ΠΈΠΌΠΎΠΉ для создания силы Ρ‚ΠΎΠΊΠ° Π² ΠΎΠ΄ΠΈΠ½ Π°ΠΌΠΏΠ΅Ρ€ Ρ‡Π΅Ρ€Π΅Π· сопротивлСниС
    Ом.

    ΠšΠΎΠ½Ρ‚Ρ€ΠΎΠ»ΡŒΠ½Ρ‹ΠΉ вопрос

    Π§Ρ‚ΠΎ Ρ‚Π°ΠΊΠΎΠ΅ сопротивлСниС?

    Π‘ΠΎΠΏΡ€ΠΎΡ‚ΠΈΠ²Π»Π΅Π½ΠΈΠ΅ β€” это противостояниС Ρ‚ΠΎΠΊΡƒ.

    Π§Ρ‚ΠΎ Ρ‚Π°ΠΊΠΎΠ΅ основная Π΅Π΄ΠΈΠ½ΠΈΡ†Π° сопротивлСния ΠΈ ΠΊΠ°ΠΊ ΠΎΠ½Π° опрСдСляСтся?

    Π•Π΄ΠΈΠ½ΠΈΡ†Π΅ΠΉ сопротивлСния являСтся Ом (Ом). Один Ом β€” это Π²Π΅Π»ΠΈΡ‡ΠΈΠ½Π° сопротивлСния, которая ΠΎΠ³Ρ€Π°Π½ΠΈΡ‡ΠΈΠ²Π°Π΅Ρ‚ силу Ρ‚ΠΎΠΊΠ° Π΄ΠΎ 1 Π°ΠΌΠΏΠ΅Ρ€Π° ΠΏΡ€ΠΈ ΠΏΡ€ΠΈΠ»ΠΎΠΆΠ΅Π½ΠΈΠΈ ΠΎΠ΄Π½ΠΎΠ³ΠΎ Π²ΠΎΠ»ΡŒΡ‚Π°. 1 Ом = 1 А/Π’.

    ΠžΠΏΡ€Π΅Π΄Π΅Π»ΠΈΡ‚Π΅ ΠΊΠ°ΠΆΠ΄ΠΎΠ΅ ΠΈΠ· ΡΠ»Π΅Π΄ΡƒΡŽΡ‰ΠΈΡ… Π·Π½Π°Ρ‡Π΅Π½ΠΈΠΉ Ρ‡Π΅Ρ€Π΅Π· Π΄Π²Π° Π΄Ρ€ΡƒΠ³ΠΈΡ…: Ρ‚ΠΎΠΊ, напряТСниС ΠΈ сопротивлСниС.

    1 Π’ β€” это сила, нСобходимая для Ρ‚ΠΎΠ³ΠΎ, Ρ‡Ρ‚ΠΎΠ±Ρ‹ Π²Ρ‹Π·Π²Π°Ρ‚ΡŒ Ρ‚ΠΎΠΊ силой 1 Π°ΠΌΠΏΠ΅Ρ€ Ρ‡Π΅Ρ€Π΅Π· сопротивлСниС 1 Ом.
    1 А β€” это сила Ρ‚ΠΎΠΊΠ°, Π²ΠΎΠ·Π½ΠΈΠΊΠ°ΡŽΡ‰Π°Ρ ΠΏΡ€ΠΈ ΠΏΡ€ΠΈΠ»ΠΎΠΆΠ΅Π½ΠΈΠΈ 1 Π²ΠΎΠ»ΡŒΡ‚Π° ΠΊ ΡΠΎΠΏΡ€ΠΎΡ‚ΠΈΠ²Π»Π΅Π½ΠΈΡŽ 1 Ом.
    1 Ом β€” это сопротивлСниС, ΠΊΠΎΡ‚ΠΎΡ€ΠΎΠ΅ ΠΎΠ³Ρ€Π°Π½ΠΈΡ‡ΠΈΠ²Π°Π΅Ρ‚ Ρ‚ΠΎΠΊ Π΄ΠΎ 1 Π°ΠΌΠΏΠ΅Ρ€Π° ΠΏΡ€ΠΈ ΠΏΡ€ΠΈΠ»ΠΎΠΆΠ΅Π½ΠΈΠΈ 1 Π²ΠΎΠ»ΡŒΡ‚Π°.

    Β 

    Π’Ρ‹ нашли apk для Android? Π’Ρ‹ ΠΌΠΎΠΆΠ΅Ρ‚Π΅ Π½Π°ΠΉΡ‚ΠΈ Π½ΠΎΠ²Ρ‹Π΅ бСсплатныС ΠΈΠ³Ρ€Ρ‹ ΠΈ прилоТСния для Android.

    ЭлСктричСский Ρ‚ΠΎΠΊ, напряТСниС ΠΈ элСктричСская ΠΌΠΎΡ‰Π½ΠΎΡΡ‚ΡŒ

    Π§Ρ‚ΠΎ Ρ‚Π°ΠΊΠΎΠ΅ элСктричСский Ρ‚ΠΎΠΊ?

    Π’Π΅Ρ€ΠΌΠΈΠ½ «элСктричСский Ρ‚ΠΎΠΊΒ» ΠΎΠ·Π½Π°Ρ‡Π°Π΅Ρ‚ ΠΏΠΎΡ‚ΠΎΠΊ элСктронов ΠΈΠ»ΠΈ элСктричСских зарядов. Π”Ρ€Π΅ΠΉΡ„ элСктронов ΠΏΡ€ΠΎΠΏΠΎΡ€Ρ†ΠΈΠΎΠ½Π°Π»Π΅Π½ Π²Π΅Π»ΠΈΡ‡ΠΈΠ½Π΅ ΠΏΡ€ΠΈΠ»ΠΎΠΆΠ΅Π½Π½ΠΎΠ³ΠΎ напряТСния ΠΈΠ»ΠΈ разности ΠΏΠΎΡ‚Π΅Π½Ρ†ΠΈΠ°Π»ΠΎΠ². Π§Π΅ΠΌ большС напряТСниС, Ρ‚Π΅ΠΌ большС Π±ΡƒΠ΄Π΅Ρ‚ Π΄Ρ€Π΅ΠΉΡ„ элСктронов. ΠšΠΎΠ»ΠΈΡ‡Π΅ΡΡ‚Π²ΠΎ элСктричСского Ρ‚ΠΎΠΊΠ°, ΠΊΠΎΡ‚ΠΎΡ€Ρ‹ΠΌ ΠΎΠ±Π»Π°Π΄Π°Π΅Ρ‚ Ρ†Π΅ΠΏΡŒ, ΠΏΡ€ΠΎΠΏΠΎΡ€Ρ†ΠΈΠΎΠ½Π°Π»ΡŒΠ½ΠΎ плотности элСктронов ΠΈΠ»ΠΈ элСктричСскому заряду.

    НапряТСниС

    НапряТСниС, ΠΏΡ€ΠΈΠΊΠ»Π°Π΄Ρ‹Π²Π°Π΅ΠΌΠΎΠ΅ ΠΊ Π΄Π²ΡƒΠΌ ΠΊΠ»Π΅ΠΌΠΌΠ°ΠΌ элСктричСской Ρ†Π΅ΠΏΠΈ, ΠΈΠ½ΠΈΡ†ΠΈΠΈΡ€ΡƒΠ΅Ρ‚ ΠΏΠΎΡ‚ΠΎΠΊ элСктричСского заряда. Π‘ΠΊΠΎΡ€ΠΎΡΡ‚ΡŒ протСкания заряда ΠΏΡ€ΠΎΠΏΠΎΡ€Ρ†ΠΈΠΎΠ½Π°Π»ΡŒΠ½Π° разности ΠΏΠΎΡ‚Π΅Π½Ρ†ΠΈΠ°Π»ΠΎΠ², ΠΏΡ€ΠΈΠ»ΠΎΠΆΠ΅Π½Π½ΠΎΠΉ ΠΊ ΠΌΠ°Ρ‚Π΅Ρ€ΠΈΠ°Π»Ρƒ. ЭлСктричСскоС сопротивлСниС опрСдСляСт Π²Π΅Π»ΠΈΡ‡ΠΈΠ½Ρƒ ΠΏΠΎΡ‚ΠΎΠΊΠ° заряда ΠΈΠ»ΠΈ просто замСдляСт ΠΏΠΎΡ‚ΠΎΠΊ элСктронов Π² Ρ†Π΅ΠΏΠΈ.

    ЭлСктроэнСргия

    ЭлСктроэнСргия – это ΡΠΊΠΎΡ€ΠΎΡΡ‚ΡŒ ΠΏΠ΅Ρ€Π΅Π΄Π°Ρ‡ΠΈ элСктричСских зарядов ΠΏΠΎ элСктричСской Ρ†Π΅ΠΏΠΈ. ΠŸΠΎΡ‚Ρ€Π΅Π±Π»Π΅Π½ΠΈΠ΅ элСктроэнСргии Π² коммСрчСских ΠΈ ΠΏΡ€ΠΎΠΌΡ‹ΡˆΠ»Π΅Π½Π½Ρ‹Ρ… цСлях оцСниваСтся Π² пСрСсчСтС Π½Π° ΡΠ»Π΅ΠΊΡ‚Ρ€ΠΈΡ‡Π΅ΡΠΊΡƒΡŽ ΠΌΠΎΡ‰Π½ΠΎΡΡ‚ΡŒ.

    CC0 1.0 | ΠšΡ€Π΅Π΄ΠΈΡ‚Ρ‹ ΠΈΠ·ΠΎΠ±Ρ€Π°ΠΆΠ΅Π½ΠΈΠΉ: https://commons.wikimedia.org | GorillaWarfare

    ЭлСктричСский заряд β€” это Ρ„ΡƒΠ½Π΄Π°ΠΌΠ΅Π½Ρ‚Π°Π»ΡŒΠ½ΠΎΠ΅ свойство, ΠΊΠΎΡ‚ΠΎΡ€Ρ‹ΠΌ ΠΎΠ±Π»Π°Π΄Π°ΡŽΡ‚ Π°Ρ‚ΠΎΠΌΠ½Ρ‹Π΅ ΠΈ субатомныС частицы ΠΌΠ°Ρ‚Π΅Ρ€ΠΈΠΈ. Π­Ρ‚ΠΎ свойство элСктричСского заряда опрСдСляСт ΠΏΠΎΠ²Π΅Π΄Π΅Π½ΠΈΠ΅ частиц ΠΏΡ€ΠΈ воздСйствии элСктричСского ΠΈ ΠΌΠ°Π³Π½ΠΈΡ‚Π½ΠΎΠ³ΠΎ ΠΏΠΎΠ»Π΅ΠΉ. ЭлСктричСский заряд подчиняСтся Π·Π°ΠΊΠΎΠ½Ρƒ сохранСния энСргии, Π΅Π³ΠΎ нСльзя Π½ΠΈ ΡΠΎΠ·Π΄Π°Ρ‚ΡŒ, Π½ΠΈ ΡƒΠ½ΠΈΡ‡Ρ‚ΠΎΠΆΠΈΡ‚ΡŒ.
    ЭлСктричСскиС заряды ΠΌΠΎΠ³ΡƒΡ‚ Π±Ρ‹Ρ‚ΡŒ ΠΏΠΎΠ»ΠΎΠΆΠΈΡ‚Π΅Π»ΡŒΠ½Ρ‹ΠΌΠΈ ΠΈ ΠΎΡ‚Ρ€ΠΈΡ†Π°Ρ‚Π΅Π»ΡŒΠ½Ρ‹ΠΌΠΈ. НапримСр, элСктроны Π°Ρ‚ΠΎΠΌΠ° ΠΈΠΌΠ΅ΡŽΡ‚ ΠΎΡ‚Ρ€ΠΈΡ†Π°Ρ‚Π΅Π»ΡŒΠ½Ρ‹ΠΉ заряд, Π° ΠΏΡ€ΠΎΡ‚ΠΎΠ½Ρ‹ β€” ΠΏΠΎΠ»ΠΎΠΆΠΈΡ‚Π΅Π»ΡŒΠ½Ρ‹ΠΉ. ΠžΠ΄Π½ΠΎΠΈΠΌΠ΅Π½Π½Ρ‹Π΅ заряды Π²Ρ‹Π·Ρ‹Π²Π°ΡŽΡ‚ силу отталкивания, Π° Ρ€Π°Π·Π½ΠΎΠΈΠΌΠ΅Π½Π½Ρ‹Π΅ заряды Π²Ρ‹Π·Ρ‹Π²Π°ΡŽΡ‚ силу притяТСния. Π­ΠΊΡΠΏΠ΅Ρ€ΠΈΠΌΠ΅Π½Ρ‚Π°Π»ΡŒΠ½ΠΎ установлСно, Ρ‡Ρ‚ΠΎ элСктроны ΠΈ ΠΏΡ€ΠΎΡ‚ΠΎΠ½Ρ‹ ΠΈΠΌΠ΅ΡŽΡ‚ ΠΎΠ΄ΠΈΠ½Π°ΠΊΠΎΠ²Ρ‹Π΅ Π²Π΅Π»ΠΈΡ‡ΠΈΠ½Ρ‹ ΠΎΡ‚Ρ€ΠΈΡ†Π°Ρ‚Π΅Π»ΡŒΠ½ΠΎΠ³ΠΎ ΠΈ ΠΏΠΎΠ»ΠΎΠΆΠΈΡ‚Π΅Π»ΡŒΠ½ΠΎΠ³ΠΎ заряда соотвСтствСнно. НСйтроны Π² ядрС Π°Ρ‚ΠΎΠΌΠ° ΠΎΠ±Π»Π°Π΄Π°ΡŽΡ‚ Π½ΡƒΠ»Π΅Π²Ρ‹ΠΌ зарядом. Π•Π΄ΠΈΠ½ΠΈΡ†Π΅ΠΉ элСктричСского заряда Π² систСмС БИ являСтся ΠΊΡƒΠ»ΠΎΠ½, ΠΊΠΎΡ‚ΠΎΡ€Ρ‹ΠΉ Ρ€Π°Π²Π΅Π½ 6,24Γ—10 9 .0421 18 элСктрона.
    Дисбаланс элСктричСского заряда Π½Π° повСрхности ΠΌΠ°Ρ‚Π΅Ρ€ΠΈΠ°Π»Π° ΠΏΡ€ΠΈΠ²ΠΎΠ΄ΠΈΡ‚ ΠΊ Ρ€Π°Π·Π²ΠΈΡ‚ΠΈΡŽ статичСского элСктричСства. Π˜Π·Π±Ρ‹Ρ‚ΠΎΠΊ заряда остаСтся Π½Π° повСрхности Π΄ΠΎ Ρ‚Π΅Ρ… ΠΏΠΎΡ€, ΠΏΠΎΠΊΠ° ΠΎΠ½ Π½Π΅ Π±ΡƒΠ΄Π΅Ρ‚ ΠΏΠ΅Ρ€Π΅ΠΌΠ΅Ρ‰Π΅Π½ Ρ‡Π΅Ρ€Π΅Π· элСктричСский разряд ΠΈΠ»ΠΈ Π½Π΅ ΠΏΡ€ΠΎΠΉΠ΄Π΅Ρ‚ Ρ‡Π΅Ρ€Π΅Π· мост элСктричСской Ρ†Π΅ΠΏΠΈ. БтатичСскоС элСктричСство ΠΌΠΎΠΆΠ΅Ρ‚ Π±Ρ‹Ρ‚ΡŒ создано ΠΏΡƒΡ‚Π΅ΠΌ трСния ΠΎΠ΄Π½ΠΎΠ³ΠΎ ΠΌΠ°Ρ‚Π΅Ρ€ΠΈΠ°Π»Π° ΠΎ Π΄Ρ€ΡƒΠ³ΠΎΠΉ, Ρ‡Ρ‚ΠΎ ΠΏΡ€ΠΈΠ²ΠΎΠ΄ΠΈΡ‚ ΠΊ пСрСносу ΠΎΡ‚Ρ€ΠΈΡ†Π°Ρ‚Π΅Π»ΡŒΠ½Ρ‹Ρ… зарядов ΠΈΠ»ΠΈ элСктронов.

    Π—Π°ΠΊΠΎΠ½ Ома

    Π—Π°ΠΊΠΎΠ½ Ома устанавливаСт Π·Π°Π²ΠΈΡΠΈΠΌΠΎΡΡ‚ΡŒ ΠΌΠ΅ΠΆΠ΄Ρƒ напряТСниСм, сопротивлСниСм ΠΈ Ρ‚ΠΎΠΊΠΎΠΌ Π² элСктричСской Ρ†Π΅ΠΏΠΈ. Π—Π°ΠΊΠΎΠ½ Ома гласит, Ρ‡Ρ‚ΠΎ Π² Ρ†Π΅ΠΏΠΈ ΠΏΠ°Π΄Π΅Π½ΠΈΠ΅ напряТСния измСняСтся ΠΏΡ€ΠΎΠΏΠΎΡ€Ρ†ΠΈΠΎΠ½Π°Π»ΡŒΠ½ΠΎ силС элСктричСского Ρ‚ΠΎΠΊΠ°.

    ΠœΠ°Ρ‚Π΅ΠΌΠ°Ρ‚ΠΈΡ‡Π΅ΡΠΊΠΈ,

    Π’Β Ξ±Β I,

    , Π³Π΄Π΅ V прСдставляСт собой напряТСниС, Π° I прСдставляСт Ρ‚ΠΎΠΊ.

    ПослС Π·Π°ΠΌΠ΅Π½Ρ‹ константы ΠΏΡ€ΠΎΠΏΠΎΡ€Ρ†ΠΈΠΎΠ½Π°Π»ΡŒΠ½ΠΎΡΡ‚ΠΈ константой ΡƒΡ€Π°Π²Π½Π΅Π½ΠΈΠ΅ сводится ΠΊ

    Π’=IR,

    , Π³Π΄Π΅Β R – константа ΠΏΡ€ΠΎΠΏΠΎΡ€Ρ†ΠΈΠΎΠ½Π°Π»ΡŒΠ½ΠΎΡΡ‚ΠΈ, извСстная ΠΊΠ°ΠΊ элСктричСскоС сопротивлСниС. Π•Π΄ΠΈΠ½ΠΈΡ†Π΅ΠΉ измСрСния являСтся Ом, Ом.

    ΠŸΡ€ΠΈΠ²Π΅Π΄Π΅Π½Π½ΠΎΠ΅ Π²Ρ‹ΡˆΠ΅ ΡΠΎΠΎΡ‚Π½ΠΎΡˆΠ΅Π½ΠΈΠ΅ ΠΎΠ·Π½Π°Ρ‡Π°Π΅Ρ‚, Ρ‡Ρ‚ΠΎ напряТСниС V ΠΈ Ρ‚ΠΎΠΊ I ΠΈΠΌΠ΅ΡŽΡ‚ Π»ΠΈΠ½Π΅ΠΉΠ½ΡƒΡŽ Π·Π°Π²ΠΈΡΠΈΠΌΠΎΡΡ‚ΡŒ с сопротивлСниСм R Π² Π²ΠΈΠ΄Π΅ Π½Π°ΠΊΠ»ΠΎΠ½Π°.

    Π‘Π»Π΅Π΄ΠΎΠ²Π°Ρ‚Π΅Π»ΡŒΠ½ΠΎ, ΠΌΡ‹ ΠΌΠΎΠΆΠ΅ΠΌ ΡΠΊΠ°Π·Π°Ρ‚ΡŒ, Ρ‡Ρ‚ΠΎ Π·Π°ΠΊΠΎΠ½ Ома справСдлив Ρ‚ΠΎΠ»ΡŒΠΊΠΎ Π² Ρ‚ΠΎΠΌ случаС, Ссли напряТСниС V ΠΈ Ρ‚ΠΎΠΊ I ΠΈΠ·ΠΌΠ΅Π½ΡΡŽΡ‚ΡΡ Π»ΠΈΠ½Π΅ΠΉΠ½ΠΎ.

    АмпСр β€” Π΅Π΄ΠΈΠ½ΠΈΡ†Π° измСрСния элСктричСского Ρ‚ΠΎΠΊΠ° Π² систСмС БИ, принятая ΡƒΡ‡Π΅Π½Ρ‹ΠΌΠΈ ΠΈ ΠΈΠ½ΠΆΠ΅Π½Π΅Ρ€Π°ΠΌΠΈ Π²ΠΎ всСм ΠΌΠΈΡ€Π΅. Один Π°ΠΌΠΏΠ΅Ρ€ ΠΎΠ·Π½Π°Ρ‡Π°Π΅Ρ‚ ΠΏΠΎΡ‚ΠΎΠΊ заряда Π² ΠΎΠ΄ΠΈΠ½ ΠΊΡƒΠ»ΠΎΠ½ Π·Π° ΠΎΠ΄Π½Ρƒ сСкунду.
    ЭлСктричСский Ρ‚ΠΎΠΊ Π² Ρ†Π΅ΠΏΠΈ измСряСтся Π°ΠΌΠΏΠ΅Ρ€ΠΌΠ΅Ρ‚Ρ€ΠΎΠΌ ΠΈΠ»ΠΈ ΠΌΡƒΠ»ΡŒΡ‚ΠΈΠΌΠ΅Ρ‚Ρ€ΠΎΠΌ. АмпСрмСтр измСряСт Ρ‚ΠΎΠ»ΡŒΠΊΠΎ элСктричСский Ρ‚ΠΎΠΊ, Ρ‚ΠΎΠ³Π΄Π° ΠΊΠ°ΠΊ ΠΌΡƒΠ»ΡŒΡ‚ΠΈΠΌΠ΅Ρ‚Ρ€ для измСрСния силы Ρ‚ΠΎΠΊΠ° являСтся ΡƒΠ½ΠΈΠ²Π΅Ρ€ΡΠ°Π»ΡŒΠ½Ρ‹ΠΌ ΠΏΡ€ΠΈΠ±ΠΎΡ€ΠΎΠΌ. Он измСряСт мноТСство ΠΏΠ°Ρ€Π°ΠΌΠ΅Ρ‚Ρ€ΠΎΠ², Ρ‚Π°ΠΊΠΈΡ… ΠΊΠ°ΠΊ Ρ‚ΠΎΠΊ, напряТСниС, Ρ€Π°Π·Π½ΠΎΡΡ‚ΡŒ ΠΏΠΎΡ‚Π΅Π½Ρ†ΠΈΠ°Π»ΠΎΠ², сопротивлСниС ΠΈ Ρ‚. Π΄.

    ЭлСктричСская ΠΌΠΎΡ‰Π½ΠΎΡΡ‚ΡŒ ΠΈ Π½Π°Π³Ρ€Π΅Π²Π°Ρ‚Π΅Π»ΡŒΠ½Ρ‹ΠΉ эффСкт элСктричСского Ρ‚ΠΎΠΊΠ°

    ЭлСктроэнСргия (P) β€” это ΡΠΊΠΎΡ€ΠΎΡΡ‚ΡŒ, с ΠΊΠΎΡ‚ΠΎΡ€ΠΎΠΉ пСрСдаСтся элСктричСский заряд для выполнСния ΠΊΠΎΠ½ΠΊΡ€Π΅Ρ‚Π½ΠΎΠΉ Π·Π°Π΄Π°Ρ‡ΠΈ. Π­Ρ‚ΠΎ ΡΠΊΠΎΡ€ΠΎΡΡ‚ΡŒ ΠΏΠ΅Ρ€Π΅Π΄Π°Ρ‡ΠΈ элСктричСской энСргии, ΠΈΡΠΏΠΎΠ»ΡŒΠ·ΡƒΠ΅ΠΌΠΎΠΉ для Ρ€Π°Π±ΠΎΡ‚Ρ‹. Π Π°Π±ΠΎΡ‚Π° ΠΌΠΎΠΆΠ΅Ρ‚ Π·Π°ΠΊΠ»ΡŽΡ‡Π°Ρ‚ΡŒΡΡ Π² Ρ‚ΠΎΠΌ, Ρ‡Ρ‚ΠΎΠ±Ρ‹ Π·Π°ΠΆΠ΅Ρ‡ΡŒ ΡΠ»Π΅ΠΊΡ‚Ρ€ΠΈΡ‡Π΅ΡΠΊΡƒΡŽ Π»Π°ΠΌΠΏΠΎΡ‡ΠΊΡƒ Π² Ρ†Π΅ΠΏΠΈ ΠΈΠ»ΠΈ ΠΏΠΎΡ‚Ρ€Π΅Π±Π»ΡΡ‚ΡŒ ΡΠ»Π΅ΠΊΡ‚Ρ€ΠΎΡΠ½Π΅Ρ€Π³ΠΈΡŽ Π² домашнСм хозяйствС.
    ЭлСктричСская ΠΌΠΎΡ‰Π½ΠΎΡΡ‚ΡŒ Π΅ΡΡ‚ΡŒ ΠΎΡ‚Π½ΠΎΡˆΠ΅Π½ΠΈΠ΅ ΠΌΠ΅ΠΆΠ΄Ρƒ Π²Ρ‹ΠΏΠΎΠ»Π½Π΅Π½Π½ΠΎΠΉ Ρ€Π°Π±ΠΎΡ‚ΠΎΠΉ W ΠΈ Π½ΠΎΡ€ΠΌΠΎΠΉ Π²Ρ€Π΅ΠΌΠ΅Π½ΠΈ t. ΠœΠ°Ρ‚Π΅ΠΌΠ°Ρ‚ΠΈΡ‡Π΅ΡΠΊΠΈ это ΠΌΠΎΠΆΠ½ΠΎ Π·Π°ΠΏΠΈΡΠ°Ρ‚ΡŒ Ρ‚Π°ΠΊ:

    P=Wt

    ΠŸΡ€ΠΈ ΡƒΠΏΡ€ΠΎΡ‰Π΅Π½ΠΈΠΈ ΠΏΠΎΠ»ΡƒΡ‡Π°Π΅ΠΌ

    P=WQΓ—Qt

    Π—Π΄Π΅ΡΡŒ ΡΠΊΠΎΡ€ΠΎΡΡ‚ΡŒ пСрСноса заряда, Qt – элСктричСский Ρ‚ΠΎΠΊ, I, ΠΈ ΠΎΡ‚Π½ΠΎΡˆΠ΅Π½ΠΈΠ΅ Ρ€Π°Π±ΠΎΡ‚Ρ‹ ΠΊ заряду. ΠΏΠ΅Ρ€Π΅Π΄Π°Ρ‡Π° – ΠΏΠ°Π΄Π΅Π½ΠΈΠ΅ напряТСния, Π’. Π‘Π»Π΅Π΄ΠΎΠ²Π°Ρ‚Π΅Π»ΡŒΠ½ΠΎ, ΡƒΡ€Π°Π²Π½Π΅Π½ΠΈΠ΅ ΠΏΡ€ΠΈΠ½ΠΈΠΌΠ°Π΅Ρ‚ Π²ΠΈΠ΄

    Π =VI.

    Π­Ρ‚ΠΎ Π²Ρ‹Ρ€Π°ΠΆΠ΅Π½ΠΈΠ΅ для мощности являСтся Ρ„ΡƒΠ½Π΄Π°ΠΌΠ΅Π½Ρ‚Π°Π»ΡŒΠ½Ρ‹ΠΌ ΡΠΎΠΎΡ‚Π½ΠΎΡˆΠ΅Π½ΠΈΠ΅ΠΌ Π² соврСмСнной ΠΊΠΎΠ½Ρ†Π΅ΠΏΡ†ΠΈΠΈ элСктричСства.

    МногиС элСктричСскиС ΠΊΠΎΠΌΠΏΠΎΠ½Π΅Π½Ρ‚Ρ‹, Ρ‚Π°ΠΊΠΈΠ΅ ΠΊΠ°ΠΊ элСктродвигатСли, ΠΏΠΎΡ‚Ρ€Π΅Π±Π»ΡΡŽΡ‚ ΡΠ»Π΅ΠΊΡ‚Ρ€ΠΎΡΠ½Π΅Ρ€Π³ΠΈΡŽ, Π½ΠΎ Π² Ρ‚ΠΎ ΠΆΠ΅ врСмя ΠΈΡΠΏΡ‹Ρ‚Ρ‹Π²Π°ΡŽΡ‚ гистСрСзисныС ΠΏΠΎΡ‚Π΅Ρ€ΠΈ Π² Π²ΠΈΠ΄Π΅ Ρ‚Π΅ΠΏΠ»ΠΎΠ²ΠΎΠΉ энСргии. Π­Ρ‚ΠΈ гистСрСзисныС ΠΏΠΎΡ‚Π΅Ρ€ΠΈ обусловлСны Π²Π½ΡƒΡ‚Ρ€Π΅Π½Π½ΠΈΠΌΠΈ сопротивлСниями ΠΈ Π½Π΅ΡΠΎΠ²Π΅Ρ€ΡˆΠ΅Π½ΡΡ‚Π²Π°ΠΌΠΈ. Π­Ρ‚ΠΈ ΠΏΠΎΡ‚Π΅Ρ€ΠΈ извСстны ΠΊΠ°ΠΊ элСктричСский гистСрСзис. ΠŸΠΎΡ‚Ρ€Π΅Π±Π»Π΅Π½ΠΈΠ΅ элСктричСской энСргии Π² этих устройствах осущСствляСтся ΠΊΠ°ΠΊ мСханичСская Ρ€Π°Π±ΠΎΡ‚Π°, Π½ΠΎ Π·Π° счСт Ρ‚Π΅ΠΏΠ»ΠΎΠ²ΠΎΠΉ энСргии. ΠœΠΎΡ‰Π½ΠΎΡΡ‚ΡŒ Π²Ρ‹Ρ€Π°Π±Π°Ρ‚Ρ‹Π²Π°Π΅ΠΌΠΎΠ³ΠΎ Ρ‚Π΅ΠΏΠ»Π° ΠΌΠΎΠΆΠ½ΠΎ Ρ€Π°ΡΡΡ‡ΠΈΡ‚Π°Ρ‚ΡŒ ΠΏΠΎ Ρ„ΠΎΡ€ΠΌΡƒΠ»Π΅

    P=I2R

    Π­Ρ‚ΠΎ ΡΠΎΠΎΡ‚Π½ΠΎΡˆΠ΅Π½ΠΈΠ΅ ΡˆΠΈΡ€ΠΎΠΊΠΎ извСстно ΠΊΠ°ΠΊ Π·Π°ΠΊΠΎΠ½ ДТоуля, ΠΈΠ»ΠΈ Π½Π°Π³Ρ€Π΅Π² ДТоуля, ΠΈΠ»ΠΈ омичСский Π½Π°Π³Ρ€Π΅Π², ΠΈΠ»ΠΈ рСзистивный Π½Π°Π³Ρ€Π΅Π².

    Π’Π΅Π»ΠΈΡ‡ΠΈΠ½Π° выдСляСмого Ρ‚Π΅ΠΏΠ»Π° опрСдСляСтся Π²Ρ‹Ρ€Π°ΠΆΠ΅Π½ΠΈΠ΅ΠΌ

    H=I2Rt

    Π“Π΄Π΅Β H прСдставляСт Ρ‚Π΅ΠΏΠ»ΠΎΡ‚Π²ΠΎΡ€Π½ΡƒΡŽ ΡΠΏΠΎΡΠΎΠ±Π½ΠΎΡΡ‚ΡŒ, Π° t – Π²Ρ€Π΅ΠΌΠ΅Π½Π½ΠΎΠΉ ΠΈΠ½Ρ‚Π΅Ρ€Π²Π°Π».

    ΠšΠ°ΠΆΠ΄Ρ‹ΠΉ ΠΏΡ€ΠΎΠ²ΠΎΠ΄Π½ΠΈΠΊ с Ρ‚ΠΎΠΊΠΎΠΌ ΠΎΠ±Π»Π°Π΄Π°Π΅Ρ‚ сопротивлСниСм. Π­Ρ‚ΠΎ ΡΠΎΠΎΡ‚Π½ΠΎΡˆΠ΅Π½ΠΈΠ΅ опрСдСляСт Π²Π΅Π»ΠΈΡ‡ΠΈΠ½Ρƒ Ρ‚Π΅ΠΏΠ»Π°, Π²Ρ‹Π΄Π΅Π»ΡΡŽΡ‰Π΅Π³ΠΎΡΡ Π² Ρ€Π°Π·Π½Ρ‹Ρ… ΠΏΡ€ΠΎΠ²ΠΎΠ΄Π½ΠΈΠΊΠ°Ρ…, ΠΈΠΌΠ΅ΡŽΡ‰ΠΈΡ… Ρ€Π°Π·Π½ΠΎΠ΅ сСчСниС.

    ΠšΠΎΠ½Ρ‚Π΅ΠΊΡΡ‚ ΠΈ прилоТСния

    Π­Ρ‚Π° Ρ‚Π΅ΠΌΠ° прСподаСтся Π½Π° Ρ€Π°Π·Π»ΠΈΡ‡Π½Ρ‹Ρ… курсах Π±Π°ΠΊΠ°Π»Π°Π²Ρ€ΠΈΠ°Ρ‚Π° ΠΈ магистратуры, Ρ‚Π°ΠΊΠΈΡ… ΠΊΠ°ΠΊ:

    • Π‘Π°ΠΊΠ°Π»Π°Π²Ρ€ Ρ‚Π΅Ρ…Π½ΠΎΠ»ΠΎΠ³ΠΈΠΈ (элСктротСхника)
    • Π‘Π°ΠΊΠ°Π»Π°Π²Ρ€ Ρ‚Π΅Ρ…Π½ΠΎΠ»ΠΎΠ³ΠΈΠΈ (элСктроника)
    • Π‘Π°ΠΊΠ°Π»Π°Π²Ρ€ Ρ‚Π΅Ρ…Π½ΠΎΠ»ΠΎΠ³ΠΈΠΈ (приборостроСниС)
    • ΠœΠ°Π³ΠΈΡΡ‚Ρ€ Ρ‚Π΅Ρ…Π½ΠΎΠ»ΠΎΠ³ΠΈΠΈ (элСктротСхника)

    ΠŸΡ€Π°ΠΊΡ‚ΠΈΡ‡Π΅ΡΠΊΠΈΠ΅ Π·Π°Π΄Π°Ρ‡ΠΈ

    Вопрос 1. Какой ΠΈΠ· ΡΠ»Π΅Π΄ΡƒΡŽΡ‰ΠΈΡ… ΠΏΠ°Ρ€Π°ΠΌΠ΅Ρ‚Ρ€ΠΎΠ² ΠΏΡ€ΠΈΠ²ΠΎΠ΄ΠΈΡ‚ Π² Π΄Π²ΠΈΠΆΠ΅Π½ΠΈΠ΅ элСктроны Π² элСктричСской Ρ†Π΅ΠΏΠΈ?

    1. НапряТСниС ΠΎΡ‚ источника напряТСния
    2. Π‘ΠΎΠΏΡ€ΠΎΡ‚ΠΈΠ²Π»Π΅Π½ΠΈΠ΅
    3. ЭлСктричСскоС ΠΏΠΎΠ»Π΅
    4. ΠœΠ°Π³Π½ΠΈΡ‚Π½ΠΎΠ΅ ΠΏΠΎΠ»Π΅

    ΠžΡ‚Π²Π΅Ρ‚: Π’Π°Ρ€ΠΈΠ°Π½Ρ‚

    элСктричСская Ρ†Π΅ΠΏΡŒ.

    Q 2. Π’ ΠΊΠ°ΠΊΠΎΠΌ ΠΈΠ· ΡΠ»Π΅Π΄ΡƒΡŽΡ‰ΠΈΡ… условий ΠΌΡ‹ ΠΈΡΠΏΠΎΠ»ΡŒΠ·ΡƒΠ΅ΠΌ Π·Π°ΠΊΠΎΠ½ Ома?

    1. Когда Ρ‚ΠΎΠΊ ΠΈ напряТСниС ΠΈΠΌΠ΅ΡŽΡ‚ Π»ΠΈΠ½Π΅ΠΉΠ½ΡƒΡŽ Π·Π°Π²ΠΈΡΠΈΠΌΠΎΡΡ‚ΡŒ
    2. Когда Ρ‚ΠΎΠΊ ΠΈ напряТСниС связаны Π½Π΅Π»ΠΈΠ½Π΅ΠΉΠ½ΠΎΠΉ Π·Π°Π²ΠΈΡΠΈΠΌΠΎΡΡ‚ΡŒΡŽ
    3. Когда ΠΏΠ°Ρ€Π°ΠΌΠ΅Ρ‚Ρ€Ρ‹ Ρ‚ΠΎΠΊΠ° ΠΈΠ·ΠΌΠ΅Π½ΡΡŽΡ‚ΡΡ параболичСски
    4. НичСго ΠΈΠ· этого

    ΠžΡ‚Π²Π΅Ρ‚: Π’Π°Ρ€ΠΈΠ°Π½Ρ‚ a

    ПояснСниС: Π—Π°ΠΊΠΎΠ½ Ома ΠΌΠΎΠΆΠ΅Ρ‚ ΠΏΡ€ΠΈΠΌΠ΅Π½ΡΡ‚ΡŒΡΡ, ΠΊΠΎΠ³Π΄Π° Ρ‚ΠΎΠΊ ΠΈ напряТСниС ΠΈΠ·ΠΌΠ΅Π½ΡΡŽΡ‚ΡΡ Π»ΠΈΠ½Π΅ΠΉΠ½ΠΎ .

    Q 3. КакоС Π΄Ρ€ΡƒΠ³ΠΎΠ΅ Π½Π°Π·Π²Π°Π½ΠΈΠ΅ Π΄ΠΆΠΎΡƒΠ»Π΅Π²Π° Π½Π°Π³Ρ€Π΅Π²Π°?

    1. ΠžΠΌΠΈΡ‡Π΅ΡΠΊΠΈΠΉ Π½Π°Π³Ρ€Π΅Π²
    2. РСзистивный Π½Π°Π³Ρ€Π΅Π²
    3. Оба a и b
    4. Ни ΠΎΠ΄ΠΈΠ½ ΠΈΠ· этих

    ΠžΡ‚Π²Π΅Ρ‚: Π’Π°Ρ€ΠΈΠ°Π½Ρ‚ c

    ПояснСниС: Π”Ρ€ΡƒΠ³ΠΎΠ΅ Π½Π°Π·Π²Π°Π½ΠΈΠ΅ Π΄ΠΆΠΎΡƒΠ»Π΅Π²Π° Π½Π°Π³Ρ€Π΅Π²Π° – омичСский Π½Π°Π³Ρ€Π΅Π² ΠΈΠ»ΠΈ рСзистивный Π½Π°Π³Ρ€Π΅Π².

    Q 4. КакоС ΠΈΠ· ΡΠ»Π΅Π΄ΡƒΡŽΡ‰ΠΈΡ… элСктричСских устройств нагрСваСтся ΠΏΡ€ΠΈ Ρ€Π°Π±ΠΎΡ‚Π΅?

    1. Π­Π»Π΅ΠΊΡ‚Ρ€ΠΎΠ΄Π²ΠΈΠ³Π°Ρ‚Π΅Π»ΡŒ
    2. Π­Π»Π΅ΠΊΡ‚Ρ€ΠΎΠ³Π΅Π½Π΅Ρ€Π°Ρ‚ΠΎΡ€
    3. Π“Π΅Π½Π΅Ρ€Π°Ρ‚ΠΎΡ€ ΠΏΠ΅Ρ€Π΅ΠΌΠ΅Π½Π½ΠΎΠ³ΠΎ Ρ‚ΠΎΠΊΠ°
    4. ВсС пСрСчислСнныС

    ΠžΡ‚Π²Π΅Ρ‚: Π’Π°Ρ€ΠΈΠ°Π½Ρ‚ d

    ПояснСниС: всС Π³Π΅Π½Π΅Ρ€Π°Ρ‚ΠΎΡ€, Π³Π΅Π½Π΅Ρ€Π°Ρ‚ΠΎΡ€ ΠΈ ΡΠ»Π΅ΠΊΡ‚Ρ€ΠΎΠ΄Π²ΠΈΠ³Π°Ρ‚Π΅Π»ΡŒ 9003.

    Q 5. Π§Ρ‚ΠΎ Ρ‚Π°ΠΊΠΎΠ΅ Π΅Π΄ΠΈΠ½ΠΈΡ†Π° измСрСния элСктричСского заряда Π² систСмС БИ?

    1. КандСла
    2. Π€Π°Ρ€Π°Π΄
    3. ΠšΡƒΠ»ΠΎΠ½
    4. Ом

    ΠžΡ‚Π²Π΅Ρ‚: Π’Π°Ρ€ΠΈΠ°Π½Ρ‚ c

    ОбъяснСниС: Π΅Π΄ΠΈΠ½ΠΈΡ†Π° элСктричСского заряда БИ

    ΠœΡ‹ обСспСчим вас ΠΏΠΎΡˆΠ°Π³ΠΎΠ²Ρ‹ΠΌΠΈ Ρ€Π΅ΡˆΠ΅Π½ΠΈΡΠΌΠΈ ΠΌΠΈΠ»Π»ΠΈΠΎΠ½ΠΎΠ² Π·Π°Π΄Π°Ρ‡ ΠΈΠ· ΡƒΡ‡Π΅Π±Π½ΠΈΠΊΠΎΠ², экспСртами Π² любой области Π½Π°Π³ΠΎΡ‚ΠΎΠ²Π΅ 24 часа Π² сутки, 7 Π΄Π½Π΅ΠΉ Π² нСдСлю, ΠΊΠΎΠ³Π΄Π° Π²Ρ‹ Π·Π°ΠΏΡƒΡ‚Π°Π»ΠΈΡΡŒ, ΠΈ ΠΌΠ½ΠΎΠ³ΠΎΠ΅ Π΄Ρ€ΡƒΠ³ΠΎΠ΅.

    ΠžΠ·Π½Π°ΠΊΠΎΠΌΡŒΡ‚Π΅ΡΡŒ с ΠΏΡ€ΠΈΠΌΠ΅Ρ€ΠΎΠΌ Ρ€Π΅ΡˆΠ΅Π½ΠΈΡ вопросов ΠΈ ΠΎΡ‚Π²Π΅Ρ‚ΠΎΠ² для граТданского ΡΡ‚Ρ€ΠΎΠΈΡ‚Π΅Π»ΡŒΡΡ‚Π²Π° здСсь!

    *ВрСмя ΠΎΡ‚Π²Π΅Ρ‚Π° зависит ΠΎΡ‚ Ρ‚Π΅ΠΌΡ‹ ΠΈ слоТности вопроса. Π‘Ρ€Π΅Π΄Π½Π΅Π΅ врСмя ΠΎΡ‚ΠΊΠ»ΠΈΠΊΠ° составляСт 34 ΠΌΠΈΠ½ΡƒΡ‚Ρ‹ для ΠΏΠ»Π°Ρ‚Π½Ρ‹Ρ… подписчиков ΠΈ ΠΌΠΎΠΆΠ΅Ρ‚ Π±Ρ‹Ρ‚ΡŒ большС для Ρ€Π΅ΠΊΠ»Π°ΠΌΠ½Ρ‹Ρ… ΠΏΡ€Π΅Π΄Π»ΠΎΠΆΠ΅Π½ΠΈΠΉ.

    ΠŸΡ€ΠΎΡΡ‚Ρ‹Π΅ схСмы, Ρ‚ΠΎΠΊ, напряТСниС ΠΈ сопротивлСниС – Π“ΠΎΠ»ΡƒΠ±ΠΈΠ½Ρ‹ΠΉ насСст

    Π’ Ρ€Π°ΠΌΠΊΠ°Ρ… собСсСдования мСня Π½Π΅Π΄Π°Π²Π½ΠΎ попросили провСсти дСмонстрационный ΡƒΡ€ΠΎΠΊ ΠΏΠΎ Ρ„ΠΈΠ·ΠΈΠΊΠ΅. Класс Ρ‚ΠΎΠ»ΡŒΠΊΠΎ Π½Π°Ρ‡Π°Π» свой Ρ€Π°Π·Π΄Π΅Π» ΠΏΠΎ элСктроникС, поэтому я Ρ€Π΅ΡˆΠΈΠ» ΠΏΡ€Π΅ΠΏΠΎΠ΄Π°Π²Π°Ρ‚ΡŒ с ΠΏΠΎΠΌΠΎΡ‰ΡŒΡŽ простой ΠΈΠ½Ρ‚Π΅Ρ€Π°ΠΊΡ‚ΠΈΠ²Π½ΠΎΠΉ ΠΎΠ½Π»Π°ΠΉΠ½-схСмы, Ρ‡Ρ‚ΠΎΠ±Ρ‹ Π΄Π°Ρ‚ΡŒ ΠΊΠΎΠ½Ρ†Π΅ΠΏΡ‚ΡƒΠ°Π»ΡŒΠ½ΡƒΡŽ основу для Ρ‚ΠΎΠΊΠ°, напряТСния ΠΈ сопротивлСния. По ΠΌΠΎΠ΅ΠΌΡƒ ΠΎΠΏΡ‹Ρ‚Ρƒ, эти Ρ‚Π΅ΠΌΡ‹ часто сразу ΠΏΡ€Π΅Π΄ΡΡ‚Π°Π²Π»ΡΡŽΡ‚ΡΡ Π² абстрактной Ρ„ΠΎΡ€ΠΌΠ΅, ΠΊΠΎΠ³Π΄Π° студСнты Ρ€ΠΈΡΡƒΡŽΡ‚ ΠΏΡ€ΠΈΠ½Ρ†ΠΈΠΏΠΈΠ°Π»ΡŒΠ½Ρ‹Π΅ схСмы ΠΈ ΠΏΡ€ΠΈΠ΄ΡƒΠΌΡ‹Π²Π°ΡŽΡ‚ Ρ€Π΅ΡˆΠ΅Π½ΠΈΡ ΡƒΡ€Π°Π²Π½Π΅Π½ΠΈΠΉ, Π½Π΅ получая ΠΈΠ½Ρ‚ΡƒΠΈΡ‚ΠΈΠ²Π½ΠΎΠ³ΠΎ прСдставлСния ΠΎ Ρ‚ΠΎΠΌ, Ρ‡Ρ‚ΠΎ это Π·Π° ΠΊΠΎΠ½Ρ†Π΅ΠΏΡ†ΠΈΠΈ. Π­Ρ‚ΠΎ усугубляСтся Ρ‚Π΅ΠΌ Ρ„Π°ΠΊΡ‚ΠΎΠΌ, Ρ‡Ρ‚ΠΎ, хотя ΠΌΡ‹ ΠΌΠΎΠΆΠ΅ΠΌ Π½Π°Π±Π»ΡŽΠ΄Π°Ρ‚ΡŒ ΠΊΠΎΠ½Π΅Ρ‡Π½Ρ‹ΠΉ Ρ€Π΅Π·ΡƒΠ»ΡŒΡ‚Π°Ρ‚ элСктричСской систСмы, ΠΌΡ‹ Π½Π΅ ΠΌΠΎΠΆΠ΅ΠΌ Π²ΠΈΠ΄Π΅Ρ‚ΡŒ Π΅Π΅ Π²Π½ΡƒΡ‚Ρ€Π΅Π½Π½ΡŽΡŽ Ρ€Π°Π±ΠΎΡ‚Ρƒ.

    Π•ΡΡ‚ΡŒ Π΄Π²Π΅ Π°Π½Π°Π»ΠΎΠ³ΠΈΠΈ, ΠΊΠΎΡ‚ΠΎΡ€Ρ‹Π΅ ΠΌΠΎΠΆΠ½ΠΎ ΠΈΡΠΏΠΎΠ»ΡŒΠ·ΠΎΠ²Π°Ρ‚ΡŒ для элСктричСской Ρ†Π΅ΠΏΠΈ. Один — водная систСма, Π΄Ρ€ΡƒΠ³ΠΎΠΉ — амСриканскиС Π³ΠΎΡ€ΠΊΠΈ. Π― ΠΏΡ€ΠΎΠΉΠ΄ΡƒΡΡŒ ΠΏΠΎ ΠΎΠ±ΠΎΠΈΠΌ здСсь. Для дСмонстрационного ΡƒΡ€ΠΎΠΊΠ° я использовал амСриканскиС Π³ΠΎΡ€ΠΊΠΈ. Π¨ΠΊΠΎΠ»Π° Π½Π°Ρ…ΠΎΠ΄ΠΈΠ»Π°ΡΡŒ Π² Нью-Π™ΠΎΡ€ΠΊΠ΅, ΠΈ я Π΄ΡƒΠΌΠ°Π», Ρ‡Ρ‚ΠΎ Ρƒ ΡƒΡ‡Π΅Π½ΠΈΠΊΠΎΠ², ΠΏΠΎ большСй части, Π±ΡƒΠ΄Π΅Ρ‚ ΠΎΠΏΡ‹Ρ‚ катания Π½Π° амСриканских Π³ΠΎΡ€ΠΊΠ°Ρ…. Π’ Нью-ДТСрси Π΅ΡΡ‚ΡŒ Π²ΠΎΠ΄ΠΎΠΏΠ°Π΄ ΠŸΠ°Ρ‚Π΅Ρ€ΡΠΎΠ½, Π½ΠΎ Π±ΠΎΠ»ΡŒΡˆΠΈΠ½ΡΡ‚Π²ΠΎ людСй, с ΠΊΠΎΡ‚ΠΎΡ€Ρ‹ΠΌΠΈ я Ρ€Π°Π·Π³ΠΎΠ²Π°Ρ€ΠΈΠ²Π°Π» Π² этом Ρ€Π°ΠΉΠΎΠ½Π΅, Π½Π΅ Π·Π½Π°Π»ΠΈ ΠΎΠ± этих Π²ΠΎΠ΄ΠΎΠΏΠ°Π΄Π°Ρ…. Π― ΡƒΠ·Π½Π°Π» ΠΎΠ± этом Π²ΠΎ врСмя просмотра Ρ„ΠΈΠ»ΡŒΠΌΠ° ΠŸΠ°Ρ‚Π΅Ρ€ΡΠΎΠ½. Если Π±Ρ‹ ΡƒΡ€ΠΎΠΊ проводился Π² Π‘ΡƒΡ„Ρ„Π°Π»ΠΎ, Π³Π΄Π΅ я ΠΆΠΈΠ²Ρƒ, я Π±Ρ‹ использовал ΠΏΡ€ΠΈΠΌΠ΅Ρ€ с систСмой водоснабТСния, ΠΏΠΎΡΠΊΠΎΠ»ΡŒΠΊΡƒ Ниагарский Π²ΠΎΠ΄ΠΎΠΏΠ°Π΄ являСтся ΠΎΡ‡Π΅Π½ΡŒ Π²Π°ΠΆΠ½Ρ‹ΠΌ элСмСнтом мСстной Π³Π΅ΠΎΠ³Ρ€Π°Ρ„ΠΈΠΈ.

    Π’ΠΎΠΊ опрСдСляСт ΠΏΠΎΡ‚ΠΎΠΊ элСктричСства Π² Ρ†Π΅ΠΏΠΈ Π² Π½Π°ΠΏΡ€Π°Π²Π»Π΅Π½ΠΈΠΈ ΠΏΠΎΠ»ΠΎΠΆΠΈΡ‚Π΅Π»ΡŒΠ½ΠΎΠ³ΠΎ заряда. На самом Π΄Π΅Π»Π΅ это ΠΏΠΎΡ‚ΠΎΠΊ свободных ΠΎΡ‚Ρ€ΠΈΡ†Π°Ρ‚Π΅Π»ΡŒΠ½ΠΎ заряТСнных элСктронов, ΠΊΠΎΡ‚ΠΎΡ€Ρ‹Π΅ ΡΠΎΠ·Π΄Π°ΡŽΡ‚ Ρ‚ΠΎΠΊ, Π½ΠΎ это соглашСниС Π±Ρ‹Π»ΠΎ ΠΎΠΏΡ€Π΅Π΄Π΅Π»Π΅Π½ΠΎ Π΄ΠΎ Ρ‚ΠΎΠ³ΠΎ, ΠΊΠ°ΠΊ Π² 20 Π²Π΅ΠΊΠ΅ Π±Ρ‹Π»Π° раскрыта ΠΏΡ€ΠΈΡ€ΠΎΠ΄Π° Π°Ρ‚ΠΎΠΌΠ°. ЭлСктричСский заряд сохраняСтся, Ρ‚ΠΎ Π΅ΡΡ‚ΡŒ Π΅Π³ΠΎ нСльзя ΡΠΎΠ·Π΄Π°Ρ‚ΡŒ ΠΈΠ»ΠΈ Ρ€Π°Π·Ρ€ΡƒΡˆΠΈΡ‚ΡŒ. Одна Π΅Π΄ΠΈΠ½ΠΈΡ†Π° заряда β€” это ΠΊΡƒΠ»ΠΎΠ½ (Кл), Π° ΠΏΠΎΡ‚ΠΎΠΊ со ΡΠΊΠΎΡ€ΠΎΡΡ‚ΡŒΡŽ 1 Кл/с называСтся Π°ΠΌΠΏΠ΅Ρ€. Когда я учился Π² ΡΡ‚Π°Ρ€ΡˆΠΈΡ… классах, ΡƒΡ‡Π΅Π½ΠΈΠΊΠΈ Ρ…Π²Π°ΡΡ‚Π°Π»ΠΈΡΡŒ, сколько Π°ΠΌΠΏΠ΅Ρ€ Ρƒ ΠΈΡ… стСрСосистСм, Ρ‡Ρ‚ΠΎ бСсконСчно Ρ€Π°Π΄ΠΎΠ²Π°Π»ΠΎ Π½Π°ΡˆΠΈΡ… Ρ€ΠΎΠ΄ΠΈΡ‚Π΅Π»Π΅ΠΉ.

    Если ΠΏΠΎΡ‚ΠΎΠΊ ΠΈΠΌΠ΅Π΅Ρ‚ ΡΠΊΠΎΡ€ΠΎΡΡ‚ΡŒ 10 Π³Π°Π»Π»ΠΎΠ½ΠΎΠ² Π² сСкунду, ΠΌΡ‹ ΠΌΠΎΠΆΠ΅ΠΌ Π½Π°Π·Π²Π°Ρ‚ΡŒ это Ρ‚Π΅Ρ‡Π΅Π½ΠΈΠ΅ΠΌ. Если Π²Ρ‹ смотритС Π½Π° амСриканскиС Π³ΠΎΡ€ΠΊΠΈ ΠΈ Π½Π°Π±Π»ΡŽΠ΄Π°Π΅Ρ‚Π΅, ΠΊΠ°ΠΊ 10 Π°Π²Ρ‚ΠΎΠΌΠΎΠ±ΠΈΠ»Π΅ΠΉ ΠΏΡ€ΠΎΠ΅Π·ΠΆΠ°ΡŽΡ‚ Ρ‚ΠΎΡ‡ΠΊΡƒ Π·Π° ΠΎΠ΄Π½Ρƒ сСкунду, Ρ‚ΠΎ 10 Π°Π²Ρ‚ΠΎΠΌΠΎΠ±ΠΈΠ»Π΅ΠΉ Π² сСкунду β€” это Π΅Π΅ Ρ‚ΠΎΠΊ. Π’ΠΎ ΠΆΠ΅ самоС Π²Π΅Ρ€Π½ΠΎ ΠΈ для Ρ†Π΅ΠΏΠΈ, ΠΏΠΎΡ‚ΠΎΠΊ 10 Π΅Π΄ΠΈΠ½ΠΈΡ† заряда Π² ΠΏΡ€ΠΎΠ²ΠΎΠ΄Π΅ составляСт 10 Кл/с ΠΈΠ»ΠΈ 10 Π°ΠΌΠΏΠ΅Ρ€. ЦСпь Π΄ΠΎΠ»ΠΆΠ½Π° Π·Π°ΠΌΠΊΠ½ΡƒΡ‚ΡŒ ΠΏΠ΅Ρ‚Π»ΡŽ, Ρ‡Ρ‚ΠΎΠ±Ρ‹ Ρ‚ΠΎΠΊ ΠΏΡ€ΠΎΡ‚Π΅ΠΊΠ°Π». ΠŸΠ΅Ρ€Π΅ΠΊΠ»ΡŽΡ‡Π°Ρ‚Π΅Π»ΡŒ Π² ΠΏΠΎΠ»ΠΎΠΆΠ΅Π½ΠΈΠΈ Β«Π²ΠΊΠ»ΡŽΡ‡Π΅Π½ΠΎΒ» Π·Π°ΠΌΡ‹ΠΊΠ°Π΅Ρ‚ ΠΏΠ΅Ρ‚Π»ΡŽ ΠΈ позволяСт Ρ‚ΠΎΠΊΡƒ Ρ‚Π΅Ρ‡ΡŒ Ρ‡Π΅Ρ€Π΅Π· систСму. Π’Ρ‹ΠΊΠ»ΡŽΡ‡Π΅Π½Π½ΠΎΠ΅ ΠΏΠΎΠ»ΠΎΠΆΠ΅Π½ΠΈΠ΅ Ρ€Π°Π·Ρ€Ρ‹Π²Π°Π΅Ρ‚ ΠΏΠ΅Ρ‚Π»ΡŽ. Однако для создания Ρ‚ΠΎΠΊΠ° трСбуСтся Π½Π΅Ρ‡Ρ‚ΠΎ большСС, Ρ‡Π΅ΠΌ ΠΏΠ΅Ρ€Π΅ΠΊΠ»ΡŽΡ‡Π°Ρ‚Π΅Π»ΡŒ, ΠΈ ΠΈΠΌΠ΅Π½Π½ΠΎ здСсь появляСтся напряТСниС.

    Если ΠΎΠ±ΡŠΠ΅ΠΊΡ‚ находится Π½Π° Π·Π΅ΠΌΠ»Π΅, Π΅Π³ΠΎ ΠΏΠΎΡ‚Π΅Π½Ρ†ΠΈΠ°Π»ΡŒΠ½Π°Ρ энСргия Ρ€Π°Π²Π½Π° Π½ΡƒΠ»ΡŽ. Если ΠΌΡ‹ ΠΏΠΎΠ΄Π½ΠΈΠΌΠ΅ΠΌ ΠΎΠ±ΡŠΠ΅ΠΊΡ‚ Π½Π°Π΄ Π·Π΅ΠΌΠ»Π΅ΠΉ, ΠΎΠ½ ΠΏΠΎΠ»ΡƒΡ‡ΠΈΡ‚ ΠΏΠΎΡ‚Π΅Π½Ρ†ΠΈΠ°Π»ΡŒΠ½ΡƒΡŽ ΡΠ½Π΅Ρ€Π³ΠΈΡŽ. Π­Ρ‚Π° ΠΏΠΎΡ‚Π΅Π½Ρ†ΠΈΠ°Π»ΡŒΠ½Π°Ρ энСргия прСобразуСтся Π² ΠΊΠΈΠ½Π΅Ρ‚ΠΈΡ‡Π΅ΡΠΊΡƒΡŽ ΡΠ½Π΅Ρ€Π³ΠΈΡŽ, Ссли ΠΌΡ‹ отпускаСм ΠΎΠ±ΡŠΠ΅ΠΊΡ‚. Π’Π΅Ρ€Π½ΠΈΡ‚Π΅ΡΡŒ ΠΊ Π°Π½Π°Π»ΠΎΠ³ΠΈΠΈ с амСриканскими Π³ΠΎΡ€ΠΊΠ°ΠΌΠΈ. Какой ΠΏΠΎΡ‚Π΅Π½Ρ†ΠΈΠ°Π»ΡŒΠ½ΠΎΠΉ энСргиСй ΠΎΠ±Π»Π°Π΄Π°ΡŽΡ‚ Π°Π²Ρ‚ΠΎΠΌΠΎΠ±ΠΈΠ»ΠΈ, ΠΊΠΎΠ³Π΄Π° ΠΎΠ½ΠΈ находятся Π½Π° ΠΎΠ΄Π½ΠΎΠΌ ΡƒΡ€ΠΎΠ²Π½Π΅ с Π·Π΅ΠΌΠ»Π΅ΠΉ? ΠΡƒΠ»ΡŒ. ΠŸΠΎΠ΄ΡΡ‚Π°Π²ΠΊΠ° добавляСт ΠΏΠΎΡ‚Π΅Π½Ρ†ΠΈΠ°Π»ΡŒΠ½ΡƒΡŽ ΡΠ½Π΅Ρ€Π³ΠΈΡŽ, поднимая Π°Π²Ρ‚ΠΎΠΌΠΎΠ±ΠΈΠ»ΠΈ Π½Π° Ρ…ΠΎΠ»ΠΌ. Π¦ΠΈΠΊΠ»ΠΎΠ½ Кони-АйлСнда ΠΈΠΌΠ΅Π΅Ρ‚ высоту 85 Ρ„ΡƒΡ‚ΠΎΠ², Ρ‚ΠΎΠ³Π΄Π° ΠΊΠ°ΠΊ соврСмСнныС подставки ΠΌΠΎΠ³ΡƒΡ‚ Π΄ΠΎΡΡ‚ΠΈΠ³Π°Ρ‚ΡŒ 200-300 Ρ„ΡƒΡ‚ΠΎΠ². ΠŸΠΎΡ‚Π΅Π½Ρ†ΠΈΠ°Π»ΡŒΠ½Π°Ρ энСргия прСобразуСтся Π² ΠΊΠΈΠ½Π΅Ρ‚ΠΈΡ‡Π΅ΡΠΊΡƒΡŽ ΡΠ½Π΅Ρ€Π³ΠΈΡŽ, ΠΊΠΎΠ³Π΄Π° Π²Ρ‹ достигаСтС Π²Π΅Ρ€ΡˆΠΈΠ½Ρ‹ ΠΈ Π½Π°Ρ‡ΠΈΠ½Π°Π΅Ρ‚Π΅ ΠΏΠ°Π΄Π°Ρ‚ΡŒ. Π‘Π°Ρ‚Π°Ρ€Π΅ΠΈ Π΄Π΅Π»Π°ΡŽΡ‚ Ρ‚ΠΎ ΠΆΠ΅ самоС, добавляя ΠΏΠΎΡ‚Π΅Π½Ρ†ΠΈΠ°Π» Π² Ρ†Π΅ΠΏΡŒ. Π­Ρ‚ΠΎΡ‚ ΠΏΠΎΡ‚Π΅Π½Ρ†ΠΈΠ°Π» измСряСтся Π² Π²ΠΎΠ»ΡŒΡ‚Π°Ρ….

    ΠšΠΎΠΌΠ΅Ρ‚Π° с ΠΎΡ‚ΠΊΡ€Ρ‹Ρ‚ΠΊΠΈ 1950-Ρ… Π³ΠΎΠ΄ΠΎΠ². ΠŸΠ΅Ρ€Π²Ρ‹ΠΉ Ρ…ΠΎΠ»ΠΌ высотой 96 Ρ„ΡƒΡ‚ΠΎΠ² обСспСчил ΠΏΠΎΡ‚Π΅Π½Ρ†ΠΈΠ°Π»ΡŒΠ½ΡƒΡŽ ΡΠ½Π΅Ρ€Π³ΠΈΡŽ для ΠΏΠΎΠ΅Π·Π΄ΠΊΠΈ. По ΠΌΠ΅Ρ€Π΅ ΡƒΠΌΠ΅Π½ΡŒΡˆΠ΅Π½ΠΈΡ высоты Π² ΠΏΠ΅Ρ‚Π»Π΅ ΠΏΠΎΡ‚Π΅Π½Ρ†ΠΈΠ°Π»ΡŒΠ½Π°Ρ энСргия ΡƒΠΌΠ΅Π½ΡŒΡˆΠ°Π΅Ρ‚ΡΡ β€” Ρ‚Π°ΠΊ ΠΆΠ΅, ΠΊΠ°ΠΊ ΡƒΠΌΠ΅Π½ΡŒΡˆΠ°Π΅Ρ‚ΡΡ напряТСниС Π² ΠΏΠ΅Ρ‚Π»Π΅ Ρ†Π΅ΠΏΠΈ.

    Π’ Π°Π½Π°Π»ΠΎΠ³ΠΈΠΈ с Π²ΠΎΠ΄ΠΎΠΉ ΠΏΡ€Π΅Π΄ΡΡ‚Π°Π²ΡŒΡ‚Π΅ Ρ€ΠΎΠ²Π½Ρ‹ΠΉ ΠΊΠ°Π½Π°Π». Π’ΠΎΠΊ Π½Π΅ Ρ‚Π΅Ρ‡Π΅Ρ‚, ΠΈ фактичСски это ΠΏΡ€ΠΈΠ²ΠΎΠ΄ΠΈΡ‚ ΠΊ Π·Π°ΡΡ‚ΠΎΡŽ ΠΊΠ°Π½Π°Π»ΠΎΠ² ΠΈ опасности для Π·Π΄ΠΎΡ€ΠΎΠ²ΡŒΡ. По этой ΠΏΡ€ΠΈΡ‡ΠΈΠ½Π΅ ΠΊΠ°Π½Π°Π»Ρ‹ АмстСрдама ΠΏΡ€ΠΎΠΌΡ‹Π²Π°ΡŽΡ‚ ΠΊΠ°ΠΆΠ΄ΠΎΠ΅ ΡƒΡ‚Ρ€ΠΎ. По этой ΠΆΠ΅ ΠΏΡ€ΠΈΡ‡ΠΈΠ½Π΅ сСгмСнт Π‘ΡƒΡ„Ρ„Π°Π»ΠΎ ΠΊΠ°Π½Π°Π»Π° Π­Ρ€ΠΈ Π±Ρ‹Π» засыпан Π² Ρ‚Π΅Ρ‡Π΅Π½ΠΈΠ΅ 19-Π³ΠΎ Π²Π΅ΠΊΠ°.20-Π΅ Π³ΠΎΠ΄Ρ‹. ИмСнно Π½Π° этом ΠΎΡ‚Ρ€Π΅Π·ΠΊΠ΅ построСн И-190. Π§Ρ‚ΠΎ происходит, ΠΊΠΎΠ³Π΄Π° Π²Ρ‹ добавляСтС Ρ€Π°Π·Π½ΠΈΡ†Ρƒ Π² высотС? ВспомнитС Ниагарский Π²ΠΎΠ΄ΠΎΠΏΠ°Π΄. Он добавляСт Ρ‚Π΅ΠΊΡƒΡ‰ΡƒΡŽ ΠΈ ΠΏΠΎΡ‚Π΅Π½Ρ†ΠΈΠ°Π»ΡŒΠ½ΡƒΡŽ ΡΠ½Π΅Ρ€Π³ΠΈΡŽ, которая ΠΈΡΠΏΠΎΠ»ΡŒΠ·ΡƒΠ΅Ρ‚ΡΡ для производства гидроэлСктроэнСргии. Π’ΠΎΠ΄Π° Π² количСствС 748 000 Π³Π°Π»Π»ΠΎΠ½ΠΎΠ² Π² сСкунду ΠΏΠ°Π΄Π°Π΅Ρ‚ с высоты 180 Ρ„ΡƒΡ‚ΠΎΠ² Π½Π° 25 Ρ‚ΡƒΡ€Π±ΠΈΠ½, производящих 2,6 ΠΌΠ΅Π³Π°Π²Π°Ρ‚Ρ‚Π° энСргии.

    ГидроэлСктростанция Π ΠΎΠ±Π΅Ρ€Ρ‚Π° МозСса. Π’ΠΎΠ΄Π° отводится ΠΏΠ΅Ρ€Π΅Π΄ Π²ΠΎΠ΄ΠΎΠΏΠ°Π΄ΠΎΠΌ, ΠΈ Π΅Π΅ ΠΏΠΎΡ‚Π΅Π½Ρ†ΠΈΠ°Π»ΡŒΠ½Π°Ρ энСргия прСобразуСтся Π² ΠΊΠΈΠ½Π΅Ρ‚ΠΈΡ‡Π΅ΡΠΊΡƒΡŽ ΡΠ½Π΅Ρ€Π³ΠΈΡŽ, Π° Π·Π°Ρ‚Π΅ΠΌ Π² ΡΠ»Π΅ΠΊΡ‚Ρ€ΠΎΡΠ½Π΅Ρ€Π³ΠΈΡŽ. ΠšΡ€Π΅Π΄ΠΈΡ‚: Π“Ρ€Π΅Π³ΠΎΡ€ΠΈ Пьяновски

    Π›ΠΈΠ½ΠΈΠΈ элСктростанции ΠΌΠΎΠ³ΡƒΡ‚ ΠΈΠΌΠ΅Ρ‚ΡŒ напряТСниС Π² сотни тысяч. Врансформаторы ΠΏΠΎΠ½ΠΈΠΆΠ°ΡŽΡ‚ Π΅Π³ΠΎ Π΄ΠΎ 120 Π²ΠΎΠ»ΡŒΡ‚ ΠΏΠ΅Ρ€Π΅Π΄ Π²Ρ…ΠΎΠ΄ΠΎΠΌ Π² Π΄ΠΎΠΌ. НапряТСниС Ρ‚Π°ΠΊΠΆΠ΅ ΠΌΠΎΠΆΠ½ΠΎ Ρ€Π°ΡΡΠΌΠ°Ρ‚Ρ€ΠΈΠ²Π°Ρ‚ΡŒ ΠΊΠ°ΠΊ Π΄Π°Π²Π»Π΅Π½ΠΈΠ΅. ΠŸΠΎΠ΄ΡƒΠΌΠ°ΠΉΡ‚Π΅ ΠΎ ΠΌΠΎΠΉΠΊΠ΅ высокого давлСния. Π‘ΠΎΠ»Π΅Π΅ высокоС Π΄Π°Π²Π»Π΅Π½ΠΈΠ΅ ΠΌΠΎΠΆΠ΅Ρ‚ Π΄ΠΎΡΡ‚Π°Π²ΠΈΡ‚ΡŒ Π²ΠΎΠ΄Ρƒ дальшС. Π‘ΠΎΠ»Π΅Π΅ высокоС напряТСниС ΠΌΠΎΠΆΠ΅Ρ‚ ΠΏΠΎΡΠ»Π°Ρ‚ΡŒ искру дольшС. Π’Π°ΠΊΠΈΠΌ ΠΎΠ±Ρ€Π°Π·ΠΎΠΌ, хотя напряТСниС ΠΈ Ρ‚ΠΎΠΊ ΠΏΡ€ΠΎΠΏΠΎΡ€Ρ†ΠΈΠΎΠ½Π°Π»ΡŒΠ½Ρ‹ Π΄Ρ€ΡƒΠ³ Π΄Ρ€ΡƒΠ³Ρƒ, это Π½Π΅ ΠΎΠ΄Π½ΠΎ ΠΈ Ρ‚ΠΎ ΠΆΠ΅. Π’Π°ΠΌ Π½ΡƒΠΆΠ½ΠΎ напряТСниС, Ρ‡Ρ‚ΠΎΠ±Ρ‹ Π·Π°ΠΏΡƒΡΡ‚ΠΈΡ‚ΡŒ Ρ‚ΠΎΠΊ.

    ПослСдняя Ρ‡Π°ΡΡ‚ΡŒ Π³ΠΎΠ»ΠΎΠ²ΠΎΠ»ΠΎΠΌΠΊΠΈ β€” сопротивлСниС. Π­Ρ‚ΠΎ ΠΏΠΎΡ…ΠΎΠΆΠ΅ Π½Π° Ρ‚Ρ€Π΅Π½ΠΈΠ΅ Π½Π° амСриканских Π³ΠΎΡ€ΠΊΠ°Ρ…. Π‘Π΅Π· трСния амСриканскиС Π³ΠΎΡ€ΠΊΠΈ Π½ΠΈΠΊΠΎΠ³Π΄Π° Π±Ρ‹ Π½Π΅ ΠΎΡΡ‚Π°Π½ΠΎΠ²ΠΈΠ»ΠΈΡΡŒ, Π° двигались Π±Ρ‹ ΠΏΠΎ Π½Π΅ΠΏΡ€Π΅Ρ€Ρ‹Π²Π½ΠΎΠΌΡƒ Ρ†ΠΈΠΊΠ»Ρƒ. Π’Ρ€Π΅Π½ΠΈΠ΅ ΠΌΠ΅ΠΆΠ΄Ρƒ Π²Π°Π³ΠΎΠ½Π°ΠΌΠΈ ΠΈ Ρ€Π΅Π»ΡŒΡΠ°ΠΌΠΈ ΠΏΡ€Π΅ΠΎΠ±Ρ€Π°Π·ΡƒΠ΅Ρ‚ ΠΊΠΈΠ½Π΅Ρ‚ΠΈΡ‡Π΅ΡΠΊΡƒΡŽ ΡΠ½Π΅Ρ€Π³ΠΈΡŽ Π² Ρ‚Π΅ΠΏΠ»ΠΎ ΠΈ рассСиваСтся Π² ΠΎΠΊΡ€ΡƒΠΆΠ°ΡŽΡ‰Π΅ΠΌ Π²ΠΎΠ·Π΄ΡƒΡ…Π΅. Π‘Π»Π΅Π΄ΠΎΠ²Π°Ρ‚Π΅Π»ΡŒΠ½ΠΎ, ΠΏΠ°Ρ€ΠΎΠ²ΠΎΠ· Π΄ΠΎΠ»ΠΆΠ΅Π½ снова ΠΏΠΎΠ΄Ρ‚ΠΎΠ»ΠΊΠ½ΡƒΡ‚ΡŒ ΠΊΠ°Π±ΠΎΡ‚Π°ΠΆΠ½ΠΎΠ΅ судно Π² Π³ΠΎΡ€Ρƒ, Ρ‡Ρ‚ΠΎΠ±Ρ‹ Π½Π°Ρ‡Π°Ρ‚ΡŒ Π΅Ρ‰Π΅ ΠΎΠ΄ΠΈΠ½ рСйс ΠΏΠΎ ΠΊΠΎΠ»ΡŒΡ†Ρƒ. Π‘ΠΎΠΏΡ€ΠΎΡ‚ΠΈΠ²Π»Π΅Π½ΠΈΠ΅ Π² Ρ†Π΅ΠΏΠΈ Π΄Π΅Π»Π°Π΅Ρ‚ Ρ‚ΠΎ ΠΆΠ΅ самоС. ЭнСргия Π² Ρ†Π΅ΠΏΠΈ прСобразуСтся сопротивлСниСм ΠΏΡ€ΠΎΠ²ΠΎΠ΄Π° ΠΈ рассСиваСтся Π² Π²ΠΈΠ΄Π΅ Ρ‚Π΅ΠΏΠ»Π°. Π­Ρ‚ΠΎ ΠΏΡ€ΠΈΠ²ΠΎΠ΄ΠΈΡ‚ ΠΊ падСнию напряТСния ΠΏΠΎ ΠΌΠ΅Ρ€Π΅ прохоТдСния Ρ‚ΠΎΠΊΠ° Π² ΠΏΠ΅Ρ‚Π»Π΅. БатарСя слуТит Ρ‚ΠΎΠΉ ΠΆΠ΅ Ρ†Π΅Π»ΠΈ, Ρ‡Ρ‚ΠΎ ΠΈ Π³ΠΎΡ€ΠΊΠ° Π² ΠΊΠ°Π±ΠΎΡ‚Π°ΠΆΠ½ΠΎΠΌ суднС. Он добавляСт напряТСниС ΠΈΠ»ΠΈ ΠΏΠΎΡ‚Π΅Π½Ρ†ΠΈΠ°Π» для пСрСзапуска Ρ‚ΠΎΠΊΠ° Π² ΠΏΠ΅Ρ‚Π»Π΅.

    Π‘Π²Π΅Ρ€Ρ…ΠΏΡ€ΠΎΠ²ΠΎΠ΄ΠΈΠΌΠΎΡΡ‚ΡŒ прСдставляСт собой состояниС Π½ΡƒΠ»Π΅Π²ΠΎΠ³ΠΎ сопротивлСния. Для этого Π½ΡƒΠΆΠ½Π° ΠΎΡ‡Π΅Π½ΡŒ холодная Ρ‚Π΅ΠΌΠΏΠ΅Ρ€Π°Ρ‚ΡƒΡ€Π°. Π’ 1980-Ρ… Π³ΠΎΠ΄Π°Ρ… Π±Ρ‹Π» ΠΎΡ‚ΠΊΡ€Ρ‹Ρ‚ кСрамичСский ΠΌΠ°Ρ‚Π΅Ρ€ΠΈΠ°Π», ΠΊΠΎΡ‚ΠΎΡ€Ρ‹ΠΉ поднял ΠΈΠ·Π²Π΅ΡΡ‚Π½ΡƒΡŽ Ρ‚Π΅ΠΌΠΏΠ΅Ρ€Π°Ρ‚ΡƒΡ€Ρƒ свСрхпроводимости с 30 К Π΄ΠΎ 92 К. БрСдства массовой ΠΈΠ½Ρ„ΠΎΡ€ΠΌΠ°Ρ†ΠΈΠΈ Ρ‚ΠΎΠ³ΠΎ Π²Ρ€Π΅ΠΌΠ΅Π½ΠΈ прСдставили это ΠΊΠ°ΠΊ Π½Π°Π΄Π΅ΠΆΠ΄Ρƒ Π½Π° созданиС практичСских свСрхпроводящих систСм, ΠΊΠΎΡ‚ΠΎΡ€Ρ‹Π΅ обСспСчат Π²Ρ‹ΡΠΎΠΊΡƒΡŽ ΡΡ„Ρ„Π΅ΠΊΡ‚ΠΈΠ²Π½ΠΎΡΡ‚ΡŒ производства элСктроэнСргии. Π‘ Ρ‚Π΅Ρ… ΠΏΠΎΡ€ прогрСсс Π½Π° этом Ρ„Ρ€ΠΎΠ½Ρ‚Π΅ Π±Ρ‹Π» ΠΌΠ΅Π΄Π»Π΅Π½Π½Ρ‹ΠΌ, ΠΏΠΎ ΠΊΡ€Π°ΠΉΠ½Π΅ΠΉ ΠΌΠ΅Ρ€Π΅, с Ρ‚ΠΎΡ‡ΠΊΠΈ зрСния Π½Π΅ΠΊΠΎΡ‚ΠΎΡ€Ρ‹Ρ… ΠΎΠΆΠΈΠ΄Π°Π½ΠΈΠΉ послС этого открытия. Π’Ρ‹ ΠΌΠΎΠΆΠ΅Ρ‚Π΅ Π΄ΡƒΠΌΠ°Ρ‚ΡŒ ΠΎ свСрхпроводящСй Ρ†Π΅ΠΏΠΈ ΠΊΠ°ΠΊ ΠΎ амСриканских Π³ΠΎΡ€ΠΊΠ°Ρ…, ΠΊΠΎΡ‚ΠΎΡ€Ρ‹ΠΌ Π½Π΅ трСбуСтся энСргия для запуска ΠΊΠ°ΠΆΠ΄ΠΎΠ³ΠΎ ΠΏΠΎΡΠ»Π΅Π΄ΡƒΡŽΡ‰Π΅Π³ΠΎ Ρ†ΠΈΠΊΠ»Π° послС добавлСния Π½Π°Ρ‡Π°Π»ΡŒΠ½ΠΎΠ³ΠΎ ΠΏΠΎΡ‚Π΅Π½Ρ†ΠΈΠ°Π»Π°.

    ΠŸΡ€ΠΈΠ²Π΅Π΄Π΅Π½Π½Ρ‹ΠΉ Π²Ρ‹ΡˆΠ΅ ΠΈΠ½Ρ‚Π΅Ρ€Π°ΠΊΡ‚ΠΈΠ²Π½Ρ‹ΠΉ PhET позволяСт классу ΡΠΎΠ·Π΄Π°Π²Π°Ρ‚ΡŒ свои собствСнныС схСмы ΠΈ Π°Π½Π°Π»ΠΈΠ·ΠΈΡ€ΠΎΠ²Π°Ρ‚ΡŒ взаимосвязь ΠΌΠ΅ΠΆΠ΄Ρƒ Ρ‚ΠΎΠΊΠΎΠΌ, напряТСниСм ΠΈ сопротивлСниСм. Π Π°Π΄ΠΈ дСмонстрационного ΡƒΡ€ΠΎΠΊΠ° я использовал ΠΈΠ½Ρ‚Π΅Ρ€Π°ΠΊΡ‚ΠΈΠ²Π½Ρ‹ΠΉ класс Ρ„ΠΈΠ·ΠΈΠΊΠΈ, Ρ‚Π°ΠΊ ΠΊΠ°ΠΊ Π΅Π³ΠΎ Π½Π΅ΠΌΠ½ΠΎΠ³ΠΎ ΠΏΡ€ΠΎΡ‰Π΅ Π½Π°ΡΡ‚Ρ€ΠΎΠΈΡ‚ΡŒ ΠΈ Π·Π°ΠΏΡƒΡΡ‚ΠΈΡ‚ΡŒ, учитывая ограничСния, связанныС с дСмонстрационным ΡƒΡ€ΠΎΠΊΠΎΠΌ. Π’ долгосрочной пСрспСктивС ΠΈΠ½Ρ‚Π΅Ρ€Π°ΠΊΡ‚ΠΈΠ² PhET Π±ΠΎΠ»Π΅Π΅ Π½Π°Π΄Π΅ΠΆΠ΅Π½. И Ρ‚ΠΎ, ΠΈ Π΄Ρ€ΡƒΠ³ΠΎΠ΅ ΠΏΠΎΠ·Π²ΠΎΠ»ΠΈΡ‚ учащСмуся Ρ€Π΅Π³ΡƒΠ»ΠΈΡ€ΠΎΠ²Π°Ρ‚ΡŒ напряТСниС ΠΈ Ρ‚ΠΎΠΊ, Ρ‡Ρ‚ΠΎΠ±Ρ‹ ΡƒΠ²ΠΈΠ΄Π΅Ρ‚ΡŒ, ΠΊΠ°ΠΊ это влияСт Π½Π° схСму.

    ΠšΠ»ΡŽΡ‡Π΅Π²Ρ‹ΠΌΠΈ ΠΌΠΎΠΌΠ΅Π½Ρ‚Π°ΠΌΠΈ для изучСния классом ΡΠ²Π»ΡΡŽΡ‚ΡΡ:

    ЦСпь Π΄ΠΎΠ»ΠΆΠ½Π° ΠΏΡ€Π΅Π΄ΡΡ‚Π°Π²Π»ΡΡ‚ΡŒ собой Π·Π°ΠΌΠΊΠ½ΡƒΡ‚Ρ‹ΠΉ ΠΊΠΎΠ½Ρ‚ΡƒΡ€ ΠΎΡ‚ ΠΎΠ΄Π½ΠΎΠΉ ΠΊΠ»Π΅ΠΌΠΌΡ‹ Π±Π°Ρ‚Π°Ρ€Π΅ΠΈ Π΄ΠΎ Π΄Ρ€ΡƒΠ³ΠΎΠΉ, Ρ‡Ρ‚ΠΎΠ±Ρ‹ ΠΏΡ€ΠΎΡ‚Π΅ΠΊΠ°Π» Ρ‚ΠΎΠΊ. ΠŸΠ΅Ρ€Π΅ΠΊΠ»ΡŽΡ‡Π°Ρ‚Π΅Π»ΡŒ Π² Π²Ρ‹ΠΊΠ»ΡŽΡ‡Π΅Π½Π½ΠΎΠΌ ΠΏΠΎΠ»ΠΎΠΆΠ΅Π½ΠΈΠΈ Ρ€Π°Π·Ρ€Ρ‹Π²Π°Π΅Ρ‚ ΠΏΠ΅Ρ‚Π»ΡŽ, Π° Π² ΠΏΠΎΠ»ΠΎΠΆΠ΅Π½ΠΈΠΈ Β«Π²ΠΊΠ»ΡŽΡ‡Π΅Π½ΠΎΒ» ΠΏΠ΅Ρ‚Π»ΡŽ Π·Π°ΠΌΡ‹ΠΊΠ°Π΅Ρ‚. ΠšΠ»ΡŽΡ‡ заТигания автомобиля выполняСт Ρ‚Ρƒ ΠΆΠ΅ Ρ„ΡƒΠ½ΠΊΡ†ΠΈΡŽ.

    Π§Ρ‚ΠΎΠ±Ρ‹ Ρ‚ΠΎΠΊ ΠΏΡ€ΠΎΡ‚Π΅ΠΊΠ°Π», ΠΊ Ρ†Π΅ΠΏΠΈ Π½Π΅ΠΎΠ±Ρ…ΠΎΠ΄ΠΈΠΌΠΎ ΠΏΡ€ΠΈΠ»ΠΎΠΆΠΈΡ‚ΡŒ ΠΏΠΎΡ‚Π΅Π½Ρ†ΠΈΠ°Π» ΠΈΠ»ΠΈ напряТСниС. Π’ ΠΏΡ€ΠΎΡ‚ΠΈΠ²Π½ΠΎΠΌ случаС это Π±Ρ‹Π»ΠΎ Π±Ρ‹ ΠΏΠΎΡ…ΠΎΠΆΠ΅ Π½Π° ΠΏΠΎΠΏΡ‹Ρ‚ΠΊΡƒ ΠΏΡ€ΠΎΠΊΠ°Ρ‚ΠΈΡ‚ΡŒΡΡ Π½Π° плоских амСриканских Π³ΠΎΡ€ΠΊΠ°Ρ….

    НапряТСниС ΠΈΠ»ΠΈ ΠΏΠΎΡ‚Π΅Π½Ρ†ΠΈΠ°Π» Π±ΡƒΠ΄ΡƒΡ‚ ΠΏΠ°Π΄Π°Ρ‚ΡŒ ΠΏΠΎ ΠΌΠ΅Ρ€Π΅ прохоТдСния Ρ‚ΠΎΠΊΠ° ΠΏΠΎ ΠΊΠΎΠ½Ρ‚ΡƒΡ€Ρƒ. Π­Ρ‚ΠΎ Π°Π½Π°Π»ΠΎΠ³ΠΈΡ‡Π½ΠΎ амСриканским Π³ΠΎΡ€ΠΊΠ°ΠΌ, ΡΠ½ΠΈΠΆΠ°ΡŽΡ‰ΠΈΠΌΡΡ ΠΏΠΎ высотС (ΠΈ ΠΏΠΎΡ‚Π΅Π½Ρ†ΠΈΠ°Π»ΡŒΠ½ΠΎΠΉ энСргии) ΠΏΠΎ ΠΌΠ΅Ρ€Π΅ Ρ‚ΠΎΠ³ΠΎ, ΠΊΠ°ΠΊ ΠΎΠ½ΠΈ Π·Π°Π²Π΅Ρ€ΡˆΠ°ΡŽΡ‚ ΠΏΠΎΠ΅Π·Π΄ΠΊΡƒ, Ρ‡Ρ‚ΠΎΠ±Ρ‹ Π² ΠΊΠΎΠ½Π΅Ρ‡Π½ΠΎΠΌ ΠΈΡ‚ΠΎΠ³Π΅ ΠΎΡΡ‚Π°Π½ΠΎΠ²ΠΈΡ‚ΡŒΡΡ Π½Π° Π·Π΅ΠΌΠ»Π΅.

    ΠŸΡ€ΠΈ ΡƒΠ²Π΅Π»ΠΈΡ‡Π΅Π½ΠΈΠΈ напряТСния Ρ‚ΠΎΠΊ увСличиваСтся, Π° ΠΏΡ€ΠΈ ΡƒΠ²Π΅Π»ΠΈΡ‡Π΅Π½ΠΈΠΈ сопротивлСния Ρ‚ΠΎΠΊ ΡƒΠΌΠ΅Π½ΡŒΡˆΠ°Π΅Ρ‚ΡΡ.

    alexxlab

    Π”ΠΎΠ±Π°Π²ΠΈΡ‚ΡŒ ΠΊΠΎΠΌΠΌΠ΅Π½Ρ‚Π°Ρ€ΠΈΠΉ

    Π’Π°Ρˆ адрСс email Π½Π΅ Π±ΡƒΠ΄Π΅Ρ‚ ΠΎΠΏΡƒΠ±Π»ΠΈΠΊΠΎΠ²Π°Π½. ΠžΠ±ΡΠ·Π°Ρ‚Π΅Π»ΡŒΠ½Ρ‹Π΅ поля ΠΏΠΎΠΌΠ΅Ρ‡Π΅Π½Ρ‹ *